You are on page 1of 235

By Vishal sir

Also Important For SBI CLERK, IBPS CLERK, RRB CLERK


& Other Competitive Exams

Facebook Page- https://www.facebook.com/vishalpariharpage Youtube- https://www.youtube.com/vishalparihar


Follow
1 Vishal Sir Telegram Channel- https://t.me/englishbyvishalsirchannel Instagram- https://www.instagram.com/vishalthetrainer
By Vishal sir
Also Important For SBI CLERK, IBPS CLERK, RRB CLERK
& Other Competitive Exams

Facebook Page- https://www.facebook.com/vishalpariharpage Youtube- https://www.youtube.com/vishalparihar


Follow
2 Vishal Sir Telegram Channel- https://t.me/englishbyvishalsirchannel Instagram- https://www.instagram.com/vishalthetrainer
By Vishal sir
Also Important For SBI CLERK, IBPS CLERK, RRB CLERK
& Other Competitive Exams

Dear Students,

In these times of immense competition, success is not something that’s easier to


achieve. You have to excel in your preparation and perform exceptionally well at various
stages. And to achieve this you need the best material present out there.

With an initiative to provide you with the best, Vishal Parihar has come with this new
book 1000+ Clerk Pre for all competitive exams. The questions in this series were found by
analysis of a collection of different previous year papers from different competitive
examinations like SBI, IBPS, CAT, MAT, and various insurance examinations. Each set of
questions in this book is a high-frequency question. This means that the effort in solving the
question is well repaid by the number of times learners have a chance to encounter or solve
them. You can rely on this book to improve your solving ability that will help you to achieve
better scores in the examinations.

We would like to take this opportunity to thank our team (Bhavesh Patni, Aanchal Singh,
Rishika Jain) for their relentless efforts towards the design and the quality content of the
book.

Good Luck!

Vishal Parihar

Facebook Page- https://www.facebook.com/vishalpariharpage Youtube- https://www.youtube.com/vishalparihar


Follow
3 Vishal Sir Telegram Channel- https://t.me/englishbyvishalsirchannel Instagram- https://www.instagram.com/vishalthetrainer
By Vishal sir
Also Important For SBI CLERK, IBPS CLERK, RRB CLERK
& Other Competitive Exams

Dedication

I dedicate this book "my parents

and my family" for their relentless

support. Hope this book will help

students to achieve their goal.

Facebook Page- https://www.facebook.com/vishalpariharpage Youtube- https://www.youtube.com/vishalparihar


Follow
4 Vishal Sir Telegram Channel- https://t.me/englishbyvishalsirchannel Instagram- https://www.instagram.com/vishalthetrainer
By Vishal sir
Also Important For SBI CLERK, IBPS CLERK, RRB CLERK
& Other Competitive Exams

The information you will find in this book and any publications or other study materials published by Vishal

Parihar is for information only, which is to be used for learning English grammar.

Tips, strategies, methods of study, lessons and all the resources are only recommendations for the aspirants,

and reading any information in this book is provided ‘as is’.

The whole book is made keeping in mind, all the previous years questions asked in bank & other
relevant exams and expected to be asked in the upcoming exams. The author has made reasonable efforts to
provide current and accurate information to his readers. The author will not be held liable for any
unintentional errors or omissions that may be found.

Facebook Page- https://www.facebook.com/vishalpariharpage Youtube- https://www.youtube.com/vishalparihar


Follow
5 Vishal Sir Telegram Channel- https://t.me/englishbyvishalsirchannel Instagram- https://www.instagram.com/vishalthetrainer
By Vishal sir
Also Important For SBI CLERK, IBPS CLERK, RRB CLERK
& Other Competitive Exams

Direction: (Q. No. 1- 100): The sentence given below, have 11. (a) The murderer was convicted/ (b) and ordered to/ (c) be hung.
four parts, indicated by (a), (b), (c) and (d). Read each /(d) No error.
sentence to find out whether there is any error. If a sentence
has no error, indicate the part (e), which stands for 12. (a) The Trinamool Congress government / b) is all set to issue an
‘No error’. (Ignore the error of punctuation, if any. order changing the/ c) very nature /d) of students. e) No error.

1. There were a great many question (a) / which were (b) / left 13. Myself and Sumit (a) / will take care of (b) / the function on
unanswered (c) / at the end of the enquiry (d) / No error (e). Tuesday. (c) / No error (d)

2. a) Agriculture is the backbone of the/b) Indian economy but the 14. (a)The company has appointed / (b) consultants to help them / (c)
sad part/c) was that it is still in a rudimentary stage,/d) which increase its revenue / (d) no error.
ultimately affects the overall economy/e) No error.
15. Women after women (a) / spoke against (b) / the practice (c) / of
3. a) The proposed INO project, /b)primarily aims to study dowry. (d) / No Error (e)
atmospheric neutrinos/c) in a 1300 m deep cavern in/d) the Bodi
West Hills in Tamil Nadu /e) No error. 16. I tried to tell him a number of times (a) that it was only me at the
door but he (b) kept on shouting and suspecting (c) it to be one of
4. You cannot (a)/ master physics (b)/ without learning the (c)/ the men of the club.(d) No error (e)
formula that govern it. (d) / No Error (e).
17. (a) The IT and Innovation Fund/ b) promote Indian- Russian
5. a) The rail coach has been / b) shifted near the station/c) and cooperation/ c) in the area of IT Technologies/ d) and innovation/
would be developed/ d) as a unique restaurant/ e) No error. e) No error.

6. A shoal (a) / of wolves (b) / attacked (c) / the travelers. (d)/ No 18. (a) I, you and she / (b) were accused / (c) of committing robbery
error (e) in broad daylight / (d) no error.

7. (a) The Government cannot/ (b) be able to contain/ (c) 19. (a) He is not one of those / (b) who will help every Tom, Dick
Encephalitis by spending just 1% of GDP on Health sector. /(d) and Harry / (c) whom he meets / (d) no error.
No error.
20. (a) There are forces, / b) in our neighboring countries,/ c)which
8. a) Heavy Hails in many / b)parts of apple growing/ c) areas also are trying to misguide and/ d) mislead the youth of Kashmir/ e)
resulted in/ d) huge losses to farmers/ e) No error. No error.

9. Just to the North of India (a) / is the Himalayas (b) / that were 21. The Chief Minister said that (a)/ his party would not repeat the
once impregnable (c) / No error (d) mistakes (b)/ done by the (c)/previous government. (d)/No error
(e)
10. The price of (a)/ production in the (b)/ industrial sector (c)/ has
gone up. (d)/ No Error 22. (a) No one can / (b) challenge death / (c) isn’t it? /(d) No error.
(e).

Facebook Page- https://www.facebook.com/vishalpariharpage Youtube- https://www.youtube.com/vishalparihar


Follow
6 Vishal Sir Telegram Channel- https://t.me/englishbyvishalsirchannel Instagram- https://www.instagram.com/vishalthetrainer
By Vishal sir
Also Important For SBI CLERK, IBPS CLERK, RRB CLERK
& Other Competitive Exams

ANSWER WITH SOLUTION Ans.14. (b) ‘Company’ is a singular noun. Pronoun ‘It’ will be used at the
place of ‘them’.
Ans.1. (a) ‘A great many’ is always followed by a plural noun and a plural
verb. So the noun ‘question’ has to be changed to ‘questions’. Ans.15. (a) In cases of the form, “Noun + Preposition + Noun”, noun is
always singular. It should be ‘woman after woman’.
Ans.2. (c) Use ‘is’ in place of ‘was’.
Ans.16. (b) Error is in part (b) where a wrong pronoun ‘me’ has been used
Ans.3. (e) No error. and need to be replaced by ‘I’ to make it grammatically correct.

Ans.4. (d) Replace 'formula' by 'formulae'. Some nouns have more than Ans.17. (b) Use ‘will’ before promote.
one plural form with different meanings. Formulae - Plural,
meaning- Rules of Physics. Ans.18. (a) ‘you, she & I’ should come in place of ‘I, you and she’.

Ans.5. (a) Use of ‘a’ in place of ‘the’. Ans.19. (c) Write ‘they meet’ in place of ‘he meets’.

Ans.6. (a) Replace ‘a shoal of wolves’ by ‘a pack of wolves’. Ans.20. (a) Use ‘some’ before “forces”.

Ans.7. (b) Remove ‘be able to’ as ‘can’ and ‘could’ represent same Ans.21. (c) Replace ‘done’ by ‘made’. The correct light verb for mistake
meaning ‘as be able to’. is ‘make’ and ‘do’ is the wrong light verb here.

Ans.8. (a) Replace ‘Hails’ with ‘Hail’. Ans.22. (c) Replace ‘isn’t it’ by ‘can they’.

Ans.9. (b) Replace ‘is’ by ‘are’. ‘The Himalayas’ takes a plural verb
because, when we mention the names of these mountains, we
immediately imagine a vast range of mountains.

Ans.10. (a) Replace 'price' with 'cost'. Price denotes amount paid by the
customer, cost denotes amount paid by the
manufacturer/shopkeeper.

Ans.11. (c) Replace ‘hung’ with ‘hanged’. Hung means to suspend and
hanged means death penalty.

Ans.12. (e) No error.

Ans.13. (a) Replace ‘myself’ by ‘I’.

Facebook Page- https://www.facebook.com/vishalpariharpage Youtube- https://www.youtube.com/vishalparihar


Follow
7 Vishal Sir Telegram Channel- https://t.me/englishbyvishalsirchannel Instagram- https://www.instagram.com/vishalthetrainer
By Vishal sir
Also Important For SBI CLERK, IBPS CLERK, RRB CLERK
& Other Competitive Exams

23. They that run (a) / after money and publicity (b) / are likely to be 35. Our destination (a) / was about (b) / fifty and sixty (c) / kilometers
disappointed (c) / No error (d) to the west. (d) / No Error (e)

24. Mohit (a)/ handed over (b) / a hundred rupees note (c) / to the 36. a) The operators argued b)that telcos offers below cost tariffs/ c)
shopkeeper. (d)/ No error (e) to consumers over a period of time/ d) may harm the industry and
its financials / e) No error.
25. The period of twenty-five years (a) / have passed (b) / and still he
is without a job (c) / No error (d) 37. a) Due to certain differences / b) between the companies with the
proposed investors,/ c) both parties has decided/ d) to terminate
26. a) The rules notified by the Ministry of Labour / b)and the arrangement / e) No error.
Employment of the enforcement of the / c)2016 amendment
include/ d) some small concessions/ e) No error. 38. He has lots (a) / of money (b) / and he dare not (c) / start a
company. (d) / No Error (e)
27. Privatisation offers the most (a) / ideal situation for consumers
because (b) / Private sector is very (c) / conscious of quality (d) / 39. She ordered (a) / certain girl to (b) / make entry into (c) / the
No error (e) Manager's office. (d) / No Error (e).

28. It is disappointing not only (a)/to the players and spectators, but 40. a)One may soon be able to avail automatic transfer/ b) of
to (b)/ the organisers and advertisers as well as (c)/ who stand to provident fund accounts on switching jobs / c)and get provident
lose financially. (d)/ No error (e) fund money within / d)a single day of filing the claim/ e) No error

29. No less than (a) / hundred audiences are sitting (b) / in the hall (c) 41. a) Incidents of / b) threats of violence/ c) against politicians have/
/ and waiting for the final performance. (d) / No Error (e). d) been at the rise /e) No error.

30. I can't allow (a) / you to go out of the class (b) / unless they 42. The crucial point to (a)/ be discussed at the (b)/ meeting is how
provide (c) / further instructions. (d) / No Error (e) to (c)/ well implement the policy. (d)/ No error (e)

31. You must admire (a) / the artwork (b) / when your brother (c) / 43. Mahatma Gandhi tried to eradicate (a) / social evils with (b) /
has done it. (d) / No Error (e) tooth and nail (c) / No error (d)

32. (a) It was in 1995 / (b) that we first flew / (c) to the Canada. / (d) 44. a) A journalist later on broadcast/ b) the recording saying because
No error. he / c) had not been at the event; / d) he was not bound by the
confidentiality rule/ e) No error.
33. (a) Nakul said impatiently, / (b) "Let I do this. / (c) None of you
can do this.” / (d) no error. 45. Little care on your part (a) / would have made you (b) / more
successful than your friend (c) / No error (d)
34. a)These restrictions/ b) are intends to/c) ensure attendance/ d) at
school / e) No error. 46. The programme was (a) / almost fixed though (b) / it was his
negligence (c) / that led to disappointment. (d) / No Error (e)

Facebook Page- https://www.facebook.com/vishalpariharpage Youtube- https://www.youtube.com/vishalparihar


Follow
8 Vishal Sir Telegram Channel- https://t.me/englishbyvishalsirchannel Instagram- https://www.instagram.com/vishalthetrainer
By Vishal sir
Also Important For SBI CLERK, IBPS CLERK, RRB CLERK
& Other Competitive Exams

ANSWER WITH SOLUTION Ans.35. (c) Replace 'and' with 'or'. In this case, destination cannot be both
'fifty and sixty' kilometers at the same time. The intention here
to convey, either one of the two.

Ans.23. (a) Replace ‘they’ by ‘those’.


Ans.36. (b) Replace ‘offers’ with ‘offering’.

Ans.24. (c) Replace ‘a hundred rupees note’ with ‘a hundred rupee note’.
Ans.37. (c) Use ‘Have’ in place of ‘has’.

Ans.25. (b) Subject here is ‘The period’ which takes singular verb, so
replace ‘have’ by ‘has’. Ans.38. (c) Replace 'and' with 'but'. To show two contrasting qualities of
same subject, use of 'but' is preferred rather than 'and'.

Ans.26. (b) Use ‘Employment for the’ in place of ‘Employment of the’


Ans.39. (b) Replace ‘certain’ with ‘a certain'.

Ans.27. (a) Delete ‘most’ as ideal is an absolute word, which means ‘The
most appropriate’. Ans.40. (e) No error.

Ans.28. (b) Replace ‘but’ by ‘but also’. Whenever we use ‘not only’, the Ans.41. (d) Replace ‘at’ with ‘on’.
other part of this correlative conjunction is ‘but also’.
Ans.42. (d) Implement is the verb and well is the adverb so ‘well’ must be
Ans.29. (a) Replace 'less' with 'fewer'. 'Less' denotes 'in small degree'. placed after ‘implement’. In this way the sentence would be
'Fewer' denotes 'number'. like “how to implement the policy well”.

Ans.30. (c) Replace 'unless' with 'until'. ‘Until’ denotes sense of time, Ans.43.(b) Delete ‘with’. The sentence should be like “Mahatma Gandhi
whereas ‘Unless’ denotes condition. In this case, ‘Until’ must tried to eradicate social evils tooth and nail”.
be used.
Ans.44. (a) Remove “on”.
Ans.31. (c) Replace 'when' with 'if'. To express condition we use 'if'.'When'
would mean admire whenever brother has done the artwork, Ans.45. (a) Add ‘a’ before ‘little’. ”A little" is positive and it means the
that is not the intention here. same as "a small amount of". "Little" is negative and it means
the same as "not much".
Ans.32. (b) Use ‘when’ in place of ‘that’ as the sentence is talking about
time. Ans.46. (b) Replace 'though' with 'but'. To show contrasting qualities, 'but'
is used.
Ans.33. (b) Replace ‘Let I’ by ‘Let me’. ‘Let’ is always followed by
objective case pronouns.

Ans.34. (b) Use ‘intended’ in place of ‘intends’.

Facebook Page- https://www.facebook.com/vishalpariharpage Youtube- https://www.youtube.com/vishalparihar


Follow
9 Vishal Sir Telegram Channel- https://t.me/englishbyvishalsirchannel Instagram- https://www.instagram.com/vishalthetrainer
By Vishal sir
Also Important For SBI CLERK, IBPS CLERK, RRB CLERK
& Other Competitive Exams

47. a) The Bill required each public authority to establish / b) 59. a) By contrast, Senate Republicans/b) have significantly altered
information and facilitation center for an efficient and effective / the House’s approach /c)to the individual market, who serves
c) delivery of services and / d) redressal of grievances / e) No those /d)who buy health insurance for themselves/ e) No error.
error.
60. Hardly had she (a) / seen her aunt (b) / than she stopped (c) /
48. From June 1995 (a) / and June 2005, (b) / the city's population fighting. (d) / No Error (e)
(c) / declined by 20,000 people. (d) / No Error (e)
61. I was surprised (a) / to see her speak (b) / with somewhat anger
49. Both Sameer as well as (a) / his wife are (b) / determined (c) / to (c) / No error (d)
make changes. (d) / No Error (e)
62. a) One of the few media companies / b)who dares to criticise the
50. I haven't been (a) / to Pune before and (b) / neither my brother. government has been / c)raided by police on grounds that would
(c) / No Error (d) not/d) normally attract such heavy-handedness/ e) No error.

51. If I knew (a)/that my friend had to visit the town today, (b)/ I 63. Kajol cannot ask (a) / her father (b) / for his car without (c) / any
would have make his stay vividly reason (d) / No error. (e)
comfortable. (c)/No error (d)
64. a) It is also time for some / b) introspection with the/ c) judiciary
52. Could you (a) / give me any money (b) / to buy food? (c) / No on the manner in/d) which judges are chosen/ e) No error.
error (d)
65. Prince (a) / saw (b) / a crew of ship (c) / in the harbor. (d)/ No
53. I never remember (a) / to have read (b) / a more interesting book error (e)
in my life (c) / No error
(d) 66. The writer (a) / got his wife’s permittance (b) / to go (c) / to the
cinema. (d) / No error (e)
54. Though he was brave, (a) / he could not face the ups and downs
(b) /of life manly.(c) / No error (d) 67. She has bought (a) / a dozen of copies (b) / of English Literature
(c) / books. (d) / No Error (e).
55. My brother is elder (a) / than me although(b) / he looks younger.
(c) / No error (d) 68. Some psychologists maintain (a) / that a child who has seen
violence (b) /on television is more likely to react (c) / violent in
56. I did not (a) / steal the oranges (b) / from your orchard (c) / never situations of stress. (d) / No error (e).
did my brother. (d) / No Error (e)
69. a) The change ends up uncertainty/ b) over succession and
57. That store (a) / hadn't hardly (b) / any of the (c) / required empowers Prince Mohammad/ c) to move faster with his plan to
products. (d) / No Error (e) reduce/ d) the kingdom’s dependence on oil /e) No error.

58. a) I wish to continue / b) this hobby until I collect / c) all the 70. My son is a nature-lover. (a) / He’d rather play in the forests (b) /
currency notes / d) of all the countries / e) No error. then watch movies on the television. (c) / No error (d).

Facebook Page- https://www.facebook.com/vishalpariharpage Youtube- https://www.youtube.com/vishalparihar


Follow
10 Vishal Sir Telegram Channel- https://t.me/englishbyvishalsirchannel Instagram- https://www.instagram.com/vishalthetrainer
By Vishal sir
Also Important For SBI CLERK, IBPS CLERK, RRB CLERK
& Other Competitive Exams

ANSWER WITH SOLUTION Ans.59. (c) Use ‘which’ in place of ’who’.

Ans.60.(c) 'Hardly' is always followed by 'when'. 'When' cannot be replaced


Ans.47. (a) Replace ‘each’ with ‘every’. by any other word.

Ans.48. (b) Replace 'and' with 'to'. 'From' is always followed by 'to'. The Ans.61. (c) Replace ‘somewhat’ by ‘some’.
correct sentence should be “From June 1995 to June 2005, the
city's population declined by 20,000 people.” Ans.62. (b) Use of ‘that’ in place of ‘who’.

Ans.49. (a) Replace 'as well as' with 'and'. 'Both..and' is used in pair (Co- Ans.63. (d) Replace ‘vividly’ with ‘vivid’. Here ‘vividly’ is an adverb
relative Conjunction). which cannot qualify a noun ‘reason’ and an adjective is
needed here.
Ans.50. (c) Add 'has' after 'neither'. 'Neither' is followed by 'helping verb'
and then the 'subject' i.e. 'neither has my brother'. Ans.64. (b) Use ‘within’ in place of ‘with’.

Ans.51. (c) The correct usage is ‘Would + have + made’. So replace ‘make’ Ans.65. (c) ‘A fleet of ships’ is the right usage.
by ‘made’.
Ans66. (b) write ‘permission’ at the place of ‘permittance’.
Ans.52. (b) Replace ‘any’ by ‘some’. Use some in affirmative and any in
negative and interrogative sentences. Ans.67. (b) The correct expression should be - 'a dozen copies' or 'dozens
of copies'.
Ans.53. (a) Replace ‘never’ by ‘do not’.
Ans.68. (d) Replace ‘violent’ by ‘violently’ as violent is intended to modify
Ans.54. (c) Replace ‘manly’ by ‘manually’ as ‘Manly’ is adjective, it ‘to react’ a veb form. So the adverb violently is required.
means brave / strong.
Ans.69 (a) Remove the word ‘up’ before the uncertainty.
Ans.55. (a) Replace ‘elder’ by ’older’.
Ans.70. (c) Replace ‘then’ by ‘than’. ‘Rather-than’ is the correct correlative
Ans.56. (d) In some cases, Neither can be substituted by 'Do/Does not'. In conjunction to be used.
that case, both clauses must have 'do/does'. Replace 'never'
with 'nor'.

Ans.57. (b) 'Hardly' itself conveys a negative sense. Use of 'had not' is not
required in this case.

Ans.58. (b) Replace “Until” with “till”.

Facebook Page- https://www.facebook.com/vishalpariharpage Youtube- https://www.youtube.com/vishalparihar


Follow
11 Vishal Sir Telegram Channel- https://t.me/englishbyvishalsirchannel Instagram- https://www.instagram.com/vishalthetrainer
By Vishal sir
Also Important For SBI CLERK, IBPS CLERK, RRB CLERK
& Other Competitive Exams

83. a) We have no clue /b) with the amount of information /c)about


71. a)Effectively, a book is banned / b) without a hearing / c) The our private lives/d) that is out in the web.
book then stays banned / d)until the case is not completed / e) No 84. (a) My cousin sister is a teacher / (b) and she / (c) teaches her
error children and friends too. / (d) No error.

72. (a) Our knowledge of history does not come to our help/ (b) and 85. a)The report noted that /b)output along the manufacturing/c)
sometimes we even fail to remember/ (c) who invented America. sector rose for the sixth consecutive/d) month in June/e) No error.
/(d) No error.
86. The crucial point to (a)/ be discussed at the (b)/ meeting is how
73. She instructed me (a) / to write the (b) / application in (c) / capital to (c)/ well implement the policy. (d)/ No error (e)
letter. (d) / No Error
(e). 87. a) In 23 years, the court did not /b) interpret the right to life as a
principle of natural justice,/c) but as a fragile right that could be
74. a) We want almost/b) every car and van /c) to be zero emission /d) taken away by arbitrary legislation /e) No error.
/d) till 2050/ e) No error.
88. As such, banks play an under – appreciated role in the economy.
75. a) Patience / b) as well as / c) perseverance are / d) necessary for / (a) To some extent, bank loan officers / (b) decide whom
success / e) No error. projects, and / or businesses, / (c) are worth pursuing and are
deserving of capital. /(d) No error / (e).
76. These kind (a) / of shoes (b) / are rather expensive (c) / for her to
buy. (d)/ No error (e) 89. The teacher helped everyone / (a) of those boys / (b) in doing their
work. / (c) No error / (d)
77. Hemant could (a) / only eat raw food (b) / because of digestion
issues (c)/ since his childhood. (d) / No Error (e) 90. a) Beside allowing /b) potential investors to/c) make informed
decisions, /d) this will open up a new sector in India /e) No error.
78. Raghav has not seldom (a) / visited his parents (b) / since he left
this place (c) / No error (d). 91. Although she had been hired (a) / by the magazine to write book
reviews (b) / she knew scarcely nothing (c) / about current fiction
79. a) We ought to /b) honest/c) in our/d) dealings. (e) No Error. (d) No error (e).

80. a)Any badly located /b) city can be vulnerable /c)merely with 92. a)However, he declared /b)that despite of the/c) U.S. declaration
virtue /d)of its location/e) No error. the freedom /d)struggle for Kashmir would continue /e) No error.

81. It had been too cold (a) / the whole month and we preferred (b) / 93. (a) I have practiced so much / (b) for the singing competition that
to stay in the plains (c) / No error (d) there/ (c) is no question of / (d) mine coming second. (e) No error.

82. Darts are (a) / played by (b) / men as well (c) / as women. (d) / 94. a) Very few young trainees/ b) willingly undertakes / c) a posting
No Error (e). to a branch/ d) located in a rural area/ e) No error.

Facebook Page- https://www.facebook.com/vishalpariharpage Youtube- https://www.youtube.com/vishalparihar


Follow
12 Vishal Sir Telegram Channel- https://t.me/englishbyvishalsirchannel Instagram- https://www.instagram.com/vishalthetrainer
By Vishal sir
Also Important For SBI CLERK, IBPS CLERK, RRB CLERK
& Other Competitive Exams

ANSWER WITH SOLUTION Ans.83 (b) Use “to” in place “with”.

Ans.84. (a) Remove ‘sister’ after ‘cousin’. As ‘cousin brother’ or ‘cousin


Ans.71. (d) Remove “not” from the sentence, as after the word “until” use sister’ is the wrong usage.
of not is wrong.
Ans.85 (b) Replace “along” with “across”.
Ans.72. (c) Replace ‘invented’ with ‘Discovered’ which means find
something during a search. Ans.86. (d) Implement is the verb and well is the adverb so ‘well’ must be
placed after ‘implement’. In
Ans.73. (d) The correct usage is 'letters'. The intention here is to convey
that the complete application is to be written in block (capital) this way the sentence would be like “how to implement the policy well”.
letters.
Ans.87. (a) Replace “in” with “for”.
Ans.74. (d) Replace “till” with “by”.
Ans.88. (c) ‘Which’ should replace ‘whom’ as the sentence is talking about
Ans.75. (c) Replace “are” with “is”. When we combine two subjects with projects.
‘as well as’, verb is in accordance with the noun before ‘as well
as’. Ans.89. (c) Use ‘In doing his work’ at the place of ‘in doing their work’.

Ans.76. (a) Replace ‘kind’ by ‘kinds’. Ans.90 (a) Use “besides” in place of “beside”.

Ans.77. (b) Replace only eat by “Eat only” as ‘Only’ should be placed Ans.91. (c) ‘Scarcely nothing’ is a double negative. The sentence must be
immediately before the word it qualifies. read ‘Scarcely anything’.

Ans.78. (a) Delete ‘not’ as seldom itself means not often, rarely. Ans.92 (b) Remove “of” from the sentence.

Ans.79 (a) Add “be” after “ought to“. Ans.93. (d) ‘My’ will be used. Pronoun coming before ‘V- ing’ remains in
the possessive form.
Ans.80 (c) Use “by” in place of “with”.
Ans.94. (b) Use ‘Undertake’ in place of ‘Undertakes’.
Ans.81. (a) Replace ‘too’ by ‘very’.

Ans.82. (a) 'Darts' is a noun, which 'looks' plural, but is used as 'singular'.
The correct sentence should be ‘Darts is played by men as well
as women.’

Facebook Page- https://www.facebook.com/vishalpariharpage Youtube- https://www.youtube.com/vishalparihar


Follow
13 Vishal Sir Telegram Channel- https://t.me/englishbyvishalsirchannel Instagram- https://www.instagram.com/vishalthetrainer
By Vishal sir
Also Important For SBI CLERK, IBPS CLERK, RRB CLERK
& Other Competitive Exams

95. a) It is necessarily to maintain/ b) a record of all transactions/ c) us to see all these bright objects of the night sky. The sun, the
in case the auditors/ d) want to see it/ e) No error. moon and all those objects shining in the night sky are called
celestial bodies.
96. Dear children, / (a) enjoy / (b) during the holidays / (c) No error.
(d) Some celestial bodies are very big and hot. They are made up of
gases. They have their own heat and light, which they emit in
97. a) The agreement on/ b) which all of us have/ c) worked so hard large amounts. These celestial bodies are called stars. The sun is
will/ d) be sign tomorrow/ e) No error. a star. Countless twinkling stars in the night sky are similar to the
sun. But we do not feel their heat or light, and they look so tiny
98. a) Since then, 180 militants has been /b) killed, both in the because they are very far from us.
Kashmir/c) Valley and on /d) the LOC. / e) No error.
You must have noticed that all objects look smaller when seen
99. (a) Though Raj appears / (b) to be very tall he is / (c) just an inch
from a distance. How small an aeroplane looks when it is flying
taller than me / (d) Noerror.
at a great height! While watching the night sky, you may notice
various patterns formed by different groups of stars. These are
100. Sooraj said (a) / it was (b) / quite all right (c) / to reject the offer.
called constellations. Ursa Major or Big Bear is one such
(d) / No Error (e)
constellation. One of the most easily recognizable constellation is
Directions (101-110): Read the below passage and answer the the Saptarishi (Sapta-seven, rishi-sages). It is a group of seven
following questions which are based on the passage. Some stars that forms a part of Ursa Major Constellation. Ask someone
words are highlighted. elder in your family or neighbourhood to show you more stars,
planets and constellations in the sky. In ancient times, people
used to determine directions during the night with the help of
How wonderful it is to watch the sky after sunset! One would first
stars. The North star indicates the north direction. It is also called
notice one or two bright dots shining in the sky. Soon you would
the Pole Star. It always remains in the same position in the sky.
see the number increasing. You cannot count them anymore. The
We can locate the position of the Pole Star with the help of the
whole sky is filled with tiny shining objects – some are bright,
others dim. It seems as if the sky is studded with diamonds. They Saptarishi.
all appear to be twinkling. But if you look at them carefully you
will notice that some of them do not twinkle as others do. They Some celestial bodies do not have their own heat and light. They
simply glow without any flicker just as the moon shines. are lit by the light of the stars. Such bodies are called planets. The
word ‘planet’ comes from the Greek word “Planetai” which
Along with these bright objects, you may also see the moon on means ‘wanderers’. The earth on which we live is a planet. It gets
most of the days. It may, however, appear at different times, in all its heat and light from the sun, which is our nearest star. If we
different shapes and at different positions. You can see the full look at the earth from a great distance, say the moon, it will
moon only once in about a month’s time. It is Full moon night or appear to be shining just as the moon. The moon that we see in
Poornima. A fortnight later, you cannot see it at all. It is a New the sky is a satellite. It is a companion of our earth and moves
moon night or Amavasya. On this day, you can watch the night round it. Like our earth, there are eight other planets that get heat
sky best, provided it is a clear night. Do you wonder why can’t and light from the sun. Some of them have their moons too.
we see the moon and all those bright tiny objects during day
time? It is because the very bright light of the sun does not allow

Facebook Page- https://www.facebook.com/vishalpariharpage Youtube- https://www.youtube.com/vishalparihar


Follow
14 Vishal Sir Telegram Channel- https://t.me/englishbyvishalsirchannel Instagram- https://www.instagram.com/vishalthetrainer
By Vishal sir
Also Important For SBI CLERK, IBPS CLERK, RRB CLERK
& Other Competitive Exams

ANSWER WITH SOLUTION

Ans.95 (a) ‘necessary’ in place of ‘necessarily’.

Ans.96. (c) Write ‘yourselves’ after ‘enjoy’.

Ans.97 (d) signed’ in place of ‘sign’.

Ans.98 (a) The correct usage is “have” in place of “has”.

Ans.99. (c) As comparison is being done with the subject not with object,
subject pronoun ‘I’ will come at the place of ‘me’.

Ans.100.(c) Remove quite. 'Quite all' cannot be used together. Either 'quite
right' or 'all right' should be used.

Facebook Page- https://www.facebook.com/vishalpariharpage Youtube- https://www.youtube.com/vishalparihar


Follow
15 Vishal Sir Telegram Channel- https://t.me/englishbyvishalsirchannel Instagram- https://www.instagram.com/vishalthetrainer
By Vishal sir
Also Important For SBI CLERK, IBPS CLERK, RRB CLERK
& Other Competitive Exams

Q101. What is being referred by the word ‘diamonds’ as highlighted in Q106. Which of the followings helps the people to determine the
the first paragraph? direction during the night?
(a) planets (a) Planets
(b) stars (b) Stars
(c) Sun (c) Moon
(d) celestial bodies (d) Sun
(e) meteors (e) None of the above

Q102. Why do stars look tiny? Directions (107-108): In each of the following questions, a
(a) they are very far away from us. word which is highlighted in the passage is given. choose the
(b) they are very tiny in shape option which has a meaning which is SIMILAR to the
(c) there is a black hole in between stars and earth which distort meaning to the given word.
the appearance of stars Q107. Flicker
(d) Both (a) and (b) (a) Twinkle
(e) None of above (b) accompany
(c) begin
Q103. Why does the Earth appear to shine when it is viewed from the (d) care
moon? (e) divide
(a) The Earth emits lights of its own.
(b) The Earth is lit by the lights from the Sun. Q108. Notice
(c) The Moon provides lights to the Earth. (a) Disappoint
(d) The Earth is lit by the lights from all the stars except the Sun. (b) Fall
(e) None of the above (c) Observe
(d) Give
Q104. What is the Saptarishi? (e) Hold
(a) A star
(b) A natural satellite Directions (109-110): In each of the following questions, a
(c) A constellation word is given which is highlighted in the given passage.
(d) A planet Choose the option from among the given options which has a
(e) None of the above meaning closer to the OPPOSITE of the meaning of the given
word.
Q105. Which of the followings celestial body or the group of celestial
bodies help in locating the Pole Star? Q109. Bright
(a) The Sun (a) High
(b) The Earth (b) Noisy
(c) The moon of the Earth (c) Nothing
(d) Saptarishi (d) Large
(e) None of the above (e) Dull

Facebook Page- https://www.facebook.com/vishalpariharpage Youtube- https://www.youtube.com/vishalparihar


Follow
16 Vishal Sir Telegram Channel- https://t.me/englishbyvishalsirchannel Instagram- https://www.instagram.com/vishalthetrainer
By Vishal sir
Also Important For SBI CLERK, IBPS CLERK, RRB CLERK
& Other Competitive Exams

ANSWER WITH SOLUTION Hence, the option (c) is the correct answer.

S101. Ans. (d)


Sol. The answer to the question can be derived from the first and S105. Ans. (d)
second paragraphs. Kindly read the last five sentences of the first Sol. The answer to the question can be derived from the last
paragraph: ‘The whole sky is filled with tiny shining objects— sentence of the second-last paragraph ‘We can locate the position
some are bright, others dim. It seems as if the sky is studded with of the Pole Star with the help of the Saptarishi’.
diamonds. They all appear to be twinkling. But if you look at them Hence, the option (d) is the correct answer.
carefully you will notice that some of them do not twinkle as
others do. They simply glow without any flicker just as the moon S106. Ans. (b)
shines.’ and the last sentence of the second paragraph: ‘The sun, Sol. The answer to the question can be derived from the second-
the moon and all those objects shining in the night sky are called last paragraph ‘In ancient times, people used to determine
celestial bodies.’ directions during the night with the help of stars.’
Not all what appears in the sky twinkles. Only stars are believed Hence, the option (b) is the correct answer.
to be twinkling. Other celestial bodies like the Moon are also seen
in the sky. The word ‘diamonds’ refers to every of those objects S107. Ans. (a)
which are seen in the sky. Hence, the words ‘diamonds’ refers to Sol. Flicker [verb] means ‘(of light or a source of light) shine
the ‘celestial bodies’. unsteadily; vary rapidly in brightness’;
Hence, the option (d) is the correct answer. Twinkle- (of a star or light, or a shiny object) shine with a gleam
that changes constantly from bright to faint.
S102. Ans. (a) Accompany [verb] means ‘go somewhere with (someone) as a
Sol. The answer to the question can be derived from the last two companion’;
sentences of the third paragraph ‘Countless twinkling stars in the Begin [verb] means ‘perform or undergo the first part of (an
night sky are similar to the sun. But we do not feel their heat or action or activity)’;
light, and they look so tiny because they are very very far from Care [verb] means ‘feel concern or interest; attach importance to
us’. Hence, the option (a) is the correct answer. something’;
Divide [verb] means ‘separate or be separate into parts’;
S103. Ans. (b) From above, we find out that the word ‘Twinkle’ has a meaning
Sol. The answer to the question can be derived from the last which is similar to the word ‘flicker’.
paragraph: ‘The earth on which we live is a planet. It gets all its Hence, the option (a) is the correct answer.
heat and light from the sun, which is our nearest star. If we look
at the earth from a great distance, say the moon, it will appear to S108. Ans. (c)
be shining just as the moon.’ Hence, the option (b) is the correct Sol. Notice [verb] means ‘become aware of’;
answer. Disappoint [verb] means ‘fail to fulfil the hopes or expectations
of’;
S104. Ans. (c) Fall [verb] means ‘move from a higher to a lower level, typically
Sol. The answer to the question can be derived from the second- rapidly and without control’;
last paragraph ‘One of the most easily recognizable constellation Observe [verb] means ‘notice or perceive (something) and
is the Saptarishi (Sapta-seven, rishi-sages)’. register it as being significant’;

Facebook Page- https://www.facebook.com/vishalpariharpage Youtube- https://www.youtube.com/vishalparihar


Follow
17 Vishal Sir Telegram Channel- https://t.me/englishbyvishalsirchannel Instagram- https://www.instagram.com/vishalthetrainer
By Vishal sir
Also Important For SBI CLERK, IBPS CLERK, RRB CLERK
& Other Competitive Exams

Give [verb] means ‘freely transfer the possession of (something)


to (someone)’;
Hold [verb] means ‘carry with one’s arms or hands’;
From above, we can understand that the word ‘observe’ has a
meaning which is similar to the given word ‘notice’.
Hence, the option (c) is the correct answer.

S109. Ans. (e)


Sol. Bright [adjective] means ‘shining, glowing in appearance’;
High [adjective] means ‘of great vertical extent’;
Noisy [adjective] means ‘making or given to making a lot of
noise’;
Dull [adjective] means ‘lacking interest or excitement’;
Large [adjective] means ‘of considerable or relatively great size,
extent, or capacity’;
Nothing [adjective] means ‘having no prospect of progress; of no
value’;
From above, we can understand that the word ‘dull’ has a
meaning which is OPPOSITE to the given word ‘bright’.
Hence, the option (e) is the correct answer.

Facebook Page- https://www.facebook.com/vishalpariharpage Youtube- https://www.youtube.com/vishalparihar


Follow
18 Vishal Sir Telegram Channel- https://t.me/englishbyvishalsirchannel Instagram- https://www.instagram.com/vishalthetrainer
By Vishal sir
Also Important For SBI CLERK, IBPS CLERK, RRB CLERK
& Other Competitive Exams

Q110. Tiny (e) No error


(a) quiet
(b) mute Q114. Manu was /preposterously paid for (A)/how dexterous (B)/he
(c) quick received the visitors (C)/and entertained them (D).
(d) fast (a) preposterously paid for
(e) Huge (b) how dexterous
(c) he received the visitors
Directions (111-120): In each of the following questions, a (d) and entertained them
sentence is divided into five parts. Out of the five parts, four (e) No error
parts are named as (A), (B), (C) and (D) and one of the fifth
part is bolded. The bolded part is grammatically correct Q115. Mr. Sharma/ not only terminated coming (A)/ here but also (B)/
while there could be an error in any of the named-parts. going to any place (C)/ which is related to his past misfortune (D).
Choose among the parts (A), (B), (C) and (D) which has an (a) not only terminated coming
error and mark the correct answer. In case, all the parts are (b) here but also
grammatically correct, then choose the option (e) ‘no error’ (c) going to any place
as your answer. (d) which is related to his past misfortune
(e) no error
Q111. It is generally the case (A) /that the elite class have no sympathy
(B)/for the poor /who are the real victims (C)/of the present Q116. The BSF personnel (A)/ who were on duty (B)/ in this area /were
social set-up (D). discovered (C)/ two illegal cow smugglers (D).
(a) It is generally the case (a) The BSF personnel
(b) that the elite class have no sympathy (b) who were on duty
(c) who are the real victims (c) were discovered
(d) of the present social set-up (d) two illegal cow smugglers
(e) No error (e) No error

Q112. She wanted (A)/ certain girl (B)/ to make /entry into (C)/ the
Q117. The visitor/ told me that he (A)/ had come from Delhi (B)/ to
Professor’s chamber (D).
discuss some of the serious (C)/ issues with me (D).
(a) She wanted
(b) certain girl (a) told me that he
(c) entry into
(b) had come from Delhi
(d) the Professor’s chamber
(e) No error (c) to discuss some of the serious
(d) issues with me
Q113. She is very (A)/drunk /so she (B)/ cannot tell (C)/ you even her
name (D). (e) no error
(a) She is very
(b) so she
(c) cannot tell
(d) you even her name

Facebook Page- https://www.facebook.com/vishalpariharpage Youtube- https://www.youtube.com/vishalparihar


Follow
19 Vishal Sir Telegram Channel- https://t.me/englishbyvishalsirchannel Instagram- https://www.instagram.com/vishalthetrainer
By Vishal sir
Also Important For SBI CLERK, IBPS CLERK, RRB CLERK
& Other Competitive Exams

ANSWER WITH SOLUTION Sol. ‘Dexterous’ [adjective] means ‘showing or having skill,
especially with the hands’; skillful;
S110. Ans. (e) Instead of ‘dexterous’, ‘dexterously’ would be used because it is
Sol. tiny [adjective] means ‘very small’; qualifying the verb ‘received’.
quiet [adjective] means ‘making little or no noise’; Hence, the option (b) is the correct answer.
mute [adjective] means ‘refraining from speech or temporarily
speechless’; S115. Ans. (a)
quick [adjective] means ‘moving fast or doing something in a Sol. The usage of ‘not only’ shall be before ‘coming’, not before
short time’; ‘terminated’ because ‘but also’ is used before ‘going’. Kindly
fast [adjective] means ‘moving or capable of moving at high keep in mind that not only…but also/ either…. Or/ neither… nor
speed’; join two subjects/objects/verbs/gerunds etc.
Huge- extremely large; enormous Hence, the option (a) is the correct answer.
From above, we can understand that the word ‘Huge’ has a
meaning which is OPPOSITE to the given word ‘tiny’. S116. Ans. (c)
Hence, the option (e) is the correct answer. Sol. The usage of ‘were’ before ‘discovered’ is wrong. The
correct phrase should be ‘the BSF personnel discovered’.
Hence, the option (c) is the correct answer.
S111. Ans. (b)
Sol. Instead of ‘have’, ‘has’ would be used because ‘elite class’
because ‘the elite’, ‘the middle class’ etc. are collective nouns S117. Ans. (e)
which are singular. Sol. The given sentence is grammatically correct and contextually
meaningful and doesn’t require any correction.
Hence, the option (e) is the correct answer.
S112. Ans. (b)
Sol. Instead of ‘certain’, ‘a certain’ (or, instead of ‘boy’, ‘boys’
would be used) would be used because after ‘certain’, noun in
plural form is used while ‘a certain’ is followed by a singular
noun.
For example:
(i) a certain lamp
(ii) certain lamps

S113. Ans. (e)


Sol. The given sentence is grammatically correct and contextually
meaningful. Hence, the option (e) is the correct answer.

S114. Ans. (b)

Facebook Page- https://www.facebook.com/vishalpariharpage Youtube- https://www.youtube.com/vishalparihar


Follow
20 Vishal Sir Telegram Channel- https://t.me/englishbyvishalsirchannel Instagram- https://www.instagram.com/vishalthetrainer
By Vishal sir
Also Important For SBI CLERK, IBPS CLERK, RRB CLERK
& Other Competitive Exams

Q118. She was very much engaged (A)/ in her office-work (B)/ when exploitation as well as (124)________ of women. Even though
the thieves/ entered into her house and (C)/ took away a lot of about 50% of the world’s population consists of women, but
things including expensive jewellery (D). unfortunately most of them are denied basic rights education,
(a) She was very much engaged freedom of speech, voting power and even independent identity.
(b) in her office-work (125)__________ directed specifically against women are
(c) entered into her house and reported from all over the world. There still remain
(d) took away a lot of things including expensive jewellery (126)___________ about acceptance of women empowerment in
(e) no error the most advanced of countries, while developing nations and
Q119. Iran’s support (A) /to the terrorists of Israel (B)/should be nations under political duress are far from achieving the desired
status. In India, in theory, women enjoy a status of
/universally condemned (C) /by all (D).
(127)_________ with the men as per constitutional and legal
(a) Iran’s support
provisions. Arguably, our country has taken enormous strides
(b) to the terrorists of Israel
towards inclusion of women with the fairer gender excelling in
(c) universally condemned diverse fields, from literature to astrophysics to finance. But with
(d) by all headlines about dowry killing, female (128)_____________ and
(e) no error domestic violence still making the newspapers, put a silent
question mark behind the two words. Here, in this current age,
Q120. Madhu informed (A)/ her friend that (B)/ she had gone (C)/ to one true development and growth can only be achieved by taking
of the officer’s residences (D)/ and stayed there all day. successful strides in (129)__________ deep-rooted ideologies of
(a) Madhu informed gender bias and discrimination like the confinement of women to
(b) her friend that the private domestic realm, restrictions on their mobility, poor
(c) she had gone access to health services, nutrition, education and employment,
(d) to one of the officer’s residences and (130)________ from the public and political sphere.
(e) no error
Q121.
Direction (121-130): Given below is a paragraph consisting of (a)Voice
blanks against each number. (b)Anger
Identify the correct option among the five alternatives that (c)Resent
perfectly fits into the given blank against the respective (d)Daunted
number to make the paragraph contextually meaningful and (e)Vocalist
grammatically correct.
Q122.
Nobel Laureate Malala Yousafzai famously quoted “I raise up my (a)Afforestation
voice—not so I can shout, but so that those without a (b)Discrimination
(121)________ can be heard…we cannot succeed when half of (c)Argumentation
us are held back.”, and that sentiment precisely outlines the basis (d)Vacation
of new age women empowerment. (122)_________ against
(e)Artificial
women is rampant all over the world even in this 21st century.
(123)_________ societies in most countries are adept at

Facebook Page- https://www.facebook.com/vishalpariharpage Youtube- https://www.youtube.com/vishalparihar


Follow
21 Vishal Sir Telegram Channel- https://t.me/englishbyvishalsirchannel Instagram- https://www.instagram.com/vishalthetrainer
By Vishal sir
Also Important For SBI CLERK, IBPS CLERK, RRB CLERK
& Other Competitive Exams

ANSWER WITH SOLUTION


S118. Ans. (C)
Sol. ‘Into’ wouldn’t be used because the meaning of ‘enter’ is
‘come into/go into’. So, the usage of ‘into’ is unnecessary and
wrong.
Hence, the option (c) is the correct answer.

S119. Ans. (d)


Sol. The usage of ‘by all’ is superfluous because ‘universally’ has
a meaning which includes the sense of ‘by all’. So, saying
‘universally accepted by all’ or ‘universally acknowledged by all’
is wrong.
Hence, the option (d) is the correct answer.

S120. Ans. (d)


Sol. There is an error in the part (D). When two nouns appear
after ‘one of’ then ownership or possession isn’t reflected through
the usage of ‘ ‘s ‘ but through the usage of ‘of’.
So, instead of ‘one of the officer’s residences’, we should use
‘residence of one of the officers’.
Hence, there is an error in the part (D) and hence, the correct
answer is the option (d).

S121. Ans. (a)


Sol. Option (a) is the correct answer choice. As the paragraph is
telling about the women empowerment the line “I raise up my
voice—not so I can shout, but so that those without” itself
suggests that the correct answer choice should be option (a).

S122. Ans. (b)


Sol. Option (b) is the correct answer choice. The line “….against
women is rampant all over the world even in...” suggest that
option (b) should be the right answer choice. As the paragraph is
also talking about the discrimination against women hence option
(b) should be the correct answer choice.

Facebook Page- https://www.facebook.com/vishalpariharpage Youtube- https://www.youtube.com/vishalparihar


Follow
22 Vishal Sir Telegram Channel- https://t.me/englishbyvishalsirchannel Instagram- https://www.instagram.com/vishalthetrainer
By Vishal sir
Also Important For SBI CLERK, IBPS CLERK, RRB CLERK
& Other Competitive Exams

Q123. (a)Hierarchical (b)Matriarchal Fundamental Research Automatic Calculator? The APSARA and
(c)Patriarchal (d)Mounted TIFRAC design teams were working together and interacting
(e)Survival regularly. There was an excitement when all the subunits – viz.
the arithmetic, memory, control, and display units – operated in
Q124. (a)Convocation (b)Infiltration synchrony, and the first program could be run on the system. “It
(c)Normalization (d)Victimization was a small machine language program cumulatively adding one
(e)Diversification number to another; it looped a number of times and stopped after
a specified number of cycles. To the design team, the first Indian
Q125. (a)Crown (b)Linked computer running a ‘stored program’ was as much a milestone as
(c)Crowds (d)Classifies the first Indian reactor sustaining a chain reaction of nuclear
(e)Crimes fission! The pilot model of the computer, completed in 1956, was
a parallel, asynchronous, fixed point, and single address machine
Q126. (a)Commotion (b)Abrasion with a word length of 12 bits and a two-dimensional ferrite core
(c)Propulsion (d)Questions memory of 256 words, the essay by S. Ramani informs. “Input
(e)Emulsion and output were accomplished via paper tape and teletype. The
total power consumption of this machine was about 10 kw.”
Q127. (a)Infirmity (b)Credibility
(c)Equality (d)Burdened An important component of the TIFRAC was an ‘intangible’ item
(e)Rationality named its assembler, designed and implemented by R.
Narasimhan and Kamalakar S. Kane, recounts S. Ramani in his
Q128. (a)Germicide (b)Feticides essay. It was almost certainly the first item of system software to
(c)Galvanized (d)Harmonize be implemented in India, he notes. A snatch of information of
(e)Criticize use to novices is that, as against an application program which
has specific uses, system software is an essential part of a
Q129. (a)Eliminating (b)Circulating computer system that is necessary to implement the very complex
(c)Misshaping (d)Rehabbing functionality that computer systems need to provide. As Ramani
(e)Instigating elaborates, the TIFR computer group had recognized in 1959-60
the importance of software. “This was a time when the term
Q130. (a)Diversion (b)Extreme ‘computer software’ meant nothing to anyone outside a small
(c)Scheme (d)Realm community of specialists in the world.” Another essay, authored
(e)Exclusion by S. R. Vijayakar and Y. S. Mayya , traces the birth of TDC12,
‘the first Indian-built electronic digital computer’ commissioned
Direction (131-140): Read the following passage carefully and by Vikram Sarabhai at Bhabha Atomic Research Centre on
answer the questions given below it. Certain words are given January 21, 1969.TDC stood for Trombay Digital Computer
in bold to help you locate them while answering some of the series of ECIL (Electronics Corporation of India Ltd), and the
questions computer was the result of the work of “a small band of young
engineers and scientists, fresh from the colleges, without any
Do you know that ‘APSARA’ was the name that Jawaharlal previous knowledge of computers or guidance from people with
Nehru chose for India’s first nuclear reactor? Or, that he also hands-on experience in the field… Their only inputs were the
named the country’s first computer, TIFRAC or Tata Institute of trust and confidence that their superiors reposed in them.

Facebook Page- https://www.facebook.com/vishalpariharpage Youtube- https://www.youtube.com/vishalparihar


Follow
23 Vishal Sir Telegram Channel- https://t.me/englishbyvishalsirchannel Instagram- https://www.instagram.com/vishalthetrainer
By Vishal sir
Also Important For SBI CLERK, IBPS CLERK, RRB CLERK
& Other Competitive Exams

A predecessor to TDC12 was EAC (Electronic Analogue


Computer) machine in 1964. About ten of these computers were Q132. How is system software useful for a computer system according
sold to various engineering colleges in the country! “One of the to the context of the passage?
very interesting applications for which this computer was put to (a) Provides default medium for video distribution.
use was the design of the flyover bridge at Kemps Corner, (b) Simulation of control systems.
Mumbai, in association with Roorkee and Osmania universities. (c) It implements the very complex functionality that computer
The system was also employed for the simulation of control systems need to provide.
systems of CIRUS reactor.” (d) Acts as a memory medium within a device.
(e) Makes the bus speeds higher better.
A differently-titled essay is of Bud Mishra titled, ‘Sarve Santu
Niramaya’ (meaning, ‘May all be disease-free’). He is of the view Q133. In reference to the writings by some of the authors, which is the
that the emerging fields of computational systems biology and first Indian built electronic digital computer?
population genomics analysis could be the next important high- (a)EAC
technology areas for India to nurture. India’s ambition, he says, (b)TIFRAC
should be to continue developing technologies with the goal of (c)TDC12
establishing herself as a global superpower and a world leader, (d)APSARA
not necessarily in a military sense or even in an economic sense, (e)CIRUS
but as an example representing the ambitions of all humanity and
having its fount embedded in knowledge, science, and technology Q134. The formation of TDC12 was the outcome of
– all aimed to end human suffering. The investment in (a) A person skilled in the technique of an art or craft.
biotechnology, nanotechnology, robotics, etc. would be the (b) An IT Specialist, computer professional, or an IT professional
necessary steps in moving India in that direction, explains especially a person who has proven extensive knowledge in the
Mishra. area of computing.
(c) Experienced engineers and scientists.
Q131. Which of the following statements is true about the 1956’s pilot (d) A small band of young fresh college engineers and scientists
model of the computer? without any previous knowledge of computers.
(a)A parallel, synchronous, fixed point, and single address (e) An expert in the practical application of a science.
machine with a word length of 12 bits and a two-dimensional
ferrite core memory of 256 words. Q135. Apart from the designing of Flyover Bridge at Kemps Corner,
(b)A parallel, asynchronous, fixed point, and single address what was the other important application of EAC?
machine with a word length of 32 bits and a two-dimensional (a) Displaying the time, the zodiac, and the orbits of the sun and
ferrite core memory of 256 words. moon.
(c)A parallel, asynchronous, fixed point, and single address (b) Simulation of control systems of CIRUS reactor.
machine with a word length of 32 bits and a single-dimensional (c) Integrating a voltage with respect to time.
ferrite core memory of 256 words. (d) Carrying bits of information in groups by means of spatial
(d)A parallel, asynchronous, fixed point, and single address arrangement of signals.
machine with a word length of 12 bits and a two-dimensional (e) All of these.
ferrite core memory of 256 words.
(e) None of these.

Facebook Page- https://www.facebook.com/vishalpariharpage Youtube- https://www.youtube.com/vishalparihar


Follow
24 Vishal Sir Telegram Channel- https://t.me/englishbyvishalsirchannel Instagram- https://www.instagram.com/vishalthetrainer
By Vishal sir
Also Important For SBI CLERK, IBPS CLERK, RRB CLERK
& Other Competitive Exams

ANSWER WITH SOLUTION S130. Ans. (e)


Sol. Option (e) is the correct answer choice.
S123. Ans. (c) Realm- a kingdom
Sol. Option (c) is the correct answer choice.
Patriarchal-relating to or denoting a system of society or S131.Ans. (d)
government controlled by men Sol. Refer to paragraph 1 which mentions about the first Indian
S124. Ans. (d) computer running a stored program and thus mentions about the
Sol. Option (d) is the correct answer choice. pilot model of the computer completed in 1956 which was “a
Victimization-the action of singling someone out for cruel or parallel, asynchronous, fixed point, and single address machine
unjust treatment with a word length of 12 bits and a two-dimensional ferrite core
Hence the meaning of the given word suggests that option (d) memory of 256 words”.
should be the right answer choice.
S125. Ans. (e) S132. Ans. (c)
Sol. Option (e) is the correct answer choice. The line “…directed Sol. Option (c) is the most appropriate choice. The answer can be
specifically against women are reported from all over the world” deduced from the second paragraph of the passage where it is
itself suggests that the right answer choice to fill the blank is given that an application program has specific uses but against it
‘crimes’ hence option (e) is the correct answer choice. system software is an essential part of a computer system that is
S126. Ans. (d) necessary to implement the very complex functionality that
Sol. Option (d) is the correct answer choice. option (b), (c) and computer systems need to provide.
(e) are out of context.
Commotion- a state of confused and noisy disturbance S133. Ans. (c)
Hence the right answer choice is option (d). Sol. The answer can be traced from the second paragraph of the
S127. Ans. (c) passage itself where it is given “Another essay, authored by S. R.
Sol. Option (c) is the correct answer choice. The line “In India, in Vijayakar and Y. S. Mayya , traces the birth of TDC12, ‘the first
theory, women enjoy a status of….. with the men as per Indian-built electronic digital computer’ commissioned by
constitutional and” suggests that the ‘equality’ is the right word Vikram Sarabhai at Bhabha Atomic Research Centre on January
to make the sentence contextually correct. Hence option (c) is the 21, 1969.”
correct answer choice.
S128. Ans. (b) S134. Ans. (d)
Sol. Option (b) is the correct answer choice. Sol. Refer to some of the last lines of the second paragraph where
Feticides- destruction or abortion of a fetus, option (a) and (c) are it is mentioned that TDC stood for Trombay Digital Computer
out of the context. Hence option (b) is the right answer choice. series of ECIL (Electronics Corporation of India Ltd), and the
S129. Ans. (a) computer was the result of the work of “a small band of young
Sol. Option (a) is the correct answer choice. As the previous line, engineers and scientists, fresh from the colleges, without any
“true development and growth can only be achieved by taking previous knowledge of computers or guidance from people with
successful strides in ………deep-rooted ideologies of gender bias hands-on experience in the field. Their only inputs were the trust
and discrimination like…” suggests that deep rooted ideologies and confidence that their superiors reposed in them.
of gender bias should be eliminated. Hence option (a) is the S135. Ans.(b)
correct answer choice. Sol. Option (b) is the most suitable choice as it is mentioned in
the passage. We can refer to paragraph 3 for deducing our answer

Facebook Page- https://www.facebook.com/vishalpariharpage Youtube- https://www.youtube.com/vishalparihar


Follow
25 Vishal Sir Telegram Channel- https://t.me/englishbyvishalsirchannel Instagram- https://www.instagram.com/vishalthetrainer
By Vishal sir
Also Important For SBI CLERK, IBPS CLERK, RRB CLERK
& Other Competitive Exams

as the answer is explicitly given. So the text can be quoted Q140. Snatch
directly as, “The system was employed for the simulation of (a)Grab (b)Seize
control systems of CIRUS reactor.” (c)Immersed (d)Release
Q136.What are the next important high-technology areas mentioned in the (e)Rooted
passage where India could nurture?
(a) Computer software development to next level. Directions (141-150): Read each sentence to find out whether
(b) Computational systems biology. there is any grammatical or idiomatic error in it. The error,
(c) Population genomics analysis if any, will be in one part of the sentence. The number of that
(d) Both (a) and (b) part is the answer. If there is ‘No error’, the answer is (e).
(e) Both (b) and (c) (Ignore errors of punctuation, if any.)

Directions (137-138): Find out the word from the given Q141. She did not let the (a)/grief of her loss effect the (b)/kingdom
options which is similar in meaning to the word given in bold administration (c)/and the lives of her people. (d)/ No error (e)
in the passage.
Q142. The industry (a)/has the potential of (b)/attaining $34bn
Q137. Cumulatively export(c)/earnings till the year 2010 (d)/ No error (e)
(a)Segregated
(b)Collectively Q143. A lot of companies (a)/in Tripura have (b)/now been
(c)Distinctly focused(c)/on branding(d)/No error(e)
(d)Isolated
(e)Disassociated Q144. Epson is one (a)/ of the most (b)/respected manufacturer (c)/in the
print market (d)/No error (e)
Q138. Accomplished
(a)Incapable Q145. Unless you (a)/ work hard (b)/you will not (c)/ pass the
(b)Inept examination (d)/No error (e)
(c)Attained
(d)Unorganized Q146. Scents always have (a)/utter disregard for (b)/the luxurious life
(e)Barbaric (c)/and momentary world (d)/No error (e)

Directions (139-140): Find out the word from the given Q147. The perfume that (a)/ your brother (b)/ has bought from Italy (c)/
options which is opposite in meaning to the word given in bold is smelling good (d)/No error (e)
in the passage.
Q148. He informed (a)/ me that he (b)/ knew to play (c)/ on the
Q139. Nurture harmonium (d)/No error (e)
(a)Foster
(b)Deprive Q149. Being (a)/ a beautiful city (b)/ it charmed (c)/ all of us (d)/No error
(c)Nourish (e)
(d)Feed
(e)Develop Q150. He does (a)/ nothing but to (b)/ annoy her (c)/ younger sister
(d)/No error (e)

Facebook Page- https://www.facebook.com/vishalpariharpage Youtube- https://www.youtube.com/vishalparihar


Follow
26 Vishal Sir Telegram Channel- https://t.me/englishbyvishalsirchannel Instagram- https://www.instagram.com/vishalthetrainer
By Vishal sir
Also Important For SBI CLERK, IBPS CLERK, RRB CLERK
& Other Competitive Exams

ANSWER WITH SOLUTION


S140. Ans.(d)
S136. Ans. (e) Sol. Option (d) is the most apt choice.
Sol. Option (e) is the most appropriate answer. The answer can Snatch-quickly seize (something) in a rude or eager way
be inferred from the last paragraph of the passage where it is Immersed means involve oneself deeply in a particular activity.
given that Bud Mishra is of the view that the emerging fields of Rooted means establish deeply and firmly.
computational systems biology and population genomics analysis Option (a) and (b) are the synonyms of the given word.
could be the next important high-technology areas for India to
nurture. S141. Ans. (b)
Sol. The error is in part (b) of the sentence. ‘Affect’ should be
S137. Ans.(b) replaced by ‘effect’.
Sol. Cumulatively means in a way that increases in quantity, Effect-A change which is a result or consequence of an action or
degree, or force by successive additions. other cause.
Collectively means taken as a whole; in combination. Affect-have an effect on; make a difference to.
Distinctly means taken as a whole; in combination.
Isolated means far away from other places, buildings, or people; S142. Ans. (d)
remote. Sol. The error is in part (d) of the sentence. ‘till’ should be
Segregated means set apart from the rest or from each other; replaced by ‘by’
isolate or divide. “By” is used to describe an action or event that happens on or
Disassociated means disconnected. before a particular moment.
“Until” is used to describe a state or situation in the period of time
S138. Ans.(c) up to a particular moment.
Sol. Accomplished means achieve or complete successfully
Attained- succeed in achieving (something that one has worked S143. Ans. (d)
for) Sol. The error is in part (c) of the sentence.
Inept means having or showing no skill; clumsy. ‘Focused’ should be replaced by ‘focusing’
Barbaric means savagely cruel or uncivilized and uncultured. “Have Been + Past Participle” also known as the Present Perfect
Thus option (c) is the answer. Progressive or Present Perfect Continuous. It is used for an
activity that started in the past and still continues to the present.
S139. Ans. (b)
Sol. Option (b) is the most suitable choice. Nurture means care S144. Ans. (c)
for and protect (someone or something) while they are growing. Sol. The error is in part (c) of the sentence.
Deprive means prevent (a person or place) from having or using ‘Manufacturer’ should be replaced by ‘manufacturers’
something. The phrase "one of the" is always a plural noun, whereas use of
Hence it is the only opposite of the given word. verbs as singular or plural will entirely depend upon the subject
Foster means encourage the development of (something, of the statement, i.e. singular verb for singular subject and plural
especially something desirable. verb for plural subject.
Nourish means provide with the food or other substances
necessary for growth, health, and good condition.

Facebook Page- https://www.facebook.com/vishalpariharpage Youtube- https://www.youtube.com/vishalparihar


Follow
27 Vishal Sir Telegram Channel- https://t.me/englishbyvishalsirchannel Instagram- https://www.instagram.com/vishalthetrainer
By Vishal sir
Also Important For SBI CLERK, IBPS CLERK, RRB CLERK
& Other Competitive Exams

S145. Ans. (e)


Sol. There is no error in the given sentence. Therefore option (e)
is the correct answer choice.

S146. Ans. (a)


Sol. The error is in part (a) of the sentence.
‘Scents’ should be replaced by ‘saints’.
Scents mean a particular kind of smell whereas saints mean a
person distinguished for holiness.

S147. Ans. (d)


Sol. The error is in part (d) of the sentence.
‘is smelling’ should be replaced by ‘smells’
Verbs such as look, seem, taste, feel, smell, and sound are sensory
(sense) verbs.
subject + sense verb + adjective

S148. Ans. (c)


Sol. The error is in part (c) of the sentence.
In part (c) of the sentence ‘how’ should be used after ‘knew’
Know + how/where/when/why + infinitive

S149. Ans. (e)


Sol. There is no error in the given sentence. Therefore option (e)
is the best answer choice.
Though there is no subject of reference with ‘being’ but ‘it’ has
been used as the main subject of the sentence.
Placing ‘it’ before ‘being’ makes a perfect sense to the sentence
(it being a beautiful city = it was a beautiful city)

S150. Ans. (b)


Sol. The error is in part (b) of the sentence. ‘to’ has to be removed
after ‘but’ to make sentence grammatically correct.
‘To’ is wrongly used here after but. Whenever ‘but’ is used after
any form of ‘do’, conveying ‘except’ as a meaning and there is a
use of verb after that then ‘to’ should not used.

Facebook Page- https://www.facebook.com/vishalpariharpage Youtube- https://www.youtube.com/vishalparihar


Follow
28 Vishal Sir Telegram Channel- https://t.me/englishbyvishalsirchannel Instagram- https://www.instagram.com/vishalthetrainer
By Vishal sir
Also Important For SBI CLERK, IBPS CLERK, RRB CLERK
& Other Competitive Exams

Direction (151-160): Given below is a paragraph consisting of Q151. (a)Born (b)Bent


blanks against each number. (c)Folded (d)Daunted
Identify the correct option among the five alternative pairs (e)Frightened
that perfectly fits into the given blank against the respective
number to make the paragraph contextually meaningful and Q152. (a)Arouse (b)Around
grammatically correct. (c)Argue (d)Affluent
(e)Artificial
She was (151)__________ in the small village of Kakati in
Belgaum, Karnataka in 1778. From a very young age she received Q153. (a)Exchanged (b)Afforested
training in sword fighting, archery and horse riding and got (c)Attracted (d)Mounted
private lessons at home. Once King Mallasarja of Kittur was (e)Balanced
moving (152) ___________trying to find support from
neighbouring kingdoms to fight Tipu Sultan and approached Q154. (a)Restored (b)Rejected
Dhulappa Desai. He then saw Chennamma and was (c)Nurtured (d)Married
(153)____________ to her confidence and bravery. Though he (e)Divorced
was already (154) __________and had a son Shivalinga Rudra
Sarja, he took Chenamma as his second wife. They had a son who Q155. (a)Crowned (b)Linked
died at a young age. King Mallasarja also passed away. It is said (c)Crowd (d)Classify
that East India company’s collector and political agent Thackeray (e)Capture
wished to (155)___________ Kittur to expand the British
Empire. Rani Chennama now queen of kittur decided to take Q156. (a)Commotion (b)Abrasion
charge and rallied together her trusted men to fight against the (c)Propulsion (d)Conclusion
British. As a (156)_____________of this war British officers (e)Emulsion
including Thackeray were killed. The remaining officials and
soldiers were (157) ________________and she ensured that they Q157. (a)Sanctioned (b)Adjourned
were treated with courtesy and kindness. The British Empire was (c)Imprisoned (d)Burdened
enraged at the humiliation of defeat at the hands of a small ruler (e)Rationed
and sent bigger armies to Kittur to capture it. A fierce battle was
fought but this time due to a number of (158)___________in her Q158. (a)Followers (b)Traitors
camp she lost the battle and was captured by the British and their (c)Practitioners (d)Survivors
treasury looted. It is said that such was her indomitable courage (e)Instructors
and fearlessness that she tried to (159) __________twice from the
Kittur fort but was recaptured both times and then put in solitary Q159. (a)Escape (b)Recap
confinement for life. She spent her days performing pooja and (c)Mishap (d)Rehab
reading the holy texts till her death. In the year 1829 she died in (e)Instigate
confinement with the (160) _________ of a free Kittur in her
heart and mind. Her burial place or Samadhi is at Bailhongal Q160. (a)Supreme (b)Extreme
taluka surrounded by a small park. (c)Scheme (d)Realm
(e)Dream

Facebook Page- https://www.facebook.com/vishalpariharpage Youtube- https://www.youtube.com/vishalparihar


Follow
29 Vishal Sir Telegram Channel- https://t.me/englishbyvishalsirchannel Instagram- https://www.instagram.com/vishalthetrainer
By Vishal sir
Also Important For SBI CLERK, IBPS CLERK, RRB CLERK
& Other Competitive Exams

ANSWER WITH SOLUTION Sol. Option (c) is the correct answer choice. The line
“…..including Thackeray were killed ………The remaining
officials and soldiers were (157) ________________and she
S151. Ans. (a)
Sol. Option (a) is the correct answer choice. As the paragraph is ensured that they were treated with courtesy and kindness.”
telling about the story of the Rani Chennamma a great warrior suggests that the queen had won the war and Thackeray got killed
from the war of independence. As the very first line of the and the remaining officials were ‘imprisoned’. Therefore option
paragraph describes her birth place and the year so ‘born’ would (c) is the correct answer choice.
be the best choice to fill the blank.
S158. Ans. (b)
S152. Ans. (b) Sol. Option (b) is the correct answer choice. The line, “ A fierce
Sol. Option (b) is the correct answer choice. As the 3rd line of the battle was fought but this time due to a number of
paragraph states that King Mallasarja of Kittur was looking for (98)___________in her camp she lost the battle and was captured
help from his neighboring kingdoms to fight Tipu Sultan, and by the British and their treasury looted” suggests that while the
thus he was moving around for getting help. war took placed second time the queen lost the battle and that too
just because of some renegades inside her camp. So the ‘traitors’
S153. Ans. (c) is the correct answer choice to fill the blank (158).
Sol. Option (c) is the correct answer choice. As the next line states
that “he took Chenamma as his second wife” so the correct option S159. Ans. (a)
to fill the blank (153) should be ‘attracted’. Sol. Option (a) is the correct answer choice. As the previous line,
“….she lost the battle and was captured by the British and their
S154. Ans. (d) treasury looted…..” suggests that the queen had lost the war and
Sol. Option (d) is the correct answer choice. As the line itself she was captured therefore ‘escape’ is the correct word to fill the
suggests that the King Mallasarja of Kittur already had a son so blank (159).
the correct option to fill the blank (154) should be ‘married’.
S160. Ans. (e)
S155. Ans. (e) Sol. Option (e) is the correct answer choice. As the line, “In the
Sol. Option (e) is the correct answer choice. As the line itself year 1829 she died in confinement with the (160) _________ of
suggests that Thackeray the East India Thackeray Company’s a free Kittur in her heart and mind.” itself suggests that while
collector and political agent wanted to expand the British Empire dying the queen had a wish/dream in her heart, therefore the best
so the correct word to fill the blank (155) should be ‘capture’. answer choice to fill the blank (160) should be ‘dream”.

S156. Ans. (d)


Sol. Option (d) is the correct answer choice. As the line, “As a
(156)_____________of this war British officers including
Thackeray were killed” suggests the idea of the outcome of war
so the best answer choice to fill the blank (96) should be
‘conclusion’.

S157. Ans. (c)

Facebook Page- https://www.facebook.com/vishalpariharpage Youtube- https://www.youtube.com/vishalparihar


Follow
30 Vishal Sir Telegram Channel- https://t.me/englishbyvishalsirchannel Instagram- https://www.instagram.com/vishalthetrainer
By Vishal sir
Also Important For SBI CLERK, IBPS CLERK, RRB CLERK
& Other Competitive Exams

Direction (161-170): Read the following passage carefully and adapted their skills and themes to changing times. As a part of
answer the questions given below it. Certain words are given this effort, a group of innovative patuas established a patachitra
in bold to help you locate them while answering some of the village at Naya. Slowly, their efforts to revive their artistic
questions. heritage started paying off.

The Patuas are an artisan community found in the state of West Today, after a period of decline, the patachitra art is flourishing
Bengal, Bihar, Jharkhand and Odisha in India and parts of again in the village, with village youngsters taking up the
Bangladesh. Some Patuas are Hindus, while others are Muslims. traditional art form as a passion and profession. A pata is created
Hindu Patuas are active in the Kalighat and Kumartuli regions of by painting on a canvas made by stitching together multiple
Calcutta, along with some other parts of West Bengal, where they sheets of commercial poster paper. In earlier days, jute fibre
are reduced in number. It is believed that most Patuas are actually canvas was used. Plant-based colours and lamp black (a pigment
converts from Hinduism to Islam. Today, they practice customs made from soot) are mixed in coconut shells with the sap of the
that are both Hindu and Islamic in nature. They may have also bel tree (wood apple), which acts as a binder. After finishing, a
been Buddhist at various points in time. Today, however, the thin cotton cloth is glued to the back of the painting to provide
majority of them are impoverished Muslims who rely on longevity. Next, the completed scrolls are kept in the sun to dry.
patronage from mainly Hindus, but also increasingly from The patuas also paint wooden souvenirs, decorative hangings and
tourists who buy their painted scrolls, as Frank J. Korom has mud walls with striking natural colours.
described and analyses in his book Village of Painters: Narrative
Scrolls from West Bengal (2006. Santa Fe: Museum of New Although Chitrakars origin is difficult to be precisely determined,
Mexico Press). Scholars argue that the Patuas, originally Hindus, historical and mythological memories coincide that their
were cast out of the Hindu society for not following canonical existence is traceable to the 13th century. Different accounts
proceedings in pursuing their trade. Patuas are also known Patigar explain their standing in the Indian caste system. The Patua are a
or Chitrakar. unique community, in that their traditional occupation is the
painting and modelling of Hindu idols, yet many of them are
An intricate tapestry of music and visual art is what makes Naya Muslims. Their name Patua is a corruption of the Bengali word
more than just a village in West Bengal’s Paschim Midnapore Pota, which means an engraver. They are also widely known as
district. A quaint little village, Naya is home to around 250 patuas Chitrakar, which literally means a scroll painter. The Patuas paid
or chitrakaars, a unique community of folk artistes who are little attention to faith, while looking for patronage. Chitrakars
painters, lyricists, singers and performers all rolled into one. themselves might have converted to Islam as a strategy to avoid
These traditional painter singers specialize in the ancient folk art the oppression by a hierarchy of subcastes created during the Sen
of pata chitra, a traditional and mythological heritage of West Dynasty. This was an extremely slow process with the Patuas, as
Bengal, divided into some different aspects like Durga Pat, seen by the fact that every Patua has two names, one Hindu and
Chalchitra, Tribal Patachitra, Medinipur Patachitra, Kalighat one Muslim.
Patachitra and etc. The subject matter of Bengal Patachitra is
mostly mythological, religious stories, folk lore and social. The The Patua are found mainly in the districts of Murshidabad, In
Kalighat Patachitra, the last tradition of Bengal Patachitra is Bihar they are mainly found in the Magahi and Maithili speaking
developed by Jamini Roy. The artist of the Bengal Patachitra is regions as well as in the adjoining regions of Jharkhand. In West
called Patua, a type of narrative scroll painting. Over time, Bengal they are a Bengali speaking community, with little or no
however, interest in this art form faded out. To ensure that their knowledge of Urdu. The community is strictly endogamous, and
art form remained relevant in the contemporary world, the patuas prefers cross-cousin marriages. The Patuas visit villages and go

Facebook Page- https://www.facebook.com/vishalpariharpage Youtube- https://www.youtube.com/vishalparihar


Follow
31 Vishal Sir Telegram Channel- https://t.me/englishbyvishalsirchannel Instagram- https://www.instagram.com/vishalthetrainer
By Vishal sir
Also Important For SBI CLERK, IBPS CLERK, RRB CLERK
& Other Competitive Exams

from house to house with their bags of scrolls. They narrate


stories while unrolling the scrolls; in return of his services he is Q164. What efforts the patuas are making to ensure that their art form
paid in cash or kind. The traditional occupation of the Patua is remains relevant in the contemporary world?
scroll painting, image making and other decorative work. They (a)The patuas are adapting their skills and themes according to
paint pictures on course pieces of cloth, showing images of Hindu changing times.
gods and goddesses. These paintings are referred to as patas. Like (b) A group of innovative patuas established a patachitra village
other Muslim artisan groups in India, they have seen a decline in at Naya
their traditional occupation. The majority are now employed as (c) Patuas are organizing different cultural activities so as to make
daily wage labourers. people aware about their customs and heritage.
(d)Both (a) and (b)
Q161. According to the passage, what might be the reason that Hindu (e) None of these.
Patuas, active in regions of Calcutta have reduced in number?
(a)The interest of the people coming from abroad in this art form Q165. According to the passage how the Pata art in now a days is
is fading out. different from the earlier days?
(b)Naya a home to around 250 patuas or chitrakaars, has been (a)Earlier Plant-based colors were to be used as a binder now days
under attacked by the mob since last many years. the sap of the bel tree acts as a binder.
(c)It is believed that most Patuas have actually been converted (b) Opposite to present scenario earlier after finishing, a thin
from Hinduism to Islam. cotton cloth was to be glued to the back of the painting to provide
(d) It is believed that most Patuas have actually been converted longevity.
from Hinduism to Christianity. (c)In earlier days, jute fiber canvas was used for painting but now
(e) None of these. days a Pata is created by painting on a canvas made by stitching
together multiple sheets of commercial poster paper.
Q162. According to the passage, how the majority of converted (d)Both (b) and (c)
bankrupt Muslims are surviving financially? (e) None of these.
(a)They rely on funding from mainly Hindu families.
(b) By the funding provided by Government Q166. As per the passage what is the traditional occupation of the Patua
(c)The funding coming from the tourists who buy their painted community?
scrolls and their (a)A unique community of folk artistes where most of them are
(d)Both (a) and (c) painters, lyricists and singers
(e) None of these. (b)Though many of the people from the Patua community are
Muslims, yet their traditional occupation is painting and
Q163. Why has author called the ‘Naya’ village of West Bengal as more modelling of Hindu idols.
than just a village in district? (c) These traditional painter singers are specialized in the ancient
(a) Naya is a unique community of folk artistes who are painters, folk art of pata chitra, a traditional and mythological heritage of
lyricists, singers and performers all rolled into one. West Bengal.
(b)Naya is a village with diversify culture and heritage (d)Both (a) and (c)
(c)The village is so well developed by the colonizers that it has (e) None of these.
the best sanitation system and transport system
(d)Both (b) and (c)
(e) None of these.

Facebook Page- https://www.facebook.com/vishalpariharpage Youtube- https://www.youtube.com/vishalparihar


Follow
32 Vishal Sir Telegram Channel- https://t.me/englishbyvishalsirchannel Instagram- https://www.instagram.com/vishalthetrainer
By Vishal sir
Also Important For SBI CLERK, IBPS CLERK, RRB CLERK
& Other Competitive Exams

ANSWER WITH SOLUTION Sol. The correct answer choice is option (d). Option (c) is out of
context as there is no mention of cultural activities so option (c)
S161.Ans. (c) can be easily omitted. Option (a) and (b) can be traced from the
Sol. The correct answer choice is option (c). Option (b) is totally last lines of 2nd paragraph where it is given as “To ensure that
out of context as there is no mention of the attack by mob so it their art form remained relevant in the contemporary world, the
can be easily omitted. Option (c) can be traced from the very first patuas adapted their skills and themes to changing times. As a
paragraph of the passage where it is given as “Hindu Patuas are part of this effort, a group of innovative patuas established a
active in the Kalighat and Kumartuli regions of Calcutta, along patachitra village at Naya. Slowly, their efforts to revive their
with some other parts of West Bengal, where they are reduced in artistic heritage started paying off.”
number. It is believed that most Patuas are actually converts from
Hinduism to Islam.” S165.Ans. (c)
Sol. The correct answer choice is option (c). Option (a) and (b)
are quoted incorrectly as per given in the passage. Option (c) can
S162.Ans. (d) be traced from the second line of 3rd paragraph where it is given
Sol. The correct answer choice is option (d). Option (b) is totally as “A pata is created by painting on a canvas made by stitching
out of context as there is no mention of government funding so it together multiple sheets of commercial poster paper. In earlier
can be easily omitted. Option (a) and (c) can be traced from the days, jute fiber canvas was used.”
very first paragraph of the passage where it is given as “Today,
however, the majority of them are impoverished Muslims who S166.Ans. (b)
rely on patronage from mainly Hindus, but also increasingly from Sol. The correct answer choice is option (b). Option (a) and (c)
tourists who buy their painted scrolls, as Frank J. Korom has are out of context of the question. Option (b) can be traced from
described and analyses in his book Village of Painters: Narrative the second line of 4th paragraph where it is given as “The Patua
Scrolls from West Bengal (2006. Santa Fe: Museum of New are a unique community, in that their traditional occupation is the
Mexico Press).” painting and modelling of Hindu idols, yet many of them are
Muslims.”
S163.Ans. (a)
Sol. The correct answer choice is option (a). Option (b) and (c)
are totally out of context as there is no mention of either cultural
heritage or development of village so both the options can be
omitted easliy. Option (a) can be traced from the 2nd paragraph of
the passage where it is given as “An intricate tapestry of music
and visual art is what makes Naya more than just a village in West
Bengal’s Paschim Midnapore district. A quaint little village,
Naya is home to around 250 patuas or chitrakaars, a unique
community of folk artistes who are painters, lyricists, singers and
performers all rolled into one.”

S164.Ans. (d)

Facebook Page- https://www.facebook.com/vishalpariharpage Youtube- https://www.youtube.com/vishalparihar


Follow
33 Vishal Sir Telegram Channel- https://t.me/englishbyvishalsirchannel Instagram- https://www.instagram.com/vishalthetrainer
By Vishal sir
Also Important For SBI CLERK, IBPS CLERK, RRB CLERK
& Other Competitive Exams

Directions (167-168): Choose the word which is most nearly (a)Maize, stubborn
the SAME in meaning as the word printed in bold as used in (b)Sunset, Resource
the passage. (c)Transit, soothing
(d)Internet, erudite
Q167. Impoverished (e)Peace, commotion
(a)Bankrupt (b)Restrict
(c)Constancy (d)Affinity Q172. Neville, the British prime minister during WWII, tried to
(e)Dissipated ______Hitler and in doing so sent a clear_______: you can walk
all over us.
Q168. Patronage (a)Opaque, question
(a)Prudent (b)Excessive (b)Harmony, predictability
(c)Funding (d)Steep (c)Tortuous, confused
(e)Frugal (d)Revamp, Implicit
(e) Appease, message
Directions (169-170): Choose the word which is the
OPPOSITE in meaning as the word printed in bold as used Q173. The judge was not _________since he had been ______by the
in the passage. witness’s family.
(a)Suspense, tolerable
Q169. Intricate (b)Temperament, stimulus
(a)Shelter (c)Impartial, bribed
(b)Defend (d)Baffle, wishful
(c)Torment (e)Wreck, trivial
(d)Beset
(e)Straightforward Q174. Russians oligarchs are famous for their________, living in fancy
homes and dining in _________restaurants.
Q170. Oppression (a)Negligible, refuse
(a)Slump (b)Opulence, expensive
(b)Freedom (c)Rebel, revolt
(c)Devaluation (d)Sorrow, presume
(d)Surge (e)Prey, rest
(e)Depreciation
Q175. The unpredictable and _______nature of déjà vu makes it very
Directions (171-175): Each question below has two blanks. difficult _________to study properly.
There are five pairs of words below the sentence. Each pair is (a)Transient, phenomenon
lettered. Choose the pair of words which can be filled up in (b)Impossible, undoubtedly
the blanks in the sentence in the same order so as to complete (c) Impertinent, vaguely
the sentences meaningfully. (d)Acute, traumatism
(e)Liberal, hint
Q171. Before the_______, the library was typically where you would
find ________readers.

Facebook Page- https://www.facebook.com/vishalpariharpage Youtube- https://www.youtube.com/vishalparihar


Follow
34 Vishal Sir Telegram Channel- https://t.me/englishbyvishalsirchannel Instagram- https://www.instagram.com/vishalthetrainer
By Vishal sir
Also Important For SBI CLERK, IBPS CLERK, RRB CLERK
& Other Competitive Exams

ANSWER WITH SOLUTION Commotion- confusion


S167.Ans. (a) Erudite- having or showing great knowledge or learning.
Sol. Impoverished- make (a person or area) poor. Soothing- having a gently calming effect.
Affinity means resemblance. S172.Ans. (e)
Constancy means consistency. Sol. The most suitable set of words to fill the blanks to make the
Dissipated means dispersed. sentence grammatically correct and contextually meaningful
Bankrupt- impoverished or depleted. is “Appease, message”. Hence, option (e) is the most viable
S168.Ans. (c) answer choice.
Appease- pacify or placate (someone) by acceding to their demands.
Sol. Patronage- the support given by a patron
Tortuous- full of twists and turns
Steep means (of a price or demand) not reasonable; excessive in
Opaque- not able to be seen through; not transparent
accordance to the context of the paragraph.
Implicit- suggested though not directly expressed
Excessive means the same as steep.
S173.Ans. (c)
Frugal means economical, sparing.
Sol. The most suitable set of words to fill the blanks to make the
Prudent means acting with or showing care and thought for the
sentence grammatically correct and contextually meaningful
future.
is “Impartial, bribed”. Hence, option (c) is the most viable answer
S169.Ans. (e)
choice.
Sol. Intricate- very complicated or detailed.
Impartial- treating all rivals or disputants equally
Beset means (of a problem or difficulty) trouble (someone or
Baffle- totally bewilder or perplex
something) persistently.
Stimulus- a thing or event that evokes a specific functional
Torment means a cause of severe suffering.
reaction in an organ or tissue
Defend means resist an attack made on (someone or something);
Wreck- the destruction of a ship at sea; a shipwreck
protect from harm or danger.
Trivial- of little value or importance
Shelter means shield.
S174.Ans. (b)
S170.Ans. (b)
Sol. The most suitable set of words to fill the blanks to make the
Sol. Oppression- prolonged cruel or unjust treatment or exercise
sentence grammatically correct and contextually meaningful
of authority.
is “Opulence, expensive”. Hence, option (b) is the most viable
Depreciation means a reduction in the value of an asset over time,
answer choice.
due in particular to wear and tear.
Opulence- great wealth or luxuriousness
Surge means increase suddenly and powerfully. Thus, it is the
Prey- an animal that is hunted and killed by another for food
antonym of the given word.
Revolt- take violent action against an established government or
Devaluation means the reduction or underestimation of the worth
ruler; rebel
or importance of something.
S175.Ans. (a)
Slump means undergo a sudden severe or prolonged fall in price,
Sol. The most suitable set of words to fill the blanks to make the
value, or amount.
sentence grammatically correct and contextually meaningful
S171.Ans. (d)
is “Transient, phenomenon”. Hence, option (a) is the most viable
Sol. The most suitable set of words to fill the blanks to make the
answer choice.
sentence grammatically correct and contextually meaningful
Transient-lasting only for a short time; impermanent
is “Internet, erudite”. Hence, option (d) is the most viable answer
Phenomenon-a fact or situation that is observed to exist or
choice.
happen, especially one whose cause or explanation is in question

Facebook Page- https://www.facebook.com/vishalpariharpage Youtube- https://www.youtube.com/vishalparihar


Follow
35 Vishal Sir Telegram Channel- https://t.me/englishbyvishalsirchannel Instagram- https://www.instagram.com/vishalthetrainer
By Vishal sir
Also Important For SBI CLERK, IBPS CLERK, RRB CLERK
& Other Competitive Exams

Directions (176-180): In each of the question given below a/an Q181. pockets are electronic circuits and materials (A)/ that can collect,
idiom/phrase is given in bold which is then followed by five record, and output data(B)/ Embedded inside these (C)/ in the
options which then try to decipher its meaning as used in the journal Nature Materials(D)/ according to the study published
sentence. Choose the option which gives the correct meaning (E)/
of the phrase most appropriately in context of the given (a)BCAED (b)CABED
sentence. (c)EDCAB (d)DCAEB
(e)CAEBD
Q176. Used books are a dime a dozen and it is difficult to sell them.
(a) Return (b) Tolerate Q182. forest cover had better (A)/ with predominant (B)/ were cropland
(c) Diminish (d) Failed dominated (C)/ In general, districts (D)/ resilience than those
(e) Common districts (E)/
(a)DBAEC (b)BAECD
Q177. We had a falling-out during our holiday and we have not spoken (c)ABEDC (d)ECDAB
since. (e)CBAED
(a) Insistent (b) Skip
(c) Disagreement (d) Rigid Q183. of a jellyfish protein that later(A)/ scientists shared the 2008
(e) Lenient Nobel prize(B) / contributed to cancer studies(C)/ for the
discovery and development(D) / Shimomura and two American
Q178. The singer received a tough break when he became sick (E)/
immediately before the music contest. (a)EBCDA (b)EBDAC
(a)Impossible (b)Misfortune (c)EBACD (d)ACDBE
(c)Illicit (d)Boldness (e)ABCED
(e)Strong
Q179. I like adventure movies but above all I love horror movies. Q184. controlling the natural fracturing(A)/process of atomically-thin
(a)Especially (b)Stubborn brittle materials(B)/ devices which the team calls (C)/the key to
(c)Biased (d)Amend making such tiny (D)/ “syncells” in large quantities lies in(E)
(e)Efficient (a)CABDE (b)EDCBA
(c)BECAD (d)DCEAB
Q180. The children began to act up during the field trip. (e)ADBCE
(a)Humanity (b)Implicit
(c)Exaggerate (d)Misbehave Q185. growers’ preferred shade tree now(A)/ The exotic silver oak may
(e)Disguise be coffee(B)/ in Kodagu’s agroforest systems(C)/ but research
shows that it affects carbon(D)/ sequestration and tree
Directions (181-185): In the question given below, some diversity(E)
sentences/phrases are given which have to be arranged in a (a)DCEAB
proper sequence. Select the option which best defines the (b)EABCD
proper sequence and arranges the sentence in an appropriate (c)BADEC
way. (d)CDBAE
(e)ABECD

Facebook Page- https://www.facebook.com/vishalpariharpage Youtube- https://www.youtube.com/vishalparihar


Follow
36 Vishal Sir Telegram Channel- https://t.me/englishbyvishalsirchannel Instagram- https://www.instagram.com/vishalthetrainer
By Vishal sir
Also Important For SBI CLERK, IBPS CLERK, RRB CLERK
& Other Competitive Exams

ANSWER WITH SOLUTION output data, according to the study published in the journal Nature
Materials.”
S176. Ans. (e)
Sol. Option (e) is the correct answer choice. ‘dime a dozen’ S182. Ans. (a)
means common, easy to get and of little value. All the other words Sol. The correct sequence is DBAEC as it arranges the sentence
fail to express the meaning of the given idiom. properly in the way as follows: “In general, districts with
predominant forest cover had better resilience than those districts
S177. Ans. (c) were cropland dominated.”
Sol. Option (c) is the correct answer choice. ‘a falling-out’ means
a disagreement or quarrel with someone. All the other words fail
to express the meaning of the given idiom. S183. Ans. (b)
Insistent means insisting on or demanding something; not Sol. The correct sequence of the statement is EBDAC as it
allowing refusal. arranges the sentence as follows: “Shimomura and two American
Rigid means unable to bend or be forced out of shape; not scientists shared the 2008 Nobel prize for the discovery and
flexible. development of a jellyfish protein that later contributed to cancer
Lenient means of a punishment or person in authority) more studies.”
merciful or tolerant than expected
S184. Ans. (d)
S178. Ans. (b) Sol. Option (d) provides the correct sequence for the arrangement
Sol. Option (b) is the correct answer choice. ‘a tough break’ of the sentence. The proper arranged sentence is as follows: “The
means an unlucky event, a misfortune. All the other words fail to key to making such tiny devices, which the team calls “syncells”
express the meaning of the given idiom. in large quantities lies in controlling the natural fracturing process
of atomically-thin, brittle materials.”
S179. Ans. (a)
Sol. Option (a) is the correct answer choice. ‘above all’ means
mainly, especially. All the other words fail to express the S185. Ans. (c)
meaning of the given idiom. Sol. The correct sequence is BADEC as the sentence can be
arranged as follows: “The exotic silver oak may be coffee
growers’ preferred shade tree now, but research shows that it
S180. Ans. (d) affects carbon sequestration and tree diversity in Kodagu’s
Sol. Option (d) is the correct answer choice. ‘act up’ means to agroforest systems.”
misbehave. All the other words fail to express the meaning of the
given idiom.

S181. Ans. (b)


Sol. The correct sequence is CABED as it arranges the sentence
in the best way as follows: “Embedded inside these pockets are
electronic circuits and materials that can collect, record, and

Facebook Page- https://www.facebook.com/vishalpariharpage Youtube- https://www.youtube.com/vishalparihar


Follow
37 Vishal Sir Telegram Channel- https://t.me/englishbyvishalsirchannel Instagram- https://www.instagram.com/vishalthetrainer
By Vishal sir
Also Important For SBI CLERK, IBPS CLERK, RRB CLERK
& Other Competitive Exams

Q(186-188):Find out the word from the given option which are similar The new banks also intend to re-channel bank credit -from the big
in meaning to the word given in bold in the passage . industries to the small sectors. With the intention of promoting
rural banking, regional rural banks were established. These
Q186. Embedded aligned the local field with the rural problems. These banks are
(a)Dislodged (b)Ingrained (c)Nonintegrated not to replace the other credit-giving bodies but to supplement
(d)Defenestrated (e)Displaced them.
The Steering Committee of the Regional Rural Banks considered
Q187. Contemplate some structural changes. First of all they gave thought to the
(a)Scorn (b)Discard staffing spectrum, then to effective coordination among banks-
(c)Abandon (d)Ruminate rural, cooperatives and commercial and the possibility of bringing
(e)Desert credit within the access of weaker sections. They wanted to
recruit staff for the rural banks at lower salaries. But this type of
Q188. Disproportionate discrimination would have been unfruitful. So, it was given up.
(a)Commensurate (b)Inordinate A problem with regard to the rural banks is the creditworthiness
(c)Equitable (d)Correspondent of the poor. The Indian farmers are so poor that they cannot pay
(e)Equivalent back their loans. The rural Indian surveys make it quite clear that
practically rural Indian surveys make it quite clear that practically
Q(189-190) :Find out the word from the given option which is opposite rural farmers have no creditworthiness. Their socio-economic
in meaning to the word given in bold in the passage . mobility is almost zero. That is, why banks fear that their credit
will never be paid back.
Q189. Notions Another difficulty for the rural banks is that loans cannot be
(a)Concepts (b)Conceptions (c)Certainties processed so easily. Processing loans also entails heavy
(d)Caprices (e)Inklings expenditure. This was also going to affect their financial position.
Still the establishment of the rural banks was decided because the
Q190. Veracity social advantages were more important than the commercial
(a)Authenticity (b)Probity consideration.
(c)Fidelity (d)Reputability Rural banks definitely encourage saving. No doubt the villagers
(e)Mendacity do not have to pay income tax and they get many other
concessions, yet their saving is not significant. Despite all the
Directions (191-200): Read the following passage carefully hurdles, the rural banking system will boost up the economy of
and answer the questions given below it. Certain villages and thereby the economy of the country.
words/phrases have been printed in bold to help you locate
them while answering some of the questions. Q191. Which of the following is/are the purpose/s of setting up banks in
rural areas?
A large number of branches of banks have been set-up in villages. A. Replacing other credit-giving bodies.
The main purpose of setting up these banks is to develop the habit B. Giving loans to farmers.
of saving among the villagers and also give loans to farmers for C. Increasing the amount of savings of a villager.
boosting production in one way or the other. So, far banks had (a) Only B
been concentrated in the bigger cities and Indian villagers had (b) A and B (c) B and C
no faith in them. (d) Only A (e) All of these

Facebook Page- https://www.facebook.com/vishalpariharpage Youtube- https://www.youtube.com/vishalparihar


Follow
38 Vishal Sir Telegram Channel- https://t.me/englishbyvishalsirchannel Instagram- https://www.instagram.com/vishalthetrainer
By Vishal sir
Also Important For SBI CLERK, IBPS CLERK, RRB CLERK
& Other Competitive Exams

ANSWER WITH SOLUTION Sol. Veracity means conformity to facts; accuracy.


Mendacity means untruthfulness. Hence it is the only antonym of
S186. Ans.(b) the given word.
Sol. Embedded means implanted. Authenticity means originality.
Ingrained means firmly fixed or established; difficult to change. Probity means the quality of having strong moral principles;
Dislodged means remove from a position of power or authority. honesty and decency.
Defenestrated means remove or dismiss (someone) from a Fidelity means faithfulness to a person, cause, or belief,
position of power or authority. demonstrated by continuing loyalty and support.
Reputability means the state of having a good reputation.

S187. Ans.(d) S191. Ans.(c)


Sol. Contemplate means think about. Sol. Refer to first paragraph of the passage. “The main purpose
Ruminate means think deeply about something. of setting up these banks is to develop the habit of saving among
Scorn means a feeling and expression of contempt or disdain for the villagers and also give loans to farmers for boosting
someone or something. production in one way or the other.”
Discard means get rid of (someone or something) as no longer
useful or desirable.
Abandon means cease to support or look after (someone); desert.

S188. Ans.(b)
Sol. Disproportionate means too large or too small in comparison
with something else.
Inordinate means unusually or disproportionately large;
excessive.
Commensurate means in proportion.
Equitable means fair and impartial.
Correspondent means parallel, equivalent.

S189. Ans.(c)
Sol. Notions means ideas or beliefs.
Certainties means surety about something.
Hence it is the only answer here which is in opposite of the given
word.
Caprices means a sudden and unaccountable change of mood or
behavior.
Inklings means a slight knowledge or suspicion; a hint.

S190. Ans.(e)

Facebook Page- https://www.facebook.com/vishalpariharpage Youtube- https://www.youtube.com/vishalparihar


Follow
39 Vishal Sir Telegram Channel- https://t.me/englishbyvishalsirchannel Instagram- https://www.instagram.com/vishalthetrainer
By Vishal sir
Also Important For SBI CLERK, IBPS CLERK, RRB CLERK
& Other Competitive Exams

Q192. The structural changes made by the steering Committee were in Directions (196-198): Choose the word /group of words which
respect of is most similar in meaning to the word/group of words
(a) staffing, co-ordinating and providing access to weaker printed in bold as used in the passage.
sections
(b) building smaller buildings to house the banks Q196. Concentrated
(c) investing very little in terms of infrastructure required to start (a) Clustered (b) Rigorous
a bank (c) Attentive (e) Diluted
(d) discriminating between urban bank staff and rural bank staff (e) Intense
(e) None of the above
Q197. Access
Q193. Which of the following is possibly the most appropriate title for (a) Admittance
the passage? (b) Reach
(a) Regional Rural Banks (c) Admission
(b) The Rural Consumer (d) Entry
(c) Microfinance in Rural India (e) permission
(d) Characteristics of Indian Villages
(e) Banking Concepts in India Q198. Entails
(a) Recommends
Q194. Which of the following is not true according to the passage? b) Lasts
(a) Processing of loans by rural banks is difficult (c) Lists
(b) Staff of the rural banks is paid a lower salary as compared to (d) Involves
urban banks (e) Filters
(c) Rural banks may not make as much profit as their urban
counterparts Directions (199-200); Choose the word /group of words which
(d) Processing of loans by bank is not cheap is most opposite in meaning to the word/group of words
(e) Rural farmers are, many a time, unable to pay back the loans printed in bold as used in the passage.
they avail Q199. Boosting
(a) Reducing
Q195. Which of the following is one of the benefits of living in the (b) Managing
village, as mentioned in the passage? (c) Overwhelming
(a) People living in the villages enjoy a higher income than urban (d) Smoothening
counterparts (e) Heightening
(b) People living in villages do not have to pay income tax and
they also get other concessions Q200. Significant
(c) People living in villages have a better quality of life (a) Forgettable
(d) Villages are self-sufficient; hence they do not need outside (b) Untrustworthy
help for any activity (c) Reliable
(e) People living in villages are rarely in need of a loan (d) Irregular
(e) Little

Facebook Page- https://www.facebook.com/vishalpariharpage Youtube- https://www.youtube.com/vishalparihar


Follow
40 Vishal Sir Telegram Channel- https://t.me/englishbyvishalsirchannel Instagram- https://www.instagram.com/vishalthetrainer
By Vishal sir
Also Important For SBI CLERK, IBPS CLERK, RRB CLERK
& Other Competitive Exams

ANSWER WITH SOLUTION Sol. Boosting means to help or encourage (something) to


increase or improve. So, Reducing is the word which is opposite
S192. Ans.(a) in meaning to it.
Sol. Refer to third paragraph of the passage. “First of all they S200. Ans.(e)
gave thought to the staffing spectrum, then to effective Sol. Significant means sufficiently great or important to be
coordination among banks-rural, cooperatives and commercial worthy of attention. So, Little is the word which is opposite in
and the possibility of bringing credit within the access of weaker meaning to it.
sections.”
S193. Ans.(a)
Sol. Regional Rural Banks
S194. Ans.(b)
Sol. ‘Staff of the rural banks is paid lower salary as compared to
urban banks' is not true according to the passage.
S195. Ans.(b)
Sol. Refer to last paragraph of the passage. “No doubt the
villagers do not have to pay income tax and they get many other
concessions, yet their saving is not significant.”
S196. Ans.(a)
Sol. Concentrated means gathered together. So, Clustered is the
word which is similar in meaning to it.
S197. Ans.(b)
Sol. Access means the means or opportunity to approach or enter
a place. So, Reach is the word which is similar in meaning to it.
S198. Ans.(d)
Sol. Entails means involve (something) as a necessary or
inevitable part or consequence. So, Involves is the word which is
similar in meaning to it.
S199. Ans.(a)

Facebook Page- https://www.facebook.com/vishalpariharpage Youtube- https://www.youtube.com/vishalparihar


Follow
41 Vishal Sir Telegram Channel- https://t.me/englishbyvishalsirchannel Instagram- https://www.instagram.com/vishalthetrainer
By Vishal sir
Also Important For SBI CLERK, IBPS CLERK, RRB CLERK
& Other Competitive Exams

Directions (201-210): Read each sentence to find out whether appropriately. Find out the appropriate word in each case. If
there is any grammatical or idiomatic error in it. The error, the word highlighted in bold does not require any
if any, will be in one part of the sentence. The number of that replacement, choose (e) as your answer.
part is the answer. If there is ‘No error’, the answer is (e).
(Ignore errors of punctuation, if any.) India may have won the final of the women’s Asia Cup hockey
tournament conflicting (211) China in a penalty shootout, but it
Q201. He told me (a) / that he wrote a letter (b) / to his superior (c) / for was a fully progressive (212) victory. Throughout their
a certain reason. (d) / No Error (e) campaign in Japan, team members played out of their skin to
recording (213)the win, which has secured them a spot in next
Q202. He reminded me (a) / that he has (b) / often told me not to (c) / year’s World Cup emphatically on merit, not as wild-card
play with fire.(d) / No Error (e) entrants. The realization (214) came against formidable hurdles
(215), and in the absence of any expectations. Indian women were
Q203. I would have asked (a) / you for dinner (b) / if I had known (c) /
never the favourites, going into the tournament ranked 12th in the
that you are staying here tonight. (d) / No Error (e)
world and fourth in Asia — behind World No. 8 China, Korea
Q204. He educated not only (a) / his nephew but also (b) / set him up (c) and Japan. Incidentally, India broken(216)defending champion
/ in business. (d) / No Error (e) Japan in the semi-finals. This happened despite a degree of pre-
tournament disarray. The team lost its second coach in the space
Q205. As soon as the peon rings (a) / the first bell (b) / then all the of a year, and Harendra Singh came to the assignment just a
students assemble (c) / on the playground for prayer. (d) / No month previous(217)the Asia Cup, that too with no previous
Error (e) experience of having worked with a women’s hockey team. The
team had other issues to feud (218) with as well; the fitness and
Q206. If the tourists (a) / would have come here, (b) / I had certainly skill levels had slipped. The lack of reality (219), sadly, was clear
have (c) / taken them around. (d) / No Error (e) from the fact that there was no live telecast, not even (220) online
streaming.
Q207. The teacher remarked (a) / very angrily yesterday (b) / that the
boys have (c) / all done it very badly. (d) / No Error (e) Q211. (a)obstructing (b) regarding
(c)against (d) about
Q208. Although it is summer (a) / now, the weather at the (b) / hill (e) No improvement required
station was (c) / quite pleasant.(d) / No Error (e)
Q212. (a)venerable (b)impending
Q209. The lawyer asked me (a) / where I had (b) / kept my clothes (c) / (c) aspiring (d) deserving
before taking a dip in the river. (d) / No Error (e) (e) No improvement required

Q210. The plays of Shakespear(a) / are not only studied (b) / in European Q213. (a)annal (b) lineup
countries (c) / but also in the oriental countries. (d) / No Error (e) (c) register (d) collect
(e) No improvement required
Directions (211-220): In the following passage there are
words highlighted in bold, each of which has been numbered. Q214. (a)recognition (b) acquisition
These numbers are printed below the passage and against (c) triumph (d) completion
each, four words are suggested, one of which fits the blank (e) No improvement required

Facebook Page- https://www.facebook.com/vishalpariharpage Youtube- https://www.youtube.com/vishalparihar


Follow
42 Vishal Sir Telegram Channel- https://t.me/englishbyvishalsirchannel Instagram- https://www.instagram.com/vishalthetrainer
By Vishal sir
Also Important For SBI CLERK, IBPS CLERK, RRB CLERK
& Other Competitive Exams

ANSWER WITH SOLUTION S207. Ans. (c)


Sol. Use ‘had’ in place of ‘have’ as the reporting verb is in past
S201. Ans. (b) tense.
Sol. ‘he had written’ will be used in place of ‘he wrote’ as in S208. Ans. (c)
Direct Narration, when reporting verb is in past tense and Sol. Use ‘is’ in place of ‘was’ as the sentence is in present tense.
reported speech is in simple past tense then in indirect narration, S209. Ans. (e)
reported speech is changed to past perfect tense. Sol. The sentence is grammatically correct.
Ex. He said to me, ‘I wrote a letter’ (Direct) S210. Ans. (b)
He told me that he had written a letter (Indirect) Sol. ‘not only’ will be used after ‘studied’.
S202. Ans. (b) S211. Ans. (c)
Sol. ‘had’ will be used in place of ‘has’ as past tense is used in Sol. ‘against is the correct word as the sentence talks about the
reporting verb hence past perfect tense will be used in reported final match of the tournament between India and China, which
speech. was won by India.
S203. Ans. (d) S212. Ans. (d)
Sol. ‘were’ will be used in place of ‘are’ because reporting clause Sol. ‘deserving’ is the correct word to be replaced as the
(I had known) is in past perfect tense hence past tense will be used paragraph is glorifying the victory of India hence deserving is the
in reported speech. most suitable word.
S213. Ans. (c)
S204. Ans. (a) Sol. ‘register’ is the most appropriate word making the sentence
Sol. ‘not only’ is used before ‘educated’. meaningful. In the sentence, a phrase has been used ‘out of their
Ex. Not only Ram but also Shyam went there. skin’ which means perform exceptionally well, which is used for
S205. Ans. (c) Indian team players, who played very well and won the
Sol. The Use of ‘then’ is superfluous as after ‘As, Because, Since, tournament.
as soon as’, ‘therefore, so, then’ is not used. S214. Ans. (c)
Ex. As soon as the bell rang, the students entered the class. Sol. ‘triumph’ best suits the purpose which means a great victory
S206. Ans. (e) or achievement. Here in the above sentences, the victory of India
Sol. The sentence is grammatically correct. has been discussed hence triumph is making the sentence
meaningful.

Facebook Page- https://www.facebook.com/vishalpariharpage Youtube- https://www.youtube.com/vishalparihar


Follow
43 Vishal Sir Telegram Channel- https://t.me/englishbyvishalsirchannel Instagram- https://www.instagram.com/vishalthetrainer
By Vishal sir
Also Important For SBI CLERK, IBPS CLERK, RRB CLERK
& Other Competitive Exams

Q215. (a)confinement (b) trust flows do not respect national boundaries, the environmental
(c) rampart (d) control trends must perforce be discussed at the regional inter-country
(e) No correction required level. As the largest nation-state of our region, and the biggest
polluter whose population is the most vulnerable, India needs to
Q216. (a)finished (b) exhausted be alert to the dangerous drift. China has been resolutely tackling
(c) discouraged (d) defeated air pollution and promoting clean energy. But while Beijing’s
centralised governance mandates environmentalism-by-decree,
(e) No correction required
the subcontinental realities demand civic participation for
sustainability to work. Unfortunately, despite being a vast
Q217. (a) back (b) before democracy where people power should be in the driving seat, the
(c) after (d) exceeding Indian state not only neglects its own realm, it does not take the
(e) No improvement required lead on cross-border environmentalism.
Thus, Bihar is helping destroy the Chure/Siwalik range of Nepal
Q218. (a)clench (b) grapple to feed the construction industry’s demand for boulders and
(c) wrangle (d) argue conglomerate, even though this hurts Bihar itself through greater
(e) No improvement required floods, desertification and aquifer depletion. Air pollution is
strangling the denizens of Lahore, New Delhi, Kathmandu and
Q219. (a) expectations (b) determination Dhaka alike, but there is no collaboration. Wildlife corridors
(c) stability (d) fact across States, provinces and countries are becoming constricted
(e) No improvement required by the day, but we look the other way. The UN Environment
Programme (UNEP) has chosen India to be the ‘host country’ to
mark World Environment Day today. But when will New Delhi
Q220. (a) undeviating (b) corresponding rise to connect the dots between representative democracy and
(c) firm (d) scheduled ecological sanity?
(e) No improvement required Truth be told, the environment ministry is invariably the least
empowered in the major countries of South Asia, without clout
Direction (221-230): Read the following passage carefully and vis-à-vis line ministries, and unable to coordinate the ecological
answer the questions given below it. Certain words are given response. Governments were content once to regard
in bold to help you locate them while answering some of the environmental protection as synonymous with wildlife
questions. protection. Today they stand unprepared when the challenges
have greatly multiplied and deepened. There is distress across the
Ecological ruin is on a gallop across South Asia, with life and ecological spectrum, but one need only study the rivers and the
livelihood of nearly a quarter of the world’s population affected. atmosphere to track the inaction of governments and our
Yet, our polities are able to neither fathom nor address the weakened activism. On water, the subcontinent is running out of
degradation. The distress is paramount in the northern half of the the resource due to the demands of industrialisation and
subcontinent, roping in the swathe from the Brahmaputra basin to urbanisation, and continuation of the colonial-era irrigation
the Indus-Ganga plain. model based on flooding the fields. The economic and
demographic forces are arrayed against the rivers and their right-
Within each country, with politics dancing to the tune of populist of-way. In the hills, the Ganga in Uttarakhand and the Teesta of
consumerism, nature is without a guardian. The erosion of civility Sikkim are representative of rivers that have been converted into
in geopolitics keeps South Asian societies apart when people dry boulder tracts by ‘cascades’ of run-of-river hydroelectric
should be joining hands across borders to save our common schemes. The same fate now threatens the rivers of Nepal and
ground. Because wildlife, disease vectors, aerosols and river

Facebook Page- https://www.facebook.com/vishalpariharpage Youtube- https://www.youtube.com/vishalparihar


Follow
44 Vishal Sir Telegram Channel- https://t.me/englishbyvishalsirchannel Instagram- https://www.instagram.com/vishalthetrainer
By Vishal sir
Also Important For SBI CLERK, IBPS CLERK, RRB CLERK
& Other Competitive Exams

India’s Northeast, while the tributaries of the Indus were ‘done (d)Both (b) and (c)
in’ decades ago through water diversion. (e)None of these

Everywhere, natural drainage is destroyed by highways and Q222. Why author finds the government responsible for the paced
railway tracks elevated above the flood line, and bunds encircling ecological imbalance and destruction?
towns and cities. Reduced flows and urban/industrial effluents (a)Government in each country is following the populist
have converted our great rivers into sewers. We refuse to consider consumerism theory
drip irrigation as a solution just as we fail to acknowledge that the (b)Government in countries are neither addressing the issue nor
rivers are made to carry hundreds of tonnes of plastics daily into they are getting it.
the Bay of Bengal and the Arabian Sea. While underground (c)Government in countries are not taking the lead on cross-
aquifers are exploited to exhaustion, the popular ‘river-training’ border environmentalism.
prescription imprisons our rivers within embankments, according (d)All of the above
to the inherited Western engineering canon that does not factor in (e)None of these
the natural silt carried by rivers of the Himalaya. The would-be
high-dam builders have not adequately studied the phenomenon Q223. According to the passage why South Asian societies are unable
of Himalayan cloudbursts, nor do they find it necessary to address to discuss the environmental trends at the regional inter-country
the question: how do you de-silt a deep reservoir when it fills up level?
with sand and mud? (a)As polities are able to neither fathom nor address the
degradation
As we have seen, the highs of environmental movements are (b)As formal politeness and courtesy in behavior or speech has
invariably followed by lows, and so to exit the cycle what is totally vanished in geopolitics
needed is an “environmental system” inbuilt into the (c)Today they stand unprepared when the challenges have greatly
infrastructure of state and society. Work towards ecological multiplied and deepened
sustainability must go beyond ritual, with the path seeming to lie (d)Both (a) and (c)
in the empowerment of local government all over. Elected (e)None of these
representatives in cities and districts must be challenged to
emerge as the bulwark of environmentalism even as the Q224. Which of the following options explain the most suitable meaning
provincial and national governments are asked to rise to their of the phrasal verb ‘done in’ used?
regulatory responsibilities. When ‘organic environmentalism’ (a)To make something happen
rises from the grassroots and makes state authority accountable, (b)Extremely tired
South Asia and its peoples will be protected. At that point, no (c)Be unwilling to tolerate or be bothered with
force will be able to stop activism across the frontiers and South (d)In a situation so bad that it is impossible to get out
Asia will begin to tackle pollution and dislocation as one. (e)None of these

Q221. According to the passage, what is the drawback of the popular Q225. What are the reasons that the subcontinent is running out of the
‘river-training’ prescription under inherited Western engineering water resources?
canon? (a)Industrialization and urbanization
(a)It doesn’t consider how to de-silt a deep reservoir when it fills (b)Continuation of the old irrigation model based on flooding the
up with sand and mud fields
(b)How to save natural drainage by highways and railway tracks (c)By continuous use of run-of-river hydroelectric schemes
elevated above the flood line (d)Natural drainages are being destroyed by highways and
(c)It does not factor in the natural silt carried by rivers of the railway tracks elevated above the flood line (e)All of the above
Himalaya.

Facebook Page- https://www.facebook.com/vishalpariharpage Youtube- https://www.youtube.com/vishalparihar


Follow
45 Vishal Sir Telegram Channel- https://t.me/englishbyvishalsirchannel Instagram- https://www.instagram.com/vishalthetrainer
By Vishal sir
Also Important For SBI CLERK, IBPS CLERK, RRB CLERK
& Other Competitive Exams

ANSWER WITH SOLUTION livelihood of nearly a quarter of the world’s population affected.
Yet, our polities are able to neither fathom nor address the
degradation.” Option (a) is stated in the first line of 2nd paragraph
S215. Ans. (e) “Within each country, with politics dancing to the tune of populist
Sol. No improvement is required here. consumerism, nature is without a guardian.” Option (c) can be
S216. Ans. (d) traced from the last line of the 2nd paragraph where it is stated as
Sol. ‘defeated’ is making the sentence meaningful as the sentence “Unfortunately, despite being a vast democracy where people
is telling about semifinals whereas India won the tournament power should be in the driving seat, the Indian state not only
hence it is clear that India defeated Japan in semifinals to reach neglects its own realm, it does not take the lead on cross-border
the finals. environmentalism.”

S217. Ans. (b) S223. Ans. (b)


Sol. ‘before’ is the most appropriate word as above sentence Sol. Option (b) is the correct answer choice. Option (b) can be
indicates about the disarray (disorder) which makes us clear traced from the 2nd paragraph of passage where it is stated as “The
erosion of civility in geopolitics keeps South Asian societies apart
that the coach left the team before the Asia Cup.
when people should be joining hands across borders to save our
common ground.”
S218. Ans. (b)
Sol. ‘grapple’ best suits the purpose as it means struggle. S224. Ans. (b)
Feud means a prolonged and bitter quarrel or dispute. Sol. option (b) is the most suitable answer choice.
Wrangle means a dispute or argument, typically one that is long ‘Done in’- extremely tired.
and complicated.
S219. Ans. (a) S225. Ans. (e)
Sol. ‘expectations’ is the correct word as it is also used in above Sol. option (e) is the most suitable answer choice as all the given
sentences of the paragraph. statements are true. Option (a), (b) and (c) can be traced from the
4th paragraph where it is given as “On water, the subcontinent is
S220. Ans. (e) running out of the resource due to the demands of
Sol. No improvement is required here. industrialization and urbanization, and continuation of the
colonial-era irrigation model based on flooding the fields. The
economic and d
S221. Ans. (c)
emographic forces are arrayed against the rivers and their right-
Sol. Option (c) is the correct answer choice. Option (c) can be
of-way. In the hills, the Ganga in Uttarakhand and the Teesta of
traced from the 5th paragraph of passage where it is stated as
Sikkim are representative of rivers that have been converted into
“While underground aquifers are exploited to exhaustion, the
dry boulder tracts by ‘cascades’ of run-of-river hydroelectric
popular ‘river-training’ prescription imprisons our rivers within
schemes.” Option (d) can be traced from the 5th paragraph where
embankments, according to the inherited Western engineering
it is given as “Everywhere, natural drainage is destroyed by
canon that does not factor in the natural silt carried by rivers of
highways and railway tracks elevated above the flood line, and
the Himalaya.”
bunds encircling towns and cities. Reduced flows and
urban/industrial effluents have converted our great rivers into
S222. Ans. (d)
sewers.”
Sol. Option (d) is the correct answer choice. Option (b) can be
traced from the 1st paragraph of passage where it is stated as
“Ecological ruin is on a gallop across South Asia, with life and

Facebook Page- https://www.facebook.com/vishalpariharpage Youtube- https://www.youtube.com/vishalparihar


Follow
46 Vishal Sir Telegram Channel- https://t.me/englishbyvishalsirchannel Instagram- https://www.instagram.com/vishalthetrainer
By Vishal sir
Also Important For SBI CLERK, IBPS CLERK, RRB CLERK
& Other Competitive Exams

Directions (226-227): Choose the word which is most similar normal. However, after 10 minutes into the flight, the
in meaning to the word printed in bold as used in the passage. _____(234)_____ of these passengers began. Suddenly the
oxygen masks came down from above their seats. By this time
Q226. Gallop the aircraft was flying at a/an _____(235)_____ of about 28,000
(a)Amble (b)Stroll feet. Suddenly the _____(236)_____ started feeling
(c)Crawl (d)Hasten uncomfortable while some started to bleed from nose and ears.
(e)Lumber The panicked passengers _____(237)_____ putting oxygen
masks onto themselves while many _____(238)_____ that they
Q227. Fathom
(a)Misinterpret (b)Misunderstand were not working. After about 10-15 minutes of
(c)Neglect (d)Guess _____(239)_____ and panic the oxygen started flowing into the
(e)Understand masks, much to the _____(240)_____ of the passengers.

Q228. Perforce Q231. (a) scary (b) horrifying


(a)Inevitably (b)Unnecessarily (c) chilling (d) happy
(c)Subliminally (d)Equivocally (e) intimidating
(e)None of these
Q232. (a) flight (b) car
Directions (229-230): Choose the word which is most opposite (c) airplane (d) aircraft
in meaning to the word printed in bold as used in the passage.
(e) plane
Q229. Denizens
(a)Inhabitant (b)Resident Q233. (a) translated (b) seemed
(c)Alien (d)Townsman (c) appeared (d) looked
(e)Native (e) None of the above

Q230. Fate Q234. (a) difficulty (b) pleasure


(a)Influence (b)Misfortune (c) agony (d) misery
(c)Chance (d)Destiny (e) trouble
(e)Consequence
Q235. (a) altitude (b) height
Directions (231-240): In the following passage there are (c) building (d) elevation
blanks, each of which has been numbered. These numbers are (e) none of the above
printed below the passage and against each, five options are
given. Four of the five words fit into the blank coherently. Q236. (a) soldiers (b) passengers
Find out the word which does not fit into the blank (c) travelers (d) commuters
appropriately. (e) voyagers

It was a _____(231)_____ journey that these 166 passengers of Q237. (a) began (b) initiated
Jaipur bound Jet Airways flight number 9W 697 from Mumbai (c) commenced (d) started
on Thursday will never forget. When the _____(232)_____ took (e) stopped
off at about 6 am on Thursday everything _____(233)_____

Facebook Page- https://www.facebook.com/vishalpariharpage Youtube- https://www.youtube.com/vishalparihar


Follow
47 Vishal Sir Telegram Channel- https://t.me/englishbyvishalsirchannel Instagram- https://www.instagram.com/vishalthetrainer
By Vishal sir
Also Important For SBI CLERK, IBPS CLERK, RRB CLERK
& Other Competitive Exams

ANSWER WITH SOLUTION can’t took off. So, the correct misfit is the option (b) which is
‘car’.
S226. Ans. (d) The correct phrase should have been ‘When the flight took off’.
Sol. Option (d) is the best answer choice here. S233. Ans. (a)
Gallop- a very fast pace of running Sol. The sentence where the given blank appears seems to be
All the given options are the antonyms of the given word ‘gallop’ composed of two clauses—one subordinate clause, starting with
except option (d) ‘when’ and the other independent clause.
‘everything __________ normal’ should be an independent
S227. Ans. (e) clause. Among the given options, options (b), (c) and (d) seem to
Sol. Option (e) is the best answer choice here. fit the blank appropriately, but the option (a) doesn’t and is
Fathom-understand (a difficult problem or an enigmatic person) irrelevant as per the context of the sentence.
after much thought The correct clause should be ‘everything
All the given options are different in meaning except ‘understand’ seemed/appeared/looked normal’.
S228. Ans. (a)
Sol. Perforce- used to express necessity or inevitability.
Subliminally- below the threshold of sensation or consciousness S234. Ans. (b)
Equivocally- in a deliberately ambiguous or questionable way Sol. The tone and the context of the paragraph suggests that the
Option (a) is the best answer choice here. passengers of the airplane were in trouble and had a bad
experience. So, option (b) ‘pleasure’ is a completely misfit word.
S229. Ans. (c) The correct clause should be ‘the trouble of these passengers
Sol. Denizens- a person, animal, or plant that lives or is found in began’.
a particular place S235. Ans. (c)
All the given options are the synonyms of the given word Sol. Something was flying. That something would be flying at a
‘denizen’ except option (c), ‘alien’ height.
The correct phrase should have been ‘was flying at an altitude’.
S230. Ans. (b) Among the given options, ‘building’ which is the option (c) is a
Sol. Fate- the development of events outside a person's control,
mis-fit option and is the correct answer.
regarded as predetermined by a supernatural power.
S236. Ans. (a)
All the given options are either the synonyms or altogether
different in meaning to the given word ‘fate’, except option (b). Sol. The given passage talks about the suffering or tribulations of
the passengers of the Jet Airways flight.
S231. Ans. (d) Option (a) ‘soldiers’ is a mis-fit and is the correct answer.
Sol. The hint for the blank can be derived from a sentence of the The correct clause should have been ‘Suddenly the passengers
third paragraph ‘Suddenly the oxygen masks came down from started feeling…’.
above their seats.’ The sentence suggests that the journey wasn’t
a happy journey. The words ‘scary, horrifying, chilling and S237. Ans. (e)
intimidating’ are synonyms. So, the correct mis-fit for the word Sol. The tone of the sentence and the context of the passage
is ‘happy’. Hence, option (d) is the correct answer. suggests that the passengers would have started putting on the
oxygen mask. So, the option (e) which is ‘stopped’ is a mis-fit
S232. Ans. (b) word and is the correct answer.
Sol. Something took off. The given passage talks about an The correct clause should have been ‘The panicked passengers
airplane belonging to the Jet Airways. A car is something which started putting oxygen masks’.

Facebook Page- https://www.facebook.com/vishalpariharpage Youtube- https://www.youtube.com/vishalparihar


Follow
48 Vishal Sir Telegram Channel- https://t.me/englishbyvishalsirchannel Instagram- https://www.instagram.com/vishalthetrainer
By Vishal sir
Also Important For SBI CLERK, IBPS CLERK, RRB CLERK
& Other Competitive Exams

Q238. (a) complained (b) criticized Q244. On Monday, the government _________that it would merge
(c) praised (d) whined Bank of Baroda (Bank of Baroda), Vijaya Bank and Dena Bank
(e) moaned to form the third largest _________in India.
(a) announced, lender (b) decided, investor
Q239. (a) hue and cry (b) disturbance (c) decided, retailor (d) sought, bank
(c) uproar (d) enjoyment (e) received, lender
(e) ruckus
Q245. The Union cabinet on Wednesday approved a proposal for the
Q240. (a) consolation (b) comfort complete electrification of Indian Railways in the next four years,
(c) ease (d) relief reducing dependence on imported fossil fuel and saving revenue
(e) problem for the national carrier.
(a) postponed, railways (b) announced, car
Directions (241-245): Each question below has two blanks. (c) discussed, transport (d) told, railways
There are five pairs of words below the sentence. Each pair is (e) approved, carrier
lettered. Choose the pair of words which can be filled up in
the blanks in the sentence in the same order so as to complete Directions (246-250): In the following questions, a sentence is
the sentences meaningfully. divided into four parts with one of the parts of each sentence
highlighted thus suggesting the grammatically correct part of
Q241. With just three months left in state assembly elections, Rashtriya the sentence. Out of the four other parts, choose the part of
Swayamsevak Sangh (RSS) Sarsanghchalak Dr Mohan Rao the sentence which contains grammatical or contextual error
Bhagwat will be on a 10-day _______of Rajasthan where he will in it. If the given sentence is both grammatically correct and
_______ in Jodhpur, Nagaur and Jaipur. contextually meaningful, choose option (e) i.e., “No error” as
(a) pleasure, stay (b) stay, sleep your answer.
(c) tour, camp (d) running, run
(e) visit, sleep Q246. The move will be cost (A)/ the government 12,134 crore, (B)/ rail
minister Piyush Goyal said (C)/ after a meeting of the (D)/
Q242. Jet Airways (India) Ltd ______ it would ________ its 49% stake Cabinet Committee on Economic Affairs (CCEA).
in Jet Privilege Pvt. Ltd, the frequent-flyer loyalty programmed (a) A (b) B (c) C (d) D (e) No error
subsidiary.
(a) walked, sell (b) slept, eat Q247. The release of Manto (A)/ directed by Nandita Das / and with
(c) asserted, play (d) said, monetize Nawazuddin Siddiqui in the lead (B)/, is momentous event in the
(e) asserted, manages life (C)/ of the great Urdu writer’s family (D).
(a) A (b) B (c) C (d) D
Q243. Solar capacity installations ________the country is at 25,000 (e) No error
megawatts (MW), or ________ of the 2022 target of 100,000
MW, according to Mercom India research. Q248. No wonder then that for some time now, (A)/ media and technology
(a) on, one-eighth (b) in, one-fourth companies for the world (B)/ have looked at (C)/ Artificial
(c) below, two-third (d) at, excessive Intelligence (AI)-led algorithms / as the saviour—the demon-
(e) in, one-eighth slayers (D).
a) A (b) B (c) C (d) D (e) No error

Facebook Page- https://www.facebook.com/vishalpariharpage Youtube- https://www.youtube.com/vishalparihar


Follow
49 Vishal Sir Telegram Channel- https://t.me/englishbyvishalsirchannel Instagram- https://www.instagram.com/vishalthetrainer
By Vishal sir
Also Important For SBI CLERK, IBPS CLERK, RRB CLERK
& Other Competitive Exams

ANSWER WITH SOLUTION The option which provides correct and relevant alternatives to the
S238. Ans. (c) blank is the option (d).
Sol. The context of the sentence where the blank appears suggest S243. Ans. (b)
that the oxygen masks weren’t working. So, the commuters Sol. The context of the sentence suggests that the blank would be
would have ‘complained’ about it. filled by a preposition to impart a correct relationship between
The words ‘criticized, whined and moaned’ are also appropriate ‘Solar capacity installations is at 25,000 megawatts (MW)’ and
for the blank, but the option (c) which is ‘praised’ is completely ‘the country’. The phrase ‘______ the country’ is behaving as an
irrelevant and a misfit. adverb-phrase. Among the given alternatives for the first blank,
Hence, option (c) is the correct answer. the correct preposition to be used is ‘in’.
The correct clause should have been ‘while many complained…’ Now, 25,000 megawatts (MW) is one-fourth of the 100,000 MW.
S239. Ans. (d) So, the correct answer is the option (b).
Sol. The correct words to describe the situation faced by the S244. Ans. (a)
passengers when oxygen masks weren’t working, and the Sol. Kindly focus on the clause ‘it (government) would merge
passengers were facing life-threatening inconvenience are ‘hue Bank of Baroda, Vijaya Bank and Dena Bank to form the third
and cry, disturbance, uproar, ruckus among others’. Option (d) largest ______ in India’. Three banks would be merged to give a
‘enjoyment’ is a completely irrelevant option and is a mis-fit. larger bank. Notice that the word ‘lender’ is a synonym for the
Hence, the correct answer is option (d). word ‘bank’. For the first blank, both ‘announced’ and ‘decided’
The correct phrase should have been ‘After about 10-15 minutes are appropriate.
of uproar’. Considering the above, the correct option which imparts correct
S240. Ans. (e) and relevant words to the blanks is the option (a).
Sol. Flowing of oxygen into the oxygen masks would and should Hence, option (a) is the correct answer.
have brought relief to the passengers. S245. Ans. (e)
Words like ‘consolation, comfort, and ease’ are also appropriate. Sol. The phrase ‘reducing dependence on imported fossil fuel…’
suggests that the proposal was approved. For the second blank,
The word ‘problem’ is completely irrelevant and a misfit as per
the context of the sentence. the appropriate words are ‘carrier’ and ‘railways’.
Hence, the correct answer is option (e). S246. Ans. (a)
The correct phrase would have been ‘much to the relief of the Sol. There is an error in the part (A). The erroneous phrase is ‘will
passengers’. be’. The correct phrase will be ‘will cost’. So, the correct answer
S241. Ans. (c) is option (a).
Sol. The context of the sentence suggests that RSS S247. Ans. (D)
Sarsanghchalak will be visiting Rajasthan where he stay there for Sol. There is an error in the part (D), and the erroneous part is ‘is
days. Among the given options, option (c) provides the best momentous event’. The correct phrase would be ‘is a momentous
alternatives to the blank and is the correct answer. event’. ‘Momentous event’ is a noun-phrase (noun type- common
S242. Ans. (d)The word ‘manages’ for the second blank doesn’t impart noun) and should be preceded by an article.
complete sense to the sense, but an incomplete sense. Jet Airways Hence, the correct answer is option (d).
would manage its 49% stake… If the stakes were already S248. Ans. (B)
belonged to Jet Airways, what is the need to state that it would Sol. There is an error in the part (B), and the error is in the phrase
manages its own stake. That’s already understood. ‘companies for the world’. Incorrect preposition ‘for’ is used in
the phrase. The correct preposition to be used is ‘around’.
Hence, the correct answer is option (b).

Facebook Page- https://www.facebook.com/vishalpariharpage Youtube- https://www.youtube.com/vishalparihar


Follow
50 Vishal Sir Telegram Channel- https://t.me/englishbyvishalsirchannel Instagram- https://www.instagram.com/vishalthetrainer
By Vishal sir
Also Important For SBI CLERK, IBPS CLERK, RRB CLERK
& Other Competitive Exams

Q249. Like every weekday (A)/ since the past two years, (B)/ she made in the exhibition. Tribal Woman is another piece that depicts the
her way / to platform number 7 (C)/ and waited for the full frontal face of a figure. Made with crumpled rice paper, the
Churchgate-bound train from Virar (D). painting prompts the viewer to take note of its textural
(a) A (b) B (c) C (d) D (e) No error complexity.
The ochres here are however blended with deep browns and rusty
Q250. All six writers (A)/ who made it to the finals (B)/ of the Booker’s reds and emit a fiery glow that emanates from the centre of the
prize (C)/ are from the UK, (D)/ the US, and Canada. woman’s face. “The color yellow ochre is the first thing that you
would notice when you visit places that are occupied by tribal
(a) A (b) B (c) C (d) D (e) No error
folks. “The forests and sand take on such a tone at around four or
five in the evening,” he said. The piece’s tactility is further
Direction (251-257): Read the following passage carefully and
enhanced by the decorative patterns that crowd the background
answer the questions given below it. Certain words are given
of the otherwise softly contoured figure.
in bold to help you locate them while answering some of the
questions.
Whether through the forms themselves or through what they
exemplify, Saravanan has portrayed generic aspects of tribal life.
Walking into S Saravanan’s ongoing exhibition at Artworld: His paintings Tribals, Love, and Mother and Daughter among
Sarala’s Art Centre, is like entering a sandstorm; shades of yellow others, all delineate figures that are reflected in the metal relief
ochre engulf you. In what he considers his ‘Tryst With Forms’, works displayed at the other end of the gallery. Here, religious
the show is his first one-man exhibit that largely consists of and mythological figures are given due consideration: Radha,
paintings and metal reliefs put together over the last year. At first Krishna, Ganesha and Buddha all form part of the artist’s
glance — and even more so on close inspection — his abstract pantheon of tribal deities, in line with the general theme of the
sensibility confounds the viewer. Only by standing afar can one show. The works as such, deal more with forms — as they are
really see the figures — all angular and Picasso-esque — that meant to — rather than content. What would have bolstered
comprise his works. Dream for instance, is one of the larger Saravanan’s remarkable tryst with forms is an anchoring into
works in this show, depicting the full frontal face of a figure, the specificities of the regions that undoubtedly inspire him.
long eyelashes and circular patterns near the earlobes hinting at a
feminine disposition. The figure however, is more of an outline,
Q251. Why has the author called walking into S Saravanan’ s ongoing
a contour — skeletal in its inclination and not possessing the
exhibition at Artworld is like entering into a sandstorm?
weight that is commonly accorded to skin. Be it the kaleidoscopic
patterns that permeate the piece or the rendering of the figure in (a)As the ochres here are blended with deep browns and rusty
itself , there is a Cubist inclination to break the form — perhaps reds and emit a fiery glow that emanates from the centre of the
even reveal what is under. woman’s face.
Yet, there is an insistence, an overpowering urge to touch the (b)Shades of yellow ochre engulf you.
surface, like an old wound that demands to be caressed in order (c)The color yellow ochre is the first thing that you would notice
to aid memories. Made with texture white — a multipurpose resin when you visit places that are occupied by tribal folks.
medium that aids in the development of textures — on canvas, (d)Both (b) and (c) (e)None of these
the piece helps compose a movement, a back and forth, between
two sensibilities; that of sight (the figure itself) and touch (the
medium). This dance that pushes the viewer to physically engage
with the works is mostly experienced with Saravanan’s paintings

Facebook Page- https://www.facebook.com/vishalpariharpage Youtube- https://www.youtube.com/vishalparihar


Follow
51 Vishal Sir Telegram Channel- https://t.me/englishbyvishalsirchannel Instagram- https://www.instagram.com/vishalthetrainer
By Vishal sir
Also Important For SBI CLERK, IBPS CLERK, RRB CLERK
& Other Competitive Exams

ANSWER WITH SOLUTION


S249. Ans. (b)
Sol. There is an error in the part (B) of the sentence, and the
erroneous phrase is ‘since the past two years’. The usage of
‘since’ is incorrect, and the correct word to be used here is ‘for’.
Hence, the correct answer is option (b).

S250. Ans. (e)


Sol. The given sentence is grammatically correct and contextually
meaningful. The correct answer is option (e).

S251. Ans. (b)


Sol. Option (b) is the correct answer choice. Option (b) can be
traced from the very line of the 1st paragraph of passage where it
is stated as “Walking into S Saravanan’s ongoing exhibition at
Artworld: Sarala’s Art Centre, is like entering a sandstorm;
shades of yellow ochre engulf you.”

Facebook Page- https://www.facebook.com/vishalpariharpage Youtube- https://www.youtube.com/vishalparihar


Follow
52 Vishal Sir Telegram Channel- https://t.me/englishbyvishalsirchannel Instagram- https://www.instagram.com/vishalthetrainer
By Vishal sir
Also Important For SBI CLERK, IBPS CLERK, RRB CLERK
& Other Competitive Exams

Q252. According to the passage, which of the following statements


supports the ‘Tactile beauty’ term for the work done by S Q255. What has supported Saravanan’s remarkable love with this tribal
Saravanan? art form?
(a)The piece’s tactility is further enhanced by the decorative (a) An anchoring into specificities of the regions.
patterns that crowd the background of the otherwise softly (b) His love for the tribal people.
contoured figure. (c)He is one of the people from the same tribe .
(b)The piece helps compose a movement, a back and forth, (d)Both (a) and (c)
between two sensibilities; that of sight and touch. This pushes the (e)None of these
viewer to physically engage with the works.
(c) An overpowering urge to touch the surface, like an old wound Q256. Choose the word which is most similar in meaning to the word
that demands to be caressed in order to aid memories. ‘Disposition’ printed in bold as used in the passage.
(d)All (a),(b) and (c) (a)Nature
(e)None of these (b)Distribution
(c)Transfer
Q253. According to the passage how S Saravanan’s abstract sensibility (d)Allocation
confounds the viewer? (e)Disposal
(a)The show is his first one-man exhibit that largely consists of
paintings and metal reliefs put together over the last year. Q257. Choose the word which is most similar in meaning to the given word
(b)The full frontal face of a figure, the long eyelashes and circular ‘Emanates’ printed in bold as used in the passage.
patterns near the earlobes hinting at a feminine disposition, one (a)Neglect
can dream of for instance, is one of the larger works in this show. (b)Halt
(c)Here, religious and mythological figures are given due (c)Disregard
consideration all form part of the artist’s pantheon of tribal (d)Ensue
deities, in line with the general theme of the show. (e)Repress
(d)Only by standing afar can one really see the figures — all
angular and Picasso-esque — that comprise his works Directions (258-263): In each of the questions given below a
(e)None of these sentence is given with one blank. Below each sentence FOUR
words are given out of which two can fit the sentence. Five
Q254. What are the characteristic features of the S Saravanan’s options are given with various combinations of these words.
portraits? You must choose the combination with the correct set of
(a)They exemplify generic aspects of tribal life. words which can fit in the given sentence.
(b)Religious and mythological figures are given due
consideration: Radha, Krishna, Ganesha and Buddha all form part Q258. The Trump administration worked to move ahead on its top
of the artist’s pantheon of tribal deities, in line with the general __________ priority.
theme of the show. (A)Diplomatic
(c)Tribal Woman is another piece that depicts the full frontal face (B)Rough
of a figure. Made with crumpled rice paper, the painting prompts (C) Consular
the viewer to take note of its textural complexity. (D)Obsessed
(d)Both (a) and (b) (a) A-B (b) B-C (c) A-D (d) A-C (e) B-D
(e)None of these

Facebook Page- https://www.facebook.com/vishalpariharpage Youtube- https://www.youtube.com/vishalparihar


Follow
53 Vishal Sir Telegram Channel- https://t.me/englishbyvishalsirchannel Instagram- https://www.instagram.com/vishalthetrainer
By Vishal sir
Also Important For SBI CLERK, IBPS CLERK, RRB CLERK
& Other Competitive Exams

ANSWER WITH SOLUTION pantheon of tribal deities, in line with the general theme of the
show.”
S252. Ans. (d) S255. Ans. (a)
Sol. Option (d) is the correct answer choice. Sol. Option (a) is the correct answer choice. Option (a) can be
Tactile- of or connected with the sense of touch. traced from the last paragraph of passage where it is stated as
Option (a) and (b) can be traced from the 2nd paragraph of “What would have bolstered Saravanan’s remarkable tryst with
passage where it is stated as “Yet, there is an insistence, an forms is an anchoring into specificities of the regions that
overpowering urge to touch the surface, like an old wound that undoubtedly inspire him.”
demands to be caressed in order to aid memories. Made with
texture white — a multipurpose resin medium that aids in the S256. Ans. (a)
development of textures — on canvas, the piece helps compose a Sol. Option (a) is the correct answer choice.
movement, a back and forth, between two sensibilities; that of Disposition- a person's inherent qualities of mind and character
sight (the figure itself) and touch (the medium). This dance that All the other given options are synonyms of the given word but
pushes the viewer to physically engage with the works is mostly in context of the law. So the suitable answer choice is option (a).
experienced with Saravanan’s paintings in the exhibition.” S257. Ans. (d)
Option (c) can be traced from the last lines of the 2nd paragraph Sol. Option (d) is the correct answer choice.
““The forests and sand take on such a tone at around four or five Emanates- (of a feeling, quality, or sensation) issue or spread out
in the evening,” he said. The piece’s tactility is further enhanced from (a source)
by the decorative patterns that crowd the background of the Ensue- happen or occur afterwards or as a result.
otherwise softly contoured figure.” Halt-bring or come to an abrupt stop.
Repress- subdue (someone or something) by force.
S253. Ans. (b) Q258. Ans.(d)
Sol. Option (b) is the correct answer choice. Option (b) can be Sol. Diplomatic means political.
traced from the 1st paragraph of passage where it is stated as Consular means ‘of or concerning diplomacy’;
“Only by standing afar can one really see the figures — all Both of these options fit most appropriately in the given as before
angular and Picasso-esque — that comprise his works. Dream for priority that must be at a top position it must be related to
instance, is one of the larger works in this show, depicting the full something which is politically related considering President and
frontal face of a figure, the long eyelashes and circular patterns its matters Donald Trump.
near the earlobes hinting at a feminine disposition.” Rough and Obsessed do not relate to the context.
S254. Ans. (d)
Sol. Option (d) is the correct answer choice. Option (a) and
Option (b) can be traced from the last paragraph of passage where
it is stated as “Whether through the forms themselves or through
what they exemplify, Saravanan has portrayed generic aspects of
tribal life. His paintings Tribals, Love, and Mother and Daughter
among others, all delineate figures that are reflected in the metal
relief works displayed at the other end of the gallery. Here,
religious and mythological figures are given due consideration:
Radha, Krishna, Ganesha and Buddha all form part of the artist’s

Facebook Page- https://www.facebook.com/vishalpariharpage Youtube- https://www.youtube.com/vishalparihar


Follow
54 Vishal Sir Telegram Channel- https://t.me/englishbyvishalsirchannel Instagram- https://www.instagram.com/vishalthetrainer
By Vishal sir
Also Important For SBI CLERK, IBPS CLERK, RRB CLERK
& Other Competitive Exams

Q259. The high level of asset prices today ____________the earlier Direction (264-269): In each of the questions given below a
trend in house prices. sentence is given which is then divided into five parts out of
(A)Mockeries (B)Mirrors which last part is correct. There are errors in some parts of
(C)Reflects (D)Refracts the sentence out of which only one part is correct. You must
(a) C-D (b) B-C choose the part as your answer.
(c) A-D (d) A-C
(e) B-D Q264. One of biggest (A)/problem we face at (B)/ our country is our(C)/
inability for (D)/work in
Q260. The rights of a citizen who has not been charged with a criminal teams(E).
act ought to be ____________ at all costs. (a)One of biggest (b)problem we face at
(A)Marred (B)Protected (c)our country is our (d)inability for
(C)Sabotaged (D)Safeguarded (e)All are incorrect
(a) B-D (b) A-D
(c) A-B (d) A-C Q265. The market condition when (A)/good and services are (B)/ not
(e) B-C free available(C) /and thus the prices is(D)/ relatively high is
seller’s market. (E)
Q261. Some laws are considered to be harmful to economic growth as (a)The market condition when
they breed ____________. (b)good and services are
(A)Inadequacy (B)Artifice (c)not free available
(C)Ingenuity (D) Inefficiency (d)and thus the prices is
(a) A-B (b) B-C (e)All are incorrect
(c) A-D (d) A-C
(e) B-D Q266. The Head of Government (A)/of countries (B) /which are member
(C) /of the Commonwealth meet for the (D) /awareness on social
Q262. The _______________ over the proposed Foreign Ministers’ issues. (E)
meeting has brought setback to India-Pakistan ties. (a)The head of Government
(A)Concord (B)Acrimony (b)of countries
(C)Animosity (D)Amenity (c)which are member
(a) B-C (b) A-B (d) of the Commonwealth meet for the
(c) B-D (d) A-C (e)All are incorrect
(e) C-D
Q267. New knowledge and new ways (A)/ for understanding the world
Q263. In Delhi, pollution levels, every winter, is ______________ (B)/, of good or bad(C)/, have always been (D)/part of every
reaching a stage of calamity. society.(E)
(A)Apace (B)Gradually (a)New knowledge and new ways
(C)Slowly (D)Forcibly (b)for understanding the world
(a) A-B (b) C-D (c)of good or bad
(c) A-C (d) B-D (d)have always been
(e) B-C (e)All are incorrect

Facebook Page- https://www.facebook.com/vishalpariharpage Youtube- https://www.youtube.com/vishalparihar


Follow
55 Vishal Sir Telegram Channel- https://t.me/englishbyvishalsirchannel Instagram- https://www.instagram.com/vishalthetrainer
By Vishal sir
Also Important For SBI CLERK, IBPS CLERK, RRB CLERK
& Other Competitive Exams

ANSWER WITH SOLUTION Concord means agreement or harmony between people or groups
S259. Ans. (b) Amenity means the pleasantness or attractiveness of a place.
Sol. Mirrors and reflects are the synonyms which fit most suitably S263. Ans.(e)
in the given blank.The sentence means that the high level of Sol. Gradually means by degrees.
prices which assets have today portray the earlier trend which Slowly means at a slow speed; not quickly.
house prices used to have. Apace means swiftly.
Mockeries means ridicule or derision. Forcibly means using force or violence.
Refracts means (of water, air, or glass) make (a ray of light) S264. Ans.(c)
change direction when it enters at an angle. Sol. There are errors in parts (A), (B) and (D). There is no error
Hence these are not relevant to the context of the sentence. in part (C) .
S260. Ans.(a) Part (A) must be ‘one of the biggest’. Usage of article ‘the’ is
Sol. Protected and safeguarded both are the synonyms which necessary before superlative degree of the word ‘big’.
mean protect from harm or damage with an appropriate measure. Part (B) must be ‘problem we face in’
Here these two are the appropriate words because the sentence Part (D) must be ‘inability to’.
speaks of citizens who have not been charged with a criminal act Hence option (c) is the correct answer.
so, the rights of these citizens must be protected and safeguarded S265. Ans.(a)
at any cost. Sol. There are errors in parts ( B) ,(C) and (D).
Marred means ‘impair the quality or appearance of; spoil’. Part (A) is free of error.
Sabotaged means ‘deliberately destroy, damage, or obstruct The correction is as follows:
(something), especially for political or military advantage.’ Part (B) must be goods and services.
These are inappropriate words to be used in the sentence. Part (C) must be not freely available.
S261. Ans.(c) Part (D) must be and thus the prices are.
Sol. Inadequacy means the state or quality of being inadequate; S266. Ans.(d)
lack of the quantity or quality required. Sol. Only part (D) of the sentence is correct. Parts (A), (B) and
Inefficiency means he state of not achieving maximum (C) are erroneous.
productivity; failure to make the best use of time or resources. In the part (A), the Heads of Government must be used as there
Here these two are the words that can be used as economic growth are more than one country. So, there should be more than one
will be hampered since some laws hold or breed inefficiency that head.
is inadequacy. In the part (B), of the countries must be used. The usage of article
Artifice means clever or cunning devices or expedients, ‘the’ is required.
especially as used to trick or deceive others. In the part (C), which are members must be used because we are
Ingenuity means the quality of being clever, original, and not talking about a singular country instead a plural noun that is,
inventive. countries so its members.
S262. Ans.(a) S267. Ans.(a)
Sol. Acrimony means bitterness or ill feeling. Sol. Only part (A) is correct.
Animosity means strong hostility. In the part (B) the preposition ‘for’ must be replaced by ‘of’
These both are the synonyms. Here these two words will fit in the whereas in the part (C) the preposition ‘of’ must be replaced by
sentence as the setback in India and Pakistan relations can only ‘for’. In the part (D), ‘have’ must be replaced with ‘has’ since this
be due to the bitterness or acrimony in any proposal. verb takes the form of new knowledge and new ways.

Facebook Page- https://www.facebook.com/vishalpariharpage Youtube- https://www.youtube.com/vishalparihar


Follow
56 Vishal Sir Telegram Channel- https://t.me/englishbyvishalsirchannel Instagram- https://www.instagram.com/vishalthetrainer
By Vishal sir
Also Important For SBI CLERK, IBPS CLERK, RRB CLERK
& Other Competitive Exams

Q268. The Partition and the (A) /bloodletting who accompanied (B) /our (d)Saving
Independence take up(C)/ many of the(D) /energy of our (e)Riding
founders.(E)
(a)The partition and the Q273. That old car is a dead ringer for the one we used to own.
(b)bloodletting who accompanied (a)Varied
(c)our Independence take up (b)Mismatched
(d)many of the (c)Fabricated
(e)All are incorrect (d)Mastered
(e)Duplicate
Q269. Giving unresponsive political parties (A)/, citizens can access (B)
/centres for power(C) /and privilege by (D)/through a vibrant civil Q274. I'd like to do dry run with this speech before I present it to the
society.(E) whole company.
(a)Giving unresponsive political parties (a) Rehearsal
(b)citizens can access (b) Operation
(c)centres for power (c) Interpret
(d)and privilege by (d) Fulfill
(e)All are incorrect (e) Complete

Directions (270-275): In each of the question given below a/an Q275. I did not appreciate the clerk's tongue-in-cheek comment about
idiom/phrase is given in bold which is then followed by five annoying customers.
options which try to decipher its meaning as used in the (a)Anger
sentence. Choose the option which gives the meaning of the (b)Temper
phrase most appropriately in context of the given sentence. (c)Humor
(d)Opposition
Q270. The police were on the criminal's trail but in the end he managed (e)Pampering
to give them the slip.
(a)Escape (b)Postpone Q276. Which of the following words has a meaning CLOSER to the
(c)Trial (d)Caught opposite to the word ‘INTRUIGING’?
(e)Inspect (a)Interesting
(b) Fascinating
Q271. They go to the beach when they should be hitting the books and (c) Attracting
then they wonder why they get low marks. (d) Boring
(e) Tempting
(a)Scrutinizing (b)Studying
(c)Reflecting (d)Chew over
Q277. Which of the following words has a meaning CLOSER to the
(e)Exploring word ‘CELESTIAL’?
(a) Propitiate
Q272. You should hold your horses before buying this car. (b) Astronomical
(a)Invest (c) Pacify
(b)Driving (d) Constitute
(c)Patient (e) Mollify

Facebook Page- https://www.facebook.com/vishalpariharpage Youtube- https://www.youtube.com/vishalparihar


Follow
57 Vishal Sir Telegram Channel- https://t.me/englishbyvishalsirchannel Instagram- https://www.instagram.com/vishalthetrainer
By Vishal sir
Also Important For SBI CLERK, IBPS CLERK, RRB CLERK
& Other Competitive Exams

ANSWER WITH SOLUTION S273. Ans.(e)


Sol. “Dead ringer” is an idiom which means duplicate or an exact
S268. Ans.(a) copy; a person or thing that closely resembles another. It means
Sol. Except the part (a), all other options are erroneous. "an exact duplicate" and derives from 19th-century horse-racing
Since the partition and bloodletting are uncountable nouns it will slang for a horse presented "under a false name and pedigree";
take a conjunction ‘that’ and not ‘who’ in part (B). "ringer" was a late nineteenth-century term for a duplicate,
Furthermore in the part (C), our Independence took up. This is so usually with implications of dishonesty, and "dead" in this case
because these events have already happened in the past. means "precise", as in "dead center".
In the part (D), ‘many of the must’ be replaced by ‘much of the’.
S274. Ans. (a)
S269. Ans.(b) Sol. “Dry run” is an idiom which means a rehearsal of a
Sol. Option (b) is the only choice which is the correct option. performance or procedure before the real one. Used in terms of
Rest all of the options are incorrect. performances as well as a corporate jargon which means to give
The part (A) must have given unresponsive political parties. something a trial before it is actually launched. Rest all the
The part (C) must be ‘centres of power’. options fail to satisfy the given idiom.
The part (D) must be ‘and privilege’.
S275. Ans.(c)
Sol. “Tongue-in-cheek” is an idiom which means Jocular or
S270. Ans.(a) humorous, though seeming or appearing to be serious. Thus the
Sol. The most appropriate phrase/word that deciphers the sentence means I did not appreciate the clerk’s humor comment
meaning of the highlighted idiom/phrase is “escape” as “to give about annoying customers.
them a slip” means to “escape”.Thus the sentence means that the
police were on the criminal’s trail but in the end he managed to S276. Ans. (d)
Sol. Among the given options, the option (d) has a meaning which
escape from the police.All the other words do not convey the
is OPPOSITE to the word ‘intriguing’ and rest are the options
appropriate meaning. which have the same meaning as the given word, and Hence,
option (d) is the correct answer choice.
S271. Ans.(b)
Sol. “Hitting the books” means to study especially in time of tests S277. Ans. (b)
and exams. Here the sentence means implying the meaning of the Sol. Celestial- positioned in or relating to the sky, or outer space
idiom as “They go to the beach when they should be studying real as observed in astronomy.
hard and then they wonder why they don’t get good marks.” Propitiate- win or regain the favour of (a god, spirit, or person)
by doing something that pleases them.
Among the given options, the option (b) has a meaning which is
S272. Ans.(c) closer to the word ‘Celestial’ and hence, is the correct answer.
Sol. ’Hold your horses’ means be patient. The idiom can be used
with any person/subject—hold one's horses—so you could say,
"I've got to hold my horses," "they better hold their horses," "he
can't hold his horses," etc. It can be used when someone is asking
you, or putting pressure on you, to do something.

Facebook Page- https://www.facebook.com/vishalpariharpage Youtube- https://www.youtube.com/vishalparihar


Follow
58 Vishal Sir Telegram Channel- https://t.me/englishbyvishalsirchannel Instagram- https://www.instagram.com/vishalthetrainer
By Vishal sir
Also Important For SBI CLERK, IBPS CLERK, RRB CLERK
& Other Competitive Exams

Directions (278-279): Choose the word which is most nearly were cast out of the Hindu society for not following canonical
the SAME in meaning as the word given below. proceedings in pursuing their trade. Patuas are also known Patigar
or Chitrakar.
Q278. Abated An intricate tapestry of music and visual art is what makes Naya
(a)Irresolute (b)Subside more than just a village in West Bengal’s Paschim Midnapore
(c)Indecisive (d)Increase district. A quaint little village, Naya is home to around 250 patuas
(e)Intensify
or chitrakaars, a unique community of folk artistes who are
Q279. Conducive painters, lyricists, singers and performers all rolled into one.
(a)Counterfeit (b)Fictitious These traditional painter singers specialize in the ancient folk art
(c)Favorable (d)Spurious of pata chitra, a traditional and mythological heritage of West
(e)Illusory Bengal, divided into some different aspects like Durga Pat,
Chalchitra, Tribal Patachitra, Medinipur Patachitra, Kalighat
Directions (280): Choose the word which is most nearly the Patachitra and etc. The subject matter of Bengal Patachitra is
OPPOSITE in meaning as the word given below. mostly mythological, religious stories, folk lore and social. The
Kalighat Patachitra, the last tradition of Bengal Patachitra is
Q280. Normalcy developed by Jamini Roy. The artist of the Bengal Patachitra is
(a)Perplex (b)Confound called Patua, a type of narrative scroll painting. Over time,
(c)Baffle (d)Insanity however, interest in this art form faded out. To ensure that their
(e)Disconcert art form remained relevant in the contemporary world, the patuas
Direction (281-290): Read the following passage carefully and adapted their skills and themes to changing times. As a part of
answer the questions given below it. Some words are given in this effort, a group of innovative patuas established a patachitra
bold to help you locate them while answering some of the village at Naya. Slowly, their efforts to revive their artistic
questions. heritage started paying off.

Today, after a period of decline, the patachitra art is flourishing


The Patuas are an artisan community found in the state of West
again in the village, with village youngsters taking up the
Bengal, Bihar, Jharkhand and Odisha in India and parts of
traditional art form as a passion and profession. A pata is created
Bangladesh. Some Patuas are Hindus, while others are Muslims.
by painting on a canvas made by stitching together multiple
Hindu Patuas are active in the Kalighat and Kumartuli regions of
sheets of commercial poster paper. In earlier days, jute fibre
Calcutta, along with some other parts of West Bengal, where they
canvas was used. Plant-based colours and lamp black (a pigment
are reduced in number. It is believed that most Patuas are actually
made from soot) are mixed in coconut shells with the sap of the
converts from Hinduism to Islam. Today, they practice customs
bel tree (wood apple), which acts as a binder. After finishing, a
that are both Hindu and Islamic in nature. They may have also
thin cotton cloth is glued to the back of the painting to provide
been Buddhist at various points in time. Today, however, the
majority of them are impoverished Muslims who rely on longevity. Next, the completed scrolls are kept in the sun to dry.
patronage from mainly Hindus, but also increasingly from The patuas also paint wooden souvenirs, decorative hangings and
mud walls with striking natural colours.
tourists who buy their painted scrolls, as Frank J. Korom has
described and analyses in his book Village of Painters: Narrative
Although Chitrakars origin is difficult to be precisely determined,
Scrolls from West Bengal (2006. Santa Fe: Museum of New
historical and mythological memories coincide that their
Mexico Press). Scholars argue that the Patuas, originally Hindus,
existence is traceable to the 13th century. Different accounts

Facebook Page- https://www.facebook.com/vishalpariharpage Youtube- https://www.youtube.com/vishalparihar


Follow
59 Vishal Sir Telegram Channel- https://t.me/englishbyvishalsirchannel Instagram- https://www.instagram.com/vishalthetrainer
By Vishal sir
Also Important For SBI CLERK, IBPS CLERK, RRB CLERK
& Other Competitive Exams

explain their standing in the Indian caste system. The Patua are a (a) They rely on funding from mainly Hindu families.
unique community, in that their traditional occupation is the (b) By the funding provided by government.
painting and modelling of Hindu idols, yet many of them are (c) The funds from the tourists who buy their painted scrolls.
Muslims. Their name Patua is a corruption of the Bengali word (d) Both (a) and (c)
Pota, which means an engraver. They are also widely known as (e) None of these.
Chitrakar, which literally means a scroll painter. The Patuas paid
little attention to faith, while looking for patronage. Chitrakars Q283. Why has author called the ‘Naya’ village of West Bengal as more
themselves might have converted to Islam as a strategy to avoid than just a village in district?
the oppression by a hierarchy of subcastes created during the Sen (a) Naya is a unique community of folk artistes who are painters,
Dynasty. This was an extremely slow process with the Patuas, as lyricists, singers and performers all rolled into one.
seen by the fact that every Patua has two names, one Hindu and (b)Naya is a village with diversified culture and heritage
one Muslim. The Patua are found mainly in the districts of (c)The village is so well developed by the colonizers that it has
Murshidabad, In Bihar they are mainly found in the Magahi and the best sanitation system and transport system
Maithili speaking regions as well as in the adjoining regions of (d)Both (b) and (c)
Jharkhand. In West Bengal they are a Bengali speaking (e) None of these.
community, with little or no knowledge of Urdu. The community
is strictly endogamous, and prefers cross-cousin marriages. The Q284. What are the efforts being made by Patuas to ensure that their art
Patuas visit villages and go from house to house with their bags form remains relevant in the contemporary world?
of scrolls. They narrate stories while unrolling the scrolls; in (a)The Patuas are adapting their skills and themes according to
return of his services he is paid in cash or kind. The traditional changing times.
occupation of the Patua is scroll painting, image making and other (b) A group of innovative Patuas established a Patachitra village
decorative work. They paint pictures on course pieces of cloth, at Naya .
showing images of Hindu gods and goddesses. These paintings (c) Patuas are organizing different cultural activities so as to make
are referred to as patas. Like other Muslim artisan groups in India, people aware about their customs and heritage.
they have seen a decline in their traditional occupation. The (d)Both (a) and (b)
majority are now employed as daily wage labourers. (e) None of these.

Q281. According to the passage, what might be the reason that Hindu Q285. According to the passage, how is the Pata art nowadays different
Patuas, active in regions of Calcutta have reduced in number? from that of earlier days?
(a)The interest of the people coming from abroad in this art form (a)Earlier Plant-based colors were to be used as a binder now days
is fading out. the sap of the bel tree acts as a binder.
(b)Naya a home to around 250 patuas or chitrakaars, has been (b) Opposite to present scenario earlier after finishing, a thin
under attacked by the mob since last many years. cotton cloth was to be glued to the back of the painting to provide
(c)It is believed that most Patuas have actually been converted longevity.
from Hinduism to Islam. (c)In earlier days, jute fiber canvas was used for painting but now
(d) It is believed that most Patuas have actually been converted days a Pata is created by painting on a canvas made by stitching
from Hinduism to Christianity. together multiple sheets of commercial poster paper.
(e) None of these. (d)Both (b) and (c)
Q282 How is the majority of converted bankrupt Muslim surviving (e) None of these.
financially?

Facebook Page- https://www.facebook.com/vishalpariharpage Youtube- https://www.youtube.com/vishalparihar


Follow
60 Vishal Sir Telegram Channel- https://t.me/englishbyvishalsirchannel Instagram- https://www.instagram.com/vishalthetrainer
By Vishal sir
Also Important For SBI CLERK, IBPS CLERK, RRB CLERK
& Other Competitive Exams

ANSWER WITH SOLUTION


S282.Ans. (d)
S278. Ans. (b) Sol. The correct answer choice is option (d). Option (b) is totally
Sol. Option (b) is the most appropriate choice to be suited as the out of context as there is no mention of government funding so it
answer of the question. can be easily omitted. Option (a) and (c) can be traced from the
Abated- (of something unpleasant or severe) become less intense very first paragraph of the passage where it is given as “Today,
or widespread. however, the majority of them are impoverished Muslims who
Subside- become less intense, violent, or severe. rely on patronage from mainly Hindus, but also increasingly from
Irresolute means showing or feeling hesitancy; uncertain. tourists who buy their painted scrolls, as Frank J.Korom has
Indecisive means Not providing a clear and definite result. described and analyses in his book Village of Painters: Narrative
Scrolls from West Bengal (2006. Santa Fe: Museum of New
S279. Ans. (c) Mexico Press).”
Sol. Option (c) is the most viable choice.
S283.Ans. (a)
Conducive- making a certain situation or outcome likely or
Sol. The correct answer choice is option (a). Option (b) and (c)
possible.
Counterfeit means imitate fraudulently. are completely out of context as there is no mention of either
Fictitious means not real or true; imaginary or fabricated. cultural heritage or development of village so both the options
Spurious means (of a line of reasoning) apparently but not can be omitted easily. Option (a) can be traced from the 2nd
actually valid. paragraph of the passage where it is given as “An intricate
Illusory means based on illusion; not real. tapestry of music and visual art is what makes Naya more than
just a village in West Bengal’s Paschim Midnapore district. A
quaint little village, Naya is home to around 250 patuas or
chitrakaars, a unique community of folk artistes who are painters,
S280. Ans. (d) lyricists, singers and performers all rolled into one.”
Sol. Option (d) is the most appropriate choice. S284.Ans. (d)
Normalcy- the condition of being normal; the state of being usual, Sol. The correct answer choice is option (d). Option (c) is out of
typical, or expected.
context as there is no mention of cultural activities so option (c)
Perplex means completely baffling; very puzzling.
can be easily omitted. Option (a) and (b) can be traced from the
Confound means causing surprise or confusion in (someone),
especially by not according with their expectations. last lines of 2nd paragraph where it is given as “To ensure that
Baffle means impossible to understand; perplexing. their art form remained relevant in the contemporary world, the
Disconcert means causing one to feel unsettled. Patuas adapted their skills and themes to changing times. As a
part of this effort, a group of innovative Patuas established a
S281.Ans. (c) Patachitra village at Naya. Slowly, their efforts to revive their
Sol. The correct answer choice is option (c). Option (b) is totally artistic heritage started paying off.”
out of context as there is no mention of the attack by mob so it S285.Ans. (c)
can be easily omitted. Option (c) can be traced from the very first Sol. The correct answer choice is option (c). Option (a) and (b)
paragraph of the passage where it is given as “Hindu Patuas are are quoted incorrectly as per given in the passage. Option (c) can
active in the Kalighat and Kumartuli regions of Calcutta, along be traced from the second line of 3rd paragraph where it is given
with some other parts of West Bengal, where they are reduced in as “A pata is created by painting on a canvas made by stitching
number. It is believed that most Patuas are actually converts from together multiple sheets of commercial poster paper. In earlier
Hinduism to Islam.” days, jute fiber canvas was used.”

Facebook Page- https://www.facebook.com/vishalpariharpage Youtube- https://www.youtube.com/vishalparihar


Follow
61 Vishal Sir Telegram Channel- https://t.me/englishbyvishalsirchannel Instagram- https://www.instagram.com/vishalthetrainer
By Vishal sir
Also Important For SBI CLERK, IBPS CLERK, RRB CLERK
& Other Competitive Exams

(d)Beset
Q286. As per the passage what is the traditional occupation of the Patua (e)Straightforward
community?
(a)A unique community of folk artistes where most of them are Q290. Oppression
painters, lyricists and singers (a)Slump
(b)Though many of the people from the Patua community are (b)Latitude
Muslims, yet their traditional occupation is painting and (c)Devaluation
modeling of Hindu idols. (d)Surge
(c) These traditional painter singers are specialized in the ancient (e)Depreciation
folk art of Pata chitra, a traditional and mythological heritage of
West Bengal. Directions (291-295): Each question below has two blanks.
(d)Both (a) and (c) There are five pairs of words below the sentence. Each pair is
(e) None of these. lettered. Choose the pair of words which can be filled up in
the blanks in the sentence in the same order so as to complete
Directions (287-288): Choose the word which is most nearly the sentences meaningfully.
the SAME in meaning as the word printed in bold as used in
the passage. Q291. Before the_______, the library was typically where you would
find ________readers.
Q287. Impoverished (a)Evolution, hard
(a)Bankrupt (b)Sunset, Resource
(b)Prosperous (c)Transit, soothing
(c)Constancy (d)Internet, scholarly
(d)Affinity (e)Peace, commotion
(e)Dissipated
Q292. Neville, the British prime minister during WWII, tried to
Q288. Patronage ______Hitler and in doing so sent a clear_______: you can walk
(a)Prudent all over us.
(b)Excessive (a)Opaque, question
(c)Funding (b)Harmony, predictability
(d)Steep (c)Torture, confused
(e)Frugal (d)Revamp, Implicit
(e) Appease, message
Directions (289-290): Choose the word which is the
OPPOSITE in meaning as the word printed in bold as used Q293. The judge was not _________since he had been ______by the
in the passage. witness’s family.
(a)Suspense, tolerable
Q289. Intricate (b)Temperament, stimulus
(a)Shelter (c)Impartial, bribed
(b)Defend (d)Baffle, wishful
(c)Torment (e)Great, trivial

Facebook Page- https://www.facebook.com/vishalpariharpage Youtube- https://www.youtube.com/vishalparihar


Follow
62 Vishal Sir Telegram Channel- https://t.me/englishbyvishalsirchannel Instagram- https://www.instagram.com/vishalthetrainer
By Vishal sir
Also Important For SBI CLERK, IBPS CLERK, RRB CLERK
& Other Competitive Exams

Surge means increase suddenly and powerfully. Thus, it is the


ANSWER WITH SOLUTION antonym of the given word.
Devaluation means the reduction or underestimation of the worth
S286.Ans. (b) or importance of something.
Sol. The correct answer choice is option (b). Option (a) and (c) Slump means undergo a sudden severe or prolonged fall in price,
are out of context of the question. Option (b) can be traced from value, or amount.
the second line of 4th paragraph where it is given as “The Patuas
are a unique community, in that their traditional occupation is the S291.Ans. (d)
painting and modelling of Hindu idols, yet many of them are Sol. The most suitable set of words to fill the blanks to make the
Muslims.” sentence grammatically correct and contextually meaningful
S287.Ans. (a) is “Internet, scholarly”. Hence, option (d) is the most viable
Sol. Impoverished- make (a person or area) poor. answer choice.
Affinity means resemblance. Commotion- confusion
Constancy means consistency. Scholarly- having or showing great knowledge or learning.
Dissipated means dispersed. Soothing- having a gently calming effect.
Bankrupt- impoverished or depleted.
S288.Ans. (c) S292.Ans. (e)
Sol. Patronage- the support given by a patron Sol. The most suitable set of words to fill the blanks to make the
Steep means (of a price or demand) not reasonable; excessive in sentence grammatically correct and contextually meaningful
accordance to the context of the paragraph. is “Appease, message”. Hence, option (e) is the most viable
Excessive means the same as steep. answer choice.
Frugal means economical, sparing. Appease- pacify or placate (someone) by acceding to their
Prudent means acting with or showing care and thought for the demands.
future. Torture- inflict severe pain on.
Opaque- not able to be seen through; not transparent
S289.Ans. (e) Implicit- suggested though not directly expressed
Sol. Intricate- very complicated or detailed. S293.Ans. (c)
Beset means (of a problem or difficulty) trouble (someone or Sol. The most suitable set of words to fill the blanks to make the
something) persistently. sentence grammatically correct and contextually meaningful
Torment means a cause of severe suffering. is “Impartial, bribed”. Hence, option (c) is the most viable answer
Defend means resist an attack made on (someone or something); choice.
protect from harm or danger. Impartial- treating all rivals or disputants equally
Shelter means shield. Baffle- totally bewilder or perplex
S290.Ans. (d) Stimulus- a thing or event that evokes a specific functional
Sol. Oppression- prolonged cruel or unjust treatment or exercise reaction in an organ or tissue
of authority. Wreck- the destruction of a ship at sea; a shipwreck
Depreciation means a reduction in the value of an asset over time, Trivial- of little value or importance
due in particular to wear and tear.

Facebook Page- https://www.facebook.com/vishalpariharpage Youtube- https://www.youtube.com/vishalparihar


Follow
63 Vishal Sir Telegram Channel- https://t.me/englishbyvishalsirchannel Instagram- https://www.instagram.com/vishalthetrainer
By Vishal sir
Also Important For SBI CLERK, IBPS CLERK, RRB CLERK
& Other Competitive Exams

Q294. Russian oligarchs are famous for their________, living in fancy (a)Impossible
homes and dining in _________restaurants. (b)Misfortune
(a)Negligible, refuse (c)Forbidden
(b)Opulence, expensive (d)Boldness
(c)Rebel, revolt (e)Strong
(d)Sorrow, presume
(e)Prey, rest Q299. I like adventure movies but above all I love horror movies.
(a)Especially
Q295. The unpredictable and _______nature of déjà vu makes it very (b)Stubborn
difficult _________to study properly. (c)Biased
(a)Transient, phenomenon (d)Amend
(b)Impossible, undoubtedly (e)Efficient
(c) Impertinent, vaguely
(d)Acute, traumatism Q300. The children began to act up during the field trip.
(e)Liberal, hint (a)Humanity
(b)Implicit
Directions (296-300): In each of the question given below a/an (c)Exaggerate
idiom/phrase is given in bold which is then followed by five (d)Misbehave
options which then try to decipher its meaning as used in the (e)Unlicense
sentence. Choose the option which gives the correct meaning
of the phrase most appropriately in context of the given Directions (301-305): In the questions given below, some
sentence. sentences/phrases are given which have to be arranged in a
proper sequence. Select the option which best defines the
Q296. Used books are a dime a dozen and it is difficult to sell them. proper sequence and arranges the sentence in an appropriate
(a) Return way.
(b) Tolerate
(c) Diminish Q301. markets have been (A)/quite volatile (B)/Indian equity (C)/a lot
(d) Failed from the highs (D)/and even corrected (E)/
(e) Common (a)BCAED
(b)CABED
Q297. We had a falling-out during our holiday and we have not spoken (c)EDCAB
since. (d)DCAEB
(a) Insistent (e)CAEBD
(b) Skip
(c) Disagreement Q302. forest cover had better (A)/ with predominant (B)/ were cropland
(d) Rigid dominated (C)/ in general, districts (D)/ resilience than those
(e) Lenient districts (E)/
(a)DBAEC (b)BAECD
Q298. The singer received a tough break when he became sick (c)ABEDC (d)ECDAB
immediately before the music contest. (e)CBAED

Facebook Page- https://www.facebook.com/vishalpariharpage Youtube- https://www.youtube.com/vishalparihar


Follow
64 Vishal Sir Telegram Channel- https://t.me/englishbyvishalsirchannel Instagram- https://www.instagram.com/vishalthetrainer
By Vishal sir
Also Important For SBI CLERK, IBPS CLERK, RRB CLERK
& Other Competitive Exams

ANSWER WITH SOLUTION


S300. Ans. (d)
Sol. Option (d) is the correct answer choice. ‘Act up’means to
S294.Ans. (b)
Sol. The most suitable set of words to fill the blanks to make the misbehave. All the other words fail to express the meaning of the
sentence grammatically correct and contextually meaningful given idiom.
is “Opulence, expensive”. Hence, option (b) is the most viable Implicit means suggested though not directly expressed.
answer choice. Exaggerate means represent (something) as being larger, better,
Opulence- great wealth or luxuriousness or worse than it really is.
Prey- an animal that is hunted and killed by another for food.
Revolt-take violent action against an established government or S301. Ans. (b)
ruler; rebel Sol. The correct sequence is CABED as it arranges the sentence
in the best way as follows: “Indian equity markets have been quite
S295.Ans. (a) volatile and even corrected a lot from the highs.”
Sol. The most suitable set of words to fill the blanks to make the
sentence grammatically correct and contextually meaningful S302. Ans. (a)
is “Transient, phenomenon”. Hence, option (a) is the most viable Sol. The correct sequence is DBAEC as it arranges the sentence
answer choice. properly in the way as follows: “In general, districts with
Transient-lasting only for a short time; impermanent predominant forest cover had better resilience than those districts
Phenomenon-a fact or situation that is observed to exist or were cropland dominated.”
happen, especially one whose cause or explanation is in question.
S296. Ans. (e)
Sol. Option (e) is the correct answer choice. ‘Dime a dozen’
means common, easy to get and of little value. All the other words
fail to express the meaning of the given idiom.
S297. Ans. (c)
Sol. Option (c) is the correct answer choice. ‘A falling-out’
means a disagreement or quarrel with someone. All the other
words fail to express the meaning of the given idiom.
Insistent means insisting on or demanding something; not
allowing refusal.
Rigid means unable to bend or be forced out of shape; not
flexible.
Lenient means of a punishment or person in authority) more
merciful or tolerant than expected
S298. Ans. (b)
Sol. Option (b) is the correct answer choice. ‘A tough break’
means an unlucky event, a misfortune. All the other words fail to
express the meaning of the given idiom.
S299. Ans. (a)
Sol. Option (a) is the correct answer choice. ‘Above all’ means
mainly, especially. All the other words fail to express the
meaning of the given idiom.

Facebook Page- https://www.facebook.com/vishalpariharpage Youtube- https://www.youtube.com/vishalparihar


Follow
65 Vishal Sir Telegram Channel- https://t.me/englishbyvishalsirchannel Instagram- https://www.instagram.com/vishalthetrainer
By Vishal sir
Also Important For SBI CLERK, IBPS CLERK, RRB CLERK
& Other Competitive Exams

Q303. is making it available (A)/only bringing (B) /to anyone in the Directions (311-315): Read the following passage carefully
world (C)/compute power but (D) / Cloud is not (E)/ and answer the questions given below it. Certain words have
(a)EBCDA (b)EBDAC (c)EBACD been printed in bold to help you locate them, while answering
(d)ACDBE (e)ABCED some of the questions.

Q304. to significantly boost (A)/ the e-wallet industry(B)/amongst PPIs The Alaska pipeline starts at the frozen edge of the Arctic Ocean.
(C)/ The interoperability (D)/ is surely going (E) It stretches southward across the largest and northernmost state
(a)CABDE (b)EDCBA (c)BECAD in the United States, ending at a remote ice-free seaport village
(d)DCEAB (e)ADBCE nearly 800 miles from where it begins. It is massive in size and
extremely complicated to operate. The steel pipe crosses
Q305. is proposed (A)/the transaction (B)/ equity and debt (C)/ to be windswept plains and endless miles of delicate tundra that tops
financed (D)/ by a mix of (E) the frozen ground. It weaves through crooked canyons, climbs
(a)DCEAB (b)EABCD (c)BADEC sheer mountains, plunges over rocky crags, makes its way
(d)CDBAE (e)ABECD through thick forests, and passes over or under hundreds of rivers
and streams. The pipe is 4 feet in diameter, and up to 2 million
Directions (306-308): Choose the word which is most nearly barrels (or 84 million gallons) of crude oil can be pumped through
the SAME in meaning as the words given below. it daily. Resting on H-shaped steel racks called "bents", long
sections of the pipeline follow a zigzag course high above the
Q306. Rebuffed frozen earth. Other long sections drop out of sight beneath spongy
(a)Please (b)Delight (c)Rejected or rocky ground and return to the surface later on. The pattern of
(d)Obscure (e)Omen the pipeline's up-and-down route is determined by the often harsh
demands of the arctic and subarctic climate, the tortuous lay of
Q307. Ostensibly
the land, and the varied compositions of soil, rock, or permafrost
(a)Foster (b)Nurture (c)Inborn
(d)Apparently (e)Nourish (permanently frozen ground). A little more than half of the
pipeline is elevated above the ground. The remainder is buried
Q308. Niceties anywhere from 3 to 12 feet, depending largely upon the type of
(a)Middle (b)Precisions (c)Ecstasy terrain and the properties of the soil. One of the largest in the
(d)Clink (e)Felicity world, the pipeline cost approximately $8 billion and is by far the
biggest and most expensive construction project ever undertaken
Directions (309-310): Choose the word which is the by private industry. In fact, no single business could raise that
OPPOSITE in meaning as the words given below. much money, so 8 major oil companies formed a consortium in
order to share the costs. Each company controlled oil rights to
Q309. Concessions particular shares of land in the oil fields and paid into the pipeline-
(a)Terminate (b)End (c)Halt construction fund according to the size of its holdings. Today,
(d)Stop (e)Denial
despite enormous problems of climate, supply shortage,
Q310. Hamstrung equipment breakdowns, labor disagreements, treacherous terrain,
(a)Bitterness (b)Spite (c)Unkindness a certain amount of mismanagement, and even theft, the Alaska
(d)Help (e)Malevolence pipeline has been completed and is operating.

Facebook Page- https://www.facebook.com/vishalpariharpage Youtube- https://www.youtube.com/vishalparihar


Follow
66 Vishal Sir Telegram Channel- https://t.me/englishbyvishalsirchannel Instagram- https://www.instagram.com/vishalthetrainer
By Vishal sir
Also Important For SBI CLERK, IBPS CLERK, RRB CLERK
& Other Competitive Exams

ANSWER WITH SOLUTION


S309. Ans.(e)
Sol. The correct answer choice is option (e).
S303. Ans. (b) Concessions- the action of conceding or granting something.
Sol. The correct sequence of the statement is EBDAC as it Denial- the action of denying something.
arranges the sentence as follows: “Cloud is not only bringing
compute power but is making it available to anyone in the world.”
S304. Ans. (d) S310. Ans.(d)
Sol. Option (d) provides the correct sequence for the arrangement Sol. The correct answer choice is option (d).
of the sentence. The proper arranged sentence is as follows: “The Hamstrung- severely restrict the efficiency or effectiveness of.
interoperability amongst PPIs is surely going to significantly Spite- hostile behaviour; unfriendliness or opposition
boost the e-wallet industry.” Malevolence- the state or condition of being malevolent; hostility

S305. Ans. (c)


Sol. The correct sequence is BADEC as the sentence can be
arranged as follows: “The transaction is proposed to be financed
by a mix of equity and debt.”
S306. Ans.(c)
Sol. The correct answer choice is option (c).
Rebuffed- reject (someone or something) in an abrupt or
ungracious manner.
Obscure- not clearly expressed or easily understood
Omen- an event regarded as a portent of good or evil

S307. Ans.(d)
Sol. The correct answer choice is option (d).
Ostensibly- as appears or is stated to be true, though not
necessarily so; apparently.
Foster-encourage the development of (something, especially
something desirable)
Inborn- existing from birth
Apparently- as far as one knows or can see.

S308. Ans. (b)


Sol. Niceties- accuracy or precision
Ecstasy- an overwhelming feeling of great happiness or joyful
excitement
Clink- a sharp ringing sound, such as that made by striking metal
or glass
Felicity- a sharp ringing sound, such as that made by striking
metal or glass

Facebook Page- https://www.facebook.com/vishalpariharpage Youtube- https://www.youtube.com/vishalparihar


Follow
67 Vishal Sir Telegram Channel- https://t.me/englishbyvishalsirchannel Instagram- https://www.instagram.com/vishalthetrainer
By Vishal sir
Also Important For SBI CLERK, IBPS CLERK, RRB CLERK
& Other Competitive Exams

Q311. The Alaskan pipeline ends e. RUIN


a. north of Alaska
b. at a seaport village Q317. In the following question, out of the four alternatives, choose
c. after passing through canyons and rivers the word which is opposite in meaning to the given word.
d. at a tundra covered village VIRTUE
e. None of these a. WILES b. CURSE
c. VICE d. CUNNING
Q312. What is the capacity of the Alaskan pipeline? e. Rude
a. 2 million gallons of crude oil
b. 4 million barrels of crude oil Q318. Four words are given, out of which only one word is spelt
c. 84 million gallons of crude oil correctly. Choose the correctly spelt word.
d. 84 billion barrels of crude oil a. Tranquility b. Tranquillity
e. None of these c. Trankquility d. Trankwility
e. Tranquelity
Q313. What are "bents"?
a. Zigzag shape of pipeline Q319. In the following questions, one part of the sentence may have an
b. Pipeline's up and down route error. Find out which part of the sentence has an error. If the
c. The section of the pipeline that drops out of sight sentence is free from error, click the "No error" option.
d. The H-shaped steel racks Could she cite (A)/ any precedent in support (B)/ for her case?
e. None of these (C)/ No Error (D)
a. A
Q314. How was the fund for pipeline - construction generated? b. B
a. 8 major oil companies joined hands to share the cost c. C
b. 8 major oil companies borrowed $8 billion. d. D
c. A single private company raised $8 billion
d. Oil rights were sold to 8 major oil companies Q320. In the following questions, one part of the sentence may have an
e. None of these error. Find out which part of the sentence has an error. If the
sentence is free from error, click the "No error" option.
Q315. Which of the following were not problems faced while The General Manager of the industry has felt (A)/ that there is
constructing the pipeline? no use of (B)/ discussing about the problems with the laborers.
a. Supply shortages (C)/No Error (D)
b. Treacherous terrain a. A b. B
c. Lack of funds c. C d. D
d. Equipment breakdown
e. None of these Q321. In the following questions, one part of the sentence may have an
error. Find out which part of the sentence has an error. If the
Q316. In the following question, out of the four alternatives, choose sentence is free from error, click the "No error" option.
the word which best expresses the meaning of the given word. She enquired from the stranger (A)/ who was he and (B)/ what
INSOLENT he wanted from her. (C)/ No Error (D)
a. DISTASTEFUL a. A b. B
b. IMPATIENT c. C d. D
c. DIABOLIC
d. RUDE

Facebook Page- https://www.facebook.com/vishalpariharpage Youtube- https://www.youtube.com/vishalparihar


Follow
68 Vishal Sir Telegram Channel- https://t.me/englishbyvishalsirchannel Instagram- https://www.instagram.com/vishalthetrainer
By Vishal sir
Also Important For SBI CLERK, IBPS CLERK, RRB CLERK
& Other Competitive Exams

ANSWER WITH SOLUTION Explanation: ‘Virtue’ is the behavior showing high moral
standards. While ‘vice’ is immoral or wicked behavior. Hence,
Ans 311. at a seaport village option(c.) is the correct answer.
Explanation: The supporting lines for finding the answer to this
Ans 318. Tranquillity
question are given below-
Explanation: ‘Tranquility’ means the state of being calm. While
“The Alaska pipeline starts at the frozen edge of the Arctic Ocean.
others are meaningless.
It stretches southward across the largest and northernmost state
in the United States, ending at a remote ice-free seaport village
Ans 319. C
nearly 800 miles from where it begins.”
Explanation: The correct sentence out of the given fragments will
be-“Could she cite any precedent in support of her case?”
Ans 312. 84 million gallons of crude oil
Explanation: The supporting lines for finding the answer to this
Ans 320. C
question are given below-
Explanation: The correct sentence formation will be-“The
“The pipe is 4 feet in diameter, and up to 2 million barrels (or 84
General manager of the industry has felt that there is no use of
million gallons) of crude oil can be pumped through it daily.”
discussing problems with the laborers.”
Ans 313. The H-shaped steel racks
Explanation: The supporting lines for finding the answer to this Ans 321. B
Explanation: Find the correct sentence for the given fragments
question are given below-
below-
“Resting on H-shaped steel racks called "bents", long sections of
“She enquired from the stranger, who he was and what he wanted
the pipeline follow a zigzag course high above the frozen earth.”
from her.”
Ans 314. 8 major oil companies joined hands to share the cost
Explanation: Find the supporting lines below to answer the
question-
“In fact, no single business could raise that much money, so 8
major oil companies formed a consortium in order to share the
costs. Each company controlled oil rights to particular shares of
land in the oil fields and paid into the pipeline-construction fund
according to the size of its holdings.”

Ans 315. Lack of funds


Explanation: Find the supporting lines below to answer the
question-
“One of the largest in the world, the pipeline cost approximately
$8 billion and is by far the biggest and most expensive
construction project ever undertaken by private industry. In fact,
no single business could raise that much money, so 8 major oil
companies formed a consortium in order to share the costs.”
Ans 316. RUDE

Ans 317. VICE

Facebook Page- https://www.facebook.com/vishalpariharpage Youtube- https://www.youtube.com/vishalparihar


Follow
69 Vishal Sir Telegram Channel- https://t.me/englishbyvishalsirchannel Instagram- https://www.instagram.com/vishalthetrainer
By Vishal sir
Also Important For SBI CLERK, IBPS CLERK, RRB CLERK
& Other Competitive Exams

Q322. The sentences given with blanks are to be filled with an c. Betray somebody
appropriate word(s). Four alternatives are suggested for each d. Have a selfish interest
question. For each question, choose the correct alternative.
We acted on a __________ impulse. Q327 .In each of the questions, four alternatives are given for the
a. momentary Idiom/Phrase. Choose the alternative which best expresses the
b. momentous meaning of the Idiom/Phrase.
c. memorable Pick to pieces
d. meritorious a. Study something superficially
b. Complete a work entirely
Q323. The sentences given with blanks are to be filled with an c. Analyze critically
appropriate word(s). Four alternatives are suggested for each d. Select only what you need.
question. For each question, choose the correct alternative.
Do your best and we'll back you ______. Q328. Out of the four alternatives, choose the one which can be
a. on substituted for the given words/sentences.
b. in A person who lays too much stress on bookish-learning
c. out a. Pervert
d. up b. Pedant
c. Philosopher
Q324. The sentences given with blanks are to be filled with an d. Scholar
appropriate word(s). Four alternatives are suggested for each
question. For each question, choose the correct alternative. Q329. Out of the four alternatives, choose the one which can be
Sanjay __________his mother in the morning every day. substituted for the given words/sentences.
a. calls in Postponement or delay permitted in the suffering of a penalty or
b. calls up the discharge of an obligation.
c. calls off a. Respite
d. calls down b. Spire
c. Splurge
Q325 .In each of the questions, four alternatives are given for the d. Scourge
Idiom/Phrase. Choose the alternative which best expresses the
meaning of the Idiom/Phrase. Q330. Out of the four alternatives, choose the one which can be
Put two and two together substituted for the given words/sentences.
a. Bad at mathematics Deviation from the right course
b. Poor financial condition a. Imagination b. Amalgamation
c. Reason logically c. Illumination d. Aberration
d. Forget something
Q331. A sentence/a part of the sentence is underlined. Four alternatives
Q326 .In each of the questions, four alternatives are given for the are given to the underlined part which will improve the
Idiom/Phrase. Choose the alternative which best expresses the sentence. Choose the correct alternative. In case no
meaning of the Idiom/Phrase. improvement is needed, choose "No improvement".
An axe to grind He likes to drive his car at a speed of eighty kilometers each
a. Attack aggressively hour.
b. Suffer a lot a. every hour b. an hour
c. hourly d. No improvement

Facebook Page- https://www.facebook.com/vishalpariharpage Youtube- https://www.youtube.com/vishalparihar


Follow
70 Vishal Sir Telegram Channel- https://t.me/englishbyvishalsirchannel Instagram- https://www.instagram.com/vishalthetrainer
By Vishal sir
Also Important For SBI CLERK, IBPS CLERK, RRB CLERK
& Other Competitive Exams

ANSWER WITH SOLUTION Ans 331. an hour


Explanation: The sentence after correction will be-“He likes to
drive his car at a speed of eighty kilometers an hour.”
Ans 322. momentary
Explanation: The best-suited word for the given sentence will be
‘momentary’ because the impulse is always momentary.

Ans 323. up
Explanation: ‘Back up’ means ‘support’. Hence, the correct
answer will be-“Do your best and we’ll back you up.”

Ans 324. calls up


Explanation: The correct sentence with the appropriate word will
be-“Sanjay calls up his mother in the morning every day.”

Ans 325. Reason logically


Explanation: The idiom ‘Put two and two together’ is used for
guessing the truth about a situation from you have seen or heard.
Hence, option(c.) is the correct answer.

Ans 326. Have a selfish interest


Explanation: ‘An axe to grind’ is used to representing a selfish or
ulterior aim. Hence, option(d.) is the most suitable answer.

Ans 327. Analyze critically


Explanation: The ‘Pick to pieces’ idiom means ‘to make an effort
to find flaws or negative aspects’. Hence, option(c.) is the most
suitable answer.

Ans 328. Pedant


Explanation: A Pedant is a person, who is excessively concerned
with minor details and rules or with displaying academic
learning.

Ans 329. Respite


Explanation: Respite is a short period of rest or relief from
something difficult or unpleasant.

Ans 330. Aberration


Explanation: Aberration is a departure from what is normal,
usual, or expected.

Facebook Page- https://www.facebook.com/vishalpariharpage Youtube- https://www.youtube.com/vishalparihar


Follow
71 Vishal Sir Telegram Channel- https://t.me/englishbyvishalsirchannel Instagram- https://www.instagram.com/vishalthetrainer
By Vishal sir
Also Important For SBI CLERK, IBPS CLERK, RRB CLERK
& Other Competitive Exams

Q332. A sentence/a part of the sentence is underlined. Four alternatives Below in each question some phrases are given, find the
are given to the underlined part, which will improve the phrase which is not really contributing to the main theme of
sentence. Choose the correct alternative. In case no the sentence or find the odd phrase out and rearrange the
improvement is needed, choose "No improvement". remaining phrases to make a coherent sentence. If the given
His argument against his opponent duly brought jeers from the phrases are correct in order as it is then choose option (E). If
crowd. the correct sequence is the one which is not given then
a. invective choose option (D) as your choice.
b. praise
c. controversy
d. No improvement Q336 .that which is tangible and which is bound to perish (A)/ we
tread a fine line (B)/ and devotion, never fixating on (C)/ the
Q333. A sentence/a part of the sentence is underlined. Four alternatives seeker of truth (D)/ treads on the path of love(E)
are given to the underlined part which will improve the 1. EDCA
sentence. Choose the correct alternative. In case no 2. AEBC
improvement is needed, choose "No improvement". 3. DECA
4. None of these
Do you know the time when the train departs? 5. No correction required
a. which
b. by Q.337 the stimuli to create outside (A)/ we are not ready to accept
c. that impermanence as a reality (B)/ the witness and the ego is (C)/
d. No improvement
because we have still not become (D)/ still running the show.
Q334. A sentence/a part of the sentence is underlined. Four alternatives
are given to the underlined part, which will improve the (E)
sentence. Choose the correct alternative. In case no 1. BDCE
improvement is needed, choose "No improvement". 2. EDAC
I'm staying with some friends who are owning a farm. 3. CEDB
a. will be owning 4. None of these
b. own 5. No correction required
c. have been owning
d. No improvement Q.338 to drugs or alcohol, but in attempting to feel good, the side
effects (A)/ to calm themselves, many turn (B)/ and joy that
Q335. A sentence/a part of the sentence is underlined. Four alternatives washes away our anxiety (C)/ impact their health and wellness
are given to the underlined part which will improve the (D)/ and the lives of those around them. (E)
sentence. Choose the correct alternative. In case no
1. EACB
improvement is needed, choose "No improvement".
2. DEAB
An old friend, may I give you an advice? 3. BADE
a. give you some advice 4. None of these
b. give you an advise 5. No correction required
c. offer you an advice
d. No improvement

Facebook Page- https://www.facebook.com/vishalpariharpage Youtube- https://www.youtube.com/vishalparihar


Follow
72 Vishal Sir Telegram Channel- https://t.me/englishbyvishalsirchannel Instagram- https://www.instagram.com/vishalthetrainer
By Vishal sir
Also Important For SBI CLERK, IBPS CLERK, RRB CLERK
& Other Competitive Exams

ANSWER WITH SOLUTION Clause (B) will be the first part of the sentence.
Hence option (c) is the correct choice.
Ans 332. invective
Explanation: The sentence after correction will be-“His invective
against his opponent duly brought jeers from the crowd.”

Ans 333. No improvement


Explanation: The sentence complies with all grammar rules.
Hence, it needs no improvement.

Ans 334. own


Explanation: ‘own’ word cannot be used as verb. Hence, the
sentence after correction will be-“I’m staying with some friends
who own a farm.”

Ans 335. Give you some advice


Explanation: The correct sentence will be-“An old friend, May I
give you some advice.”

Answer 336: 3
Solution: The sequence of the phrases after rearrangement is
DECA. We can easily point out that clauses (E) and (C) are
interlinked with each other as ‘love’ and ‘devotion’ is
contributing the similar sense. Clause (B) fails to get linked with
the other clauses of the sentence. Hence option (c) is the correct
and appropriate choice.

Answer337: 1
Solution: The correct order of the phrases is BDCE making the
sentence meaningful. Phrase (A) does not provide any meaning
with respect to the other phrases and hence is irrelevant here.
Phrases (B) and (D) can easily be identified as interlinked as ‘to+
accept’ will act like verb which must be followed by a noun or
pronoun (impermanence). Hence option (a) is the correct choice
to be made.

Answer338: 3
Solution: Clauses BADE form a coherent sentence after
rearrangement and clause (C) bears no resemblance here as the
sentence (phrases B, D and E) is referring to the third person
(themselves, those) whereas clause (c) is referring to ‘our’.

Facebook Page- https://www.facebook.com/vishalpariharpage Youtube- https://www.youtube.com/vishalparihar


Follow
73 Vishal Sir Telegram Channel- https://t.me/englishbyvishalsirchannel Instagram- https://www.instagram.com/vishalthetrainer
By Vishal sir
Also Important For SBI CLERK, IBPS CLERK, RRB CLERK
& Other Competitive Exams

Q.339 sense of self, it cannot be (A)/ of existence of the fragmented offered them as a special gesture. India’s requests for advanced
(B)/ since ego is the nucleus (C)/ annihilated from bodymind technologies routinely get entangled in the foundational question
complex (D)/ it may work like an aid. (E) — has any other country that is not a military ally been given this
1. CBAD 2. BCED 3. DEAB particular technology? Whenever the answer is ‘no’, its request
4. None of these 5. No correction required could be denied. For instance, a new plane for the travel of India’s
Prime Minister, being negotiated between India and American
Q.340 consciousness that we experienced before deity was only an manufacturer Boeing, will come without a lot of advanced
excuse (A)/ the change that occurred in our (B)/consciousness or communication equipment unless both countries manage to
the altered state of (C)/ we have to acknowledge that (D)/ the conclude a treaty that governs its use. Hence one can argue that
ego loses its limitations. (E) there are benefits of signing up as a military ally of the U.S.
1. DBCA 2. DEBA 3. BDEC The counter to this argument is that given the drastic changes in
4. None of these 5. No correction required U.S. position across several crucial issues, India might have done
Directions (341-350): Read the following passage carefully well by never aligning with it as an ally. The U.S. had been
and answer the questions given below it. Certain words have pushing India to open its markets more to global trade, but has
been printed in bold to help you locate them, while answering now suddenly turned against the same, under Mr. Trump. The
some of the questions. U.S. under Barack Obama put tremendous pressure on India to
While Indian foreign policy has evolved over the decades, what ratify the Paris Agreement. But his successor has announced a
has not changed is the concept of strategic autonomy, which is withdrawal from the pact and ordered an end to all measures for
that India would not join any military alliance, would always keep its implementation.
its choices open and would choose what is good for it depending Even before Mr. Trump, if one considers the last two decades of
on the situation at a particular moment. Some commentators have accelerated engagement between the two countries, the U.S. has
ridiculed strategic autonomy as a rigid ideological position that made abrupt U-turns on many foreign policy issues, much to
has prevented India from achieving more in the international India’s discomfort. It has alternated between trying to befriend
arena. Some have said that India should have become an ally of and confront China — something that continues under Mr.
the U.S. several decades ago, and by not doing so, it had limited Trump; it has sought to ignore Pakistan, punish it and then woo it
its potential. Strategic autonomy has recently been at the with money and weapons; it has tried to contain Iran and then
forefront of discussions largely due to India’s ever-tightening open up to it and, now, contain it again; and it has given
embrace with the U.S. As the two countries inch closer to one conflicting signals on Afghanistan. President Obama wanted
another, will India be able to maintain its autonomy of choice and India to take a tougher stand against the military junta in
independence? Undoubtedly, the U.S. is crucial to India’s Myanmar, and then went ahead for a rapprochement with them
progress as a key source of technology and capital and as the himself. Strategic autonomy has allowed India to have its own
foremost destination for its students and jobseekers in various policies towards these countries to a great extent, in the midst of
sectors. Many advocates of continuing expansion of India-U.S. the flux that the U.S. often contributes to Mr. Modi and his key
ties say that strategic autonomy is useless and counterproductive. adviser in the initial years, S. Jaishankar, did not use the phrase
The U.S. shares its most advanced technologies and intelligence strategic autonomy in the beginning. But in his speech at the
only with its closest allies. The NATO allies and Israel are Shangri-La Dialogue in June 2018, Mr. Modi said: “It is a
topmost in this pecking order. For instance, only they have been measure of our strategic autonomy that India’s Strategic
given F-35 fighter planes, the most advanced of America’s fighter Partnership, with Russia, has matured to be special and
planes yet. The Guardian-series Unmanned Aerial Vehicles have privileged.” The speech itself was an elucidation of the age-old
been sold only to NATO allies till date, and now India has been policy of India’s strategic autonomy.

Facebook Page- https://www.facebook.com/vishalpariharpage Youtube- https://www.youtube.com/vishalparihar


Follow
74 Vishal Sir Telegram Channel- https://t.me/englishbyvishalsirchannel Instagram- https://www.instagram.com/vishalthetrainer
By Vishal sir
Also Important For SBI CLERK, IBPS CLERK, RRB CLERK
& Other Competitive Exams

ANSWER WITH SOLUTION


Answer339: 1
Solution: The correct sequence of the phrases formed is CBAD.
We can get the hint from phrase (C) that is forming the first part
of the sentence as it is starting from ‘since’ for telling the reason
whereas phrase (E) is irrelevant here which is talking about the
some ‘aid’ which is not mentioned in the other parts. Hence
option (a) is correct.

Answer340: 1
Solution: Phrases DBCA form a coherent sentence making the
sentence meaningful. Phrase (D) will form the first part of the
sentence as started with pronoun (we). Moreover, the word ‘ego’
finds no relevance in the sentence. Hence phrase (E) will not be
a part of the sentence formed. Therefore option (a) is the correct
choice here.

Facebook Page- https://www.facebook.com/vishalpariharpage Youtube- https://www.youtube.com/vishalparihar


Follow
75 Vishal Sir Telegram Channel- https://t.me/englishbyvishalsirchannel Instagram- https://www.instagram.com/vishalthetrainer
By Vishal sir
Also Important For SBI CLERK, IBPS CLERK, RRB CLERK
& Other Competitive Exams

Q341 .In what way has strategic autonomy been ridiculed by some of (d) As U.S. has made abrupt U-turns on many foreign policy
the commentators? issues much to India’s comfort.
(a) They say that it is continuing expansion for India and is over- (e) All of these.
productive.
(b)They commented that due to it India has been offered a special Q345. Which of the following are the countries mentioned in the
gesture. passage through which U.S. has caused discomfort to India in
(c)They said that it prevented India from achieving more in the terms of foreign policy issues?
international arena. (a) Afghanistan, China, Japan and Myanmar
(d)They commented that due to it drastic changes can be seen in (b) Myanmar, Japan, Iran and Afghanistan
U.S.’s position. (c) Japan, Pakistan, Iran and Afghanistan
(e) None of these. (d) China, Pakistan, Iran and Myanmar
(e) None of these.
Q342. Which option best explains the essence of strategic autonomy of
India as described in the passage? Directions (346-348): Choose the word which is most nearly
(a)India would not join any military alliance. the SAME in meaning as the word printed in bold as used in
(b)It would be open for choices and will choose what is good for the passage.
it based on the situation.
(c)It is in the state or condition of being ruled, governed, or under Q346. Ridiculed
the sway of another, as in a military occupation. (a)Exalted
(d)Both (a) and (b) (b)Respected
(e)Both (b) and (c) (c)Mocked
(d)Approved
Q343. What benefit could India avail by signing up as a closest military (e)Praised
ally with U.S.?
(a) India’s market would never get open to global trade. Q347. Forefront
(b) As U.S shares its most advanced technologies and intelligence (a)Aback
only with its closest allies. (b)Lead
(c) India would get a tougher stand against military attacks from (c)Backdrop
South America. (d)Basis
(d) India would get formal consent of Paris agreement. (e)Ambit
(e) All of these.
Q348. Embrace
Q344. Why is it questionable whether India will be able to maintain its (a)Eschew
strategic autonomy in future? (b)Alienate
(a) As U.S. has turned against pushing India to open its markets (c)Exclude
to global trades. (d)Comprehend
(b)As India’s Prime Minister recently visited UK for the second (e)Renounce
time as an official Guest of Government.
(c) Because of India’s ever tightening embrace with the U.S.

Facebook Page- https://www.facebook.com/vishalpariharpage Youtube- https://www.youtube.com/vishalparihar


Follow
76 Vishal Sir Telegram Channel- https://t.me/englishbyvishalsirchannel Instagram- https://www.instagram.com/vishalthetrainer
By Vishal sir
Also Important For SBI CLERK, IBPS CLERK, RRB CLERK
& Other Competitive Exams

ANSWER WITH SOLUTION S345. Ans.(d)


Sol. Option (d) is the most appropriate choice to be suited as the
S341. Ans.(c) answer. The answer can be deduced from the last paragraph of
Sol. The answer can be deduced from the first paragraph of the the passage where it is illustrated in detail that “Even before Mr.
passage where it is specifically mentioned that some Trump, if one considers the last two decades of accelerated
commentators have derided strategic autonomy as a rigid engagement between the two countries, the U.S. has made abrupt
ideological position that has prevented India from achieving U-turns on many foreign policy issues, much to India’s
more in the international arena. discomfort. It has alternated between trying to befriend and
confront China — something that continues under Mr. Trump; it
S342. Ans.(d) has sought to ignore Pakistan, punish it and then woo it with
Sol. Option (d) is the most appropriate choice to be suited as the money and weapons; it has tried to contain Iran and then open
answer as it consists of both the options (a) and (b).The answer up to it and, now, contain it again; and it has given conflicting
can be deduced from the first line of the first paragraph itself signals on Afghanistan. President Obama wanted India to take a
where it is given that despite the evolving foreign policy over tougher stand against the military junta in Myanmar, and then
decades, India hasn’t changed its concept of strategic autonomy went ahead for a rapprochement with them himself.”
of not joining any military alliance and keeping its choices open
and choosing what is good for it depending on the situation in the S346. Ans. (c)
particular moment. Sol. Option (c) is the most appropriate choice.
Ridiculed means subject to contemptuous and dismissive
S343. Ans.(b) language or behavior.
Mocked means tease or laugh at in a scornful or contemptuous manner.
Sol. Option (b) is the most suitable answer. The answer can be
inferred from the second paragraph of the passage where it is Exalted means to raise in rank, honor, power, character, quality.
given that “The U.S. shares its most advanced technologies and
intelligence only with its closest allies. The NATO allies and S347. Ans.(b)
Israel are topmost in this pecking order.” Sol. Option (b) is the most suitable choice.
In the same paragraph, the last sentence is given which provides Forefront means the leading or most important position or place.
a conclusion and hence an ending to the paragraph that is that one Lead means first position or primacy.
can argue that there are benefits of signing up as a military ally of Aback means towards or situated to the rear; back
the U.S. Backdrop means lie behind or beyond; serve as a background to.
Ambit means the scope, extent, or bounds of something.
S344. Ans. (c)
Sol. Option (c) is the most appropriate choice. The answer can be S348. Ans. (d)
inferred from the first paragraph of the passage where it is Sol. Option (d) is the most apt choice.
mentioned that strategic autonomy has recently been at the Embrace means include, comprise or encompass in terms of
forefront of discussions largely due to India’s ever-tightening context.
embrace with the U.S. Furthermore the author asks a question that Hence, comprehend is the suitable word to be used here.
as the two countries inch closer to one another, will India be able Eschew means abstain from.
to maintain its autonomy of choice and independence. Alienate means estranged.
Renounce means formally declare one's abandonment of (a claim,
right, or possession).

Facebook Page- https://www.facebook.com/vishalpariharpage Youtube- https://www.youtube.com/vishalparihar


Follow
77 Vishal Sir Telegram Channel- https://t.me/englishbyvishalsirchannel Instagram- https://www.instagram.com/vishalthetrainer
By Vishal sir
Also Important For SBI CLERK, IBPS CLERK, RRB CLERK
& Other Competitive Exams

Directions (349-350): Choose the word which is the OPPOSITE in (e)ABCED


meaning as the word printed in bold as used in the passage.
Q353 .it is the (A) /combination of wit (B)/ analysis that makes (C) / his
articles so readable (D)/and political (E)/
Q349. Entangled
(a)DCEAB
(a)Complicated
(b)EABCD
(b)Free
(c)BADEC
(c)Perplexed
(d)CDBAE
(d)Confused
(e)ABECD
(e)Concluded
Q354. fabric is sufficient (A)/ to cover the (B)/one meter of (C)/ exterior
Q350. Ratify
of an (D)/ 18-in-diameter hatbox (E)/
(a)Approve
(a)CABDE
(b)Endorse
(b)EDCBA
(c)Sanction
(c)BECAD
(d)Veto
(d)DCEAB
(e)Confirm
(e)ADBCE
Directions (351-355): In the question given below, some
Q355. was predicated (A)/ on the assumption (B)/ that the economy (C)/
sentences/phrases are given which have to be arranged in a
this project (D)/was expanding (E)
proper sequence. Select the option which best defines the
(a)ABECD
proper sequence and arranges the sentence in an appropriate
(b)CDEAB
way.
(c)BAEDC
(d)DABCE
(e)ECBDA
Q351. is responsible for(A)/ low- intensity rainfall (B)/during the
monsoon(C) / in northwest and north-central India
(D)/groundwater recharge(E)/ Directions (356-360): Each question below has one blank,
(a)BCAED which is indicating that something has been omitted. Find out
(b)CABED which option can be used to fill up the blank in the sentence
(c)EDCAB in the same sequence to make it meaningfully complete.
(d)DCAEB
(e)CAEBD
Q356. Mass violence __________________to unsettle the very
foundations of government.
Q352. food hamburger (A) / widely regarded (B)/ restaurant chain(C) /
(a)Inserted
as the quality fast (D) /burger King is (E)/
(b)Braved
(a)EBCDA
(c)Befriended
(b)EBDAC
(d)Threatened
(c)EBACD
(e)Essential
(d)ACDBE

Facebook Page- https://www.facebook.com/vishalpariharpage Youtube- https://www.youtube.com/vishalparihar


Follow
78 Vishal Sir Telegram Channel- https://t.me/englishbyvishalsirchannel Instagram- https://www.instagram.com/vishalthetrainer
By Vishal sir
Also Important For SBI CLERK, IBPS CLERK, RRB CLERK
& Other Competitive Exams

ANSWER WITH SOLUTION Sol. The appropriate option here is option is (d).The arranged
sentence will be “This project was predicated on the assumption
S349. Ans. (b) that the economy was expanding.”
Sol. Entangled means involve (someone) in difficulties or
complicated circumstances from which it is difficult to escape. S356. Ans. (d)
Complicated means complex. Sol. Option (d) is the most suitable choice to be filled in the blank.
Perplexed means make (someone) feel completely baffled. As contextually only threatened word fits in the blank.
Threatened means endangered.
S350. Ans. (d) Befriended means act as or become a friend to (someone),
Sol. Ratify means sign or give formal consent to (a treaty, especially when they are in need of help or support.
contract, or agreement), making it officially valid.
Veto means rejection.
Endorse means declare one's public approval or support of.
Sanction means a threatened penalty for disobeying a law or rule.
Hence option (d) is the most appropriate choice.

S351. Ans. (a)


Sol. The appropriate option is (a).The corrected sentence is
“Low-intensity rainfall during the monsoon is responsible for
groundwater recharge in northwest and north-central India.”

S352. Ans. (b)


Sol. The correct option is (b).The proper arranged sentence is
“Burger King is widely regarded as the quality fast food
hamburger restaurant chain.”

S353. Ans. (e)


Sol. The most correct option is option (e).The arranged sentence
is “It is the combination of wit and political analysis that makes
his articles so readable.”

S354. Ans.(a)
Sol. The appropriate choice here is option (a).The proper
arranged sentence is “One meter of fabric is sufficient to cover
the exterior of an 18-in-diameter hatbox.”

S355. Ans. (d)

Facebook Page- https://www.facebook.com/vishalpariharpage Youtube- https://www.youtube.com/vishalparihar


Follow
79 Vishal Sir Telegram Channel- https://t.me/englishbyvishalsirchannel Instagram- https://www.instagram.com/vishalthetrainer
By Vishal sir
Also Important For SBI CLERK, IBPS CLERK, RRB CLERK
& Other Competitive Exams

Q357. The minister said that such statements would______________ Q361. His mother asked him when will he have his lunch.
lower the morale of our soldiers. (a)would he have
(a)Curiously (b)would he has
(b)Crookedly (c)he would have
(c)Definitely (d)might he have
(d)Unclearly (e)No correction required
(e)Defectively
Q362. I would not have gone alone if you were told me of the danger.
Q358. The witness ______________ the suspect as short, dark-skinned (a)had been told me of
and bearded (b)would have told me of
(a)Committed (c)had told me with
(b)Described (d)had told me of
(c)Praised (e)No correction required
(d)Disappointed
(e)Pacified Q363 .If a person studied a period of history; he would have wondered
how things had happened at that time.
Q359. He remained strongly_______________ that an agreement could (a)a person had studied
be reached. (b)a person would study
(a)Inactive (c)a person could have studied
(b)Complicated (d)a person has studied
(c)Generous (e)No correction required
(d)Optimistic
(e)Playful Q364. You can learn the principles of government from a reading of
history.
(a)a view of history
Q360. __________________ there were many inconveniences for the (b)a study of history
Amarnath pilgrims. (c)a glimpses of history
(a)Cowardly (d)lookings of history
(b)Unimportantly (e)No correction required
(c)Unfortunately
(d)Scholarly Q365. Seeing that she is very tired, I walked out of the room and let
(e)Heavily her sleep.
(a)Seeing that she are very tired
Directions (361–365): Which of the phrases (a), (b), (c) and (b)Seeing that she was very tired
(d) given below should replace the phrase given in bold in the (c)Seeing she is tired
following sentences to make the sentence grammatically (d)Seeing her to be tired
correct? If the sentence is correct as it is and there is no (e)No correction required.
correction required mark (e) i.e. 'No correction required' as
the answer.

Facebook Page- https://www.facebook.com/vishalpariharpage Youtube- https://www.youtube.com/vishalparihar


Follow
80 Vishal Sir Telegram Channel- https://t.me/englishbyvishalsirchannel Instagram- https://www.instagram.com/vishalthetrainer
By Vishal sir
Also Important For SBI CLERK, IBPS CLERK, RRB CLERK
& Other Competitive Exams

ANSWER WITH SOLUTION Sol. The most suitable option here is option (c).’He would have’
will be used because in indirect narration when reporting verb is
S357. Ans.(c) in past tense , reported speech is also in past tense and in
Sol. Option (c) is the most suitable choice to be filled in the blank. subordinate clause we put words in order of Subject +Verb. For
The sentence would be “The minister said that such statements example,
She asked me why I was late.
would definitely lower the morale of our soldiers.”
Curiously means in a strange and unusual way.
Crookedly means in a way that does not form a straight line or S362. Ans. (d)
that has many bends. Sol. Option (d) is the most appropriate choice as the replacement
Defectively means characterized by subnormal intelligence or of the bold part. ’Had told me of’ will be used as in order to
behavior. express the unfulfilled condition or desire we use
If+Subject+had+V3 or Had+Subject+V3,
S358. Ans. (b) Subject+Would/Could/Might/Should+Have+V3 .
Sol. The correct answer is option (b). The sentence will be “The
witness described the suspect as short, dark-skinned and S363. Ans. (a)
bearded.” Sol. The most appropriate option for the phrase replacement is
Committed means pledged or bound to a certain course or policy; option (a).’If a person had studied’ will be used instead of ‘If a
dedicated. person studied’ because in order to portray the past of unreal
Described means gave a detailed account of. situation , the structure is ‘If+S+had+V3,S+would+have+V3’
Praised means expressed warm approval or admiration of. .For example ,
If you had played well , you would have won the match.
Pacified means quelled the anger.

S359. Ans. (d) S364. Ans. (b)


Sol. The correct answer is option (d). Optimism is the quality Sol. The most appropriate option is option (b). ‘A study of
which can make an agreement to be reached. history’ will be used instead of ‘a reading of history’ because read
Optimistic means hopeful and confident about the future. means look at and comprehend the meaning of (written or printed
Complicated means introduce complications. matter) by interpreting the characters or symbols of which it is
Generous means showing kindness towards others. composed. But study means to investigate or examine, as by
Playful means lively , high-spirited. observation, research, etc. For example;
(i)She is reading your letter.
S360. Ans. (c) (ii)He is studying astrology these days.
Sol. The correct answer is option (c).Unfortunately there were
many inconveniences for the Amarnath pilgrims. S365. Ans.(b)
Unfortunately means sadly, regrettably. Sol. The most suitable choice here is option (b). ‘Seeing that she
Cowardly means lacking courage was very tired’ will be used instead of ‘Seeing that she is very
Scholarly means involving or relating to serious academic study. tired’ because in the Main clause the subject ‘I’ and the verb
Heavily means laboriously. ‘walked’ used with it in past tense confirms us that both the
incidences are in past.
S361. Ans. (c)

Facebook Page- https://www.facebook.com/vishalpariharpage Youtube- https://www.youtube.com/vishalparihar


Follow
81 Vishal Sir Telegram Channel- https://t.me/englishbyvishalsirchannel Instagram- https://www.instagram.com/vishalthetrainer
By Vishal sir
Also Important For SBI CLERK, IBPS CLERK, RRB CLERK
& Other Competitive Exams

Directions (366-370): In the following passage, some of the


words have been left out, each of which is indicated by a Q368. (a)Abduct
number. Find the suitable word from the options given (b)Isolate
against each number and fill up the blanks with appropriate (c)Crack
words to make the paragraph meaningfully complete. (d)Converge
(e)Evacuate
Religion does not consist in ______(366)________ or dogmas. It
is not what you read, nor what dogmas you believe in, but what Q369. (a)Revealed
you realize. No great master ever taught that external forms were (b)Deliberate
necessary for _______(367)_________. The power of attaining it (c)Diagnostic
is within us. We live and move in God. Books never make (d)Effortless
religions, but religions make books. No book ever created God, (e)Augmented
but God inspired great books. No book ever created a soul. The
end of religion is in realizing God in the soul. The one universal Q370. (a)Width
truth in all religions is to realize God. Ideals and methods may (b)Quantity
differ, but that is the central point. There may be a thousand (c)Adversity
different radii, but they all _______(368)___________ to the one (d)Durability
center, and that is the realization of God: something behind this (e)Indifference
world of sense, this world of eternal eating and drinking and
talking nonsense, this world of false shadows and selfishness. Directions (371-380): Read the following passage carefully
Aman may have never entered a church or mosque, nor and answer the questions given below it. Certain words have
performed any religious ceremony, but if he feels God within and been printed in bold to help you locate them, while answering
is thereby lifted above the vanities of the world, that man is a holy some of the questions.
man. Many years ago, I visited a sage. We talked of the Vedas,
Bible, Quran and other ________(369)________ books. At the Youth represents the most dynamic segment of the population
close of our talk, he asked me to pick up a book that was on the and are the future of the Nation. India is one of the youngest
table. It contained forecast of rainfall during the year. He asked nations in the world. About 65% of India’s population is under
me to read it. I read out the ________(370)________ of rain that 35 years of age. The focus of the Government is on youth-led
was to fall. “Now take the book and squeeze it,” he said and I did. development. The Youth should be “active drivers” and not
“Why, not a drop of water has come out,” he exclaimed. “Until merely the “passive recipients” of development. The vision of our
the water comes out, it is all book, book, book.” country lies in the hands of our youths. They are filled with
tremendous and towering ambitions. It will be a great wastage
Q366. (a)Rudeness (b)Doctrines (c)Impatience of human resources if these youths are not given an opportunity
(d)Extravagance (e)Trustworthiness to exercise their talent. This beautiful land needs these youths in
order for our soil to become a brighter one.
Q367. (a)Extinction
(b)Enhancement The primary role of young people is to get a good education in
(c)Salvation order to become better citizens of tomorrow. They need to learn
(d)Revision skills to do the job that their country’s economy needs. They also
(e)Modification need to know how to read, write, think, understand, analyze, and

Facebook Page- https://www.facebook.com/vishalpariharpage Youtube- https://www.youtube.com/vishalparihar


Follow
82 Vishal Sir Telegram Channel- https://t.me/englishbyvishalsirchannel Instagram- https://www.instagram.com/vishalthetrainer
By Vishal sir
Also Important For SBI CLERK, IBPS CLERK, RRB CLERK
& Other Competitive Exams

discuss the issues their country faces. The entire success of the life. The ones whom are destroying their future, they tend to listen
nation depends on the youths. However, in order for continuous to their fellow youths. They will make them understand the
success to take place; it is the Government’s responsibility to importance of a good education. The wise youths that we have
provide the youth with proper facilities for, getting equipped with out there should be taken into consideration. Some of them,
the knowledge of the modern era. Youths are fighters. They fight though educated are unemployed.
for an identity in society, equality, the homeless, bullying,
unemployment, exploitation, poverty and other problems which They should be given an opportunity to expose their intelligence
the world faces today. All of them hope for a world full of to the world and make themselves into someone. Some
opportunities, so great minds can conquer them and become organizations and other firm should assist the youths; so they can
better individuals. They need good morals and values to handle make our land a great and educated one. They will make a huge
conflicts in a positive way. We have aspiring doctors, difference in society and the entire world. The country will be
entrepreneurs, scientists, and who knows; maybe the next recognized. The youths just need the support from their fellow
president. citizens and they will perform their duties. In conclusion, the role
of the youth in the nation building is crucial. They are problem
Illiteracy is major around the world. About ninety percent of the solvers, have a positive influence on other young people and the
people who reside in villages are illiterate. The reason is their nation, and are extremely ambitious. They have the ability to
utter ignorance and lack of initiative to lead them in the right create an identity for themselves and move the nation forward.
direction. Someone has to pull them up. The role of the youth is However, they will not be able to do this without the support of
the most important in today’s time. They have underplayed their Government and fellow youths. So the youths can make
themselves in the field of the politics. Youth are strong forces in their beautiful land flourish and shine in success.
social movements.
Youths are problem solvers. Our nations need them to resolve Q371 .How can government help the youth in attaining the continuous
most of our problems. The nation is facing a lot of problems, and success for the nation?
they are capable of solving them. They just need to be given a (a) By combating offences like smuggling, black-marketing and
chance to prove themselves. Youths have the power to unite hoarding.
individuals in the six ethnic groups. Racism is an ongoing issue (b) Getting youth equipped with proper facilities and knowledge
around the world. Individuals are fighting against each other of the modern era.
because of the complexion of their skin and the texture of their (c) By making them understand a specific project and its
hair. Religion is another issue, the youths can convince their importance to the society.
fellow man to live in peace and love. All of us are one and we (d) By politicizing many university campuses.
should not allow these little differences to push us away from (e) All of these.
each other.
Q372. How are youths described as fighters in the passage?
There is also a lot of crime taking place. Women are being killed (a)With the help of NCC training they fight in armed forces.
by their abusive husbands. Person’s homes, businesses, are being (b)They fight for exploitation, poverty, bullying and
broken into. All of this crime and violence needs to stop. The unemployment.
youth once more has the ability to bring about a change in their (c)They fight for an identity in society.
country. Youths seems to have the ability to face any challenges (d)Both (b) and (c)
and problems. They have a positive influence on their fellow (e)None of these
young people. They are able to teach them the positive things in

Facebook Page- https://www.facebook.com/vishalpariharpage Youtube- https://www.youtube.com/vishalparihar


Follow
83 Vishal Sir Telegram Channel- https://t.me/englishbyvishalsirchannel Instagram- https://www.instagram.com/vishalthetrainer
By Vishal sir
Also Important For SBI CLERK, IBPS CLERK, RRB CLERK
& Other Competitive Exams

ANSWER WITH SOLUTION Sol. Option (b) is the most appropriate choice. As only the
quantity of rain can be suited in the blank that was to fall. Width
S366. Ans. (b) of rain cannot be measured.
Sol. Doctrines will be used in the blank. Adversity means a difficult or unpleasant situation.
Doctrines means beliefs or convictions. Durability means the ability to withstand wear, pressure, or
Extravagance means lack of restraint in spending money or using damage.
resources. Indifference means lack of distinction.
Trustworthiness means the ability to be relied on as honest or
truthful. S371. Ans. (b)
Sol. The most suitable answer is option (b).Refer to second
S367. Ans. (c) paragraph where it is mentioned that the entire success of the
Sol. Salvation means preservation or deliverance from harm, ruin, nation depends on the youths. However, in order for continuous
or loss. success to take place; it is the Government’s responsibility to
Extinction means disappearance. provide the youth with proper facilities for, getting equipped with
Modification means alteration. the knowledge of the modern era.

S368. Ans. (d) S372. Ans. (d)


Sol. The appropriate answer is option (d).As it is mentioned that Sol. Option (d) is the most suitable choice. The answer can be
there may be a thousand different radii, but they all converge to deduced from the second paragraph of the passage where it is
the one center. Hence to the one center can only be converge and explicitly mentioned that “youths are fighters and they fight for
not diverge or the rest other options. an identity in society, equality, the homeless, bullying,
Abduct means take (someone) away illegally by force or unemployment, exploitation, poverty and other problems which
deception; kidnap. the world faces today.” Hence option (d) consists of all the
Isolate means cause (a person or place) to be or remain alone or specifics mentioned in the option (d).
apart from others.
Crack means break or cause to break without a complete
separation of the parts.
Evacuate means remove (someone) from a place of danger to a
safer place.

S369. Ans.(a)
Sol. The most suitable option is (a).
Revealed means made (previously unknown or secret
information) known to others.
Diagnostic means a distinctive symptom or characteristic.
Augmented means increased.

S370. Ans. (b)

Facebook Page- https://www.facebook.com/vishalpariharpage Youtube- https://www.youtube.com/vishalparihar


Follow
84 Vishal Sir Telegram Channel- https://t.me/englishbyvishalsirchannel Instagram- https://www.instagram.com/vishalthetrainer
By Vishal sir
Also Important For SBI CLERK, IBPS CLERK, RRB CLERK
& Other Competitive Exams

Q373. What is the reason behind the illiteracy of the ninety percent of (a)Passive
the people who reside in the villages? (b)Lively
(a) Lack of initiative to lead them in proper direction. (c)Stagnant
(b) Complete ignorance. (d)Lethargic
(c) The basic issue of religion. (e)Powerless
(d) Both (b) and (c)
(e) Both (a) and (b) Q378. Tremendous
(a)Insignificant
Q374. How does the youth have the ability to bring about a change in (b)Infinitesimal
their country? (c)Minute
(a)Youths can convince their fellow man to live in peace and love. (d)Colossal
(b)Youths have the power to unite individuals in the six ethnic (e)Marginal
groups.
(c)They have a positive influence on fellow young people. Directions (379-380): Choose the word which is the
(d)They are strong forces in social movements. OPPOSITE in meaning as the word printed in bold as used
(e) All of these. in the passage.

Q375. What is the reason that individuals of different ethnic groups are Q379. Ambitious
fighting against each? (a)Ambiguous
(a)Because of the enslavement (b)Critical
(b)As some public figures in the American Black community (c)Deploy
have championed the supremacy of their own race. (d)Lazy
(c)As the people are causing geographical division around the (e)Determined
world
(d)Because of the complexion of their skin and the texture of their Q380. Crucial
hair (a)Trivial (b)Vital
(e) None of these. (c)Significant (d)Fundamental
(e)Essential
Q376. What is the most appropriate theme of the passage?
(a)Dynamics of Making Country Work Directions (381-385): In the question given below, some
(b)Cultural Racism as a Rhetorical Strategy sentences/phrases are given which have to be arranged in a
(c)Racial Inequality proper sequence. Select the option which best defines the
(d)Role of Youth in Nation Building proper sequence and arranges the sentence in an appropriate
(e)Fellow youth and Beautiful India way.

Directions (377-378): Choose the word which is most nearly Q381. of factors and hence (A)/ for each case (B)/ upon a large number
the SAME in meaning as the word printed in bold as used in (C)/historical outcomes depend (D)/prediction is difficult (E)/
the passage. (a)BCAED (b)CABED
(c)EDCAB (d)DCAEB
Q377 .Dynamic (e)CAEBD

Facebook Page- https://www.facebook.com/vishalpariharpage Youtube- https://www.youtube.com/vishalparihar


Follow
85 Vishal Sir Telegram Channel- https://t.me/englishbyvishalsirchannel Instagram- https://www.instagram.com/vishalthetrainer
By Vishal sir
Also Important For SBI CLERK, IBPS CLERK, RRB CLERK
& Other Competitive Exams

ANSWER WITH SOLUTION Stagnant means showing no activity; dull and sluggish.
Lethargic means inert.
S373. Ans. (e) Powerless means helpless.
Sol. The most appropriate answer here is option (e).The answer
can be inferred from the third paragraph of the passage where it S378. Ans. (d)
is given that illiteracy is major around the world and about ninety Sol. Tremendous means very great in amount, scale, or intensity.
percent of the people who reside in villages are illiterate. Colossal means extremely large or great.
Furthermore the reason behind it is their utter ignorance and lack Insignificant means too small or unimportant to be worth
of initiative to lead them in the right direction. Someone has to consideration.
pull them up. Infinitesimal means extremely small.
Marginal means minor and not important; not central.
S374. Ans. (e)
Sol. The appropriate answer is option (e).All of these options can S379. Ans. (d)
be inferred from the last three paragraphs of the passage. The six Sol. Ambitious means having or showing a strong desire and
ethnic groups of India are Negrito, Proto - Australoids or Austrics determination to succeed
,Mongoloids, Mediterranean or Dravidian ,Western Critical means expressing adverse or disapproving comments or
Brachycephals and Nordic Aryans. judgements.
Hence it is the only opposite of the given word.
S375. Ans. (d) Ambiguous- open to more than one interpretation; not having one
Sol. The most suitable choice here is option (d).This answer can obvious meaning
be traced from the third paragraph of the passage where it is
mentioned as “Racism is an ongoing issue around the world. S380. Ans. (a)
Individuals are fighting against each other because of the Sol. Crucial means main, of much importance.
complexion of their skin and the texture of their hair.” Trivial means of little value or importance.
Hence option (d) is the best choice for the answer. Vital means absolutely necessary; essential.
Significant means notable , noteworthy.
S376. Ans. (d) Fundamental means elementary, root.
Sol. The whole passage is based upon the role of the youth in Essential means absolutely necessary, extremely important.
nation building as it revolves around this topic. The opening line
of the passage itself is that the youth represents the most dynamic S381. Ans. (d)
segment of the population and are the future of the Nation. Hence Sol. Option (d) is the correct choice. The sentence can be
option (d) is the most appropriate choice that can be suited as the arranged as follows, “Historical outcomes depend upon a large
theme of the passage. number of factors and hence prediction is difficult for each case.”

S377. Ans. (b)


Sol. Dynamic means (of a person) positive in attitude and full of
energy and new ideas.
Lively means energetic.
Passive means yielding.

Facebook Page- https://www.facebook.com/vishalpariharpage Youtube- https://www.youtube.com/vishalparihar


Follow
86 Vishal Sir Telegram Channel- https://t.me/englishbyvishalsirchannel Instagram- https://www.instagram.com/vishalthetrainer
By Vishal sir
Also Important For SBI CLERK, IBPS CLERK, RRB CLERK
& Other Competitive Exams

Q382. amount of damage rises (A)/ as civilization (B)/ people to do a (a)Converged


massive (C)/ becomes more complex (D)/the ability of a few (E)/ (b)Climatic
(a)DBAEC (c)Deeper
(b)BAECD (d)Managerial
(c)ECABD (e)Preclude
(d)ECDAB
(e)CBAED Q387. Modern science and technology _______________ human
beings to control natural forces more effectively.
Q383. market may experience (A)/the real estate (B)/ interest rates are (a)Populates
reduced (C)/ a recession if (D)/a boom during (E)/ (b)Empower
(a)DCEAB (c)Interprets
(b)EABCD (d)Adjourns
(c)BAEDC (e)Reschedule
(d)CDBAE
(e)ABECD Q388. The customer also threatened to __________________ his
business relationship with the employer.
Q384. we need sustenance (A)/ cohesion and connection (B) /but we (a)Aspect
also need social (C)/of all sorts (D)/ and a viable habitat (E) (b)Ponder
(a)CABDE (c)Terminate
(b)AECBD (d)Appease
(c)BECAD (e)Increase
(d)DCEAB
(e)ADBCE Q389. For college students to do a part-time job will exert a
_________________ influence on their personality and life.
Q385. tipped the balance (A)/ two quick goals from Robson (B)/the (a)Disfiguring
teams were (C)/ evenly matched until (D)/ in favor of England (b)Yearning
(E) (c)Blameless
(a)DCEAB (d)Profound
(b)EABCD (e)Aloof
(c)BADEC
(d)CDBAE Q390. The reduction in white blood cell counts lowers the
(e)ABECD ________________ to disease.
(a)Reaction
Directions (386-390): Each question below has one blank, (b)Acceptance
which is indicating that something has been omitted. Find out (c)Resistance
which option can be used to fill up the blank in the sentence (d)Magnificence
to make it meaningfully complete. (e)Reliability

Q386. Religious commitments go much ____________ than intellectual


explanations.

Facebook Page- https://www.facebook.com/vishalpariharpage Youtube- https://www.youtube.com/vishalparihar


Follow
87 Vishal Sir Telegram Channel- https://t.me/englishbyvishalsirchannel Instagram- https://www.instagram.com/vishalthetrainer
By Vishal sir
Also Important For SBI CLERK, IBPS CLERK, RRB CLERK
& Other Competitive Exams

ANSWER WITH SOLUTION Empower means make (someone) stronger and more confident,
especially in controlling their life and claiming their rights.
S382. Ans. (c) Interprets means explain the meaning of (information or actions).
Sol. Option (c) is the correct choice which rearranges the sentence Adjourns means put off or postpone (a resolution or sentence)
most suitably. The sentence can be arranged as follows, “The Reschedule means change the time of (a planned event).
ability of a few people to do a massive amount of damage rises
as civilization becomes more complex.” S388. Ans. (c)
Sol. Option (c) is the most suitable choice to be filled in the blank.
S383. Ans. (c) According to the sentence and the usage , terminate is the most
Sol. The appropriate option for the arrangement of the sentence appropriate word to be used as the employer was threatened to
is option (c).The sentence can be arranged as follows: “The real end the business relationship he had with the customer. Rest all
estate market may experience a boom during a recession if the words do not fit in the blank.
interest rates are reduced.” Terminate means bring to an end.
Aspect means the side of a building facing a particular direction.
S384. Ans. (b) Ponder means think about (something) carefully, especially
Sol. The correct choice for the arrangement of the sentence is before making a decision or reaching a conclusion.
option (b).Furthermore the sentence can be arranged as follows: But ponder will take the preposition ‘over’. So it does not fit here.
“We need sustenance and a viable habitat, but we also need social Appease means assuage or satisfy (a demand or a feeling).
cohesion and connection of all sorts.”
S389. Ans. (d)
S385. Ans. (d) Sol. Profound is the most appropriate option to be filled in the
Sol. Option (d) is the correct choice for the arrangement of the blank. This is so as being a college student doing a part time job
sentence. The sentence can be arranged as follows: “The teams too can only have a positive effect which is described by the word
were evenly matched until two quick goals from Robson tipped profound among all of the given options.
the balance in favor of England.” Profound means very great or intense.
Disfiguring means spoil the appearance of.
S386. Option (c) is the most appropriate choice to be filled in the Yearning means a feeling of intense longing for something.
sentence. According to the context of the sentence deeper is the Aloof means not friendly or forthcoming ; cool and distant.
only word that fits in the blank.
Converging means tending to meet at a point. S390. Ans. (c)
Managerial means relating to management or managers. Sol. Resistance is the most suitable option that fits in the blank
Preclude means prevent from happening; make impossible. among all of the given options. As lowering resistance to disease
is the result of reduction in white blood cell count.
S387. Ans. (b) Resistance means the refusal to accept or comply with something.
Sol. Option (b) is the most appropriate choice to be filled in the Magnificence means glory, opulence.
blank. According to the context of the sentence, the word that fits Reliability means the quality of being trustworthy or of
most suitably in the blank is empower. This is because modern performing consistently well.
science and technology has empowered which means boosted
human beings to control natural forces more effectively.

Facebook Page- https://www.facebook.com/vishalpariharpage Youtube- https://www.youtube.com/vishalparihar


Follow
88 Vishal Sir Telegram Channel- https://t.me/englishbyvishalsirchannel Instagram- https://www.instagram.com/vishalthetrainer
By Vishal sir
Also Important For SBI CLERK, IBPS CLERK, RRB CLERK
& Other Competitive Exams

Directions (391–395): Which of the phrases (a), (b), (c) and (b)Lend me any rupees
(d) given below should replace the phrase given in bold in the (c)Lend me a few rupees
following sentences to make the sentence grammatically (d)Bought me few rupees
correct? If the sentence is correct as it is and there is no (e)No correction required
correction required mark (e) i.e. 'No correction required' as
the answer. Directions (396-400): In the following passage, some of the
words have been left out, each of which is indicated by a
Q391. There is no one who I esteem more highly than your father. number. Find the suitable word from the options given
(a)That I esteem more against each number and fill up the blanks with appropriate
(b)Whom I esteem very words to make the paragraph meaningfully complete.
(c)Whom I esteem more
(d)Which I esteem very The irony of our age is that we’re working towards machines
(e)No correction required becoming human, and humans becoming machines. Technology
doesn’t have the answer to everything health-related. This is an
Q392. The use of animals in scientific tests raise some difficult ethical intoxicating viewpoint that several industry giants hold. But our
questions. lives are a __________(396)___________ between mind, body
(a) did raised some difficult and spirit. Technology can never replace self-knowledge and self-
(b) raises some difficult intimacy, a lack of which is the very root of chronic physical
(c) rises some difficult illness as well as emotional trauma. Ancient intelligence of your
(d) being raised some ancestors is still stored in your DNA, and you carry a
(e) No correction required _______(397)________ piece of the cosmos within you. It
becomes dangerous when metrics and technological identities are
Q393. More ammonium sulphate solution is being recovered with _______(398)__________ on you. We were never made to fit
through the process of distilling oil shale. into external moulds , and we are too beautifully complex to live
(a) Recovering on in technical strait jackets. Throughout history, humans have had
(b) Having recovered from the experience of knowing more than what’s
(c) Having been recovered from ______(399)_________ by their five senses – an implicit
(d) Being recovered in knowledge, energy sensitivities, intuition – and the more they
(e) No correction required open to these concepts, the more strongly they feel this enigmatic
force. We have been losing touch with the
Q394. There was too many people trying to get into the football ____(400)______________ information that mind-body-spirit is
stadium. constantly trying to convey, and this wreaks havoc in our lives.
(a)There was too much We are divinely human – we have knowledge, wisdom and power
(b)There were too many people stored inside our personhood – which technology cannot replace.
(c)There were too much Our bodies never lie. Desire, ambition, art, creativity, gut
(d)It was too many instincts, love, empathy, ability to create beauty, to rise from the
(e)No correction required ashes of our losses – are just some of the tools in our arsenal
which are unmatched.
Q395. You could have lend me few rupees in that hour of need?
(a)Borrow me many rupees

Facebook Page- https://www.facebook.com/vishalpariharpage Youtube- https://www.youtube.com/vishalparihar


Follow
89 Vishal Sir Telegram Channel- https://t.me/englishbyvishalsirchannel Instagram- https://www.instagram.com/vishalthetrainer
By Vishal sir
Also Important For SBI CLERK, IBPS CLERK, RRB CLERK
& Other Competitive Exams

ANSWER WITH SOLUTION without a preceding 'a', means "very few" or "none at all". On the
other hand, a few is used to indicate "not a large number". The
S391. Ans. (c) difference is subtle, yet there are instances where the two can
Sol. Option (c) is the most suitable replacement of the bold part. mean completely opposite things.
The word who can only be used when it is the subject of a verb.
Whom is like him, her, and them because it is never the subject
of a verb. Whom is an object. Thus whom I esteem more will be
used.

S392. Ans. (b)


Sol. Option (b) is the most appropriate choice to be used as the
replacement of the bold part. In a simple present sentence where
he, she, it or a name is the subject, verbs can end in -s or -es
depending on the way the verb end. Option (c) is not correct as
Rise is intransitive verb and does not take an object. What this
means is that you use the verb rise when something moves
upwards by itself. Raise, on the other hand, is a transitive verb
that requires that the subject act upon an object. In other words,
something raises something else.

S393. Ans.(d)
Sol. Option (d) is the most suitable choice to be used as the
replacement of the bold part. The correct solution will be “More
ammonium sulphate solution is being recovered in the process of
distilling oil shale.” With and through will not go together as two
prepositions do not go hand in hand with reference to the context
of the sentence. Hence ‘being recovered in’ will be used.

S394. Ans. (b)


Sol. Option (b) is the most appropriate choice. ‘There were too
many people’ will be used instead of ‘There was too many
people’ because the verb that is used after ‘there’ depends on the
number or person of noun or pronoun. For example;
There is a table lamp on the table.
There are five books on the table.

S395. Ans.(c)
Sol. Option (c) is the correct choice. ‘Lend me few rupees’ will
be replaced by ‘lend me few rupees’ because few, when used

Facebook Page- https://www.facebook.com/vishalpariharpage Youtube- https://www.youtube.com/vishalparihar


Follow
90 Vishal Sir Telegram Channel- https://t.me/englishbyvishalsirchannel Instagram- https://www.instagram.com/vishalthetrainer
By Vishal sir
Also Important For SBI CLERK, IBPS CLERK, RRB CLERK
& Other Competitive Exams

Q396. (a)Purchase (b)Negotiation Atharvaveda : Salutations to Mother Earth. Leading lifestyles that
(c)Scolding (d)Dishonest are harmonious and sustainable are a part of our ethos. Once we
(e)Resolving realize how we are flag bearers of a rich tradition, it will
automatically have a positive impact on our actions.
Q397. (a)Tedious (b)Abandoning
(c)Vital (d)Contentious The second aspect is public awareness. We need to talk, write,
(e)Swift debate, discuss and deliberate as much as possible on questions
relating to the environment. At the same time, it is vital to
Q398. (a)Embarassed (b)Located encourage research and innovation on subjects relating to the
(c)Collided (d)Annoyed environment. This is when more people will know about the
(e)Imposed pressing challenges of our times and ways to mitigate them.
When we as a society are aware of our strong links with
Q399. (a)Perceived (b)Consoled environmental conservation and talk about it regularly, we will
(c)Surrendered (d)Extreme automatically be proactive in working towards a sustainable
(e)Insisted environment. We see proactiveness in the Swachh Bharat
Mission, which is directly linked to a sustainable future. With the
Q400. (a)Unkind (b)Humble blessings of the people of India, over 85 million households now
(c)Crucial (d)Ruined have access to toilets for the first time. Over 400 million Indians
(e)Civilized no longer have to defecate in the open. Sanitation coverage is up
from 39% to 95%. These are landmark efforts in the quest of
Directions (401-408): Read the following passage and answer reducing the strain on our natural surroundings. Our country is
the questions that follow. devoting unparalleled attention to new and renewable sources of
energy. Over the last four years, this sector has become more
Human beings and nature have a very special relationship. accessible and affordable. The Ujala Yojana has led to the
Mother Nature has nurtured and nourished us. The first distribution of nearly 31 crore LED bulbs. The costs of LED
civilizations were established on the banks of rivers. Societies bulbs have reduced and so have the electricity bills and the CO2
that live in harmony with nature flourish and prosper. Today, emissions.
human society stands at an important crossroads. The path that While the world is talking about climate change, the call for
we take hereon will not only determine our well-being but also climate justice has also reverberated from India. Climate justice
that of the generations who will inhabit our planet after us. The is about safeguarding the rights and interests of the poor and
imbalances between our greed and necessities have led to grave marginalized sections of society, who are often the biggest
ecological imbalances. We can either accept this, go ahead with sufferers from the menace of climate change. It is up to us to take
things as if it is business as usual, or we can take corrective on the mantle of global responsibility towards a sustainable
actions. future. The world needs to shift to a paradigm of environmental
Three things will determine how we as a society can bring a philosophy that is anchored in environmental consciousness
positive change. The first is internal consciousness. For that, there rather than merely in government regulations. Individuals and
is no better place to look than our glorious past. Respect for nature organizations who are working assiduously in this direction have
is at the core of India’s traditions. The Atharvaveda contains the become the harbingers of a monumental change in our society.
Prithvi Sukta, which contains unparalleled knowledge about Together, we will create a clean environment that will be the
nature and the environment. It is beautifully written in cornerstone of human empowerment!

Facebook Page- https://www.facebook.com/vishalpariharpage Youtube- https://www.youtube.com/vishalparihar


Follow
91 Vishal Sir Telegram Channel- https://t.me/englishbyvishalsirchannel Instagram- https://www.instagram.com/vishalthetrainer
By Vishal sir
Also Important For SBI CLERK, IBPS CLERK, RRB CLERK
& Other Competitive Exams

ANSWER WITH SOLUTION Insisted means persisted in (doing something).

S396. Ans. (b) S400. Ans. (c)


Sol. Negotiation is the appropriate word to be used in the blank. Sol. Option (c) is the most appropriate choice to be filled in the
As in the line the author means to say that our lives are a blank. Crucial means of great importance. This word suits as our
discussion aimed at reaching an agreement between mind, body mind body spirit is constantly trying to convey some important
and spirit. information but we lose track of it which creates havoc in our
Scolding means rebuking angrily. lives.
Resolving means settling or finding a solution to. Humble means having or showing a modest or low estimate of
one's importance.
S397. Ans. (c) Ruined means reduced (a building or place) to a state of decay,
Sol. Option (c) is the most appropriate word to be used in the collapse, or disintegration.
blank. Civilized means well mannered.
Vital means absolutely necessary; essential.
Tedious means too long, slow, or dull; tiresome or monotonous.
Abandoning means ceasing to support or look after (someone);
deserting.
Contentious means causing or likely to cause an argument;
controversial.
Swift means happening quickly or promptly.

S398. Ans. (e)


Sol. Option (e) is the most appropriate word to be filled in the
blank.Rest all the words do not fit in the context of the sentence.
Imposed means forced on someone.
Embarassed means awkward.
Located means discover the exact place or position of.
Collided means came into conflict or opposition.
Annoyed means slightly angry; irritated

S399. Ans. (a)


Sol. The most suitable option to be filled in the blank is option
(a).Our senses can only perceive implicit knowledge, energy
sensitivities and intuition. Rest all of the options are irrelevant in
context of the paragraph.
Perceived means became aware or conscious of (something).
Consoled means comforted.
Surrendered means capitulated.
Extreme means very large in amount or degree:

Facebook Page- https://www.facebook.com/vishalpariharpage Youtube- https://www.youtube.com/vishalparihar


Follow
92 Vishal Sir Telegram Channel- https://t.me/englishbyvishalsirchannel Instagram- https://www.instagram.com/vishalthetrainer
By Vishal sir
Also Important For SBI CLERK, IBPS CLERK, RRB CLERK
& Other Competitive Exams

Q401. According to the author, what has led to the serious ecological Q406. What can be the most appropriate title of the passage?
imbalance in nature? (a)Climate Justice
(a) Poor and marginalized section of the society. (b)In Harmony with Mother Nature
(b) The cost of LED bulbs and Co2 emissions (c)Imbalance in nature
(c) Volcanic eruptions. (d)Human empowerment
(d) Imbalance between our greed and necessities. (e)Environmental Conservation
(e) All of these.
Q407. What is the antonym of the given word?
Q402. What are some of the specifics of Atharva Veda? Flourish
(a) Description about working towards a sustainable (a)Thrive
environment. (b)Prosper
(b) How to bring effective changes in the society. (c)Develop
(c) Exceptional knowledge about nature and the environment. (d)Bloom
(d) Elaboration of human empowerment. (e)Shrink
(e) Consciousness about human mind.
Q408. What is the synonym of the given word?
Q403. When shall the subjects related to environment be encouraged? Innovation
(a) Over-harvesting of natural resources. (a)Stagnation
(b) When there will be public awareness, discussions, talks, (b)Novelty
debates on questions relating to the environment. (c)Deterioration
(c) Choosing energy efficient appliances. (d)Vocation
(d) By promoting climate justice. (e)Imitation
(e) All of these.
Direction (409-413): In each of the questions given below a
Q404. What are some of the landmark efforts in the quest to reduce sentence is given which is then divided into five parts. There
strain on environment as mentioned in the passage? may or may not be error in any one part of the sentence .You
(a) Leading lifestyles that are sustainable and harmonious. must choose that part as your answer.
(b) Focus on new and renewable sources of energy.
(c) Sanitation coverage is up from 39% to 95%. Q409 .River water (a)/ pollution is (b)/ often indicate (c) / by algae (d)/
(d) Both (b) and (c) distribution (e)
(e) Both (a) and (b)
Q410 The first year (a)/of the child’s (b) /life is characterized (c) /in rapid
Q405. What is/are the factor/(s) to which the methodology of (d)/ physical growth (e)
environmental philosophy is anchored?
(a)Government Regulations Q411. A ray of light (a)/ passing through (b) / the centre of a (c)/ thin lens
(b)Environmental Consciousness . keep (d)/ its original direction (e)
(c) Safeguarding the rights and interests of the poor.
(d)Both (a) and (b) Q412. The teacher was (a)/ happy when (b) /he entered the (c) /class
(e)Both (b) and (c) which comprises (d)/of a hundred students. (e)

Facebook Page- https://www.facebook.com/vishalpariharpage Youtube- https://www.youtube.com/vishalparihar


Follow
93 Vishal Sir Telegram Channel- https://t.me/englishbyvishalsirchannel Instagram- https://www.instagram.com/vishalthetrainer
By Vishal sir
Also Important For SBI CLERK, IBPS CLERK, RRB CLERK
& Other Competitive Exams

S401. Ans. (d) S406. Ans. (b)


Sol. Option (d) is the appropriate choice. Refer to the first Sol. Option (b) is the most suitable title of the passage. This is
paragraph where it is mentioned that the path that we take hereon because whole passage revolves around it and hence we need a
will not only determine our well-being but also that of the clean environment for human empowerment as is said by the first
generations who will inhabit our planet after us. The imbalances line of the passage that human beings and nature have a very
between our greed and necessities have led to grave ecological special relationship.
imbalances. S407. Ans. (e)
S402. Ans. (c) Sol. Flourish means grow or develop in a healthy or vigorous
Sol. Option (c) is the only suitable answer here. The answer can way, especially as the result of a particularly congenial
be directly quoted from second paragraph where the lines are environment.
given as,“The Atharvaveda contains the Prithvi Sukta, which Shrink means shorten.
contains unparalleled knowledge about nature and the Thrive means prosper.
environment. It is beautifully written in Atharvaveda: Salutations Prosper means the same .
to Mother Earth.” Bloom means produce.
S408. Ans. (b)
S403. Ans.(b) Sol. Innovation means transformation.
Sol. The most appropriate answer is option (b).The answer can be Novelty means the quality of being new, original, or unusual.
referred from third paragraph of the passage where it is Stagnation means the state of not flowing or moving.
mentioned that “We need to talk, write, debate, discuss and Deterioration means degradation.
deliberate as much as possible on questions relating to the Vocation means a strong feeling of suitability for a particular
environment. At the same time, it is vital to encourage research career or occupation.
and innovation on subjects relating to the environment. This is Imitation means the action of using someone or something as a
when more people will know about the pressing challenges of our model.
times and ways to mitigate them.” S409. Ans.(c)
S404. Ans. (d) Sol. ‘is often indicate’ is wrong. Instead ‘is often indicated’ will
Sol. Option (d) is the appropriate choice as it consists of both the be used because after be(is/are/am/was/were/be/been) ,the
options (b) and (c).Both of these options are mentioned in the original form of verb is not used instead past participle form or
third paragraph where it is given that sanitation coverage is up present participle form is used.
from 39% to 95%. These are landmark efforts in the quest of S410. Ans. (d)
reducing the strain on our natural surroundings. Our country is Sol. Option (d) has an error. ‘by’ will be used instead of ‘in’
devoting unparalleled attention to new and renewable sources of because in passive voice by+ Agent is used.
energy. Over the last four years, this sector has become more (i) Ram beat Sita.[Active]
accessible and affordable. (ii) Sita was beaten by Ram.[Passive]
S405. Ans.(d) S411. Ans.(d)
Sol. Option (d) is the most appropriate choice. This can be Sol. There is error in part (d) of the sentence. ‘keeps’ will be used
inferred from the last paragraph of the passage where it is given instead of ‘keep’ because the subject of sentence is ‘A ray ‘ and
“The world needs to shift to a paradigm of environmental this is singular.
philosophy that is anchored in environmental consciousness S412. Ans. (d)
rather than merely in government regulations.” Sol. Instead of comprises, ‘comprised’ should be used.

Facebook Page- https://www.facebook.com/vishalpariharpage Youtube- https://www.youtube.com/vishalparihar


Follow
94 Vishal Sir Telegram Channel- https://t.me/englishbyvishalsirchannel Instagram- https://www.instagram.com/vishalthetrainer
By Vishal sir
Also Important For SBI CLERK, IBPS CLERK, RRB CLERK
& Other Competitive Exams

Q413. When I shall get back (a)/ I shall (b)/ pay of the money (c)/ that I. (d)Distinct
borrowed (d)/ from you last month (e) (e)Helped
Directions (414-420): In the following passage there are
blanks, each of which has been numbered. These numbers are Q417. (a)Equality
printed below the passage and against each, some words are (b)Compare
suggested, one of which fits the blank appropriately. Find out (c)Disparity
the appropriate word in each case. (d)Practicality
(e) Order
Educational institutions usually deal with gender issues in a
textbook mode, ____________(414)equality and mythologizing Q418. (a)Explicit
modernity. In the typical ethos of a social science classroom, no (b)Direct
______________(415) is possible with male attitudes towards (c)Specific
women either. Nor can the terror internalized by girls at an early (d)Embedded
age be _______________(416) and discussed. Lessons on gender (e)Precise
______________(417) mention prejudices and stereotypes, but
they seldom include the ones ______________(418) in religious Q419. (a)Hampered
practices and festive rituals. While education has improved the (b)Employed
distribution of eligibility and job opportunities for women, it has (c)Observed
made little impact on male aggression and self-righteousness. The (d)Protected
potential that education has for improving male sanity has been (e)Separated
severely ______________(419) by the unprecedented and easy
access provided by digital devices to pornography, including Q420. (a)Endured
child pornography. The situation is quite dire. A helpline was set (b)Expressed
up by the National Commission for Protection of Child Rights (c)Wrecked
(NCPCR) as a facility for children who have been abused. It was (d)Emptied
_______________(420) by callers looking for pornography. (e)Inundated
Apparently, the state apparatus has no immediate answers to offer
for a social phenomenon growing at a wild pace. Directions (421-425): There are two different sentences with
a blank space in each sentence. Choose the word from the
Q414. (a)Benefitting (b)Preaching given options which fits appropriately in both the blanks
(c)Facilitating adding a proper and logical meaning to the sentences.
(d)Anticipating
(e)Abstracting Q421. (I)You thus take a positive view of life and find it ____________.
(II) Solitude can be ______________ even sweet just for
Q415. (a)Engagement (b)Amending cherishing hope of love.
(c)Empathy (d)Inferring
(e)Furthering (a)Tidy (b)Light
(c)Endurable (d)Harsh
Q416. (a)Forced (b)Included (e)Foul
(c)Acknowledged

Facebook Page- https://www.facebook.com/vishalpariharpage Youtube- https://www.youtube.com/vishalparihar


Follow
95 Vishal Sir Telegram Channel- https://t.me/englishbyvishalsirchannel Instagram- https://www.instagram.com/vishalthetrainer
By Vishal sir
Also Important For SBI CLERK, IBPS CLERK, RRB CLERK
& Other Competitive Exams

S413. Ans. (a) Precise means marked by exactness and accuracy of expression
Sol. ‘When I shall get back’ will not be used, instead ‘when I get or detail.
back’ will be used. Hence there is error in part (a) of the sentence.
S419. Ans.(a)
S414. Ans. (b) Sol. Option (a) is the most apt choice. Hampered means hinder or
Sol. Preaching must be the word that must come in the blank as impede the movement or progress of. Here the statement that
educational institutions that deal with gender issues have includes the given word means that education which has the
incorporated equality and modernity only through textbook potential to improve the male minds and their ability to think and
mode. So option (b) is the appropriate choice. behave in a normal and rational manner is hampered and lessened
by the easy access provided by child pornography. All other
S415. Ans. (a) words do not fit contextually.
Sol. Option (a) is the most suitable choice for the blank.
Engagement is the word to be filled because the line refers about S420. Ans.(e)
involvement of talks between males and females in a classroom Sol. Option (e) is the most suitable choice to be filled in the blank
which is impossible due to male attitudes towards women. in the given statement. Inundated means overwhelm (someone)
with things or people to be dealt with.
S416. Ans. (c) Endured means remain in existence; last.
Sol. Option (c) is the appropriate choice. Acknowledged is the Wrecked means exhausted or damaged. Both are contextually
word that must fill the blank most suitably. Acknowledged means inappropriate.
admitted or accepted as true, valid, or legitimate. Here the
statement S421. Ans.(c)
means that the terror experienced by the girls at an early stage Sol. Endurable is the word that must come in both the blanks of
cannot be recognized and discussed .Rest all the words are the given statements. Since both the sentences are positive, they
contextually incorrect. cannot take negative words like harsh and foul.
Endurable means bearable.
S417. Ans.(c) Tidy means arranged neatly and in order.
Sol. Option (c) is the appropriate choice to be filled in the blank. Harsh means unpleasantly rough or jarring to the senses.
Disparity means a great difference. Foul means wicked or immoral.
Lessons on gender difference mention prejudices and stereotypes
i.e. a widely held but fixed and oversimplified image or idea of a
particular type of person or thing. Equality means same
consideration. Practicality and order and contextually are wrong
words to use here.

S418. Ans. (d)


Sol. Embedded means fixed firmly and deeply in surroundings.
In the given statement it is the only word that fits most suitably
in the blank. Explicit means stated clearly and in detail, leaving
no room for confusion or doubt.
Specific means clearly defined or identified.

Facebook Page- https://www.facebook.com/vishalpariharpage Youtube- https://www.youtube.com/vishalparihar


Follow
96 Vishal Sir Telegram Channel- https://t.me/englishbyvishalsirchannel Instagram- https://www.instagram.com/vishalthetrainer
By Vishal sir
Also Important For SBI CLERK, IBPS CLERK, RRB CLERK
& Other Competitive Exams

Q422. (I)You will need to drain the central heating system before you Directions (426-430): In the question given below, some parts
__________ the radiator. are given which have to be arranged in a proper sequence.
(II) These measures in no way _____________the need for Select the option which best defines the proper sequence and
regular safety checks. arranges the parts to form a meaningful sentence.
(a)Replace
(b)Refund Q426. primary heat transport system(A)/normally the cooling (B) /of the
(c)Drain nuclear fuel (C) /is done by the (D) /heavy water of the (E)/
(d)Disappear (a)ABCDE
(e)Hamper (b)BCDEA
(c)CABDE
Q423. (I)The exact nature of black holes continues to _________ (d)DBEAC
scientists. (e)ACEDB
(II)The exam questions were made so as to ____________ you
completely. Q427. size and usage of (A)/ they diversified with spread (B)/ of the
(a)Explain grasslands during(C)/ the mid-Pliocene epoch only(D)/kangaroo
(b)Baffle teeth revealed that (E)/
(c)Relieve (a)DACBE
(d)Indicate (b)ABCDE
(e)Charge (c)BACDE
(d)AEBCD
Q424. (I) A good _____________ story is a battle of wits between (e)EBDAC
author and reader.
(II) He was a man of ____________ - erased from the history Q428. methods found a response to questions of (A)/ biological systems
books. using computational (B)/some university researchers (C)/ why
(a)Gloss life exists, diversifies and fills the earth (D)/ studying properties
(b)Repulsion of complex (E)/
(c)Account (a)BACED
(d)Mystery (b)CEBAD
(e)Understanding (c)DECAB
(d)EBCAD
Q425. (I) Circumstances are the rulers of the weak, instruments of the (e)BACDE
__________.
(II) A fool can ask more questions than seven ___________ men Q429. in rates in the near future (A)/ no possibility of a cut
can answer. (B)/,indicating that there was(C)/ its stance from neutral to
(a)Victim calibrated tightening (D) /the central bank changed (E)/
(b)Wise (a)DEABC
(c)Eminent (b)CDBEA
(d)Servile (c)BADCE
(e)Gentle (d)ABCDE
(e)EDCBA

Facebook Page- https://www.facebook.com/vishalpariharpage Youtube- https://www.youtube.com/vishalparihar


Follow
97 Vishal Sir Telegram Channel- https://t.me/englishbyvishalsirchannel Instagram- https://www.instagram.com/vishalthetrainer
By Vishal sir
Also Important For SBI CLERK, IBPS CLERK, RRB CLERK
& Other Competitive Exams

S422. Ans. (a) Sol. Option (b) is the most appropriate choice. The correct
Sol. Replace is the most appropriate word to be filled in both of sequence of the statement is BCDEA. The sentence will be as
the given blanks. follows : “Normally the cooling of the nuclear fuel is done by the
Replace will be used as it means taking the place of. heavy water of the primary heat transport system.”
Refund is not apt as neither of the statement talks about the
money. S427. Ans.(d)
Drain means deprive of strength or vitality. Sol. Option (d) is the most suitable choice which provides the
Hamper means hinder or impede the movement or progress of. correct arrangement of the sentence. The correct sequence of the
Both drain and hamper are contextually wrong to use. statement is AEBCD. The above sentence is as follows: “Size and
usage of kangaroo teeth revealed that they diversified with spread
S423. Ans.(b) of the grasslands during the mid-Pliocene epoch only.”
Sol. Baffle is the most appropriate word to be filled in both the
blanks of the statements. S428. Ans.(b)
Baffle means to confuse, perplex. Sol. Option (b) is the most appropriate choice which gives the
In both of the given statements confusion is referred that the black best arrangement for the statement. The sentence will be arranged
holes confuse scientists in statement 1. While in statement 2 , the as follows: “Some university researchers studying properties of
exam questions were made to confuse some person completely. complex biological systems using computational methods found
a response to questions of why life exists, diversifies and fills the
S424. Ans.(d) earth.”
Sol. Mystery is the most suitable word to be used in both of the
blanks of the statements. Mystery means something that is S429. Ans.(e)
difficult or impossible to understand or explain. Sol. Option (e) is the most apt choice which arranges the sentence
Gloss means try to conceal or disguise (something unfavorable) in a proper way. The sentence is as follows: “The central bank
by treating it briefly or representing it misleadingly. changed its stance from neutral to calibrated tightening,
Repulsion means a feeling of intense distaste or disgust. indicating that there was no possibility of a cut in rates in the near
Account means a report or description of an event or experience. future.”

S425. Ans.(b)
Sol. Option (b) is the most relevant answer in reference to both
the statements. This is so because both the statements refer about
the men which can be wise because a comparison has to be used
in the sentences by referring to the weak and the wise and then a
fool and wise.
Victim is someone who is tricked or duped.
Eminent means famous and respected within a particular sphere.
Servile means referring to slave.
Gentle means moderate in action, effect, or degree; not strong or
violent.

S426. Ans.(b)

Facebook Page- https://www.facebook.com/vishalpariharpage Youtube- https://www.youtube.com/vishalparihar


Follow
98 Vishal Sir Telegram Channel- https://t.me/englishbyvishalsirchannel Instagram- https://www.instagram.com/vishalthetrainer
By Vishal sir
Also Important For SBI CLERK, IBPS CLERK, RRB CLERK
& Other Competitive Exams

peace or truce. This idea is a modern myth because the Greeks


Q430. our personal beliefs and tastes implying (A) / does not mean we never suspended their wars. The truce did allow those religious
give up who we are (B)/ that we constantly evaluate these pilgrims who were travelling to Olympia to pass through warring
beliefs(C)/ belonging to this community (D)/ on the touchstone territories unmolested because they were protected by Zeus.
of constitutional morality (E) According to the earliest records, only one athletic event was held
(a)ACBDE (b)ACBED (c)DBACE in the ancient Olympics — a footrace of about 183 m (200 yd),
(d)DABEC (e)EADBC or the length of the stadium. A cook, Coroibus of Elis, was the
Direction (431-440): Read the following passage carefully and first recorded winner. The first few Olympics had only local
answer the questions given below it. Certain words are given appeal and were limited to one race on one day; only men were
in bold to help you locate them while answering some of the allowed to compete or attend. A second race — twice the length
questions. of the stadium — was added in the 14th Olympics, and a still
longer race was added to the next competition, four years later.
The Olympic Games are an international sports festival that When the powerful, warlike Spartans began to compete, they
began in ancient Greece. The original Greek games were staged influenced the agenda. The 18th Olympiad included wrestling
every fourth year for several hundred years, until they were and a pentathlon consisting of running, jumping, spear throwing
abolished in the early Christian era. The revival of the Olympic (the javelin), discus throwing, and wrestling. Boxing was added
Games took place in 1896, and since then they have been staged at the 23rd Olympiad, and the Games continued to expand, with
every fourth year, except during World War I and World War II the addition of chariot racing and other sports. In the 37th
(1916, 1940, 1944). Olympiad (632 B.C.) the format was extended to five days of
competition. The growth of the Games fostered
Perhaps the basic difference between the ancient and modern "professionalism" among the competitors, and the Olympic ideals
Olympics is that the former was the ancient Greeks' way of waned as royalty began to compete for personal gain, particularly
saluting their gods, whereas the modern Games are a manner of in the chariot events.
saluting the athletic talents of citizens of all nations. The origin The ideology of nationalism, which swept the world during the
of the Olympics is shrouded in mystery and legend; one of the early 20th century, left its mark on the Olympics. Athletic
most popular myths identifies Heracles and his father Zeus as the nationalism was brought to a peak by Nazi Germany, which
progenitors of the Games. According to legend, it was Heracles staged the 1936 Games in Berlin and used the Olympics to
who first called the Games "Olympic" and established the custom propagandize its cause. Hitler saw the Games as an opportunity
of holding them every four years. The myth continues that after to promote his government and ideals of racial supremacy and the
Heracles completed his twelve labours, he built the Olympic official Nazi party paper, the Völkischer Beobachter, wrote in the
Stadium as an honour to Zeus. The original Olympics featured strongest terms that Jews should not be allowed to participate in
competition in music, oratory, and theater performances as well. the Games. When threatened with a boycott of the Games by
It is certain that during the midsummer of 776 B.C. a festival was other nations, Hitler appeared to allow athletes of other ethnicities
held at Olympia on the highly civilized eastern coast of the from other countries to participate. However, German Jewish
Peloponnesian peninsula. That festival remained a regularly athletes were barred or prevented from taking part by a variety of
scheduled event, taking place during the pre-Christian golden age methods and Jewish athletes from other countries (notably the
of Greece. As a testimony to the religious nature of the Games US) seem to have been side-lined in order not to offend the Nazi
(which were held in honor of Zeus, the most important god in the regime. The Germans built a powerful team through nationalized
ancient Greek pantheon), all wars would cease during the training and scientific advances and dominated the Games in
contests. This cessation of hostilities was known as the Olympic terms of medals won. The political overtones of the Olympics did

Facebook Page- https://www.facebook.com/vishalpariharpage Youtube- https://www.youtube.com/vishalparihar


Follow
99 Vishal Sir Telegram Channel- https://t.me/englishbyvishalsirchannel Instagram- https://www.instagram.com/vishalthetrainer
By Vishal sir
Also Important For SBI CLERK, IBPS CLERK, RRB CLERK
& Other Competitive Exams

not lessen with the fall of Nazi Germany. In 1956, Egypt, Iraq, (b)The growth of the Games fostered "nationalism" among the
and Lebanon boycotted the Melbourne Games to protest the competitors.
Anglo-French seizure of the Suez Canal, and the Netherlands, (c)In the 73rd Olympiad (632 B.C.) the format was extended to
Spain, and Switzerland boycotted as well to protest the USSR's five days of competition.
invasion of Hungary. In Mexico City in 1968, two African (d)The first few Olympics had only local appeal and were limited
American runners used the victory pedestal to protest U.S. racial to one race on one day; only men were allowed to compete or
policies. In the Munich Olympics in 1972, 11 Israeli athletes were attend.
massacred by Palestinian terrorists. And in 1976 in Montreal, 33 (e) None of these.
African nations, to be represented by about 400 athletes,
boycotted the Games to protest South Africa's apartheid policies. Q434. What was the reason that all wars used to cease during the
Olympics?
Q431. According to the passage how the ancient Olympic Games differ (a)As the powerful warlike Spartans started taking part in the
from the modern ones? competition
(a) The ancient Greek games were staged every fourth year (b)The As all the men used to take part in the competition so none
whereas the modern one takes place at every 5th year. was left there to lead the war. (c)Because of their religious nature
(b)The ancient Greek games as compared to the modern ones had as the games used to held in honor of Zeus, the most important
large number of participants. god in the ancient Greek pantheon
(c)The former was the ancient Greeks' way of saluting their gods, (d)Both (a) and (b)
whereas the modern Games are a manner of saluting the athletic (e) None of these.
talents of all nations.
(d)The ancient Greek games as compared to the modern ones had Q435. Why Egypt, Iraq, and Lebanon had to boycott the 1956
participants from only western countries. Melbourne Olympics?
(e) None of these. (a)To protest the Anglo-French seizure of the Suez Canal
(b)Because of the war with Turkey and Greece
Q432. Which of the following options is false according to the passage? (c)United Kingdom, and France had seized the waterways of
(a)The Olympic Games are an international sports festival that Mediterranean sea
began in ancient Greece. (d)To protest the USSR's invasion of Hungary
(b)It is certain that after Heracles completed his twelve labours, (e) None of these.
he built the Olympic Stadium as an honour to Zeus.
(c)As a testimony to the religious nature of the Games all wars Q436. What was the main motive of Hitler behind the staging of 1936
would cease during the contests. Games in Berlin?
(d)Nazi Germany staged the 1936 Games in Berlin and used the (a)Hitler wanted to promote Athletic nationalism in Germany
Olympics to propagandize its cause. (b)He wanted Jews to participate in the Games.
(e) None of these. (c)He wanted the Germany to build a powerful team through
nationalized training and scientific advances and dominate in the
Q433. Which of the following options is true according to the passage? Games.
(a)The Germans built a powerful team through nationalized (d) Hitler saw the Games as an opportunity to promote his
training and scientific advances and dominated the Games in government and ideals of racial supremacy
terms of number of participants. (e) None of these.

Facebook Page- https://www.facebook.com/vishalpariharpage Youtube- https://www.youtube.com/vishalparihar


Follow
100 Vishal Sir Telegram Channel- https://t.me/englishbyvishalsirchannel Instagram- https://www.instagram.com/vishalthetrainer
By Vishal sir
Also Important For SBI CLERK, IBPS CLERK, RRB CLERK
& Other Competitive Exams

S430. Ans.(c) important god in the ancient Greek pantheon), all wars would
Sol. Option (c) is the most suitable choice. The proper arranged cease during the contests. This cessation of hostilities was known
sentence will be as follows: “Belonging to this community does as the Olympic peace or truce.”
not mean we give up who we are, our personal beliefs and tastes
implying that we constantly evaluate these beliefs on the S435. Ans. (a)
touchstone of constitutional morality.” Sol. The correct answer choice is option (a). Option (b) is totally
out of context as there is no mention of war with Turkey or
S431. Ans. (c) Greece. Option (a) can be traced from the last paragraph of the
Sol. The correct answer choice is option (c). Option (c) and (d) passage where it is given as “In 1956, Egypt, Iraq, and Lebanon
are totally out of context as there is no mention of either number boycotted the Melbourne Games to protest the Anglo-French
of participants or participant countries. Option (c) can be traced seizure of the Suez Canal.”
from the 2nd paragraph of the passage where it is given as
“Perhaps the basic difference between the ancient and modern S436. Ans. (d)
Olympics is that the former was the ancient Greeks' way of Sol. The correct answer choice is option (d). Option (a) and (b)
saluting their gods, whereas the modern Games are a manner of are totally different from the facts given in the paragraph. Option
saluting the athletic talents of citizens of all nations.” (d) can be traced from the starting lines of last paragraph of the
passage where it is given as “Athletic nationalism was brought to
S432. Ans. (b) a peak by Nazi Germany, which staged the 1936 Games in Berlin
Sol. The correct answer choice is option (b) as all the given and used the Olympics to propagandize its cause. Hitler saw the
options are true except option (b). Passage has mentioned that Games as an opportunity to promote his government and ideals
“The myth continues that after Heracles completed his twelve of racial supremacy”
labours, he built the Olympic Stadium as an honour to Zeus.” The
given option (b) is a myth as per the given passage but here in
option it is given as a certain fact thus option (b) stands as the
false statement.

S433. Ans. (d)


Sol. The correct answer choice is option (d) as all the given
options are false except option (d). Passage has mentioned that
“The first few Olympics had only local appeal and were limited
to one race on one day; only men were allowed to compete or
attend. A second race twice the length of the stadium was added
in the 14th Olympics, and a still longer race was added to the next
competition, four years later.”

S434. Ans. (c)


Sol. The correct answer choice is option (c). Option (b) is totally
out of context. Option (c) can be traced from the 3rd paragraph of
the passage where it is given as “As a testimony to the religious
nature of the Games (which were held in honor of Zeus, the most

Facebook Page- https://www.facebook.com/vishalpariharpage Youtube- https://www.youtube.com/vishalparihar


Follow
101 Vishal Sir Telegram Channel- https://t.me/englishbyvishalsirchannel Instagram- https://www.instagram.com/vishalthetrainer
By Vishal sir
Also Important For SBI CLERK, IBPS CLERK, RRB CLERK
& Other Competitive Exams

Directions (437-438): Choose the word which is most nearly alternative among the five options given below each question.
the SAME in meaning as the word printed in bold as used in If all the sentences are correct, choose (e) as your answer.
the passage.
Q441. (i) The fall in the Indian rupee is the single largest factor affecting
Q437. Offend foreign flows in the Indian market.
(a)Please (ii) Alphonso exporters said the tag will help establish a separate
(b)Delight market for the fruit in future.
(c)Displease (iii) The increase in MSP for Rabi crops comes just ahead of the
(d)Obscure RBI monetary policy announcement.
(e)Omen
(a)Only (i) is correct
Q438 .Massacred (b)Only (iii) is correct
(a)Foster (c)Both (i) and (ii) are correct
(b)Nurture (d)None is correct
(c)Inborn (e)All are correct
(d)Slaughter
(e)Nourish Q442. (i) Cardamom is grown by small farmers and it is a plant that
cannot take water stagnation.
Directions (439-440): Choose the word which is the (ii) There is an increase of aspirations among the community
OPPOSITE in meaning as the word printed in bold as used practising shifting cultivation.
in the passage. (iii)A failure to put the right person at the right place could prove
expansion for the organization.
Q439. Cease
(a)Terminate (a)Only (i) is correct
(b)End (b)Only (iii) is correct
(c)Halt (c)Both (i) and (ii) are correct
(d)Stop (d)None is correct
(e)Start (e)All are correct

Q440. Hostilities Q443. (i)Jesse had mockingly pointing out all of Nancy’s faults in front
(a)Bitterness of everyone, publically degrading the poor girl.
(b)Spite (ii)Test day can be one of pure apprehension as many students
(c)Unkindness worried about their test scores.
(d)Friendliness (iii) In the sciences, even questionable examples of research fraud
(e)Malevolence are harshly punished.
(a)Only (i) is correct
Directions (441-445): There are three sentences given in each (b)Only (iii) is correct
question. Find the sentence(s) which is/are grammatically (c)Both (i) and (ii) are correct
correct and mark your answer choosing the best possible (d)None is correct
(e)All are correct

Facebook Page- https://www.facebook.com/vishalpariharpage Youtube- https://www.youtube.com/vishalparihar


Follow
102 Vishal Sir Telegram Channel- https://t.me/englishbyvishalsirchannel Instagram- https://www.instagram.com/vishalthetrainer
By Vishal sir
Also Important For SBI CLERK, IBPS CLERK, RRB CLERK
& Other Competitive Exams

S437. Ans. (c) “Test day can be one of pure apprehension as many students
Sol. The correct answer choice is option (c). worry about their test scores” Would be the correct structure for
Obscure- not clearly expressed or easily understood the given sentence.
Omen- an event regarded as a portent of good or evil “Jesse had mockingly pointing out all of Nancy’s faults in front
of everyone, publically degrading the poor girl” Would be the
S438. Ans. (d) correct structure for the given sentence.
Sol. The correct answer choice is option (d).
Massacred- deliberately and brutally kill (many people)
Foster-encourage the development of (something, especially
something desirable)
Inborn- existing from birth

S439. Ans. (e)


Sol. The correct answer choice is option (e).
Cease- come or bring to an end

S440. Ans. (d)


Sol. The correct answer choice is option (d).
Hostilities- the quality of being friendly; affability
Spite- hostile behaviour; unfriendliness or opposition
Malevolence- the state or condition of being malevolent; hostility

S441. Ans. (e)


Sol. All the given three sentences are grammatically correct. Thus
the best answer choice is option (e).

S442. Ans. (a)


Sol. Best answer choice is option (a) as both the given sentences
(ii) and (iii) are grammatically incorrect.
“There is an increase of aspirations among the communities
practising shifting cultivation” would be the right structure for the
given (ii) sentence.
“A failure to put the right person at the right place could prove
expansive for the organization.” Would be the correct structure
for the given (iii) sentence.

S443. Ans. (b)


Sol. Best answer choice is option (b) as both the given sentences
(i) and (ii) are grammatically incorrect.

Facebook Page- https://www.facebook.com/vishalpariharpage Youtube- https://www.youtube.com/vishalparihar


Follow
103 Vishal Sir Telegram Channel- https://t.me/englishbyvishalsirchannel Instagram- https://www.instagram.com/vishalthetrainer
By Vishal sir
Also Important For SBI CLERK, IBPS CLERK, RRB CLERK
& Other Competitive Exams

Q444. (i)Irish cuisine makes great used of potatoes, but ironically, the (b)Resurrected
potato is not endemic to Ireland. (c)Remembered
(ii)After the visit team scored nine times, the home team’s fans (d)Retreat
were disheartened, some leaving the game early. (e)Revolved
(iii)Many first time skydivers say that describing the acting of
falling from the sky is elusive. Q448. Prerna couldn’t ___________between a good wine and a bad
(a)Only (i) is correct wine, so she avoided wine tastings.
(b)Only (iii) is correct (a)Detrimental
(c)Both (i) and (ii) are correct (b)Determination
(d)None is correct (c)Discriminate
(e)All are correct (d)Realized
(e)Fight
Q445. (i)Martin Luther King Jr. was a tireless advocate for the rights of
African-Americans in the United States. Q449. Himanshu preferred flavorless and mild beers, but Bhavesh
(ii)My little brother is so malleable that I can convince him to preferred a beer with more _________flavor.
sneak cookies from the cupboard for me. (a)Proposal
(iii)Since the police sergeant had to train the pair of new hires, (b)Productive
progress in his own case was impede. (c)Ceased
(a)Only (i) is correct (d)Robust
(b)Only (iii) is correct (e)Converse
(c)Both (i) and (ii) are correct
(d)None is correct Q450. There is an ___________difference between those who say they
(e)All are correct can get the job done and those who actually get the job done.
(a)Apprehend
Directions (446-450): Each question below has one blank, (b)Alter
which is indicating that something has been omitted. Find out (c)Considerate
which option can be used to fill up the blank in the sentence (d)Attribute
in the same sequence to make it meaningfully complete. (e)Appreciable

Q446. The sand bags placed on the river formed an __________barrier, Directions (451-455): In the question given below, some
protecting the town from flooding. sentences/phrases are given which have to be arranged in a
(a)Treacherous proper sequence. Select the option which best defines the
(b)Impermeable proper sequence and arranges the sentence in an appropriate
(c)Impertinent way.
(d) Improper
(e)Exhaustive Q451. forever and life was (A)/ not transitory (B)/ If we lived (C)/
appreciate life less or more (D)/ do you think we could (E)/
Q447. After much thought, Ted _________not to travel abroad this (a)BCAED (b)CABED
summer because he didn’t have much money in his account (c)EDCAB (d)DCAEB
(a)Resolved (e)CAEBD

Facebook Page- https://www.facebook.com/vishalpariharpage Youtube- https://www.youtube.com/vishalparihar


Follow
104 Vishal Sir Telegram Channel- https://t.me/englishbyvishalsirchannel Instagram- https://www.instagram.com/vishalthetrainer
By Vishal sir
Also Important For SBI CLERK, IBPS CLERK, RRB CLERK
& Other Competitive Exams

S444. Ans. (d) S449. Ans. (d)


Sol. Best answer choice is option (d) as none of the given Sol. The most appropriate option to be filled in the blank is option
sentences is grammatically incorrect. (d).
“Irish cuisine makes great use of potatoes, but ironically, the Robust- strong and healthy; vigorous
potato is not endemic to Ireland” Would be the correct structure Converse- engage in conversation
for the given sentence.
“After the visiting team scored nine times, the home team’s fans S450. Ans. (e)
were disheartened, some leaving the game early” Would be the Sol. The most appropriate option to be filled in the blank is option
correct structure for the given sentence. (e).
“Many first time skydivers say that describing the act of falling Appreciable- large enough to be noticed
from the sky is elusive” Would be the correct structure for the Apprehend- arrest (someone) for a crime
given sentence. Considerate- careful not to inconvenience or harm others
Attribute- regard something as being caused by
S445. Ans. (c)
Sol. Best answer choice is option (c) as both the given sentences S451. Ans. (b)
(i) and (ii) are grammatically correct. Sol. The correct sequence is CABED as it arranges the sentence
“Since the police sergeant had to train the pair of new hires, in the best way as follows: “If we lived forever and life was not
progress in his own case was impeded” Would be the correct transitory, do you think we could appreciate life less or more?”
structure for the given sentence.

S446. Ans. (b)


Sol. The most appropriate option to be filled in the blank is option
(b).
Impermeable- not allowing fluid to pass through
Impertinent - not showing proper respect; rude
Treacherous- guilty of or involving betrayal or deception

S447. Ans. (a)


Sol. The most appropriate option to be filled in the blank is option
(a).
Retreat- move back or withdraw
Resolved- firmly determined to do something
Resurrected- restore (a dead person) to life

S448. Ans. (c)


Sol. The most appropriate option to be filled in the blank is option
(c).
Discriminate- recognize or perceive the difference
Detrimental- tending to cause harm

Facebook Page- https://www.facebook.com/vishalpariharpage Youtube- https://www.youtube.com/vishalparihar


Follow
105 Vishal Sir Telegram Channel- https://t.me/englishbyvishalsirchannel Instagram- https://www.instagram.com/vishalthetrainer
By Vishal sir
Also Important For SBI CLERK, IBPS CLERK, RRB CLERK
& Other Competitive Exams

Q452. augmented menu will (A)/ the restaurant’s (B)/ and increase its correction required mark (e) i.e. 'No correction required' as
profits (C)/ Ideally (D)/ expand its customers (E)/ the answer.
(a)DBAEC
(b)BAECD Q456. While some people are good for converting opportunities into
(c)ABEDC desirable outcomes, others are not.
(d)ECDAB (a) are good at converting opportunities
(e)CBAED (b) is good at converting opportunities
(c) behaving good when converting opportunities
Q453. for income tax evasion (A)/ in many illicit activities (B) / a (d) are goods in converting opportunities
relatively minor offence (C)/ he was finally arrested (D) /though (e) No correction required
he was engaged (E)/
(a)EBCDA Q457. I could barely seeing the site since it was dark.
(b)EBDAC (a) barely seen the site
(c)EBACD (b) barely see the site
(d)ACDBE (c) barely see site
(e)ABCED (d) barely the site
(e) No correction required
Q454. but processed sugar in (A)/ large quantities is equally bad (B)/
smoking is detrimental (C)/ many know that (D)/ to your health Q458. I had been not expected to meet him it was quite an accidental
(E) meeting.
(a)CABDE (a) I had no expected to
(b)EDCBA (b) I had no expectation to
(c)BECAD (c) I had not expected to
(d)DCEAB (d) He had not expected to
(e)ADBCE (e) No correction required

Q455. pertinent to the question of(A)/such monetary figures are Q459. Two years been passed since my father died.
not(B)/while the salaries of the players(C)/might draw attention (a) years are passed
in the media(D)/who plays the best on the field(E) (b) years were passed
(a)DCEAB (c) years had passed
(b)EABCD (d) years have passed
(c)BADEC (e) No correction required
(d)CDBAE
(e)ABECD Q460. The police have registered a complaint against four persons
and have arrested two of them.
Directions (456 – 460): Which of the phrases (a), (b), (c) and (a) have received a complaint against
(d) given below should replace the phrase given in bold in the (b) have lodged a complaint against
following sentences to make the sentence grammatically (c) have followed a complaint against
correct? If the sentence is correct as it is and there is no (d) has registered a complaint against
(e) No correction required

Facebook Page- https://www.facebook.com/vishalpariharpage Youtube- https://www.youtube.com/vishalparihar


Follow
106 Vishal Sir Telegram Channel- https://t.me/englishbyvishalsirchannel Instagram- https://www.instagram.com/vishalthetrainer
By Vishal sir
Also Important For SBI CLERK, IBPS CLERK, RRB CLERK
& Other Competitive Exams

Sol. Option (c) provides the correct choice for making the
S452. Ans. (a) sentence grammatically correct and contextually meaningful.
Sol. The correct sequence is DBAEC as it arranges the sentence The correct sentence structure should be “I had not expected to
properly in the way as follows: “Ideally, the restaurant’s meet him it was quite an accidental meeting.”
augmented menu will expand its customers and increase its
profits.” S459. Ans. (d)
Sol. Option (d) provides the correct choice for making the
S453. Ans. (b) sentence grammatically correct and contextually meaningful.
Sol. The correct sequence of the statement is EBDAC as it The correct sentence structure should be “Two years have passed
arranges the sentence as follows: “Though he was engaged in since my father died.”
many illicit activities, he was finally arrested for income tax
evasion, a relatively minor offence.”
S460. Ans. (e)
S454. Ans. (d) Sol. Option (e) provides the correct choice as an answer as there
Sol. Option (d) provides the correct sequence for the arrangement is no correction required in the given sentence.
of the sentence. The proper arranged sentence is as follows:
“Many know that smoking is detrimental to your health, but
processed sugar in large quantities is equally bad.”

S455. Ans. (d)


Sol. The correct sequence is CDBAE as the sentence can be
arranged as follows: “While the salaries of the players might draw
attention in the media, such monetary figures are not pertinent to
the question of who plays the best on the field.”

S456. Ans. (a)


Sol. Option (a) provides the correct choice for making the
sentence grammatically correct and contextually meaningful.
It should be you are good ‘at’, but weak ‘in’ doing something.
Good "in" used when the phrase is followed by verb, whereas
Good 'at' is used when the phrase is followed by a noun. E.g. - A
person is good in writing poems, A person is good at poetry.

S457. Ans. (b)


Sol. Option (b) provides the correct choice for making the
sentence grammatically correct and contextually meaningful.
The correct sentence structure should be “I could barely see the
site since it was dark.”

S458. Ans. (c)

Facebook Page- https://www.facebook.com/vishalpariharpage Youtube- https://www.youtube.com/vishalparihar


Follow
107 Vishal Sir Telegram Channel- https://t.me/englishbyvishalsirchannel Instagram- https://www.instagram.com/vishalthetrainer
By Vishal sir
Also Important For SBI CLERK, IBPS CLERK, RRB CLERK
& Other Competitive Exams

Direction (461-470): Read the following passage carefully and around the world already have renewable energy contributing
answer the questions given below it. Certain words are given more than 20 percent of energy supply. National renewable
in bold to help you locate them while answering some of the energy markets are projected to continue to grow strongly in the
questions coming decade and beyond. Some places and at least two
countries, Iceland and Norway generate all their electricity using
Renewable energy is energy that is congregated from renewable renewable energy already, and many other countries have the set
resources, which are naturally replenished on a human timescale, a goal to reach 100% renewable energy in the future. For
such as sunlight, wind, rain, tides, waves, and geothermal heat. example, in Denmark the government decided to switch the total
Renewable energy often provides energy in four important areas: energy supply (electricity, mobility and heating/cooling) to 100%
electricity generation, air and water heating/cooling, renewable energy by 2050.
transportation, and rural (off-grid) energy services.
While many renewable energy projects are large-scale, renewable
Based on REN21's 2017 report, renewables contributed 19.3% to technologies are also suited to rural and remote areas and
humans' global energy consumption and 24.5% to their developing countries, where energy is often crucial in human
generation of electricity in 2015 and 2016, respectively. This development. Former United Nations Secretary-General Ban Ki-
energy consumption is divided as 8.9% coming from traditional moon has said that renewable energy has the ability to lift the
biomass, 4.2% as heat energy (modern biomass, geothermal and poorest nations to new levels of prosperity. As most of
solar heat), 3.9% hydroelectricity and 2.2% is electricity from renewables provide electricity, renewable energy deployment is
wind, solar, geothermal, and biomass. Worldwide investments in often applied in conjunction with further electrification, which
renewable technologies constituted to more than US$286 billion has several benefits: Electricity can be converted to heat (where
in 2015, with countries such as China and the United States necessary generating higher temperatures than fossil fuels), can
heavily investing in wind, hydro, solar and biofuels. Globally, be converted into mechanical energy with high efficiency and is
there are an estimated 7.7 million jobs associated with the clean at the point of consumption. In addition to that
renewable energy industries, with solar photovoltaics being the electrification with renewable energy is much more efficient and
largest renewable employer. As of 2015 worldwide, more than therefore leads to a significant reduction in primary energy
half of all new electricity capacity installed was renewable. requirements; because most renewables don't have a steam cycle
with high losses (fossil power plants usually have losses of 40 to
Renewable energy resources prevail over wide geographical 65%). Renewable energy systems are rapidly becoming more
areas, in contrast to other energy sources, which are concentrated efficient and cheaper. Their share of total energy consumption is
in a limited number of countries. Rapid deployment of renewable increasing. Growth in consumption of coal and oil could end by
energy and energy efficiency is resulting in significant energy 2020 due to increased uptake of renewables and natural gas.
security, climate change mitigation, and economic benefits. The
results of a recent review of the literature deduced that as Renewable energy resources and significant opportunities for
greenhouse gas (GHG) emitters begin to be held liable for energy efficiency exist over wide geographical areas, in contrast
damages resulting from GHG emissions resulting in climate to other energy sources. Rapid deployment of renewable energy
change, a high value for liability mitigation would provide and energy efficiency, and technological diversification of energy
powerful incentives for deployment of renewable energy sources, would result in significant energy security and economic
technologies. In international public opinion surveys there is benefits. It would also reduce environmental pollution such as air
strong support for promoting renewable sources such as solar pollution caused by burning of fossil fuels and improve public
power and wind power. At the national level, at least 30 nations health, reduce premature mortalities due to pollution and save

Facebook Page- https://www.facebook.com/vishalpariharpage Youtube- https://www.youtube.com/vishalparihar


Follow
108 Vishal Sir Telegram Channel- https://t.me/englishbyvishalsirchannel Instagram- https://www.instagram.com/vishalthetrainer
By Vishal sir
Also Important For SBI CLERK, IBPS CLERK, RRB CLERK
& Other Competitive Exams

associated health costs that amount to several hundred billion


dollars annually only in the United States. Renewable energy Q464. Who has said that “renewable energy has the ability to lift the
sources, that derive their energy from the sun, either directly or poorest nations to new levels of prosperity”?
indirectly, such as hydro and wind, are expected to be capable of (a)The present United Nations Secretary-General António
supplying humanity energy for almost another 1 billion years, at Guterres
which point the predicted increase in heat from the sun is (b)Former United Nations Secretary-General Ban Ki-moon
expected to make the surface of the earth too hot for liquid water (c)UNEP head Erik Solheim
to exist. (d)A Norwegian socialist politician Kristin Halvorsen
(e) None of these
Q461. According to the passage what are the important areas where
Renewables contribute in energy? Q465. What is/are the advantage/s of electrification with renewable
(a)Electricity generation energy?
(b)Transportation (a)Electricity can be converted to heat generating higher
(c)Off-grid energy services temperatures than fossil fuels
(d)Transportation (b)Electricity can be converted into mechanical energy with high
(e)All of these efficiency
(c)Electricity can be converted into chemical energy with high
Q462. Which of the following sentence (s) is not true in the context to efficiency
the passage? (d)Both (a) and (b)
(a)Renewable energy is energy that is collected from renewable (e) None of these.
resources, which are artificially replenished on a human
timescale. Q466. According to the passage what could be the favorable outcomes
(b)There are an estimated 7.7 million jobs associated with the of the Rapid deployment of renewable energy and energy
renewable energy industries. efficiency?
(c)Worldwide investments in renewable technologies amounted (a)Reduction in environmental pollution such as air pollution
to less than US$286 billion in 2015. caused by burning of fossil fuels
(d)Both (a) and (c) (b)Reduction in premature mortalities due to pollution
(e) None of these. (c)Would result in significant energy security
(d)All (a) (b) and (c)
Q463. As mentioned in the passage how Renewable energy resources (e) None of these.
are different from the other energy resources?
(a)Renewable energy resources are less efficient than other Directions (467-468): Find out the word from the given
energy resources. options which is similar in meaning to the word given in bold
(b)Renewable energy resources are easy to find whereas there is in the passage.
scarcity of other resources.
(c)Renewable energy resources exist over large geographical Q467. Congregated
areas, whereas the other energy sources are concentrated in a (a)Segregated (b)Collected
limited number of countries. (c)Dispersed
(d)Both (a) and (c) (d)Isolated
(e) None of these. (e)Disassociated

Facebook Page- https://www.facebook.com/vishalpariharpage Youtube- https://www.youtube.com/vishalparihar


Follow
109 Vishal Sir Telegram Channel- https://t.me/englishbyvishalsirchannel Instagram- https://www.instagram.com/vishalthetrainer
By Vishal sir
Also Important For SBI CLERK, IBPS CLERK, RRB CLERK
& Other Competitive Exams

S461. Ans. (e) caused by burning of fossil fuels and improve public health,
Sol. Option (e) can be traced from the very 1st paragraph of the reduce premature mortalities due to pollution”
passage where it is given as “Renewable energy often provides
energy in four important areas: electricity generation, air and S467. Ans. (b)
water heating/cooling, transportation, and rural (off-grid) energy Sol. Option (b) is the correct answer choice.
services.” Congregated- gather into a crowd or mass
Segregated means set apart from the rest or from each other;
S462. Ans. (d) isolate or divide.
Sol. Option (d) is the correct answer choice. Replace ‘artificially’ Dispersed- distribute or spread over a wide area
by ‘naturally’ and ‘less’ by ‘more’ to make options (a) and (c) Isolated means far away from other places, buildings, or people;
true in context to the passage respectively. remote
Disassociated means disconnected.
S463. Ans. (c)
Sol. Option (c) is the correct answer choice. Option (c) can be
traced from the very first line of the 3rd paragraph.

S464. Ans. (b)


Sol. Option (b) is the correct answer choice. Option (b) can be
traced from the second line of the 4th paragraph where it is given
as “Former United Nations Secretary-General Ban Ki-moon has
said that renewable energy has the ability to lift the poorest
nations to new levels of prosperity.”

S465. Ans. (d)


Sol. Both the Option (a) and (b) are correct. Option (a) and (b)
can be traced from the 4th paragraph where it is given as “As most
of renewables provide electricity, renewable energy deployment
is often applied in conjunction with further electrification, which
has several benefits: Electricity can be converted to heat (where
necessary generating higher temperatures than fossil fuels), can
be converted into mechanical energy with high efficiency and is
clean at the point of consumption.”

S466. Ans. (d)


Sol. Option (d) is the correct answer choice. All the Options (a)
(b) and (c) can be traced from very last paragraph where it is
given as “Rapid deployment of renewable energy and energy
efficiency, and technological diversification of energy sources,
would result in significant energy security and economic benefits.
It would also reduce environmental pollution such as air pollution

Facebook Page- https://www.facebook.com/vishalpariharpage Youtube- https://www.youtube.com/vishalparihar


Follow
110 Vishal Sir Telegram Channel- https://t.me/englishbyvishalsirchannel Instagram- https://www.instagram.com/vishalthetrainer
By Vishal sir
Also Important For SBI CLERK, IBPS CLERK, RRB CLERK
& Other Competitive Exams

Q468. Constituted Q471. While I was (a)/ returning from (b)/ the party someone (c)/
(a)Incapable attacked on me with a knife (d)/ No error (e)

(b)Inept
(c)Comprise Q472. His wife (a)/is more (b)/prettier (c)/than his sister (d)/ No error
(e)
(d)Unorganized
(e)Barbaric
Q473. She regulated (a)/me to (b)/made him learn (c)/how to cook (d)/
No error (e)
Directions (469-470): Find out the word from the given
options which is opposite in meaning to the word given in bold
in the passage. Q474. People have the freedom (a)/ of speech but at the (b)/ same time
each of them (c)/ have to remember their fundamental duties (d)/
No error (e)
Q469. Prevail
(a)Occur
Q475. Being the best (a)/Tennis player he was (b)/invited to play
(b)Extinct (c)/a friendly match on Sunday (d)/ No error (e)
(c)Nourish
(d)Feed Q476. His behavior (a)/is too much (b)/painful (c)/for his sister (d)/ No
(e)Develop error (e)

Q470. Deduced Q477. The class listened (a)/to the teacher (b)/with (c)/carefully (d)/ No
(a)Conclude error (e)

(b)Inferred
(c)Immersed Q478. Since she was not (a)/aware of the effect (b)/she might has
(c)/done that work (d)/ No error (e)
(d)Commence
(e)Rooted
Q479. No player (a)/in the cricket team (b)/is as bright as (c)/his brother
(d)/ No error (e)
Directions (471-480): Read each sentence to find out whether
there is any grammatical or idiomatic error in it. The error,
if any, will be in one part of the sentence. The number of that Q480. This is the (a)/only one of his (b)/poems that is (c)/worth reading
part is the answer. If there is ‘No error’, the answer is (e). (d)/ No error (e)
(Ignore errors of punctuation, if any.)

Facebook Page- https://www.facebook.com/vishalpariharpage Youtube- https://www.youtube.com/vishalparihar


Follow
111 Vishal Sir Telegram Channel- https://t.me/englishbyvishalsirchannel Instagram- https://www.instagram.com/vishalthetrainer
By Vishal sir
Also Important For SBI CLERK, IBPS CLERK, RRB CLERK
& Other Competitive Exams

S468. Ans. (c) e.g.


Sol. Constituted- be (a part) of a whole “He requested me to did it for him” –incorrect
Inept means having or showing no skill; clumsy. “He requested me to do it for him” –correct
Barbaric means savagely cruel or uncivilized and uncultured.
Hence option (c) is the answer. S474. Ans. (d)
S469. Ans. (b) Sol. The error is in the part (d) of the sentence. ‘have’ should be
Sol. Option (b) is the most suitable choice. replaced by ‘has’ because ‘each of’, ‘either of’, always takes the
Prevail- be widespread or current in a particular area or at a singular verb.
particular time
Extinct- no longer in existence S475. Ans. (e)
Hence, it is the only option which is most nearly opposite to the Sol. The correct answer choice is part (e) of the sentence, as the
given word. sentence is grammatically and contextually correct.
Nourish means provide with the food or other substances
necessary for growth, health, and good condition. S476. Ans. (b)
S470. Ans. (d) Sol. The error is in part (b) of the sentence. ‘too much’ should be
Sol. Option (d) is the most apt choice. replaced by ‘much too’ because rule says:
Deduced- arrive at (a fact or a conclusion) by reasoning; draw as ‘too much + noun’
a logical conclusion. ‘much too + unpleasant adjectives’
Immersed means involve oneself deeply in a particular activity. Here in the given sentence ‘painful’ is an adjective.
Commence - begin
Option (a) and (b) are the synonyms of the given word. S477. Ans. (c)
Sol. The error is in part (c) of the sentence. Use of ‘with’ is
S471. Ans. (d) superfluous here, because ‘carefully’ is an adverb of manner
Sol. The error is in part (d) of the sentence. ‘on’ is incorrectly which should not be followed by a preposition.
used here reason being, when ‘attack’ is used as a verb it should
be followed by an object. S478. Ans. (c)
e.g. “China attacked India in 1962.” Sol. The error is in part (c) of the sentence. ‘has’ should be
replaced by ‘have’ because rule says, modal auxiliary (might)
S472. Ans. (b) always takes first form of the verb after it.
Sol. The error is in part (b) of the sentence. ‘more’ is incorrectly S479. Ans. (a)
used here reason being, ‘more prettier’ is a double comparative, Sol. The error is in part (a) of the sentence. ‘Other’ should be
and in a sentence double comparative or double superlative can’t followed by ‘No’, here comparison is between two people of
be used. same group, as rule says
e.g. “he is wiser than you” is a correct whereas “he is more wiser “No other + noun (singular) + ……. So/as + adjective(positive)
than you” is wrong. + as …..”
S480. Ans. (e)
S473. Ans. (c) Sol. The correct answer choice is the option (e) because the
Sol. The error is in part (c) of the sentence. ‘made’ should be sentence is grammatically and contextually correct.
replaced by ‘make’ because when ‘to’ is used as an infinitive then
it always takes the singular form of verb.

Facebook Page- https://www.facebook.com/vishalpariharpage Youtube- https://www.youtube.com/vishalparihar


Follow
112 Vishal Sir Telegram Channel- https://t.me/englishbyvishalsirchannel Instagram- https://www.instagram.com/vishalthetrainer
By Vishal sir
Also Important For SBI CLERK, IBPS CLERK, RRB CLERK
& Other Competitive Exams

Direction (481-490): Given below is a paragraph consisting of (e)Frightening


blanks against each number.
Identify the correct option among the five alternative pairs Q482. (a)Aroused (b)Extended
that perfectly fits into the given blank against the respective (c)Argued (d)Eliminated
number to make the paragraph contextually meaningful and (e)Artificial
grammatically correct. Q483. (a)Disturbance (b)Diligence
Abu Abdullah Muhammad Ibn Battuta was a Moroccan Muslim (c)Distance (d)Mounted
scholar and traveler. He was known for his ___(481)___and (e)Balanced
undertaking excursions called the Rihla. His journeys lasted for a
period of almost thirty years, covering nearly the whole of the Q484. (a)Restored (b)Rejected
known Islamic world and beyond. They ____(482)_____ from (c)Nurtured (d)Experience
North Africa, West Africa, Southern Europe and Eastern Europe (e)Divorced
in the West, to the Middle East, Indian subcontinent, Central
Asia, Southeast Asia and China in the East, a ____(483)____ Q485. (a)Crowned (b)Linked
readily surpassing that of his predecessors. After his travels he (c)Vague (d)Variance
returned to Morocco and gave his account of the ___(484)____ (e)Voyage
to Ibn Juzay. He first began his ___(485)_____ by exploring the
lands of the Middle East. Thereafter he sailed down the Red Sea Q486. (a)Commissioned (b)Abrasion
to Mecca. He crossed the huge Arabian Desert and (c)Propulsion (d)Traveled
____(486)_____ to Iraq and Iran. In 1330, he set of again, down (e)Emulsion
the Red Sea to Aden and then to Tanzania. Then in 1332, Ibn
Battuta decided to visit India. He was ___(487)____ open Q487. (a)Sanctioned (b)Adjourned
heartedly by the Sultan of Delhi. There he was appointed to the (c)Greeted (d)Burdened
position of a judge. He stayed in India for a period of 8 years and (e)Rationed
then left for China. Finally, Battuta returned home to Tangier in
1355. It is ______(488)_____ whether Ibn Battuta visited all the Q488. (a)Answerable (b)Questionable
places that he described. In order to provide a comprehensive (c)Practical (d)Survival
description of places in the Muslim world, Ibn Battuta probably (e)Instructors
_______(489)____ on hearsay evidence and made extensive use
of accounts by earlier travelers. Ibn Battuta reported that he Q489. (a)Relied (b)Recapped
experienced culture shock in some of the regions he visited. (c)Misshaped (d)Retired
Among Turks and Mongols, he was astonished at the way women (e)Renovated
behaved. They were allowed freedom of speech. After the
completion of the Rihla in 1355, little is known about Ibn Q490. (a)Applauded
Battuta’s life. He was ______(490)____ a judge in Morocco and (b)Referred
died in 1368. The Rihla provides an important account of many (c)Attacked
areas of the world in the 14th century. (d)Sanctioned
(e)Appointed
Q481. (a)Traveling (b)Actuating
(c)Folded (d)Daunting

Facebook Page- https://www.facebook.com/vishalpariharpage Youtube- https://www.youtube.com/vishalparihar


Follow
113 Vishal Sir Telegram Channel- https://t.me/englishbyvishalsirchannel Instagram- https://www.instagram.com/vishalthetrainer
By Vishal sir
Also Important For SBI CLERK, IBPS CLERK, RRB CLERK
& Other Competitive Exams

S481. Ans. (a) Sol. Option (d) is the correct answer choice. As the previous line,
Sol. Option (a) is the correct answer choice. As the paragraph is “Thereafter he sailed down the Red Sea to Mecca” suggests the
telling the story of Ibn Battuta who was a Moroccan Muslim idea of the journey through one place to another moreover the
scholar and traveler. As line “His journeys lasted for a period of paragraph is talking about the voyage of Ibn Battuta around the
almost thirty years………” of the paragraph describes that he was world, hence ‘travelled’ is the right answer choice.
a traveller and was known for his travelling hence, the option (a)
is the correct answer choice. S487. Ans. (c)
Sol. Option (c) is the correct answer choice. The line “…..open
S482. Ans. (b) heartedly by the Sultan of Delhi” suggests that Ibn was welcomed
Sol. Option (b) is the correct answer choice. As the line “North in India by the then sultan of Delhi, hence option (c), ‘greeted’ is
Africa, West Africa, Southern Europe and Eastern Europe in the the correct answer choice.
West, to the Middle East, Indian subcontinent, Central Asia,
Southeast Asia….” suggests the pattern of Ibn Battuta’s travel. S488. Ans. (b)
Hence, the option (b) is the correct answer choice. Sol. Option (b) is the correct answer choice. The line, “whether
Ibn Battuta visited all the places that he described” suggests that
S483. Ans. (c) the writer of the passage has mentioned a doubt about his journey
Sol. Option (c) is the correct answer choice. As the previous line it’s not clear whether he had visited all the places that he has
“North Africa, West Africa, Southern Europe and Eastern Europe described in his words. Hence the correct option to fill the blank
in the West, to the Middle East, Indian subcontinent, Central is ‘Questionable’.
Asia, Southeast Asia….” suggests that the correct answer choice
should be option (c). Distance is the only option which fits S489. Ans. (a)
contextually correct in the given sentence. Sol. Option (a) is the correct answer choice. As the previous line,
“Ibn Battuta probably …….on hearsay (means of uncertain,
S484. Ans. (d) indefinite, or unclear character or meaning) evidence and made
Sol. Option (d) is the correct answer choice. As the line “After extensive use of accounts by earlier travelers” suggests that Ibn
his travels he returned to Morocco and gave his account of the had derived a lot of assumptions and the conclusions depending
……..to Ibn Juzay.” itself suggests that after Ibn was done with on the experience of the older travellers which were probably
his travelling he would have shared his experience with Ibn only the rumors else was not based on any facts. Hence option
Juzay. (a), ‘relied’ is the correct answer choice to filled the given blank.

S485. Ans. (e) S490. Ans. (e)


Sol. Option (e) is the correct answer choice. As the paragraph is Sol. Option (e) is the correct answer choice. As the line, “………a
talking about the journey of Ibn Battuta around the world, hence judge in Morocco and died in 1368” suggests that before he died
‘Voyage’ is the correct answer choice to fill the blank. he was working as a judge in his last days of life. Hence option
Voyage- a long journey involving travel by sea or in space (e), ‘appointed’, is the correct answer choice.
Variance- the fact or quality of being different, divergent, or
inconsistent
Vague- of uncertain, indefinite, or unclear character or meaning

S486. Ans. (d)

Facebook Page- https://www.facebook.com/vishalpariharpage Youtube- https://www.youtube.com/vishalparihar


Follow
114 Vishal Sir Telegram Channel- https://t.me/englishbyvishalsirchannel Instagram- https://www.instagram.com/vishalthetrainer
By Vishal sir
Also Important For SBI CLERK, IBPS CLERK, RRB CLERK
& Other Competitive Exams

Directions (491-498): Read the following passage carefully Question 492


and answer the questions given below it. Certain words are Which of the following correctly explains the meaning of the
given in bold to help you locate them while answering some phrase, 'in vain' as used in the story ?
of the questions. A were painful B were successful
Once there was a king who had been trying really hard to capture C were in order D were untruthful
an elephant, but that prize had proved elusive. All the hunters in E were simple
the kingdom had tried but their efforts were 'in vain'. So the king
announced that anyone who could capture the elephant would get Question 493
half his kingdom. The tortoise heard about this and went to the Which of the following could be a suitable title of the story ?
king to accept the challenge. The king was very amused. 'All my A The Elephant who Became the Most Popular King
hunters have failed to capture the elephant and you think you can B The Wise King
succeed ?', asked the king. The tortoise insisted that he was up to C The Clever Tortoise
the task and promised to deliver that elephant to the king within D The Villagers and their Love for the Elephant
forty eight hours. The tortoise then dug a big hole, big enough to E The Selfish Villagers
hold the elephant along a path leading to the village. Then he
covered the hole with sticks and leaves so that it was not visible Question 494
unless inspected closely. When this was done, the tortoise went Which of the following is most nearly the opposite in meaning as
in search of the elephant. When the tortoise met the elephant, he the word 'Excited' as used in the story ?
told him, 'You know you are the largest animal in the forest and A Bored
you should be a king ?' The elephant had never considered this B Interested
before but he thought it was not a bad idea. The tortoise told the C Affected
elephant that the villagers had decided to make the largest animal D Feeling
their king and were all expecting the elephant to come to the E Merciless
village and be crowned as their king. The more the elephant
heard, the more excited he became. The tortoise adorned the Question 495
elephant with colourful beads and beating a gong, he sang songs Which of the following statements is true in the context of the
praising the elephant while he led the way to the village. Soon story ?
they approached the trap and the tortoise being lighter and smaller A The king, refused to share his kingdom with the tortoise.
walked over the trap. The elephant who was following him fell B The tortoise kept his word by providing the king with what he
through the sticks and leaves into the deep hole and thus helped desired.
the tortoise in achieving his goal well within the time limit. C It was the king's idea that helped the tortoise achieve his target.
D The elephant was popular for his beautiful skin.
Question 491 E None of the given statements is true.
As mentioned in the story, despite the king doubting his ability,
the tortoise accepted the challenge . Question 496
A. he was forced to do so by the villagers. As mentioned in the story, the elephant followed tortoise's advice
B. to prove to the king that only a tortoise and his family could because :
complete the challenge. A. for him, the tortoise was the wisest animal in the village
C. he wanted the entire kingdom to himself. B. he really believed that the villagers wanted him as the king
A Only A B A and B C. he had always wanted to become rich and powerful
C Only B D Only C A Only A B A and
E None of these C A and B D Only B
E All of these

Facebook Page- https://www.facebook.com/vishalpariharpage Youtube- https://www.youtube.com/vishalparihar


Follow
115 Vishal Sir Telegram Channel- https://t.me/englishbyvishalsirchannel Instagram- https://www.instagram.com/vishalthetrainer
By Vishal sir
Also Important For SBI CLERK, IBPS CLERK, RRB CLERK
& Other Competitive Exams

Answer 491: E

Answer 492: E

Answer 493: C

Answer 494: A

Answer 495: B

Answer 496: D

Facebook Page- https://www.facebook.com/vishalpariharpage Youtube- https://www.youtube.com/vishalparihar


Follow
116 Vishal Sir Telegram Channel- https://t.me/englishbyvishalsirchannel Instagram- https://www.instagram.com/vishalthetrainer
By Vishal sir
Also Important For SBI CLERK, IBPS CLERK, RRB CLERK
& Other Competitive Exams

Question 497 Question 501


Which of the following is most nearly the same in meaning as the If you were given a (a) chance to live in (b) some other country,
word 'Insisted' as used in the story ? which (c) one would you have chosen ? (d) No error (e)
A Requested A If you were given a
B Revived B chance to live in
C Repeated C some other country
D Invited D which one would you have chosen
E Showed E No error

Question 498 Question 502


Which of the following is most nearly the opposite in meaning as Having lived to ( a) Paris for two (b) years, Prakasii under stands
the word 'Praising' as used in the story ? (c) Fret reasonably well.(d) No error (e)
A Insulting A Having lived to
B Concealing B Paris for two
C Hiding C years, Prakasii under stands
D Refusing D Fret reasonably well
E Removing E No error

Directions (499-508): Instructions Read each sentence to find Question 503


out whether there is any grammatical error in it. The error, Reema decided not to (a ) watch the movie because (b) she had
if any, will ben one part of the sentence. The number of that to study for the (c) quiz to be hold the next day(d)No error(e)
part is the answer. If there is ‘No error’, the answer is e:. A Reema decided not to
(Ignore the errors of punctuation, if any) B watch the movie because
C she had to study for the
Question 499 D quiz to be hold the next day
People who play (a) their radios too (b) loud have no (c) E No error
considerations others. (d) No error (e)
A People who play Question 504
B their radios too Athletes do various (a) kinds of warm (b) up activities before (c)
C loud have no they start to running.(d) No error (e)
D considerations others A Athletes do various
E No error B kinds of warm
C up activities before
Question 500 D they start to running
The novel has neither (a ) interesting (b) plot. nor any (c) E No error
interesting character. (d) No error (e)
A The novel has neither Question 505
B interesting Ramesh will lives (a) with Ram until he (b) finds a separate (c)
C plot. nor any house for login self.(d)No error (e)
D interesting character A Ramesh will lives Downloaded from
No error B with Ram until he
C finds a separate
D house ior lgiin self E No error

Facebook Page- https://www.facebook.com/vishalpariharpage Youtube- https://www.youtube.com/vishalparihar


Follow
117 Vishal Sir Telegram Channel- https://t.me/englishbyvishalsirchannel Instagram- https://www.instagram.com/vishalthetrainer
By Vishal sir
Also Important For SBI CLERK, IBPS CLERK, RRB CLERK
& Other Competitive Exams

Answer 497: C

Answer 498: A

Answer 499: E

Answer 500: E

Answer 501: E

Answer 502: A

Answer 503: D

Answer 504: D

Answer 505: A

Facebook Page- https://www.facebook.com/vishalpariharpage Youtube- https://www.youtube.com/vishalparihar


Follow
118 Vishal Sir Telegram Channel- https://t.me/englishbyvishalsirchannel Instagram- https://www.instagram.com/vishalthetrainer
By Vishal sir
Also Important For SBI CLERK, IBPS CLERK, RRB CLERK
& Other Competitive Exams

Question 506 D preparation


Cows, goats and (a) such domestic (b) animals together is E significance
(c)called stock. (d) No error (e)
A Cows, goats and Question 510
B such domestic He appeared on stage…………………… a narrator of the drama.
C animals together is A as
D called stock B what
E No error C about
D whatever
Question 507 E always
Students are likely to (a) work on the evenings. (b) during the
weekends, (c) or summer breaks. (d) No error (e) Question 511
A Students are likely to It is required that you fill out these two………………… to
B work on the evenings register for the job.
C during the weekends A forms
D or summer breaks B applications
E No error C relevance
D statements
Question 508 E views
Being-able to (a)remember a lot of informations is (b) not the
same as (c) being able to think. (d) No error (e) Question 512
A Being-able to As a last…………………. we had to accept these terms and
B remember a lot of informations is conditions.
C not the same as A resort
D being able to think B step
E No error C attraction
D spot
Instructions In the following questions each sentence has a E means
blank indicating that something has been omitted. Choose the
word that best fits the meaning of the sentence as a whole Question 513
Even ……………..the book doesn’t draw from political
Question 509 characters; the possible coincidence is not missing.
It is hard to believe the……………. of operations involved in A while
this activity. B though
A magnitude C until
B size D when
C scale E if

Facebook Page- https://www.facebook.com/vishalpariharpage Youtube- https://www.youtube.com/vishalparihar


Follow
119 Vishal Sir Telegram Channel- https://t.me/englishbyvishalsirchannel Instagram- https://www.instagram.com/vishalthetrainer
By Vishal sir
Also Important For SBI CLERK, IBPS CLERK, RRB CLERK
& Other Competitive Exams

Answer 506: C

Answer 507: B

Answer 508: B

Answer 509: E

Answer 510: A

Answer 511: B

Answer 512: A

Answer 513: B

Facebook Page- https://www.facebook.com/vishalpariharpage Youtube- https://www.youtube.com/vishalparihar


Follow
120 Vishal Sir Telegram Channel- https://t.me/englishbyvishalsirchannel Instagram- https://www.instagram.com/vishalthetrainer
By Vishal sir
Also Important For SBI CLERK, IBPS CLERK, RRB CLERK
& Other Competitive Exams

Instructions In the given passage there are blanks, each of Direction (519-523): Read the passage carefully and answer
which has been numbered. Against each five words are the questions given below it.
suggested, one of which fits the blank appropriately. Find out India went past two grim Covid-19 milestones on Wednesday—
the appropriate word in each case. crossing the 50,000 mark overall and breaching the 3,000-figure
in its daily tally for the first time. With 3,602 new cases—the
One World Trade Centre is viewed as a statement of hope, a most in a day till now—country‟s tally rose to 52,935.
marvel of persistence and a miracle of logistics. As years passed Alarmingly, while it took five days to go from the 30,000-mark
after the tragedy at the site at which it was since constructed and to the 40,000-mark, the next 10,000 cases were added in just three
the delays kept mounting. Americans began to _____514_____ days. It was on May 3 that the country had crossed 40,000 cases
what’s taking so long ? Have we lost the capacity to rebuild ? The while the 30,000-mark was breached on April 28. However, at
answer in part was the sheer _____515_____ of the project – 93, the death toll on Wednesday was less than half of the 199
10000 workers at-tempting one of the most difficult construction casualties recorded across the country the previous day.
projects ever in one of the most densely populated cities on Earth Maharashtra alone accounts for 36.6% of the total 1777 casualties
_____516_____ the funds allotted for the project were estimated in India so far. Delhi recorded its maximum number of cases in a
as $ 1.5 billion when he design was unveiled but the price tag just day (428), pushing its count to 5,532. With one more death, the
kept going up. Other _____517_____ included the weather in the toll in the national capital now is 65. Gujarat reported 28 more
harsh sun of summer the steel beams could reach temperatures deaths, 25 of these from Ahmedabad alone, taking the toll to 396.
that were not enough to singe skin added to which a hurricane With 380 fresh cases, the state‟s count is 6,625. As the situation
_____518_____ the construction site. The monument may not be remained grim in Ahmedabad, the administration banned sale of
all things to all people, but its completion signifies that ambition grocery, vegetables and fruits till May 15. Only milk and
coupled with determination of people in the face of odds is intact medicines sale will be allowed. Municipal commissioner Mukesh
and will always win the day. Kumar said in a notification: “Super spreaders like those working
in shops like vegetables, fruits, grocery, provisions,
Question 514 supermarkets, ice-cream parlours and people engaged in home
A understand B anger C wonder delivery of these items as well as app based food delivery services
D sense E questions become a potent source of infection for many.” Tamil Nadu saw
another big surge in infections (771). As many as 324 of the cases
Question 515 were in Chennai and 188 in Ariyalur, with the Koyambedu cluster
A complexity B delight C knowing spreading across nearly two dozen districts.
D drop E obsession
519. Tamil Nadu saw a big surge in infections by how much?
Question 516 a) 171 b) 770 c) 771 d) 155
A When B Despite C Instead
D Exclusive E Moreover 520. How many deaths were reported by Gujarat?
a) 24 b) 26 c) 28 d) 23
Question 517
A advantages B information C challenges 521. What is the total number of cases reported in Gujarat?
D attempts E crisis a) 380 b) 6652 c) 6625 d) 6605
Question 518
A formed B affected C predicted 522. What percent of the total casualties in India does Maharashtra
D hazard E flooded account for?
a) 35.3% b) 36.2% c) 36.6% d) 34.2%

Facebook Page- https://www.facebook.com/vishalpariharpage Youtube- https://www.youtube.com/vishalparihar


Follow
121 Vishal Sir Telegram Channel- https://t.me/englishbyvishalsirchannel Instagram- https://www.instagram.com/vishalthetrainer
By Vishal sir
Also Important For SBI CLERK, IBPS CLERK, RRB CLERK
& Other Competitive Exams

Answer 514: C
Answer 515: A
Answer 516: E
Answer 517: C
Answer 518: E

(519) Answer: C Solution: It is clearly mentioned in the passage that,


„Tamil Nadu saw another big surge in infections (771)‟.
Therefore the correct answer is option c).

520) Answer: C Solution: It is clearly mentioned in the passage that,


„Gujarat reported 28 more deaths, 25 of these from Ahmedabad
alone, taking the toll to 396.‟ Therefore the correct answer is
option c).

521) Answer: C Solution: It is clearly mentioned in the passage that,


„Gujarat reported 28 more deaths, 25 of these from Ahmedabad
alone, taking the toll to 396. With 380 fresh cases, the state‟s
count is 6,625.‟ Therefore the correct answer is option c).

522) Answer: C Solution: It is clearly mentioned in the passage that,


„Maharashtra alone accounts for 36.6% of the total 1777
casualties in India so far.‟ Therefore the correct answer is option
c).

Facebook Page- https://www.facebook.com/vishalpariharpage Youtube- https://www.youtube.com/vishalparihar


Follow
122 Vishal Sir Telegram Channel- https://t.me/englishbyvishalsirchannel Instagram- https://www.instagram.com/vishalthetrainer
By Vishal sir
Also Important For SBI CLERK, IBPS CLERK, RRB CLERK
& Other Competitive Exams

523. What is the most suitable title for the passage?


a) India breaches 50,000-mark, hits new peak of 3,600 cases in 1 Directions (529-530): In the following questions, a sentence
day has been given with a word omitted. Find the word from the
b) India breaches 51,000-mark, hits new peak of 3,600 cases in 1 options that fits in the blank and mark it as your answer. If
day none of the words fit in the blank then mark E as the answer.
c) India breaches 50,000-mark, hits new peak of 3,610 cases in 1
day 529. For India to meet its goal to fully _______ all adults by the year
d) None of the above end, it needs to vaccinate over a crore every day but has
consistently averaged a little less than 80 lakh a day since the 2.5
Direction (524-528): In each of the questions given below a crore doses on September 17.
sentence is given which is then divided into five parts out of A. Infect B. Administered C. Inoculate
which one bold part is correct. There are no errors in three D. Vaccination E. None of these
out of four remaining parts and therefore only one of the
parts other than the bold one is incorrect. You must choose 530. With students’ queues drying up and over 70-90 per cent seats
the grammatically incorrect part as your answer. Choose e if going _______, private engineering colleges across the country
you find out there is no error. have been collapsing like ninepins in the past around eight years.
A. Abegging B. Annoyed
524. The entire business has since been a)/ taken over by another C. Embracing D. Disintegrate
dealer after b)/ he defaulted on bank loans and c)/ a resolution E. None of these
process is currently underway, said d)/ the entrepreneur who
decline to be named e)/ Directions (531-540): Read the following passage and answer
A.b B.a C.d D.e E.No error the following questions. Some words are highlighted to help
you answer some of the questions.
525. It is obvious that these changes will a)/ be challenged in the
Supreme b)/ Court, though many eminent lawyers c)/ are Safe and rewarding — small savings schemes offered by the post
persuaded about the constitution d)/ validity of the legislation
office, and by some banks can make for a neat fit in your fixed
approved by Parliament e)/
income portfolio. With government backing, these schemes are
A.c B.d C.a D.e E.No error
as safe as they get. Given their no-risk profile, the interest rates
526. Opposition parliamentarians in New a)/ Delhi were preparing to offered by many of these schemes are very attractive — higher
reject b)/ any dilution of the Article, which c)/ was enacted by a than what most banks offer on fixed deposits. Besides, many of
executive order, d)/ without any Parliamentary endorsement e)/ these schemes enjoy tax breaks which increase the effective
A.b B.c C.d D.e E.No error returns.
While you can invest in small savings schemes any time of the
527. The company continues to have a a)/ strategic direction and a year, now seems a particularly opportune time to do so. That’s
suitable way)/ forward, a person of eminence or c)/ a reputed firm because interest rates on these schemes are ruling high and could
will soon be d)/ appointed as strategic corporate adviser e)/ possibly decline in the coming quarters.
A.b B.a C.d D.e E.No error From April 2016, interest rates on small savings schemes are
528. The increase in surcharge on the income tax a)/ outgo of certain being reset on a quarterly basis. The idea is to align the rates on
classes of taxpayers b)/ earning above specified limits that hit c)/
these schemes with those on government securities (G-Secs). So,
certain FPIs organised as trusts has now come in force d)/ with
the President signing the Finance Bill e)/ as G-Sec rates move up or down, the rates on small savings
A.b B.c C.d D.a E.No error schemes are also supposed to move up and down every quarter.

Facebook Page- https://www.facebook.com/vishalpariharpage Youtube- https://www.youtube.com/vishalparihar


Follow
123 Vishal Sir Telegram Channel- https://t.me/englishbyvishalsirchannel Instagram- https://www.instagram.com/vishalthetrainer
By Vishal sir
Also Important For SBI CLERK, IBPS CLERK, RRB CLERK
& Other Competitive Exams

But in practice, the quarterly reset mechanism is not always (b) Account can be closed as per the convenience of account
implemented, especially when G-Sec rates are on the decline. For holder
two quarters in a row (January to March 2019, and the recent (c) Rates will be same during the entire tenure of investment
April to June 2019), the rates on most small savings schemes have (d) Premature closure facility is available for small savings
been left untouched, despite a significant decline in G-Sec rates. scheme accounts
As a result, the rates on small savings schemes today are very (e) Accounts can be transferred from one person to another
attractive compared with many fixed income options. For
instance, the interest rate on the Senior Citizen Savings Scheme Q532. Why has been the policy to reset interests quarterly introduced
(SCSS) is 8.7%, the Sukanya Samriddhi Yojana (SSY) gets 8.5%, for Small Savings Schemes (SSS)?
while the PPF and the NSC both earn 8%. With tax breaks the (a) To transfer interest amounts to the costumer’s bank account
effective returns are much higher. after each quarter.
Going by its recent statements, the RBI may continue with its (b) To encourage small savings and investment on G-Secs
policy rate cuts, which could also reflect on the G-Sec rates and among the public
eventually on the rates on small savings schemes. (c) Interest can be drawn through auto credit into savings
Even if the RBI does not cut rates further, the Centre may still account standing at same post office
choose to moderate rates on small savings schemes to make up (d) To align rates of SSS with government securities interest
for the pending cuts. So, it makes sense to lock into high rates rates
now, rather than wait and run the risk of settling for lower rates. (e) To offer extension facility after maturity
That said, invest only in fixed rate schemes where the rate at the
start stays the same until maturity. Q533. Why are small savings schemes a “neat fit for investment
Small savings schemes can be variable rate or fixed rate portfolio”?
products. The popular PPF and the girl-child oriented SSY are (a) Any number of accounts can be opened in any number of
variable rate products in which rates applicable on the investment branches.
_____(A)______. So, new rates announced for each quarter will (b) The rates of interest are higher than most other deposit
apply to the accumulated corpus until then. Ergo: it will not schemes offered by banks
really help to rush into investments in these schemes just to take (c) many of these schemes enjoy tax breaks that pegs up their
advantage of higher rates now. effective returns sharply
But in fixed rate products, the rate at the start of the investment (d) Both (b) and (c)
stays until maturity. New rates announced each quarter will apply (e) All (a), (b) and (c)
only to investments made in the quarter and will hold till their
maturity. This category comprises the NSC, SCSS, Kisan Vikas Q534. Why did the author state that it was the favourable time to invest
Patra (KVP), Post office monthly income scheme (POMIS) and in Small Savings Scheme?
Post office time and recurring deposits. (a) These deposits will qualify for tax rebate
(b) Interest rates on these schemes were high and could possibly
Q531. According to the information given in the passage above, why is decline in coming quarters
it beneficial to invest in fixed rate schemes like NSC, SCSS, (c) The government will not be offering premature closure
KVP, etc.? facility for these accounts from next quarter
(d) A depositor will not be able to operate more than one
(a) These accounts offer variable interest rates, considering the account in individual capacity or jointly with spouse
economic background of teh account holder (e) None of the above

Facebook Page- https://www.facebook.com/vishalpariharpage Youtube- https://www.youtube.com/vishalparihar


Follow
124 Vishal Sir Telegram Channel- https://t.me/englishbyvishalsirchannel Instagram- https://www.instagram.com/vishalthetrainer
By Vishal sir
Also Important For SBI CLERK, IBPS CLERK, RRB CLERK
& Other Competitive Exams

523) Answer: A Solution: In option b) the mention of „51000‟ is Abegging – unnoticed, unappreciated, unused
incorrect. It should be ‟50,000‟. In option c) the mention of
S531. Ans. (c)
„3610‟ is incorrect, it should be ‟36,000‟. Therefore only the title
Sol. Refer to the last paragraph of the passage given above, which
provided in option a) is correct. Thus the correct answer is option
a) states “But in fixed rate products, the rate at the start of the
524. Answer: D Private sector lender ICICI Bank, launched a virtual investment stays until maturity. New rates announced each
property exhibition that digitally showcases real estate projects quarter will apply only to investments made in the quarter and
from key cities across the country. Called Home Utsav, the will hold till their maturity. This category comprises the NSC,
exhibition is available for everyone, including ICICI Bank’s SCSS, Kisan Vikas Patra (KVP), Post office monthly income
customers, and those who are not customers of the bank. They scheme (POMIS) and Post office time and recurring deposits.”
can also avail exclusive offers such as attractive interest rates, From the given lines, we can infer that it is better to invest in
special processing fees and digital sanction of loans on buying a fixed rates schemes as the rates will be same throughout the
property through this exhibition,” the lender stated in a statement, investment tenure therefore, it is better to invest when rates are
adding that its customers can get further benefits such as pre- high. Hence, option (c) is the mist suitable answer choice.
approved and insta series of products.
S532. Ans. (d)
525. Answer: D The World Day of Remembrance for Road Traffic
Victims takes place on the third Sunday in November every year. Sol. Refer to the second paragraph of the passage given above,
This year it falls on Nov 15th . The theme for WDoR in 2020 is which states, “ The idea is to align the rates on these schemes
“First Responders”, recognizing the selfless men and women who with those on government securities (G-Secs). So, as G-Sec rates
rescue, care for and support victims of road trauma move up or down, the rates on small savings schemes are also
supposed to move up and down every quarter.” Hence, option (d)
526. Answer: C Indian Olympic quota winner Yashaswini Singh is the most suitable answer choice.
Deswal won the 10m air pistol event in the 5th edition of the S533. Ans. (d)
International Online Shooting Championship (IOSC). Sol. Refer to the first paragraph of the passage given above,
Yashaswini, who had won a gold at last year’s ISSF World Cup which states, “Given their no-risk profile, the interest rates
in Rio de Janeiro, shot the best qualification score of 577. She had offered by many of these schemes are very attractive — higher
previously won the gold in the fourth edition of IOSC held in than what most banks offer on fixed deposits. Besides, many of
May. The second place was grabbed by Ahmed Nabi
these schemes enjoy tax breaks which increase the effective
527. Answer: E Peruvian politician Francisco Sagasti has been sworn
returns.” From the given lines, we can conclude that owing to tax
in as the country's interim President.
saving benefits and high interest rates, investing in SSS is fruitful.
528. Answer: D Export – Import (EXIM) Bank of India is the premier Hence, option (d) is the most suitable answer choice.
export finance institution in India S534. Ans. (b)
529. Answer: C; inoculate.The word that fits in the blank is – Sol. Refer to the first paragraph of the passage given above,
inoculate. It means vaccinate, inject, or administer a vaccine. The which states, “While you can invest in small savings schemes
word should be in present tense as the sentence is in present tense. any time of the year, now seems a particularly opportune time to
530. Answer: A; abegging do so. That’s because interest rates on these schemes are ruling
The word that fits in the blank is – abegging. The sentence gives high and could possibly decline in the coming quarters.” From
a sense of seats unnoticed in private engineering colleges and these lines, we can infer that option (b) is the most suitable
students are disappearing from admission queues. Only option A answer choice.
fits the blank.

Facebook Page- https://www.facebook.com/vishalpariharpage Youtube- https://www.youtube.com/vishalparihar


Follow
125 Vishal Sir Telegram Channel- https://t.me/englishbyvishalsirchannel Instagram- https://www.instagram.com/vishalthetrainer
By Vishal sir
Also Important For SBI CLERK, IBPS CLERK, RRB CLERK
& Other Competitive Exams

Q535. What has been the contradiction stated in the passage given (a) sweep
above? (b) surge
(a) At the time of transfer of certificate, old certificates will not (c) scutter
be discharged for SSS whereas, they will be discharged for G- (d) scamper
Secs. (e) plummet
(b) The interests on G-secs increased for two continuous
quarters with no change in rates of small savings scheme Q540. VARIABLE
accounts
(c) Small Saving Schemes can be purchased from any bank (a) assiduous
whereas there is no such facility for government securities (b) protean
(d) The interest rates on government securities fell for two (c) ardent
continuous quarters with no change in rates of small saving (d) dedicated
schemes (e) None of these (e) unswerving
Q536. Which of the following phrases can be filled in the blank (A) to
make a grammatically correct and contextually meaningful Directions (541-545): Rearrange the following six sentences in
statement? a proper sequence to form a meaningful paragraph and then
(a) ensures the safety of your investment answer the questions given below.
(b) keep changing throughout the tenure
(c) are endangering the system’s health and viability (A)However, in order for continuous success to take place; it is
(d) stressing the need for greater accountability the Government’s responsibility to provide the youth with proper
(e) ultimately undermines the very rule of law facilities for, getting equipped with the knowledge of the modern
era.
Directions (537-538): For each of the following words, (B) Its focus is on laws, policies and legal frameworks that have
choose the word which has a meaning which is OPPOSITE been developed with the participation of young people and which
to the meaning of the given words. help them realize their full potential.
Q537. ACCUMULATED (C)They also need to know how to read, write, think, understand,
(a) shielded (b) strengthened analyze, and discuss the issues their country faces.
(c) resonated (d) dissipated (D)The entire success of the nation depends on the youths.
(e) accrued (E)The primary role of young people is to get a good education
in order to become better citizens of tomorrow.
Q538. RUSH (F)They need to learn skills to do the job that their country’s
(a) bustle (b) scurry economy needs.
(c) gallop (d) dawdle
(e) bolt Q541. Considering statement (E) “The primary role of young people
is to get a good education in order to become better citizens
Directions (539-540): For each of the following words, of tomorrow” as the first sentence of the paragraph then which
choose the word which has a meaning which is SIMILAR to one among the following becomes the last sentence to make the
the meaning of the given words. paragraph coherent?
Q539. DECLINE (a) A (b) C (c) B (d) E (e) D

Facebook Page- https://www.facebook.com/vishalpariharpage Youtube- https://www.youtube.com/vishalparihar


Follow
126 Vishal Sir Telegram Channel- https://t.me/englishbyvishalsirchannel Instagram- https://www.instagram.com/vishalthetrainer
By Vishal sir
Also Important For SBI CLERK, IBPS CLERK, RRB CLERK
& Other Competitive Exams

S535. Ans. (d) or amount.” Since they are synonymous with each other, option
Sol. Refer to the 4th paragraph of the passage given above, which (e) is the most suitable answer choice.
states, “But in practice, the quarterly reset mechanism is not Scamper: run with quick light steps, especially through fear or
always implemented, especially when G-Sec rates are on the excitement.
decline. For two quarters in a row (January to March 2019, and Scutter: (especially of a small animal) move hurriedly with short
the recent April to June 2019), the rates on most small savings steps.
schemes have been left untouched, despite a significant decline in Surge: a sudden powerful forward or upward movement,
G-Sec rates.” From the given lines, we can infer that the especially by a crowd or by a natural force such as the tide.
statement given in option (d) is correct as per the information Sweep: move or remove (dirt or litter) by brushing it away.
given in the passage. Hence, option (d) is the most suitable S540. Ans. (b)
answer choice. Sol. “VARIABLE” means “not consistent or having a fixed
S536. Ans. (b) pattern; liable to change” and “PROTEAN” means “tending or
Sol. Here, phrase given in option (a) will make a grammatically able to change frequently or easily.” Since, they are
incorrect statement and therefore, can be omitted. Also, phrase synonymous with each other, option (b) is the most suitable
given in option (c) can be omitted as it is out of context. Here, answer choice.
only the phrase given in option (b) makes a grammatically correct Assiduous: showing great care and perseverance.
and contextually meaningful statement. Hence, option (d) is the Ardent: very enthusiastic or passionate.
most suitable answer choice. Unswerving: not changing or becoming weaker; steady or
S537. Ans. (d) constant.
Sol. “ACCUMULATED” [verb] means “gather together or S541. Ans. (a)
acquire an increasing number or quantity of” and Sol. As provided in the question, statement (E) is the first
“DISSIPATE” [verb] means “disperse or scatter.” Since, they statement of the paragraph in which there is a mention of the
are opposite to each other, option (d) is the most suitable answer prerequisite for the youths to become good citizens in future.
choice. Accrued: accumulate or receive Sentence (E) and (F) forms a coherent pair as sentence (F) has
resonated: produce or be filled with a deep, full, reverberating talked that youths need to learn the skills to get themselves into
sound. strengthened: make or become stronger. the job and thus contributing to the country’s economy. Sentence
shielded: protect from a danger, risk, or unpleasant experience. (C) is consecutively following sentence (F) as it has mentioned
S538. Ans. (d) Sol. “RUSH” means “move with urgent haste” and the other things that youths should follow to know their country
“DAWDLE” means “move slowly and idly in a particular more. Moreover, sentence (D) and (A) are forming a contextually
direction.” Since, they are opposite to each other, option (d) is logical pair, the clue can be inferred from the joining word
the most suitable answer choice. “success.” Sentence (D) is talking about the entire nation’s
Gallop: a very fast pace of running by a person. success and sentence (A) is talking about how to achieve that
Bolt: fasten (something) to something else with a long pin that success on continuous pace. Thus, sentence (A) becomes the last
screws into a nut. sentence of the paragraph as it describes about the role of
Scurry: move hurriedly with short quick steps. government in order to provide the facilities to the youth to
Bustle: move in an energetic and busy manner. achieve that success. However, sentence (B) fails to connect with
the theme of the paragraph as it is describing about import duties
S539. Ans. (e) levied in India in various sectors. Therefore, with the elimination
Sol. “DECLINE” means “become smaller, fewer, or less; of sentence (B) the sequence thus formed is EFCDA. Hence,
decrease” and “PLUMMET” means “decrease rapidly in value option (a) becomes the most suitable answer choice.

Facebook Page- https://www.facebook.com/vishalpariharpage Youtube- https://www.youtube.com/vishalparihar


Follow
127 Vishal Sir Telegram Channel- https://t.me/englishbyvishalsirchannel Instagram- https://www.instagram.com/vishalthetrainer
By Vishal sir
Also Important For SBI CLERK, IBPS CLERK, RRB CLERK
& Other Competitive Exams

Q542. Considering statement (E) “The primary role of young people [II] He would conceive an unintelligible _________ to a
is to get a good education in order to become better citizens particular alley and perform a great circuit rather than see the
of tomorrow” as the first sentence of the paragraph, then which hateful place.
among the following does not coherently relate to the paragraph (a) delight
after the rearrangement? (b) elation
(a) D (b) C (c) aversion
(c) A (d) B (d) function
(e) F (e) cognizance

Q543. Which one of the following statements should consecutively Q547. [I] Some have supposed that in this passage seven teachers are
follow statement (F)? named, others that there are only five, and various
(a) A (b) B _____________ have been hazarded as to what persons were
(c) C (d) D meant.
(e) E [II] These _____________ were made in the pre-scientific era of
philology
Q544. Which one of the following is the FOURTH statement of the (a) conjectures
paragraph after rearrangement? (b) feint
(a) F (b) B (c) homage
(c) A (d) D (d) case
(e) E (e) evidence

Q545. Considering statement (E) “The primary role of young people Q548. [I] While many people love technology, there are just as many
is to get a good education in order to become better citizens who show _____________ towards it.
of tomorrow” as the first statement of the paragraph, choose the [II] Even though I am a great swimmer, I have a great deal of
correct sequence of the paragraph after rearrangement. __________ about swimming because my uncle drowned.
(a) EDCBA (a) illumination
(b) ECDAB (b) glee
(c) EAFDC (c) triumph
(d) EFCDA (d) significance
(e) ECADB (e) ambivalence

Direction (546-550): There are two different sentences with a Q549. [I] Although there were many ________ risks in the project, he
blank space in each sentence. Choose the word from the given made a decision to plunge in it and emerge successful.
options which fits appropriately in both the blanks adding a [II] I can sense an ___________ protest brewing amongst
proper and logical meaning to the sentences. students of the university
(a) illusory
Q546. [I] Cotton seed in those days was the object of so much _______ (b) inherent
that the planter burned it or threw it into running streams, as was (c) imminent
most convenient. (d) impassive
(e) ingenious

Facebook Page- https://www.facebook.com/vishalpariharpage Youtube- https://www.youtube.com/vishalparihar


Follow
128 Vishal Sir Telegram Channel- https://t.me/englishbyvishalsirchannel Instagram- https://www.instagram.com/vishalthetrainer
By Vishal sir
Also Important For SBI CLERK, IBPS CLERK, RRB CLERK
& Other Competitive Exams

S542. Ans. (d) achieve that success. However, sentence (B) fails to connect with
Sol. As provided in the question, statement (E) is the first the theme of the paragraph as it is describing about import duties
statement of the paragraph in which there is a mention of the levied in India in various sectors. Therefore, with the elimination
prerequisite for the youths to become good citizens in future. of sentence (B) the sequence thus formed is EFCDA. Thus,
Sentence (E) and (F) forms a coherent pair as sentence (F) has option (c) is the correct answer choice.
talked that youths need to learn the skills to get themselves into
the job and thus contributing to the country’s economy. Sentence S544. Ans. (d)
(C) is consecutively following sentence (F) as it has mentioned Sol. As provided in the question, statement (E) is the first
the other things that youths should follow to know their country statement of the paragraph in which there is a mention of the
more. Moreover, sentence (D) and (A) are forming a contextually prerequisite for the youths to become good citizens in future.
logical pair, the clue can be inferred from the joining word Sentence (E) and (F) forms a coherent pair as sentence (F) has
“success.” Sentence (D) is talking about the entire nation’s talked that youths need to learn the skills to get themselves into
success and sentence (A) is talking about how to achieve that the job and thus contributing to the country’s economy. Sentence
success on continuous pace. Thus, sentence (A) becomes the last (C) is consecutively following sentence (F) as it has mentioned
sentence of the paragraph as it describes about the role of the other things that youths should follow to know their country
government in order to provide the facilities to the youth to more. Moreover, sentence (D) and (A) are forming a contextually
achieve that success. However, sentence (B) fails to connect with logical pair, the clue can be inferred from the joining word
the theme of the paragraph as it is describing about import duties “success.” Sentence (D) is talking about the entire nation’s
levied in India in various sectors. Therefore, with the elimination success and sentence (A) is talking about how to achieve that
of sentence (B) the sequence thus formed is EFCDA. Therefore, success on continuous pace. Thus, sentence (A) becomes the last
only option (d) i.e., statement (B) does not follow the theme of sentence of the paragraph as it describes about the role of
the paragraph as it is describing about the import duties levied by government in order to provide the facilities to the youth to
India in various sectors. achieve that success. However, sentence (B) fails to connect with
the theme of the paragraph as it is describing about import duties
S543. Ans (c) levied in India in various sectors. Therefore, with the elimination
Sol. As provided in the question, statement (E) is the first of sentence (B) the sequence thus formed is EFCDA. Therefore,
statement of the paragraph in which there is a mention of the option (d) is the correct choice.
prerequisite for the youths to become good citizens in future.
Sentence (E) and (F) forms a coherent pair as sentence (F) has S545. Ans. (d)
talked that youths need to learn the skills to get themselves into Sol. As provided in the question, statement (E) is the first
the job and thus contributing to the country’s economy. Sentence statement of the paragraph in which there is a mention of the
(C) is consecutively following sentence (F) as it has mentioned prerequisite for the youths to become good citizens in future.
the other things that youths should follow to know their country Sentence (E) and (F) forms a coherent pair as sentence (F) has
more. Moreover, sentence (D) and (A) are forming a contextually talked that youths need to learn the skills to get themselves into
logical pair, the clue can be inferred from the joining word the job and thus contributing to the country’s economy. Sentence
“success.” Sentence (D) is talking about the entire nation’s (C) is consecutively following sentence (F) as it has mentioned
success and sentence (A) is talking about how to achieve that the other things that youths should follow to know their country
success on continuous pace. Thus, sentence (A) becomes the last more. Moreover, sentence (D) and (A) are forming a contextually
sentence of the paragraph as it describes about the role of logical pair, the clue can be inferred from the joining word
government in order to provide the facilities to the youth to “success.” Sentence (D) is talking about the entire nation’s

Facebook Page- https://www.facebook.com/vishalpariharpage Youtube- https://www.youtube.com/vishalparihar


Follow
129 Vishal Sir Telegram Channel- https://t.me/englishbyvishalsirchannel Instagram- https://www.instagram.com/vishalthetrainer
By Vishal sir
Also Important For SBI CLERK, IBPS CLERK, RRB CLERK
& Other Competitive Exams

success and sentence (A) is talking about how to achieve that


success on continuous pace. Thus, sentence (A) becomes the last S549. Ans. (b)
sentence of the paragraph as it describes about the role of Sol. The blank can be filled with the word ‘inherent’ making the
government in order to provide the facilities to the youth to sentence meaningful. Inherent means existing in something as a
achieve that success. However, sentence (B) fails to connect with permanent, essential or characteristic attribute. Other words are
the theme of the paragraph as it is describing about import duties irrelevant and do not fit in the context of the sentence here. Hence
levied in India in various sectors. Therefore, with the elimination option (b) is the correct choice.
of sentence (B) the sequence thus formed is EFCDA. Hence, Illusory means based on illusion; not real.
option (d) becomes the most suitable answer choice. Imminent means about to happen.
Impassive means not feeling or showing emotion.
S546. Ans. (c) Ingenious means (of a person) clever, original, and inventive.
Sol. The blank can be filled with the word ‘aversion’ making the
sentence meaningful. Aversion means a strong dislike or
disinclination. Other words are irrelevant and do not fit in the
context of the sentence here. Hence option (c) is the correct
choice.
Delight means take great pleasure in.
Elation means great happiness and exhilaration.
Cognizance means knowledge or awareness

S547. Ans. (a)


Sol. The blank can be filled with the word ‘conjectures’ making
the sentence meaningful. Conjectures means an opinion or
conclusion formed on the basis of incomplete information. Other
words are irrelevant and do not fit in the context of the sentence
here. Hence option (a) is the correct choice.
Feint means make a deceptive or distracting movement,
especially during a fight.
Homage means special honour or respect shown publicly.

S548. Ans. (e)


Sol. The blank can be filled with the word ‘ambivalence’ making
the sentence meaningful. Ambivalence means the state of having
mixed feelings or contradictory ideas about something or
someone. Other words are irrelevant and do not fit in the context
of the sentence here. Hence option (e) is the correct choice.
Illumination means lighting or light.
Glee means great delight, especially from one's own good fortune
or another's misfortune.
Triumph means a great victory or achievement.

Facebook Page- https://www.facebook.com/vishalpariharpage Youtube- https://www.youtube.com/vishalparihar


Follow
130 Vishal Sir Telegram Channel- https://t.me/englishbyvishalsirchannel Instagram- https://www.instagram.com/vishalthetrainer
By Vishal sir
Also Important For SBI CLERK, IBPS CLERK, RRB CLERK
& Other Competitive Exams

Q550. [I] The interviewer asked me to _________ my potential India’s rural households. As per the United Nations Development
contributions to the company. Programme (UNDP) recommended “Energy Plus” approach,
[II] When I get nervous, it is very hard for me to ___________ supply of electricity only for lighting is a necessary but not
my thoughts. sufficient condition for rural livelihood development. This
(a) articulate (b) languish framework emphasizes on energy access in combination with
(c) linguistic (d) perceive productive use of electricity for income generation and livelihood
(e) perforate upliftment. However, to use electricity directly for income-
generation activities, ownership of appliances plays an important
Directions (551-560): Read the following passage to answer role, apart from market availability, financial and technical
these questions given below it. Certain words phrases have assistance. Appliance ownership, in turn, depends on the
been printed in bold to help you locate them while answering household’s economic status and on the quality and availability
some of the questions: of power supply. This makes the problem more challenging.
Further, lack of access to energy at home and for income-
Access to affordable electricity for each and every household is a generating activities is associated with higher levels of poverty,
necessary condition for social and economic development. low productivity, heavy workload, women’s safety issues, missed
However, rural electrification received attention in the educational opportunities and high exposure to health risks.
development agenda mostly in the last one-and-a-half decades. In
2005, the Central government launched the Rajiv Gandhi The cost of power supply to rural areas is also significantly high.
Grameen Vidyutikaran Yojana (RGGVY) which subsumed all As a majority of the rural households cannot afford high cost
other ongoing schemes related to rural electrification. The supply, utilities are reluctant to supply the required quality and
scheme focused on electrification of villages through quantity of electricity in these areas. This is apart from the issue
implementation of decentralized distributed generation (DDG). of capacity constraint in terms of power generation/purchase.
RGGVY was later included in the Deen Dayal Upadhyaya Gram However, implementing some appropriate measures such as
Jyoti Yojana (DDUGJY) (recently renamed the Saubhagya smart meters, infrastructure development, franchisee
scheme), which additionally focuses on feeder separation, arrangements with local self-help-groups (for more effective
improvement of sub-transmission and distribution network, and billing, monitoring and collection) may improve the situation to
metering to reduce losses. All these schemes have delivered some extent. The recent Saubhagya scheme addresses some of
results and now only a few villages are left that have yet to these issues. It aims to improve environment, public health,
achieve the target of 100% electrification. As per the latest education and connectivity with the help of last-mile power
government statistics, only 910 villages are yet to be electrified, connections across India along with providing electricity
which account for 5% of India’s un-electrified villages (as on connections to over 40 million families in rural and urban areas
April 2015), excluding some uninhabited villages. However, the by December. Households out of reach of the national electricity
performance of rural household electrification is not that grid are proposed to be provided with solar power packs along
encouraging. Around 35 million households—approximately with battery banks with the Rural Electrification Corporation as
11% of the total rural households—are yet to be electrified. the nodal agency.

The success of rural electrification should not be measured only Q551. According to the passage, Saubhagya Scheme aims to
on the basis of connections provided, but also on the basis of (a) Supply of electricity through dedicated feeders.
provision of reliable and quality power supply during peak hours. (b) improvement of metering and distribution network to reduce
Both these are still persistent problems faced by a majority of losses.

Facebook Page- https://www.facebook.com/vishalpariharpage Youtube- https://www.youtube.com/vishalparihar


Follow
131 Vishal Sir Telegram Channel- https://t.me/englishbyvishalsirchannel Instagram- https://www.instagram.com/vishalthetrainer
By Vishal sir
Also Important For SBI CLERK, IBPS CLERK, RRB CLERK
& Other Competitive Exams

(c) Access to affordable electricity through implementation of Directions (556- 558): Choose the word which is most same
decentralized distributed generation. in meaning of the word printed in bold in context of the
(d) Both (b) and (c) passage.
(e) All are correct
Q556. Subsume
Q552. What are the problem(s) faced by Indian rural population (a) abundant (b) variegate
nowadays? (c) integrate (d) comprise
(I) Lack of sufficient appliances to access the electricity. (e) consistent
(II) Lack of reliable and quality power supply.
(III) Lack of connection provided for electrification. Q557. Reluctant
(a) Only (I) (a) ardent
(b) Both (I) and (II) (b) anonymous
(c) Both (II) and (III) (c) grudging
(d) Both (I) and (III) (d) suggest
(e) All are correct (e) enthusiastic

Q553. The need of “energy plus” approach is to Q558. Uplift


(a) use it for educational opportunities. (a) extenuate
(b) avoiding high health risks. (b) pacify
(c) use electricity for income generation activities. (c) abet
(d) provide women safety. (d) mollify
(e) All (e) elevate

Q554. How the problems can be resolved for easy access of electricity Directions (559- 560): Choose the word which is most
to rural population? opposite in meaning of the word printed in bold in context of
(a) By developing infrastructure the passage.
(b) By installing smart and effective machinery to the system.
(c) By launching some schemes aiming to improve rural Q559. Reliable
electrification. (a) conciliate
(d) By effective monitoring of the power supply. (b) dodgy
(e) All of the above. (c) persistence
(d) persuade
Q555. The tone of the author in context to the passage is (e) incite
(a) sarcastic
(b) didactic Q560. Exposure
(c) cynical (a) candor
(d) critical (b) dire
(e) Analytical (c) subdue
(d) safety
(e) proficient

Facebook Page- https://www.facebook.com/vishalpariharpage Youtube- https://www.youtube.com/vishalparihar


Follow
132 Vishal Sir Telegram Channel- https://t.me/englishbyvishalsirchannel Instagram- https://www.instagram.com/vishalthetrainer
By Vishal sir
Also Important For SBI CLERK, IBPS CLERK, RRB CLERK
& Other Competitive Exams

associated with higher levels of poverty, low productivity, heavy


S550. Ans. (a) workload, women’s safety issues, missed educational
Sol. The blank can be filled with the word ‘articulate’ making the opportunities and high exposure to health risks.”
sentence meaningful. Articulate means having or showing the
ability to speak fluently and coherently. Other words are S554. Ans. (e)
irrelevant and do not fit in the context of the sentence here. Hence Sol. All the given statements are correct with respect to the third
option (a) is the correct choice. paragraph of the passage.
Languish means be forced to remain in an unpleasant place or
situation. S555. Ans. (e)
Linguistic means relating to language or linguistics. Sol. The author’s tone is ‘analytical’ as the author gave a detailed
Perceive means become aware or conscious of (something); come treatment of the issue after undergoing deep analysis of the causes
to realize or understand. and effects.
Perforate means pierce and make a hole or holes in.
S551. Ans. (e) S556. Ans. (d)
Sol. According to paragraph 1, Saubhagya scheme, previously Sol. Subsumed means to consider or include. Hence it has same
named as Rajiv Gandhi Grameen Vidyutikaran Yojana focuses meaning as comprise.
on feeder separation, improvement of sub-transmission and
distribution network, and metering to reduce losses in addition to S557. Ans. (c)
electrification of villages through implementation of Sol. Reluctant means unwilling and hesitant; disinclined. Hence
decentralized distributed generation (DDG). it has same meaning as grudging.
Hence option (e) is the correct choice.
S558. Ans. (e)
552. Ans. (c) Sol. Uplift means to lift up, elevate. Hence it has same meaning
Sol. In reference to second paragraph of the passage, we can infer as elevate.
that India’s rural household are facing non- reliable power supply Extenuate means acting in mitigation to lessen the seriousness of
at peak hours and lack of connection. guilt or an offence.
Hence option (c) is the correct choice. Mollify means appease the anger or anxiety of.
Refer the first few lines of second paragraph “The success of rural Abet means to help or encourage a person or thing to do
electrification should not be measured only on the basis of something.
connections provided, but also on the basis of provision of S559. Ans. (b)
reliable and quality power supply during peak hours.” Sol. Reliable means consistently good in quality or performance;
able to be trusted. Hence its opposite is dodgy.
S553. Ans. (e) Incite meaning encourage.
Sol. All the given options are correct. Conciliate means stop (someone) being angry or discontented;
Refer third paragraph of the passage which indicates that “energy placate.
plus” approach is not just providing electricity for lighting but S560. Ans. (d)
also for other facilities. Sol. Exposure means the state of having no protection from
Hence option (e) is the correct choice. something harmful. Hence it has opposite meaning as safety.
“This makes the problem more challenging. Further, lack of Candor means the quality of being open and honest; frankness.
access to energy at home and for income-generating activities is Dire means extremely serious or urgent.

Facebook Page- https://www.facebook.com/vishalpariharpage Youtube- https://www.youtube.com/vishalparihar


Follow
133 Vishal Sir Telegram Channel- https://t.me/englishbyvishalsirchannel Instagram- https://www.instagram.com/vishalthetrainer
By Vishal sir
Also Important For SBI CLERK, IBPS CLERK, RRB CLERK
& Other Competitive Exams

(Directions 561-570): In each of the question given below a/an (a) impair
idiom/phrase is given in bold which is then followed by five (b) taper
options which then tries to decipher its meaning as used in the (c) expatiate
sentence. Choose the option which gives the meaning of the (d) slump
phrase most appropriately in context of the given sentence. (e) doleful

Q561. All I knew about were Antony's own financial dealings, which Q566. We had to calm her down and encourage her to keep her head.
were always above board. (a) composed
(a) subservience (b) jubilant
(b) priority (c) consumed
(c) pattern (d) demented
(d) guideline (e) elated
(e) honest
Q567. Those socialist niceties always turn to dog eat dog.
Q562. It took a considerable amount of polish and elbow grease before (a) entranced
the brass shone like new. (b) diplomat
(a) accent (c) arbitrator
(b) gist (d) ferocious
(c) bulge (e) untraced
(d) toil
(e) brim Q568. My aunt used to say I had a black dog on my shoulder when I
was in a temper as a littlun.
Q563. Alex opponents never missed an opportunity to cast aspersions (a) melancholy
on his professionalism. (b) responsibility
(a) limit (c) authority
(b) abuse (d) immaturity
(c) annex (e) answerability
(d) appreciate
(e) adore Q569. The person went scot-free even though there were many people
convinced of his crime because the evidence against him was
Q564. You should always try and shoot for being above par so that you circumstantial.
know you made something for your time. (a) offended (b) unscathed
(a) exquisite (c) spoiled (d) operative
(b) mediocre (e) hidden
(c) navigable
(d) participate Q570. We had to be going lickety-split in order to reach our destination
(e) dwindle on time.
(a) hurriedly (b) extremely
Q565. When she heard us mocking her, she cast down her book and (c) recklessly (d) foolishly
stormed out of the room. (e) dangerously

Facebook Page- https://www.facebook.com/vishalpariharpage Youtube- https://www.youtube.com/vishalparihar


Follow
134 Vishal Sir Telegram Channel- https://t.me/englishbyvishalsirchannel Instagram- https://www.instagram.com/vishalthetrainer
By Vishal sir
Also Important For SBI CLERK, IBPS CLERK, RRB CLERK
& Other Competitive Exams

S561. Ans. (e) without altering the context of the sentence. Hence, option (b)
Sol. The correct word that decipher the meaning of the phrase becomes the most suitable answer choice.
“above board” is “honest” as ‘above board’ means in the open; Annex means add as an extra or subordinate part, especially to a
without dishonesty, concealment, or fraud. Hence, the word document.
‘honest’ provides the intended meaning to the sentence without Adore means love and respect (someone) deeply.
altering the context of the sentence. Hence, option (e) becomes
the most suitable answer choice. S564. Ans. (a)
Subservience means willingness to obey others unquestioningly. Sol. The correct word that decipher the meaning of the phrase
Priority means the fact or condition of being regarded or treated “above par” is “exquisite” as ‘above par’ means
as more important than others. better than average or normal. Hence, the word ‘exquisite’
provides the intended meaning to the sentence without altering
S562. Ans. (d) the context of the sentence. Hence, option (a) becomes the most
Sol. The correct word that decipher the meaning of the phrase suitable answer choice.
“elbow grease” is “toil” as ‘elbow grease’ means vigorously Navigable means (of a website) easy to move around in.
applied physical labor. Hence, the word ‘toil’ provides the Dwindle means diminish gradually in size, amount, or strength.
intended meaning to the sentence without altering the context of
the sentence. Hence, option (d) becomes the most suitable answer S565. Ans. (e)
choice. Sol. The correct word that decipher the meaning of the phrase
Accent means a special or particular emphasis. “cast down” is “doleful” as ‘cast down’ means to make
Gist means the substance or general meaning of a speech or text. discouraged or dejected. Hence, the word ‘doleful’ provides the
Bulge means a rounded swelling which distorts an otherwise flat intended meaning to the sentence without altering the context of
surface. the sentence. Hence, option (e) becomes the most suitable answer
Brim means be full to the point of overflowing. choice.
Impair means weaken or damage (something, especially a faculty
S563. Ans. (b) or function).
Sol. The correct word that decipher the meaning of the phrase Taper means diminish or reduce in thickness towards one end.
“cast aspersions” is “abuse” as ‘cast aspersions’ means an attack Expatiate means speak or write in detail about.
on the reputation or integrity of someone or something. Hence, Slump means sit, lean, or fall heavily and limply.
the word ‘abuse’ provides the intended meaning to the sentence

Facebook Page- https://www.facebook.com/vishalpariharpage Youtube- https://www.youtube.com/vishalparihar


Follow
135 Vishal Sir Telegram Channel- https://t.me/englishbyvishalsirchannel Instagram- https://www.instagram.com/vishalthetrainer
By Vishal sir
Also Important For SBI CLERK, IBPS CLERK, RRB CLERK
& Other Competitive Exams

S566. Ans. (a) Sol. The correct word that decipher the meaning of the phrase
Sol. The correct word that decipher the meaning of the phrase “black dog” is “melancholy” as ‘black dog’ is a metaphorical
“keep heads” is “composed” as ‘keep heads’ means to remain representation of melancholy or depression. Hence, the word
calm and sensible when in an awkward situation that might cause ‘melancholy’ provides the intended meaning to the sentence
a person to panic or go out of control. Hence, the word without altering the context of the sentence. Hence, option (a)
‘composed’ provides the intended meaning to the sentence becomes the most suitable answer choice.
without altering the context of the sentence. Hence, option (a)
becomes the most suitable answer choice. S569. Ans. (b)
Jubilant means feeling or expressing great happiness and Sol. The correct word that decipher the meaning of the phrase
triumph. “scot-free” is “unscathed” as ‘scot-free’ means without suffering
Demented means behaving irrationally due to anger, distress, or any punishment or injury. Hence, the word ‘unscathed’ provides
excitement. the intended meaning to the sentence without altering the context
Elated means make (someone) ecstatically happy. of the sentence. Hence, option (b) becomes the most suitable
answer choice.
S567. Ans. (d) Offended means resentful or annoyed, typically as a result of a
Sol. The correct word that decipher the meaning of the phrase perceived insult.
“dog eat dog” is “ferocious” as ‘dog eat dog’ is used to express Operative means functioning or having effect.
your disapproval of a situation where everyone wants to succeed
and is willing to harm other people in order to do so. Hence, the S570. Ans. (a)
word ‘ferocious’ provides the intended meaning to the sentence Sol. The correct word that decipher the meaning of the phrase
without altering the context of the sentence. Hence, option (d) “lickety-split” is “hurriedly” as ‘lickety-split’ means as fast as
becomes the most suitable answer choice. possible. Hence, the word ‘hurriedly’ provides the intended
Entranced means fill (someone) with wonder and delight, meaning to the sentence without altering the context of the
holding their entire attention. sentence. Hence, option (a) becomes the most suitable answer
Arbitrator means an independent person or body officially choice.
appointed to settle a dispute. Recklessly means without regard to the danger or the
Ferocious means savagely fierce, cruel, or violent. consequences of one's actions; rashly.

S568. Ans. (a)

Facebook Page- https://www.facebook.com/vishalpariharpage Youtube- https://www.youtube.com/vishalparihar


Follow
136 Vishal Sir Telegram Channel- https://t.me/englishbyvishalsirchannel Instagram- https://www.instagram.com/vishalthetrainer
By Vishal sir
Also Important For SBI CLERK, IBPS CLERK, RRB CLERK
& Other Competitive Exams

Directions (571-575): In the following questions, a sentence is (b) budgetary allocation to the
divided into five parts with last part of each sentence is (c) water resource ministry towards
highlighted in bold suggesting the grammatically correct part (d) revival of Ganga is a welcome move
of the sentence. Out of the four other parts, choose the part of (e) no error
the sentence which contains grammatical or contextual error
in it. If the given sentence is both grammatically correct and Q575. Ganesh and his (A)/ friend were (B)/ walking towards the (C)/
contextually meaningful, choose option (e) i.e., “No error” as
station when they (D)/ met his common friend. (E)
your answer.
(a) Ganesh and his
(b) friend were
Q571. Frederick found suitable clothing to (A)/ his guests and in turn,
(B)/ they each retired to the (C)/ one bedchamber of the (D)/ (c) station when they
(d) met his common friend.
cottage to wash and change (E)
(e) no error
(a) Frederick found suitable clothing to
(b) his guests and in turn,
Direction (576-580): Given below is a paragraph consisting of
(c) one bedchamber of the
pairs of words given in bold.
(d) cottage to wash and change
Retired people suddenly have so much time during the day that
(e) no error
they may stop bothering about their routine. Their time of waking
up, shower, breakfast and overall schedule become ___(576)___.
Q572. I started visiting schools (A)/ and talk to kids (B)/ about bullying
They may feel time is moving at a slower pace and they do not
and (C)/ what to do and (D)/ how to deal with it (E)
know what to look forward to. This may ___(577)__ to a hitherto-
(a) and talk to kids
unseen focus on their interpersonal relationships, ____(578)___
(b) about bullying and
about what their children or relatives are up to, etc, which may
(c) what to do and
lead to interference in their near and dear ones' lives. This can
(d) how to deal with it
turn into a troublesome ___(579)____ for those around them.
(e) no error
Shortcomings in their families appear magnified and become the
most important thing that they believe have to be ____(580)___
Q573. Work with team members (A)/ to ensure timelines are realistic
(B)/ and deliverables are scoped to (C)/ allow for confident as top priority.
delivery of the (D)/ highest possible quality in the time allotted.
Q576. (a) rational
(E)
(b) random
(a) Work with team members
(c) erratic
(b) to ensure timelines are realistic
(d) fanciful
(c) and deliverables are scoped to
(e) botch
(d) highest possible quality in the time allotted
(e) no error
Q577. (a) cast
(b) accelerate
Q574. A Rs 2,000 crore hike in (A)/ budgetary allocation to the (B)/
(c) lead
water resource ministry towards (C)/ revival of Ganga is a
welcome move (D)/ by this year’s Union Budget. (E) (d) step
(e) conduct
(a) A Rs 2,000 crore hike in

Facebook Page- https://www.facebook.com/vishalpariharpage Youtube- https://www.youtube.com/vishalparihar


Follow
137 Vishal Sir Telegram Channel- https://t.me/englishbyvishalsirchannel Instagram- https://www.instagram.com/vishalthetrainer
By Vishal sir
Also Important For SBI CLERK, IBPS CLERK, RRB CLERK
& Other Competitive Exams

S571. Ans. (a) it’s possessive adjective should also be plural [their]. Hence,
Sol. The error lies in the first part i.e., part (A) of the sentence. It option (d) becomes the most suitable answer choice.
is to be noted that certain words require specific preposition. We
seem to use "suitable for" when referring to a person or to an "ing" S576. Ans. (c)
verb, and "suitable to" when referring to a thing or a "to" verb. Sol. ‘erratic’ is the correct set of words making the sentence
Therefore, to make the sentence grammatically correct and meaningful.
contextually meaningful replace “to” with “for”. Hence, option The sentence is indicating some negative effect on retired
(a) is the most suitable answer choice. people’s schedule due to plenty of time during the day.
Hence a negative word must fill the blank.
S572. Ans. (a) Erratic means not even or regular in pattern or movement, which
Sol. The error lies in the second part of the sentence i.e., part (B). is similar in meaning with irregular can be used with it
Replace ‘talk’ with ‘talking’ to make the syntax of the sentence interchangeably.
grammatically correct. It is to be noted that in part (A) ‘visiting’ Botch means carry out (a task) badly or carelessly.
is a gerund while the other word in part (B) ‘talk’ is a bare
infinitive i.e., infinitive without ‘to’. Moreover, a gerund and a S577. Ans. (c)
bare infinitive cannot be joined using the conjunction ‘and’. If the Sol. ‘lead’ is the correct set of words making the sentence correct
first word is a gerund the other word to be connected using ‘and and meaningful.
must be a gerund as well. Hence, option (a) is the most suitable The sentence points to the outcome of being retired and feeling
answer choice. like time is moving at slower pace.
Hence option (c) is correct.
S573. Ans. (d)
Sol. The error lies in the last part i.e., part (E) of the sentence, ‘in’
should be replaced with ‘within’ to provide the precise contextual
meaning to the sentence. ‘In’ is specifically defined as being
inside while ‘within’ implies being inside of a given set of
parameters relative to the object it’s linking to and not actually
inside of it. Therefore, option (d) becomes the most suitable
answer choice.

S574. Ans. (e)


Sol. The given sentence is grammatically correct and contextually
meaningful and hence, does not require any corrections.
Therefore, option (e) is the most suitable answer choice.

S575. Ans. (d)


Sol. The error lies in the last part i.e., part (D) of the sentence. To
make the sentence grammatically correct and contextually
meaningful replace ‘his’ with ‘their’. It is to be noted that the
subject of the sentence [Ganesh and his friend] is plural therefore

Facebook Page- https://www.facebook.com/vishalpariharpage Youtube- https://www.youtube.com/vishalparihar


Follow
138 Vishal Sir Telegram Channel- https://t.me/englishbyvishalsirchannel Instagram- https://www.instagram.com/vishalthetrainer
By Vishal sir
Also Important For SBI CLERK, IBPS CLERK, RRB CLERK
& Other Competitive Exams

Q578. (a) uncertain him to riches far different—and far more satisfying—than he ever
(b) weird imagined. Santiago's journey teaches us about the essential
(c) deviation wisdom of listening to our hearts, of recognizing opportunity and
(d) strange learning to read the omens strewn along life's path, and, most
(e) curiosity importantly, to follow our dreams.

Q579. (a) ravel Born in Brazil as a teenager Coelho wanted to become a writer.
(b) tackle His father was an engineer and very practical towards life, so he
(c) experience didn’t understood Coelho’s passion. When his father realized that
(d) valiant he can’t manipulate Paulo, he sent him to mental institution at the
(e) resolute age of 17. There he tried his escape for three times but couldn’t
made it. At the age of 20 finally he was free. After that he joined
Q580. (a) resolved law school and abandoned his idea of becoming a writer. But
(b) dissolved soon after a year he dropped out and started life as nomads,
(c) examined traveling through South America, North America, Mexico and
(d) proposed Europe where he started using drugs. After his return to Brazil he
(e) confirmed started his career as lyricist. At that time he was got arrested by
the ruling military government for his lyrics as they found them
Directions (581-590): Read the following passage carefully leftist and dangerous.
and answer the questions given below it. Certain words are
given in bold to help you locate them while answering some After all of these in 1986 Coelho walked a long Road to Santiago
of the questions. which he calls his time of spiritual awakening, and he have
described it in his book The Pilgrimage. It was the turning point
“When you want something from all your heart, all the universe in his life which gave a very unique concept toward living a life.
will conspire in helping you to achieve it.” — The Alchemist. We “I was happy in doing the things I was doing. I was doing
all have heard this quote in different ways, and got motivation by something that gave me food and water for The Alchemist.”–
these golden words. In same way, the motivation giving is the Paulo Coelho. In 1987, The Alchemist was published with only
man who inspired billions of people and made them believe that 900 copies. But as soon his novel Brida with bigger publication
everyone must have a dream and courage of taking steps to make The Alchemist went off in 1994.The Alchemist has gone on to
them real. Of course things don’t always happen the way we wish sell over 83 million copies and became one of the best-selling
they would. There are moments in which we feel we are seeking books of the history. From a mental institution to nomad and drug
something that is not meant for us, and in those moments, it is addict and then the pilgrim of Road to Santiago de Compostela
best to relax and believe that the Universe is still working for us and now the living legend Coelho teaches us the large number of
secretly, even if we cannot comprehend it. lessons through his life. He found a unique concept of spirituality
for everyone who is in the language of the world of heaven
Paulo Coelho is the Guinness world record for most translated passing through our ears, and that language is love, humanity and
book by a living author. His masterpiece ‘The Alchemist’ has never giving up on our dreams.
been translated in 81 languages. Paulo Coelho's masterpiece tells
the mystical story of Santiago, an Andalusian shepherd boy who
yearns to travel in search of a worldly treasure. His quest will lead

Facebook Page- https://www.facebook.com/vishalpariharpage Youtube- https://www.youtube.com/vishalparihar


Follow
139 Vishal Sir Telegram Channel- https://t.me/englishbyvishalsirchannel Instagram- https://www.instagram.com/vishalthetrainer
By Vishal sir
Also Important For SBI CLERK, IBPS CLERK, RRB CLERK
& Other Competitive Exams

S578. Ans. (e)


Sol. ‘curiosity’ is the right choice.
Noun or pronoun should come before the new clause.
Curiosity is a noun having similar meaning as ‘interest’ that can
be used interchangeably.

S579. Ans. (c)


Sol. ‘experience’ is the correct set of words.
All the other words are adjective and adjective is not followed by
adjective.
Ravel means to untangle something
Valiant means possessing or showing courage or determination.
Resolute means admirably purposeful, determined, and
unwavering.

S580. Ans. (a)


Sol. ‘resolved’ is the correct set of words making the sentence
meaningful and correct. Resolved which means settle or find a
solution to, can also be used as ‘settled’ interchangeably.

Facebook Page- https://www.facebook.com/vishalpariharpage Youtube- https://www.youtube.com/vishalparihar


Follow
140 Vishal Sir Telegram Channel- https://t.me/englishbyvishalsirchannel Instagram- https://www.instagram.com/vishalthetrainer
By Vishal sir
Also Important For SBI CLERK, IBPS CLERK, RRB CLERK
& Other Competitive Exams

Q581. What does the author suggest to do in those moments when we Q585. Which book of Paulo Coelho turned out to be the turning point in
cannot comprehend the meaning of our failure of something that his life that gave a unique concept towards living a life?
is not meant for us? (a)Brida
(a) To learn a language that is of love, humanity and never giving (b)The Pilgrimage
up on our dreams. (c)The Alchemist
(b) To start life as nomads. (d)Alchemist-Graphic Novel
(c) To relax and believe that the Universe is still working for us (e)Eleven minutes
secretly.
(d) To abandon your old idea and start a new life. Q586. Why did The Alchemist go off in 1994?
(e) All of these. (a)As it held concept of religious views that could divide the
society.
Q582. In reference to the passage what is the reason for which Coelho’s (b)Because Paulo Coelho was a drug addict so his readers did not
father couldn’t understand his passion? accept his book.
(a) His father never cared enough about him. (c) As it did not provide the good stuff to read.
(b) As his father was very practical towards life. (d)Paul Coelho’s novel Brida was launched with the bigger
(c) Since he didn’t want him to follow his dreams. publication.
(d) As he didn’t want him to attain fame so soon. (e) All of these.
(e) He thought Coelho might be harmful and dangerous as he
grew up while pursuing his passion Directions (587-588): Find out the word from the given
options which is similar in meaning to the word given in bold
in the passage.
Q583. Why did Paulo’s father sent him to mental institution?
(a)As he tried his escape from home for three times but couldn’t Q587. Inspired
made it. (a)Discouraged (b)Disillusioned
(b)As Paulo was on drugs and his father wanted him to get back (c)Dissuaded (d)Motivated
to normal. (e)Inhibited
(c)As his father realized that he couldn’t manipulate Paulo away
from his passion. Q588. Comprehend
(d) To teach him large number of lessons through his life. (a)Misinterpret (b)Perceive
(e) All of these. (c)Exclude (d)Blunder
(e)Advertise
Q584. When Paulo started his career as lyricist, why was he arrested by
the ruling military government? Directions (589-590): Find out the word from the given option
(a) Because of the drug consumption found on his blood test. which is opposite in meaning to the word given in bold in the
(b) As he was suspected of relations with terrorists. passage.
(c) Since his lyrics were a copy of some other lyricist.
(d) As they found his lyrics leftist and dangerous. Q589. Essential
(e) None of these. (a)Vital (b)Trivial
(c)Necessary (d)Plot
(e)Hunt

Facebook Page- https://www.facebook.com/vishalpariharpage Youtube- https://www.youtube.com/vishalparihar


Follow
141 Vishal Sir Telegram Channel- https://t.me/englishbyvishalsirchannel Instagram- https://www.instagram.com/vishalthetrainer
By Vishal sir
Also Important For SBI CLERK, IBPS CLERK, RRB CLERK
& Other Competitive Exams

S581. Ans.(c) S586. Ans.(d)


Sol. Option (c) is the appropriate choice according to the passage. Sol. Option (d) is the most appropriate choice which is in
Refer to the first paragraph where it is mentioned that there are relevance to the context of the paragraph. Refer to last paragraph
moments in which we feel we are seeking something that is not of the passage where it is mentioned that in 1987, The Alchemist
meant for us, and in those moments, it is best to relax and believe was published with only 900 copies. But as soon his novel Brida
that the Universe is still working for us secretly, even if we cannot with bigger publication The Alchemist went off in 1994.The
comprehend it. Alchemist has gone on to sell over 83 million copies and became
one of the best-selling books of the history.
S582. Ans.(b)
Sol. Option (b) is the most suitable choice according to the S587.Ans. (d)
passage. Refer to second line of the third paragraph of the passage Sol. Option (d) is the most appropriate choice. Inspired means of
where it is mentioned that though Coelho wanted to become a extraordinary quality, as if arising from some external creative
writer from the time he was a teenager but as his father was an impulse.
engineer and very practical towards life, so he didn’t understood Motivated means influenced. Hence it is the only synonym of the
Coelho’s passion. given word.
Discouraged means having lost confidence or enthusiasm;
S583. Ans.(c) disheartened.
Sol. Option (c) is the most appropriate answer according to the Disillusioned means disappointed.
context of the passage. The answer can be deduced from the third Dissuaded means persuade (someone) not to take a particular
paragraph of the passage where it is mentioned that when his course of action.
father realized that he can’t manipulate Paulo, he sent him to Inhibited means unable to act in a relaxed and natural way
mental institution at the age of 17.This was so as Paulo wanted to because of self-consciousness or mental restraint.
become a writer from the teenage, it was his passion but his father
was an engineer and very practical towards life. S588.Ans.(b)
Sol. Comprehend means grasp mentally; understand.
S584. Ans.(d) Perceive means become aware or conscious of (something); come
Sol. Option (d) is the most correct option here. This answer can to realize or understand.
also be inferred from the third paragraph itself where it is given Blunder means a stupid or careless mistake.
that after his return to Brazil he started his career as lyricist and Advertise means publicize information about
at that time he was got arrested by the ruling military government
for his lyrics as they found them leftist and dangerous. S589. Ans.(b)
Sol. Essential- absolutely necessary; extremely important
S585. Ans. (b) Trivial -of little value or importance, is the only word which is
Sol. Option (b) is the correct option here. Refer to last paragraph opposite in meaning of the word Essential
of the passage where it is given that in 1986 Coelho walked a long Plot- a plan made in secret by a group of people to do something
Road to Santiago which he calls his time of spiritual awakening, illegal or harmful.
and he have described it in his book The Pilgrimage. It was the Rest all of the words mean the same.
turning point in his life which gave a very unique concept toward
living a life.

Facebook Page- https://www.facebook.com/vishalpariharpage Youtube- https://www.youtube.com/vishalparihar


Follow
142 Vishal Sir Telegram Channel- https://t.me/englishbyvishalsirchannel Instagram- https://www.instagram.com/vishalthetrainer
By Vishal sir
Also Important For SBI CLERK, IBPS CLERK, RRB CLERK
& Other Competitive Exams

Q590. Abandoned (b)EDCBA


(a)Deserted (c)BECAD
(b)Rejected (d)DCEAB
(c)Inhabited (e)ADBCE
(d)Dropped
(e)Discarded Q595. with vivid pictures (A)/illustrating a life (B)/the prose itself
(C)/detracts the reader’s (D)/ understanding of (E)
Directions (591-600): In the question given below, some (a)DCEAB
sentences/phrases are given which have to be arranged in a (b)EABCD
proper sequence. Select the option which best defines the (c)BADEC
proper sequence and arranges the sentence in an appropriate (d)CDBAE
way. (e)ABECD

Q591. school education to (A)/ be free of cost for (B)/we should make Q596. electrons and protons (B)/matter and electricity (E)/ certain finite
(C)/ they can study easily (D)/girls so that (E)/ units (D)/exclusively in (C)/ are concentrated (A)
(a)BCAED (a)ACBDE
(b)CABED (b)EACDB
(c)EDCAB (c)BDCEA
(d)DCAEB (d)DCEAB
(e)CAEBD (e)CAEBD

Q592. credit disbursement (A)/ pulling back on (B)/ rated Q597. the perfection of our (A)/ to the root of our being (B)/ on our
companies(C)/public Sector Banks are (D)/to lower (E)/ training in truth(C)/personality depends mostly (D)/and love,
(a)DBAEC upon ideals that go (E)/
(b)BAECD (a)CDEAB
(c)ABEDC (b)DACEB
(d)ECDAB (c)ADBEC
(e)CBAED (d)ADCEB
(e)BCDEA
Q593. childlike nature which (A)/ ego while innocence (B) / makes you
feel light(C)/ comes from a (D) /personality comes from (E)/ Q598. had been travelling for days (A)/ had an original beauty (B)/the
(a)EBCDA country through which we (C)/ with stretches of wide (D)/plains
(b)EBDAC across the wide country (E)/
(c)EBACD (a)ACBED
(d)ACDBE (b)DECAB
(e)ABCED (c)BDECA
(d)ABECD
Q594. as a consequence (A)/ of nuclear war (B)/ likelihood of (e)CABDE
extinction(C)/there is a (D)/of the human species (E)
(a)CABDE

Facebook Page- https://www.facebook.com/vishalpariharpage Youtube- https://www.youtube.com/vishalparihar


Follow
143 Vishal Sir Telegram Channel- https://t.me/englishbyvishalsirchannel Instagram- https://www.instagram.com/vishalthetrainer
By Vishal sir
Also Important For SBI CLERK, IBPS CLERK, RRB CLERK
& Other Competitive Exams

S590. Ans.(c) Sol. All of the options are incorrect except option (d) as it
Sol. Abandoned means having been deserted or left. properly arranges the sentence as follows : “The perfection of our
Inhabited means live in or occupy. personality depends mostly on our training in truth and love ,upon
Deserted means left. ideals that go to the root of our being.”
Rejected means left out.
Discarded means thrown out which is not of use. S598. Ans. (e)
Sol. Option (e) is the most appropriate choice. i.e. CABDE as it
S591. Ans. (b) provides the best sequence to arrange the sentence in a proper
Sol. The correct sequence is CABED as it arranges the sentence way as follows : “The country through which we had been
in the best way as follows: “We should make school education to travelling for days had an original beauty with stretches of wide
be free of cost for girls so that they can study easily.” plains across the wide country.”

S592. Ans. (a)


Sol. The correct sequence is DBAEC as it arranges the sentence
properly in the way as follows: “Public Sector Banks are pulling
back on credit disbursement to lower rated companies.”

S593. Ans. (b)


Sol. The correct sequence of the statement is EBDAC as it
arranges the sentence as follows: “Personality comes from ego
while innocence comes from a childlike nature which makes you
feel light.”

S594. Ans. (d)


Sol. Option (d) provides the correct sequence for the arrangement
of the sentence. The proper arranged sentence is as follows:
“There is a likelihood of extinction of the human species as a
consequence of nuclear war.”

S595. Ans. (c)


Sol. The correct sequence is BADEC as the sentence can be
arranged as follows: “Illustrating a life with vivid pictures
detracts the reader’s understanding of the prose itself.”

S596. Ans. (b)


Sol. The correct sequence is EACDB as the sentence can be
arranged as follows: “Matter and electricity are concentrated
exclusively in certain finite units, electrons and protons.”

S597. Ans. (d)

Facebook Page- https://www.facebook.com/vishalpariharpage Youtube- https://www.youtube.com/vishalparihar


Follow
144 Vishal Sir Telegram Channel- https://t.me/englishbyvishalsirchannel Instagram- https://www.instagram.com/vishalthetrainer
By Vishal sir
Also Important For SBI CLERK, IBPS CLERK, RRB CLERK
& Other Competitive Exams

Q599. its mammoth profitability (B)/to Google following the disclosure such as gamma-rays for short periods of time. But the trouble is
about (E)/a greater challenge (D)/and Microsoft posed that they need to be used for _________(607)_______periods. A
(C)/competition from Yahoo (A) recent ______ (608)__________ in the field is to for the
(a)DEACB oncologist to isolate a piece of cancer tissue from the patient, and
(b)ACDEB collaborate with a group of molecular bio-analysts to identify the
(c)BACED neo-antigen on the cancer cells. Next, the oncologist asks an
(d)CEADB immunologist collaborator to prepare the
(e)EDABC __________(609)_________ antibody molecule, which can be
________(610)__________ to the patient so as to stop recurrence
Q600. which true speech emanates (A)/space through the (B)/we also of the tumor. This is thus a therapeutic vaccine (not a preventive
create learning (D)/utter and the silence from (E)/ kind of speech vaccine such as the one against hepatitis or mumps). Some such
we (C) cancer vaccines are already in the market.
(a)ABCED Q601. (a)Common
(b)BEACD (b)Relaxing
(c)DBCEA (c)Uncontrolled
(d)EDCBA (d)Oral
(e)DBACE (e)Inorganic

Directions (601-610): In the following passage, some of the Q602. (a)Connect


words have been left out, each of which is indicated by a (b)Inborn
number. Find the suitable word from the options given (c)Skilled
against each number and fill up the blanks with appropriate (d)Scattered
words to make the paragraph meaningfully complete. (e)Balanced

Cancer is an ______(601)___________ growth and Q603. (a)Mutated


multiplication of cells in a given organ (for example, the lung or (b)Purposed
stomach), which are damaged due to ____(602)________ or (c)Seasoned
external triggers (such as smoking or high doses of radiation). (d)Sprayed
While normal cells are programmed to multiply and grow to a (e)Integrated
certain size and stay so, cancer cells, whose DNA is
______(603)_________ due to such damage, go on a Q604. (a)Rampant
_____(604)__________ growth leading to tumors, weakening (b)Easy
the body and ultimately even death. There have been a variety of (c)Proper
approaches to win over this emperor. Surgical removal of the (d)Poor
tumor has been one option, but it does not guarantee total (e)Bit
removal, nor its recurrence if the original cause is not
________(605)________. Radiation therapy using high power Q605. (a)Praised (b)Appreciated
gamma rays has also been tried, again with (c)Addressed
________(606)________ success. Many doctors have tried (d)Forgiven
combining drugs along with shining the tumor using radiation (e)Convinced

Facebook Page- https://www.facebook.com/vishalpariharpage Youtube- https://www.youtube.com/vishalparihar


Follow
145 Vishal Sir Telegram Channel- https://t.me/englishbyvishalsirchannel Instagram- https://www.instagram.com/vishalthetrainer
By Vishal sir
Also Important For SBI CLERK, IBPS CLERK, RRB CLERK
& Other Competitive Exams

S599. Ans. (b) go on a rampant or say flourishing or spreading unchecked


Sol. Option (b) is the most appropriate choice as it provides the growth. Easy, proper or poor growths are not the suitable words
correct sequence for the arrangement of the sentence as follows: to be filled in the blank given.
“Competition from Yahoo and Microsoft posed a greater
challenge to Google following the disclosure about its mammoth S605. Ans. (c)
profitability.” Sol. The only word that fits most suitably in the blank is option
(c) that is addressed. This is so because the total removal or the
S600. Ans.(c) recurrence can only be addressed .Rest all of the options are
Sol. Option (c) is the most suitable choice as it provides the irrelevant.
correct sequence for the proper arrangement of the sentence Praised means express warm approval or admiration of.
which is as follows: “We also create learning space through the Appreciated means valued.
kind of speech we utter and the silence from which true speech Convinced means completely certain about something.
emanates .”

S601. Ans. (c)


Sol. Option (c) is the most suitable choice. Since multiplication
of cells takes place unevenly and by itself the only word that can
fit the blank is uncontrolled which means not in a controlled
manner.
Inorganic means lifeless.

S602. Ans.(b)
Sol. Option (b) fits in the blank most suitably. The line after the
blank talks about the triggers which are external or they could be
inborn which means genetic due to which cancer is caused. Other
options are irrelevant and they do not fit in the blank.
Skilled means expert at something.
Scattered means throw in various random directions.

S603. Ans.(a)
Sol. Mutated is the correct option to be filled in the blank as it
means (with reference to a cell, DNA molecule, etc.) undergo or
cause to undergo change in a gene or genes. The DNA of cancer
cells cannot be purposed, seasoned, sprayed or integrated. They
can only be mutated. Hence all other options are irrelevant and
only option (a) is correct.

S604. Ans.(a)
Sol. The only word that fits the blank is rampant that is option (a)
as it is given that cancer cells whose DNA is mutated or changed

Facebook Page- https://www.facebook.com/vishalpariharpage Youtube- https://www.youtube.com/vishalparihar


Follow
146 Vishal Sir Telegram Channel- https://t.me/englishbyvishalsirchannel Instagram- https://www.instagram.com/vishalthetrainer
By Vishal sir
Also Important For SBI CLERK, IBPS CLERK, RRB CLERK
& Other Competitive Exams

Q606. (a)Obsessed (b)Disapproved (c)Uninterested nightfall along the Aqueduct, where he would rob them of their
(d)Limited (e)Worried earnings. Afterward, Alves would throw them over the edge of
the 213-foot tall structure, sending them falling to their deaths.
Q607. (a)Reluctant (b)Beneficial (c)Universal Between 1836 and 1839, he repeated this process some 70 times.
(d)Difficult (e)Sustained Local police initially attributed the deaths to copycat suicides,
which led to a temporary closure of the bridge. While the killings
Q608. (a)Approach (b)Imperfection (c)Cruelty on the Aqueduct may have come to a stop, break-ins began
(d)Arrogance (e)Casualty sprouting up among private residences after Alves formed a gang
of murderous robbers to target the wealthier residents of the city.
Q609. (a)Co-operated (b)Specific (c)Acquired The group was caught while killing four people inside the home
(d)Demonstrated (e)Initiated of a local doctor, and Alves was arrested and sentenced to death
by hanging.
Q610. (a)Advanced (b)Disguised (c)Uniformed Many regard Alves as the country’s first serial killer, and the last
(d)Concentrated (e)Injected person to die by hanging, but this simply isn’t the case. A woman
named Luisa de Jesus, who admitted to poisoning 28 children, is
Directions (611-620): Read the following passage carefully the first recorded serial killer in Portugal and was whipped,
and answer the questions given below it. Certain words are hanged, and burned in the streets of Lisbon for her crimes in 1772.
given in bold to help you locate them while answering some Alves, who was hanged to death in February of 1841, may have
of the questions. been among the last people to die as a result of capital punishment
before the country did away with the practice in 1867, but he
In spite of his 1841 execution, Diogo Alves "lives" on to this day. wasn’t the last: Around half a dozen people followed him. Still,
For years, Diogo Alves terrorized the people of Lisbon, Portugal, what was it about Alves that compelled scientists to preserve his
killing or stealing at will. Although he was executed in 1841, he head in a glass jar? For the most part, it was all about trends and
still “lives” on in a bizarre way. Indeed, today, 176 years after timing. At the time of Alves’ execution, phrenology — the belief
his death, his perfectly preserved head can be seen in a glass jar that certain mental or character traits were determined by the
at the University of Lisbon’s Faculty of Medicine. Diogo Alves shape of one’s skull — was taking off. While the “laws”
is considered by many to be Portugal’s first serial killer. He was undergirding the discipline have since been debunked,
born in Galicia in 1810 and traveled to Lisbon as a young child researchers at the time were all too excited by the possibility that
to work as a servant in the affluent homes of the capital city. It they might be able to understand what could make a person like
wasn’t long before young Alves realized that a life of crime was Alves so undeniably evil. As such, his head was removed from
better for turning a profit, and in 1836 he had himself transferred his already lifeless body and moved into the glass jar where it can
to work in a home located on the Aqueduto das Águas Livres, the still be found in today, perfectly preserved for all to see. Not much
Aqueduct of the Free Waters. Less than a half a mile long, the is known about the result of the study on Alves, as little recorded
waterway allowed suburbanites and rural farmers to traverse the evidence, if there ever was any, remains. A second skull, which
rural landscape from above, making their way into the city of belonged to Francisco Mattos Lobo, who murdered a family of
Lisbon. It was along this route that many of these unsuspecting four before throwing their dog out of a window, was examined
commuters met Diogo Alves. Despite the fact that many of the just one year after Alves’ death, in April of 1842.His head can be
workers who traveled long and far to reach the city were no more found in its own glass jar, housed right down the hall from that of
than humble farmers entering Lisbon to sell off their harvests, Diogo Alves.
Alves targeted them. Awaiting their return home, he met them by

Facebook Page- https://www.facebook.com/vishalpariharpage Youtube- https://www.youtube.com/vishalparihar


Follow
147 Vishal Sir Telegram Channel- https://t.me/englishbyvishalsirchannel Instagram- https://www.instagram.com/vishalthetrainer
By Vishal sir
Also Important For SBI CLERK, IBPS CLERK, RRB CLERK
& Other Competitive Exams

S606. Ans.(d) Sol. Option (e) is the most suitable choice. Here the antibody
Sol. Option (d) is the most appropriate answer. As the line talks molecule can be injected into the patient which means inserted or
about a trial about the radiation therapy using high power gamma introduced into the patient. Rest all other options do not fit in the
rays ,the option can only be limited success. Rest all the other blank given.
options do not fit in the blank in terms of success. Advanced means new and not yet generally accepted.
Obsessed means preoccupy or fill the mind of (someone) Disguised means give (someone or oneself) a different
continually and to a troubling extent. appearance in order to conceal one's identity.
Disapproved means have or express an unfavorable opinion. Uniformed means make of a similar form or character to another
Worried means anxious or troubled about actual or potential or others.
problems. Concentrated means wholly directed to one thing; intense.
Uninterested means not interested in or concerned about
something or someone.

S607. Ans.(e)
Sol. Option (e) is the choice to be filled in the blank. Sustained
means continuing for an extended period or without interruption.
Reluctant means unwilling and hesitant ; disinclined.
Beneficial means resulting in good; favorable or advantageous.
Universal means general.

S608. Ans. (a)


Sol. Option (a) is the most suitable choice.
Approach means a way of dealing with a situation or problem.
Imperfection means a fault, blemish, or undesirable feature.
Cruelty means cruel behavior or attitudes.
Arrogance means pride, egotism.
Casualty means a person killed or injured in a war or accident.

S609. Ans.(b)
Sol. Option (b) is the most suitable choice. Here the sentence talks
about the some particular antibody molecule which can be
defined by the only word specific in the options given. Rest all of
the options are irrelevant.
Co-operated means work jointly towards the same end.
Acquired means learned or developed.
Demonstrated means clearly show the existence or truth of
(something) by giving proof or evidence.
Initiated means caused to begin.

S610. Ans. (e)

Facebook Page- https://www.facebook.com/vishalpariharpage Youtube- https://www.youtube.com/vishalparihar


Follow
148 Vishal Sir Telegram Channel- https://t.me/englishbyvishalsirchannel Instagram- https://www.instagram.com/vishalthetrainer
By Vishal sir
Also Important For SBI CLERK, IBPS CLERK, RRB CLERK
& Other Competitive Exams

Q611. How Diogo Alves is assumed to be living on to this day even after (b) The branch of medicine involving study and treatment of the
he got executed in 1814? blood.
(a) As his spirit is seen wandering in the streets of Lisbon. (c) The study of human behavior and social organization from a
(b) As he is considered an idol in his city and worshipped for all biological perspective.
the good deeds he did. (d)Mental or character traits can be determined by the shape of
(c)His head is still preserved perfectly in a glass jar at the the human skull.
University of Lisbon’s Faculty of Medicine. (e) The use of the fingers and hands to communicate and convey
(d) His whole body is still preserved in the University of Lisbon’s ideas.
Faculty of Medicine.
(e) None of these. Q616. Why did researchers at that time remove Alves head from his
body and preserve it in a jar?
Q612. Whom did Alves target as his victims? (a) As his was a great and intelligent mind known in the history.
(a) Men and women suspected of wrong doings in Lisbon. (b) To study all normal human behaviors.
(b) Women he considered unfaithful to their husbands. (c) To serve it as a piece of antique and place it in a museum.
(c) Affluent businessmen of Lisbon prospering in their dealings. (d) As they wanted to know what could make a person so
(d) Ordinary farmers entering Lisbon to sell off their harvests. undeniably evil.
(e) Killed people out of personal hostility. (e) None of these.

Q613. After robbing farmers of their earnings, what methodology would Directions (617-618) : Find out the word from the given
Alves adopt to kill them? options which is similar in meaning to the word given in bold
(a) By torturing them and making them commit suicide from 213 in the passage.
foot tall building.
(b) By burning them alive on the top of 213 foot tall structure. Q617. Bizarre
(c) Throwing them over the edge of the 213 foot tall structure. (a)Normal (b)Regular
(d) By giving them the over excess drug mixture and making (c)Unusual (d)Ordinary
them suffer to death. (e)Standard
(e) None of these.
Q618. Affluent
Q614. How were the deaths of the farmers attributed by the local police (a)Deprived (b)Prosperous
initially? (c)Destitute (d)Needy
(a) Luisa de Jesus admitted of poisoning the farmers. (e)Penniless
(b) Murders carried out by Francisco Mattos Lobo.
(c) Suicides carried out in an imitation of one another. Directions (619-620) : Find out the word from the given
(d) That they were burned in the streets of Lisbon. option which is opposite in meaning to the word given in bold
(e) None of these. in the passage.

Q615. According to the passage, the main focus of phrenology is on Q619. Humble
(a) Seeking correlations between dreaming and current (a)Modest (b)Meek
knowledge about the functions of the brain. (c)Pretentious (d)Unassertive
(e)Blamed

Facebook Page- https://www.facebook.com/vishalpariharpage Youtube- https://www.youtube.com/vishalparihar


Follow
149 Vishal Sir Telegram Channel- https://t.me/englishbyvishalsirchannel Instagram- https://www.instagram.com/vishalthetrainer
By Vishal sir
Also Important For SBI CLERK, IBPS CLERK, RRB CLERK
& Other Competitive Exams

character traits were determined by the shape of one’s skull —


S611. Ans. (c) was taking off.”
Sol. Option (c) is the most appropriate choice to be selected as
the answer.The answer can be inferred from the first paragraph of S616. Ans.(d)
the passage where it is mentioned that although he was executed Sol. Option (d) is the correct choice. The answer can be deduced
in 1841, he still “lives” on in a bizarre way. Indeed, today, 176 from the second last paragraph of the passage where it is given
years after his death, his perfectly preserved head can be seen in that while the “laws” undergirding the discipline have since been
a glass jar at the University of Lisbon’s Faculty of Medicine debunked, researchers at the time were all too excited by the
possibility that they might be able to understand what could make
S612. Ans. (d) a person like Alves so undeniably evil.
Sol. Option (d) is the most appropriate choice. The answer can be
quoted from the text mentioned in the paragraph 2 itself which is S617. Ans. (c)
given as follows: “Despite the fact that many of the workers who Sol. Bizarre means very strange or unusual.
traveled long and far to reach the city were no more than humble Rest all the words are opposite of the given word.
farmers entering Lisbon to sell off their harvests, Alves targeted
them.” S618. Ans. (b)
Sol. Affluent means having a great deal of money; wealthy.
Prosperous means bringing wealth and success.
S613. Ans.(c) Deprived means underprivileged.
Sol. Option (c) is the most suitable choice as the form of the Destitute means extremely poor and lacking the means to provide
answer. The answer can be deduced from the second paragraph for oneself.
itself where the overall methodology of Alves of how he used to Penniless means having no money ; very poor.
kill farmers is mentioned. It is given that awaiting their return
home, he met them by nightfall along the Aqueduct, where he S619. Ans. (c)
would rob them of their earnings. Afterwards he would throw Sol. Humble means having or showing a modest or low estimate
them over the edge of the 213 foot tall structure, sending them of one's importance.
falling to their deaths. Pretentious means artificial, overambitious.
Modest means unassuming in the estimation of one's abilities or
S614. Ans. (c) achievements.
Sol. Option (c) is the most appropriate choice. The answer can be Meek means quiet, gentle, and easily imposed on; submissive.
inferred from the second paragraph of the passage itself where it Unassertive means (of a person) not having or showing a
is given that local police initially attributed the deaths to copycat confident and forceful personality.
suicides, which led to a temporary closure of the bridge.

S615. Ans.(d)
Sol. Option (d) is the correct choice. The answer can be deduced
from the 3rd paragraph of the passage where it is given that “For
the most part, it was all about trends and timing. At the time of
Alves’ execution, phrenology — the belief that certain mental or

Facebook Page- https://www.facebook.com/vishalpariharpage Youtube- https://www.youtube.com/vishalparihar


Follow
150 Vishal Sir Telegram Channel- https://t.me/englishbyvishalsirchannel Instagram- https://www.instagram.com/vishalthetrainer
By Vishal sir
Also Important For SBI CLERK, IBPS CLERK, RRB CLERK
& Other Competitive Exams

Q620. Compelled (e)All are correct.


(a)Obliged
(b)Constrained Q625. (a)Learning is an important for both people and organisations.
(c)Self-ruling (b)Good advertising can make people buy your products even
(d)Forced they sucks.
(e)Bound (c)Making people laugh is trickable.
(d)The delegation of any power, to any person, is never absolute.
Directions (621-625): In each of the questions given below, (e)All are correct.
there is a statement which is correct grammatically. Mark
that option as your answer. All of the options can also be Directions (626-630): Each question below has one blank,
correct, In that case, mark option (e) as your answer. which is indicating that something has been omitted. Find out
which option can be used to fill up the blank in the sentence
Q621. in the same sequence to make it meaningfully complete.
(a) I could clearly seen the site since it was dark.
(b) We are all atheists about most of the gods that humanity has Q626. Personal and professional _______________ sometimes conflict.
ever believed in. (a)Prevalence
(c) Seat reservation in colleges is a case of haste action. (b)Amount
(d) He is usual tense but today he appears rather restless. (c)Defense
(e) All are correct. (d)Ethics
(e)Occupy
Q622. (a)To a large extent, many cooperative societies need advice and
guidance. Q627. The party would make a pleasant _____________ in his rather
(b)The investigation was confine to manufacturing firms in the dull social life.
area. (a)Obstruction
(c)They quickly come back with pitchers laden with water. (b)Alienation
(d)This is another kind of signing behavior. (c)Distraction
(e)All are correct (d)Diversion
(e)Deliberation
Q623. (a)India national pastime is cricket.
(b)Christopher Reeve confirm a faith in life that will not be easily Q628. She knew that society would ______________ her for leaving her
forgotten. children.
(c)Forecasting the weather has always been a difficult business. (a)Treat (b)Condemn
(d)He himself didn’t have many hope of recovery. (c)Contrast (d)Lose
(e)All are correct (e)Exhaust

Q624. (a)We seldom examined the basic character of its anatomy. Q629. Priya appeared to be in a _______________mood and was deep
(b)The process continued for millions of year. in thought.
(c)The decline in write skills can be stopped. (a)Responsible (b)Tuning
(d) Financial services is up against tight liquidity and falling (c)Engaging (d)Destructive
markets. (e)Introspective

Facebook Page- https://www.facebook.com/vishalpariharpage Youtube- https://www.youtube.com/vishalparihar


Follow
151 Vishal Sir Telegram Channel- https://t.me/englishbyvishalsirchannel Instagram- https://www.instagram.com/vishalthetrainer
By Vishal sir
Also Important For SBI CLERK, IBPS CLERK, RRB CLERK
& Other Competitive Exams

S620. Ans. (c) will be learning is important. Option (c) will be ‘making people
Sol. Compelled means forced to do something. laugh is tricky.’ In option (b), even if will be used and not just
Obliged means make (someone) legally or morally bound to do even.
something.
Constrained means compelled or forced (someone) to follow a S626. Ans.(d)
particular course of action. Sol. Option (d) is the most suitable choice to be filled in the blank.
Bound means a limitation or restriction on feeling or action. Confliction can only happen in ethics.
Rest all of the options do not fit in the blank.
S621. Ans. (b) Prevalence means commonness, generality.
Sol. Option (b) is the only choice among all the given options Defense means the action of defending from or resisting attack.
which is grammatically correct. In option (a) ‘see’ will be used
instead of ‘seen’. In option (d) ‘usually’ will be used instead of S627. Ans.(d)
‘usual’. Sol. The most suitable option to be filled in the blank is option
(d) i.e. diversion. From a dull social life , diversion would mean
S622. Ans. (a) the action of turning something aside from its course.
Sol. Only option (a) is the one which is grammatically correct Obstruction means a thing that impedes or prevents passage or
among all the given options. In option (b) , ‘confined’ will be progress; an obstacle or blockage.
used instead of ‘confine’ as the sentence is referring about the Alienation means the state or experience of being alienated ,
event happened in past. Option (c) will also take the past form of isolation.
‘come’ that is ‘came’. In option (d), this is another kind of sign Distraction means a thing that prevents someone from
behavior is an appropriate usage and not ‘signing’ behavior. concentrating on something else.
Deliberation means long and careful consideration or discussion,
S623. Ans.(c) contemplation.
Sol. Option (c) is the most appropriate answer as it is the choice
which is grammatically correct. In option (a) , it must be “India’s S628. Ans.(b)
national pastime is cricket.” Option (b) will take a past form of Sol. The most appropriate option to be filled in the blank is option
verb ‘confirm’. So it will be ‘confirmed’. The correction for (b).For leaving children , society would condemn a lady which
option (d) will be ‘He himself didn’t have much hope of means express complete disapproval of; censure.
recovery”. Contrast means dissimilarity.
Exhaust means make someone feel very tired.
S624. Ans.(a)
Sol. Option (a) is the best answer choice which is grammatically S629. Ans.(e)
correct. In option (b), ’years’ will be used instead of ‘year’. The Sol. The most suitable option to be filled in the blank is option (e)
correction for option (c) is “The decline in writing skills can be i.e. introspective which means self- examining.
stopped”. In option (d) as financial services is used and not Tuning means telling something to someone.
financial service, ‘are’ will be used and not ‘is’. Engaging means participating or becoming involved in .
Destructive means causing great and irreparable damage.
S625. Ans. (d)
Sol. Option (d) is the only option which is grammatically correct.
In option (a), article ‘an’ will not be used before important, so it

Facebook Page- https://www.facebook.com/vishalpariharpage Youtube- https://www.youtube.com/vishalparihar


Follow
152 Vishal Sir Telegram Channel- https://t.me/englishbyvishalsirchannel Instagram- https://www.instagram.com/vishalthetrainer
By Vishal sir
Also Important For SBI CLERK, IBPS CLERK, RRB CLERK
& Other Competitive Exams

Q630. All the respondents should express their _______________views One thing that’s great about stories is how quickly they can
in this questionnaire. ________(636)__________ your life. Maybe you start reading
(a)Forced (b)Forthright (c)Strongly one over your lunch break and, if it’s the right one, before that
(d)Critically (e)Attacked peanut butter cup you brought for dessert even has a chance to
finish its melting shape-shift into some kind of sugary cement,
Directions (631-635): Below are given some sentences in the whole world has been destroyed around you and then
which a part is given in bold which is either an idiom or a _________(637)__________, and nothing is quite the same
phrase. Five options are given below each sentence. Select the again. This happens whether you like it or not. Great stories
option which is the appropriate meaning of the idiom or the practice this violent beauty on you in a variety of ways: some by
phrase. making an ________(638)____________ world familiar (or vice
versa), some with a slow burn , some with a voice that colonizes
Q631. It seems like they all turned a blind eye to the fact that they were your thoughts. Some do it quietly, almost without you even
creating something truly terrible. noticing, and some do it with high wire acts of imagination or
(a)Discovered (b)Organized (c)Horrified ________(639)_________ that make you into a breathless
(d)Ignored (e)Troubled witness. The trick, then, is finding the right story, one that is
Q632. Don’t spill the beans that we’ll be there by 6 for the surprise capable of such a thing. This is no easy task. Tastes differ, of
party. course, and it can be confusing to spot the small boat of a great
(a)Skip (b)Enrich (c)Reveal story on the wide sea of fiction. Ultimately what any reader can
(d)Economize (e)Disagree offer you in terms of _________(640)________ is actually the
same thing that any good writer can offer you with the story itself.
Q633. The teacher had to put up with a great deal of nonsense from the
new students. Q636. (a)Allow (b)Ruin (c)Theoretical
(a)Manifest (b)Tolerate (c)Allow (d)Climb (e)Network
(d)Explore (e)Augment
Q637. (a)Inspected (b)Dreamt (c)Rebuilt
Q634. She thinks they look down on her because she didn't go to (d)Fooled (e)Arranged
university.
(a)Despise (b)Scrutinize (c)Cherish Q638. (a)Absurd (b)Revised (c)Availed
(d)Punctuate (e)Inculcate (d)Captured (e)Great

Q635. The authorities have urged people to stock up on fuel in case Q639. (a)Acceptance (b)Consolation (c)Intellect
hostilities break out. (d)Emerging (e)Growth
(a)Alter (b)Erupt (c)Quote
(d)Desert (e)Demon Q640. (a)Simplicity
Directions (636-640): In the following passage, some of the (b)Enhancing
words have been left out, each of which is indicated by a (c)Dramatize
number. Find the suitable word from the options given (d)Guidance
against each number and fill up the blanks with appropriate (e)Escape
words to make the paragraph meaningfully complete.

Facebook Page- https://www.facebook.com/vishalpariharpage Youtube- https://www.youtube.com/vishalparihar


Follow
153 Vishal Sir Telegram Channel- https://t.me/englishbyvishalsirchannel Instagram- https://www.instagram.com/vishalthetrainer
By Vishal sir
Also Important For SBI CLERK, IBPS CLERK, RRB CLERK
& Other Competitive Exams

S630. Ans. (b) Sol. ‘Look down on’ means to look down on someone means to
Sol. Only option (b) fits appropriately in the blank. Rest all of the consider that person to be inferior or unimportant, usually when
options are irrelevant. this is not true; despise.
Forthright means candid, outspoken. Scrutinize means examine or inspect closely and thoroughly.
Critically (adverb) means in a way that expresses disapproval. Cherish means protect and care for (someone) lovingly.
Attacked means criticize or oppose fiercely and publicly. Punctuate means occur at intervals throughout (an area or period).
S631. Ans.(d) Inculcate means instil (an idea, attitude, or habit) by persistent
Sol. ‘Turn a blind eye’ means pretend not to notice. instruction.
Discovered means located. S635.Ans.(b)
Organized means arranged or structured in a systematic way. Sol. Option (b) is the most apt choice. ‘Break out’ is a phrasal
Horrified means fill with horror; shock greatly. verb which means erupt or start suddenly.
Troubled means beset by problems or difficulties. Alter means change in character or composition, typically in a
S632. Ans.(c) comparatively small but significant way.
Sol. ‘Spill the beans’ means reveal secret information Quote means repeat a statement by (someone).
unintentionally or indiscreetly. Desert means abandon (a person, cause, or organization) in a way
Skip means an act of passing over part of a sequence of data or considered disloyal or treacherous.
instructions. Demon means an evil spirit or devil, especially one thought to
Enrich means improve or enhance the quality or value of. possess a person or act as a tormentor in hell.
Economize means spend less; reduce one's expenses. S636. Ans. (b)
Disagree means have or express a different opinion. Sol. The most suitable option here is option (b) which also
S633. Ans.(b) matches with the context of the passage and the line. Theoretical,
Sol. ‘Put up with’ is a phrasal verb which means to tolerate or to climb and network are not in a right form to be used in a given
endure. Rest all of the options are irrelevant and do not suit as the blank. Hence these are grammatically incorrect. Allow does not
appropriate meaning of the given phrasal verb. fit contextually.
Manifest means clear or obvious to the eye or mind. S637. Ans. (c)
Explore means inquire into or discuss (a subject) in detail. Sol. The most appropriate option here is option (c) which suits in
Augment means make (something) greater by adding to it; the given blank as the author talks about the world stories create
increase. that is being destroyed and then rebuilt that is the originally built
S634. Ans.(a) world is broken and built again.

Facebook Page- https://www.facebook.com/vishalpariharpage Youtube- https://www.youtube.com/vishalparihar


Follow
154 Vishal Sir Telegram Channel- https://t.me/englishbyvishalsirchannel Instagram- https://www.instagram.com/vishalthetrainer
By Vishal sir
Also Important For SBI CLERK, IBPS CLERK, RRB CLERK
& Other Competitive Exams

Inspected means examined (someone or something) to ensure that Dramatize means exaggerate the seriousness or importance of (an
they reach an official standard. incident or situation).
Dreamt means indulge in daydreams or fantasies about something Escape means an act of breaking free from confinement or
greatly desired. control.
S638. Ans. (a)
Sol. The most suitable option to be filled in the blank is absurd
which means illogical. In the statement the author is talking about
making illogical world familiar, some with a voice that colonizes
your thoughts. Rest of all the options are irrelevant. Revised
means reviewed.
Availed means utilized.
Captured means taken into one's possession or control by force.
S639. Ans.(c)
Sol. Option (c) is the most suitable choice to be filled in the given
blank.
Intellect means the faculty of reasoning and understanding
objectively, especially with regard to abstract matters.
Consolation means the comfort received by a person after a loss
or disappointment.
Emerging means becoming apparent or prominent.
Growth means extension.

S640. Ans.(d)
Sol. ‘Guidance’ can only be used in the given blank. As the reader
can only offer guidance which is the same thing that any good
writer can offer you with the story itself. Rest of all the options
are irrelevant.
Simplicity means the quality or condition of being easy to
understand or do.

Facebook Page- https://www.facebook.com/vishalpariharpage Youtube- https://www.youtube.com/vishalparihar


Follow
155 Vishal Sir Telegram Channel- https://t.me/englishbyvishalsirchannel Instagram- https://www.instagram.com/vishalthetrainer
By Vishal sir
Also Important For SBI CLERK, IBPS CLERK, RRB CLERK
& Other Competitive Exams

Directions: Read the following passage carefully and answer controversies here. We may leave welfare undefined, and for
the questions given below it. Certain words are given in bold present purpose need not distinguish the community from the
to help you locate them while answering some of the state. The welfare of this whole exists, of course, nowhere outside
questions. the individuals, and the individuals again have rights and duties
only as members in the whole. This is the revived Hellenism —
or we may call it in the organic view of things — urged by
That the doctrines connected with the name of Mr Darwin are
German Idealism early in the present century.
altering our principles has become a sort of commonplace thing
to say. And moral principles are said to share in this general
transformation. Now, to pass by other subjects, I do not see why Q641. What is most probably the author's opinion of the existing moral
Darwinism need change our ultimate moral ideas. It was not to principles of the people?
modify our conception of the end, either for the community, or A. He thinks they have to be revamped in the light of Darwinism.
the individual, unless we have been holding views, which long B. He thinks that they are okay as they are and do not need any major
before Darwin were out of date. As to the principles of ethics I change.
perceive, in short, no sign of revolution. Darwinism has indeed C. He thinks that it may be a good idea to have a modicum of the
helped many to truer conception of the end, but I cannot admit immortal Darwinism in us.
that it has either originated or modified that conception. And yet D. He thinks they have to be impoverished in the light of Darwinism.
in ethics Darwinism after all perhaps be revolutionary, it may lead E. Cannot be determined from the passage
not to another view about the end, but to a different way of
regarding the relatively importance of the means. For in the Q642. According to the author, the doctrines of Mr Darwin
ordinary moral creed those means seem estimated on no rational _______.
principle. Our creed appears rather to be an irrational mixture of A. have changed our physical and moral principles.
jarring elements. We have the moral code of Christianity, B. have to be re-evaluated to correct the faults endemic in them.
accepted in part; rejected practically by all save a few fanatics. C. do not have to change our moral ideas.
But we do not realise how in its very principle the Christian ideals D. are actually new versions of old moral rules.
is false. And when we reject this code for another and in part a
E. None of these
sounder morality, we are in the same condition of blindness and
of practical confusion. It is here that Darwinism, with all the
tendencies we may group under that name, seems destined to Q643. What, according to the passage, is the Chief Good?
intervene. It will make itself felt, I believe, more and more A. Being good and kind to all fellow human beings.
effectually. It may force on us in some points a correction of our B. The greatest good of the greatest number.
moral views, and a return to a non-Christian and perhaps a C. The welfare of the community realised in its members.
Hellenic ideal. I propose to illustrate here these general D. The fulfilment of self-interest vested in an individual.
statements by some remarks on Punishment. E. Cannot be determined from the passage.

Darwinism, I have said, has not even modified our ideas of the Q644. It is implied in the passage that _______.
Chief Good. We may take that as — the welfare of the community A. a Hellenic ideal is not a proper substitute of the Christian ideal.
realised in its members. There is, of course, a question as to B. what mankind needs is a Hellenic ideal rather than a Christian
meaning to be given to welfare. We may identify that with mere one.
pleasure, or gain with mere system, or may rather view both as C. Darwinism is more Christian than Hellenic.
inseparable aspects of perfection and individuality. And the D. fanatics do not understand what Darwinism really is.
extent and nature of the community would once more be a subject E. activists do not try to comprehend the meaning of Darwinism.
for some discussion. But we are forced to enter on these

Facebook Page- https://www.facebook.com/vishalpariharpage Youtube- https://www.youtube.com/vishalparihar


Follow
156 Vishal Sir Telegram Channel- https://t.me/englishbyvishalsirchannel Instagram- https://www.instagram.com/vishalthetrainer
By Vishal sir
Also Important For SBI CLERK, IBPS CLERK, RRB CLERK
& Other Competitive Exams

641.(A)
642.(C)
643.(C)
644.(D)

Facebook Page- https://www.facebook.com/vishalpariharpage Youtube- https://www.youtube.com/vishalparihar


Follow
157 Vishal Sir Telegram Channel- https://t.me/englishbyvishalsirchannel Instagram- https://www.instagram.com/vishalthetrainer
By Vishal sir
Also Important For SBI CLERK, IBPS CLERK, RRB CLERK
& Other Competitive Exams

Q645. According to the author, the moral code of Christianity audience were male. How can we affect change in the world when
________. only half of it is invited or feel welcome to participate in the
A. is not followed by most people. conversation? Men—I would like to take this opportunity to
B. is in danger due to opposition of Darwinism. extend your formal invitation. Gender equality is your issue too.
C. is followed by a vast majority of people.
D. is totally ignored by all true Christians. Q646. Why can’t all women afford the same rights of feminism today?
E. is followed wholeheartedly by most of the people. A. Because of the patriarchal society
B. Because feminism is involved with anti-men notion
Directions: Read the following passage carefully and answer C. Because not everyone advocates for women's rights
the questions given below it. Certain words are given in bold D. Because feminism is complicated
to help you locate them while answering some of the E. Because feminism is an overrated topic
questions.
Q647. What is the difference between Gender Equality and Feminism?
I decided I was a feminist and this seemed uncomplicated to me. A. Feminism puts women at the top of the hierarchy since they give
But my recent research has shown me that feminism has become birth to the next generation.
an unpopular word. Apparently, I am among the ranks of women B. Feminism and Gender Equality are inter-related
whose expressions are seen as too strong, too aggressive, C. Feminism is an anti-men movement
isolating, antimen and, unattractive. Why is the word such an D. Both B and C
uncomfortable one? I am from Britain and think it is right that as E. None of the above
a woman I am paid the same as my male counterparts. I think it
is right that I should be able to make decisions about my own Q648. Why is Feminism an issue for men too?
body. I think it is right that women be involved on my behalf in A. Because men think they are incapable of leading the world
the policies and decision-making of my country. I think it is right B. Because men think women are more capable of leading the world
that socially I am afforded the same respect as men. But sadly, I C. Because every individual should have equal rights
can say that there is no one country in the world where all women D. Because feminism directly involves men
can expect to receive these rights. No country in the world can E. None of the above
yet say they have achieved gender equality. These rights I
consider to be human rights but I am one of the lucky ones. My Q649. Why can’t any country in the world yet say they have achieved
life is a sheer privilege because my parents didn’t love me less gender equality?
because I was born a daughter. My school did not limit me A. Because women are still restricted
because I was a girl. My mentors didn’t assume I would go less B. Because women are better in household works
far because I might give birth to a child one day. These C. Because world leaders are misogynist
influencers were the gender equality ambassadors that made me D. Because the reports show it clearly
who I am today. They may not know it, but they are the E. All of the mentioned
inadvertent feminists who are changing the world today. And we
need more of those. Q650 “Only 30 per cent of her audience were male”. Why?
And if you still hate the word—it is not the word that is important A. Feminism is an anti-men movement
but the idea and the ambition behind it. Because not all women B. Feminism is preached wrong
have been afforded the same rights that I have. In fact, C. Most men are misogynist
statistically, very few have been. In 1995, Hilary Clinton made a D. Most men see through the problem
famous speech in Beijing about women’s rights. Sadly, many of E. None of the above
the things she wanted to change are still a reality today. But what
stood out for me the most was that only 30 per cent of her

Facebook Page- https://www.facebook.com/vishalpariharpage Youtube- https://www.youtube.com/vishalparihar


Follow
158 Vishal Sir Telegram Channel- https://t.me/englishbyvishalsirchannel Instagram- https://www.instagram.com/vishalthetrainer
By Vishal sir
Also Important For SBI CLERK, IBPS CLERK, RRB CLERK
& Other Competitive Exams

645.(A)

646.(A)
Because of the patriarchal society
647.(C)
Feminism is an anti-men movement
648.(C)
Because every individual should have equal rights
649.(D)
Because the reports show it clearly
650.(E)
None of the above

Facebook Page- https://www.facebook.com/vishalpariharpage Youtube- https://www.youtube.com/vishalparihar


Follow
159 Vishal Sir Telegram Channel- https://t.me/englishbyvishalsirchannel Instagram- https://www.instagram.com/vishalthetrainer
By Vishal sir
Also Important For SBI CLERK, IBPS CLERK, RRB CLERK
& Other Competitive Exams

Directions Read the following passage carefully and answer


the questions given below it. Certain words are given in bold Q653.) What causes the supercell to tilt downward toward the ground?
to help you locate them while answering some of the a.) wind
questions. b.) warm air
A tornado is born from a powerful storm called a supercell. c.) rain and hail
Tornadoes have been reported in all states, but most tornadoes d.) lightning
happen in the central parts of America called “Tornado Alley.” In
some supercells, warm, moist air rises quickly into the Q654.) When are tornadoes most devastating?
atmosphere. Winds blowing at different speeds at different parts a.) When they hit in Tornado Alley
of the supercell produce wind shear and cause a horizontal, b.) When wind shear occurs
rotating column of air. A funnel cloud will form as the air column c.) When they register on the Fujita Scale
rotates faster and more tightly within the supercell. The rain and d.) When they hit areas with lots of people
hail within the storm cause the funnel cloud to touch the ground,
resulting in a tornado. The strength of a tornado is measured by Q655.) Which of the following would complete the analogy:
what’s called the Fujita scale. The weakest tornadoes (F0) feature Supercell : Tornado ::
winds of 40–78 miles per hour, while the strongest tornadoes (F5) d.) When they hit areas with lots of people
have winds of up to 318 miles per hour. All tornadoes can be b.) Moon : Stars
devastating, especially if they touch down in areas with lots of c.) Hurricane : Ocean
people. A tornado outbreak occurs when one storm system d.) Thunder : Rain
produces multiple tornadoes. Some tornado outbreaks can result Q656.) What is the theme of the first paragraph?
in the formation of dozens of tornadoes over several states. One a.) Historic Tornadoes
particularly powerful tornado outbreak occurred between April b.) Wind Speed
25 and April 28 of 2011, where a record 355 tornadoes in 21 states c.) The Fujita Scale
and Canada were recorded, including an F5 tornado that d.) Birth and Strength of a Tornado
completely destroyed parts of Tuscaloosa, Alabama. Much of the
destruction was caught on camera and broadcast across the Q657.) Why does the author refer to the hailstones that hit Virginia as
country and internet. The same weather system produced part of the tornado outbreak of April 2011?
hailstones that measured 4.5 inches across in southern Virginia. a.) To show that much of the damage was caught on camera
328 people were killed as a result of the outbreak, which totaled b.) To show how long the storm lasted
over $11 billion in damages. c.) The show that tornado outbreaks often happen in the spring
d.) To show how powerful and destructive the storms were
Q651.) Which is NOT true about tornadoes? Q658.) In a tornado outbreak… (select all that are true)
a.) They are born from supercells. a.) A single storm system can produce multiple tornadoes.
b.) They only occur in some states. b.) Only F5 tornadoes occur.
c.) Tornado strength is measured on the Fujita scale. c.) Storms that produce damaging hail can occur as well.
d.) Most tornadoes occur in North America in Tornado d.) Dozens of tornadoes can form in distant locations.
Alley.
Q659.) What did paragraph two include that paragraph one did not
Q652.) What states have never had tornadoes? include?
a.) Alaska and Hawaii a.) information about the Fujita scale
b.) The passage doesn't say. b.) information on when tornadoes become very destructive
c.) States outside of Tornado Alley c.) information about how tornadoes form
d.) All states have had tornadoes. d.) a specific instance

Facebook Page- https://www.facebook.com/vishalpariharpage Youtube- https://www.youtube.com/vishalparihar


Follow
160 Vishal Sir Telegram Channel- https://t.me/englishbyvishalsirchannel Instagram- https://www.instagram.com/vishalthetrainer
By Vishal sir
Also Important For SBI CLERK, IBPS CLERK, RRB CLERK
& Other Competitive Exams

651.b- They only occur in some states.


652.d- All states have had tornadoes
653. c) rain and hail
654. d.) When they hit areas with lots of people
655. d.) When they hit areas with lots of people
656. d.) Birth and Strength of a Tornado
657. d.) To show how powerful and destructive the storms were
658. d.) Dozens of tornadoes can form in distant locations.
659. d.) a specific instance

Facebook Page- https://www.facebook.com/vishalpariharpage Youtube- https://www.youtube.com/vishalparihar


Follow
161 Vishal Sir Telegram Channel- https://t.me/englishbyvishalsirchannel Instagram- https://www.instagram.com/vishalthetrainer
By Vishal sir
Also Important For SBI CLERK, IBPS CLERK, RRB CLERK
& Other Competitive Exams

Q660.) Select all that are true about the tornado outbreak of 2011. more time this year after leaving interest rates unchanged from
a.) It produced a violent hailstorm in Virginia. now. However the one element in the Fed’s policy calculus that
b.) It devastated Tuscaloosa, Alabama. eludes, in Ms. Yellen’s words, a more perfect “understanding” is
c.) It spanned two nations. the lower than anticipated trajectory of inflation. Given that the
d.) It was both deadly and costly monetary authorities in the U.S. are focused on reflating the
Answers And Solutions economy by supporting “further strengthening” in the labour
market through an accommodative stance, the central bank has to
Directions :Read the following passage carefully and answer remain vigilant in warding off any let-up in expansionary
the questions given below it. Certain words are given in bold momentum. For Indian policymakers, there are both positive and
to help you locate them while answering some of the not-so-welcome cues. While the ongoing moderate expansion
questions. moderate expansion in the U.S. bodies well for the the country’s
Quantitative easing has concluded in the world’s largest economy struggling exporters, the end of easy money conditions could
– at least for now. Almost nine years after the U.S. Federal augur a slowdown in investment inflows from abroad and
Reserve started its unprecedented programe of liquidity infusion resultant pressure on the current account deficit. The Reserve
through the purchase of asset-backed and Treasury securities in Bank of India, though, should welcome the clarity in messaging
the wake of the global financial crisis, Fed chair Janet Yellen from its U.S. counterpart.
announced on Wednesday that starting next month the central
bank would begin the normalization of its balance sheet. To their Q661. What is the main purpose of the passage?
credit, Ms Yellen and her colleagues on the Federal Open market A. To warn India of the consequences of a certain policy of US Fed.
committee have walked the talk on their June decision to unwind B. To inform the readers of the effects of Taper Tantrums.
the mammoth $4.5 trillion balance sheet. Most crucially, they C. To discuss the measures planned and the possible effects of US
have done it in a manner that precludes the risk of ‘taper tantrum’ Fed’s moves to normalize its balance sheet.
similar to that in 2013 – when chairman Ben Bernanke had hinted D. To contrast the fore cast for GDP and inflation of US economy.
at starting to turn off the tap- by setting out a slow, long driven E. To elaborate on the current precarious state of US economy.
and well calibrated timetable to shrink the Fed’s holdings. The
asset wind-down will begin with monthly reductions of a modest Q662. Which of the following is opposite in meaning to the expression
$10 billion in the three months through December. That amount “walk the talk” as used in passage.
will gradually increase in quarterly increments of $10 billion so A. Walking while talking
as to reach, in October 2018, a monthly cap of $50 billion. It will B. Speaking Fluently
continue at this level till such time the committee concludes that C. To keep the word
the size of the central bank’s assets holding is optimal for the D. Obeying an order
effective conduct of monetary policy. Interestingly, Ms. Yellen, E. To break the promise
who asserted that the balance sheet was not intended to be an
“active” policy tool in normal times, especially now that Q663. Which of the following is true with respect to the implementation
economic activity had strengthened, also placed a caveat. The of the new policy of US Fed?
Fed, she said, “would be prepared to resume reinvestments” if the A. Huge amount of money is involved
economic outlook were to deteriorate so significantly as to B. It is being conducted in hurried manner
warrant “sizeable” interest rate cuts. C. It will conclude in October 2018
Not that Ms. Yellen sees the healthy expansion under-girded by D. It is well planned to avoid past mistakes
household spending weakening, the damage from the recent E. Ben Bernanke is overseeing the implementation
hurricanes that have battered coastal regions in the U.S. real GDP
growth in 2017 to 2.4% from the 2.2% estimated in June, and
signalled that it is on course to raise the federal funds rate one

Facebook Page- https://www.facebook.com/vishalpariharpage Youtube- https://www.youtube.com/vishalparihar


Follow
162 Vishal Sir Telegram Channel- https://t.me/englishbyvishalsirchannel Instagram- https://www.instagram.com/vishalthetrainer
By Vishal sir
Also Important For SBI CLERK, IBPS CLERK, RRB CLERK
& Other Competitive Exams

660. b.) It devastated Tuscaloosa, Alabama


661. C- To discuss the measures planned and the possible effects of US
Fed’s moves to normalize its balance sheet.
662. E- To break the promise
663.D- It is well planned to avoid past mistake

Facebook Page- https://www.facebook.com/vishalpariharpage Youtube- https://www.youtube.com/vishalparihar


Follow
163 Vishal Sir Telegram Channel- https://t.me/englishbyvishalsirchannel Instagram- https://www.instagram.com/vishalthetrainer
By Vishal sir
Also Important For SBI CLERK, IBPS CLERK, RRB CLERK
& Other Competitive Exams

Q664. Which of the following is closest in meaning to the word ’caveat’ known as the International Date Line in the early spring of 1521
– in the passage? after 98 days on the Pacific Ocean. During those long days at sea,
A. Act many of Magellan’s men died of starvation and disease.
B. Caution Later, Magellan became involved in an insular conflict
C. Oppose in the Philippines and was killed in a tribal battle. Only one ship
D. Code and 17 sailors under the command of the Basque navigator
E. Praise
Elcano survived to complete the westward journey to Spain and
thus prove once and for all that the world is round, with no
Q665. Which of the following statements is implied about FOMC as per
the passage? precipice at the edge.
A. It never understands how inflation moves. 666. The 16th century was an age of great ______ exploration.
B. It has the authority to raise interest rates. a) cosmic
C. Ms. Yellen managed it better than Mr. Bernanke had b) land
D. Its initial GDP forecast is wrong c) mental
E. Its policies adversely effect the world economy, including d) common man
India’s e) None of the above
667. Magellan lost the favor of the king of Portugal when he
Directions : Read the following passage carefully and answer became involved in a political ________.
the questions that follow. a) entanglement
In the 16th century, an age of great marine and terrestrial b) discussion
exploration, Ferdinand Magellan led the first expedition to sail c) negotiation
around the world. As a young Portuguese noble, he served the d) problem
king of Portugal, but he became involved in the quagmire of e) None of the above
political intrigue at court and lost the king’s favor. After he was 668. The Pope divided New World lands between Spain and
dismissed from service by the king of Portugal, he offered to Portugal according to their location on one side or the other
serve the future Emperor Charles V of Spain. of an imaginary geographical line 50 degrees west of
A papal decree of 1493 had assigned all land in the New World Greenwich that extends in a _________ direction.
west of 50 degrees W longitude to Spain and all the land east of a) north and south
that line to Portugal. Magellan offered to prove that the East b) crosswise
Indies fell under Spanish authority. On September 20, 1519, c) easterly
Magellan set sail from Spain with five ships. More than a year d) south east
later, one of these ships was exploring the topography of South e) north and west
America in search of a water route across the continent. This ship 669. One of Magellan’s ships explored the _________ of South
sank, but the remaining four ships searched along the southern America for a passage across the continent.
peninsula of South America. Finally they found the passage they a) coastline
sought near 50 degrees S latitude. Magellan named this passage b) mountain range
the Strait of All Saints, but today it is known as the Strait of c) physical features
Magellan. d) islands
One ship deserted while in this passage and returned to Spain, so e) None of the above
fewer sailors were privileged to gaze at that first panorama of the
Pacific Ocean. Those who remained crossed the meridian now

Facebook Page- https://www.facebook.com/vishalpariharpage Youtube- https://www.youtube.com/vishalparihar


Follow
164 Vishal Sir Telegram Channel- https://t.me/englishbyvishalsirchannel Instagram- https://www.instagram.com/vishalthetrainer
By Vishal sir
Also Important For SBI CLERK, IBPS CLERK, RRB CLERK
& Other Competitive Exams

664. B- Caution
665.B- It has the authority to raise interest rates.
666. B
667. A
668. A
669. C

Facebook Page- https://www.facebook.com/vishalpariharpage Youtube- https://www.youtube.com/vishalparihar


Follow
165 Vishal Sir Telegram Channel- https://t.me/englishbyvishalsirchannel Instagram- https://www.instagram.com/vishalthetrainer
By Vishal sir
Also Important For SBI CLERK, IBPS CLERK, RRB CLERK
& Other Competitive Exams

670. Four of the ships sought a passage along a southern ______. the world-famous university. In 1911 she received the Nobel
a) coast Prize in chemistry for isolating radium. Although Marie Curie
b) inland eventually suffered a fatal illness from her long exposure to
c) body of land with water on three sides radium, she never became disillusioned about her work.
d) border Regardless of the consequences, she had dedicated herself to
e) Answer not available science and to revealing the mysteries of the physical world.
671. The Curies’ _________ collaboration helped to unlock the
Directions Read the following passage carefully and answer secrets of the atom.
the questions given below it. Certain words are given in bold
to help you locate them while answering some of the a) friendly
questions. b) competitive
Marie Curie was one of the most accomplished scientists in c) courteous
history. Together with her husband, Pierre, she discovered d) industrious
radium, an element widely used for treating cancer, and studied e) chemistry
uranium and other radioactive substances. Pierre and Marie’s 672. Marie had a bright mind and a ______ personality.
amicable collaboration later helped to unlock the secrets of the a) strong
atom. b) lighthearted
Marie was born in 1867 in Warsaw, Poland, where her father was c) humorous
a professor of physics. At an early age, she displayed a brilliant d) strange
mind and a blithe personality. Her great exuberance for learning e) envious
prompted her to continue with her studies after high school. She 673. When she learned that she could not attend the university in
became disgruntled, however, when she learned that the Warsaw, she felt _________.
university in Warsaw was closed to women. Determined to a) hopeless
receive a higher education, she defiantly left Poland and in 1891 b) annoyed
entered the Sorbonne, a French university, where she earned her c) depressed
master’s degree and doctorate in physics. d) worried
Marie was fortunate to have studied at the Sorbonne with some e) None of the above
of the greatest scientists of her day, one of whom was Pierre 674. Marie _________ by leaving Poland and traveling to France to
Curie. Marie and Pierre were married in 1895 and spent many enter the Sorbonne.
productive years working together in the physics laboratory. A a) challenged authority
short time after they discovered radium, Pierre was killed by a b) showed intelligence
horse-drawn wagon in 1906. c) behaved
Marie was stunned by this horrible misfortune and endured d) was distressed
heartbreaking anguish. Despondently she recalled their close e) Answer not available
relationship and the joy that they had shared in scientific research. 675. _________ she remembered their joy together.
The fact that she had two young daughters to raise by herself a) Dejectedly
greatly increased her distress. b) Worried
Curie’s feeling of desolation finally began to fade when she was c) Tearfully
asked to succeed her husband as a physics professor at the d) Happily
Sorbonne. She was the first woman to be given a professorship at e) Irefully

Facebook Page- https://www.facebook.com/vishalpariharpage Youtube- https://www.youtube.com/vishalparihar


Follow
166 Vishal Sir Telegram Channel- https://t.me/englishbyvishalsirchannel Instagram- https://www.instagram.com/vishalthetrainer
By Vishal sir
Also Important For SBI CLERK, IBPS CLERK, RRB CLERK
& Other Competitive Exams

670. C
671.A
672. B
673. B
674. A
675. A

Facebook Page- https://www.facebook.com/vishalpariharpage Youtube- https://www.youtube.com/vishalparihar


Follow
167 Vishal Sir Telegram Channel- https://t.me/englishbyvishalsirchannel Instagram- https://www.instagram.com/vishalthetrainer
By Vishal sir
Also Important For SBI CLERK, IBPS CLERK, RRB CLERK
& Other Competitive Exams

Directions : Read the following passage carefully and answer b) disconsolate excluding David
the questions given below it. Certain words are given in bold c) happy and disconsolate
to help you locate them while answering some of the d) sad excluding David
questions.
My brother, David, was always close to our grandmother. Both Directions: Read the following passage carefully and answer
of them shared a love of Mother Nature and of food that they had the questions given below it. Certain words are given in bold
grown themselves. Whenever his schedule permitted, he would to help you locate them while answering some of the
drop in for a short visit and a cup of coffee. One day, when he questions.
found no one home, he left a chunk of dirt on her porch. This One of the most intriguing stories of the Russian Revolution
started what was later to be known as his “calling card”. concerns the identity of Anastasia, the youngest daughter of Czar
Grandmother would come home occasionally and instantly know Nicholas II. During his reign over Russia, the czar had planned to
that Dave had been by when she spotted the chunk of dirt on her revoke many of the harsh laws established by previous czars.
porch. Although Grandmother had a poor upbringing in Italy, she Some workers and peasants, however, clamored for more rapid
managed to do well in the United States. She was always healthy social reform. In 1918, a group of these people known as
and independent and enjoyed a fulfilling life. Recently she had a Bolsheviks overthrew the government. On July 17 or 18, they
stroke and died. Everyone was saddened by her death. David was murdered the czar and what was thought to be his entire family.
disconsolate. His life-long friend was now gone. Although witnesses vouched that all the members of the czar’s
Q676. What is the opposite of the word ‘disconsolate’? family had been executed, there were rumors suggesting that
a) Devastated Anastasia had survived. Over the years, a number of women
b) Hilarious claimed to be Grand Duchess Anastasia. Perhaps the most famous
c) Exuberant claimant was Anastasia Tschaikovsky, who was also known as
d) Sombre Anna Anderson.
Q677. David would drop in for a short visit and leave a _____ as a In 1920, 18 months after the czar’s execution, this terrified young
sign on grandma’s porch if she was not at home. woman was rescued from drowning in a Berlin river. She spent
a) schedule two years in a hospital, where she attempted to reclaim her health
b) chunk of dirt and shattered mind. The doctors and nurses thought that she
c) calling card resembled Anastasia and questioned her about her background.
d) cup of coffee She disclaimed any connection with the czar’s family. Eight years
Q678. Grandmother used to be _____ . later, however, she claimed that she was Anastasia.
a) rich in Italy but poor in the United States She said that she had been rescued by two Russian soldiers after
b) in the United States but is now in Italy the czar and the rest of her family had been killed. Two brothers
c) poor earlier but became rich later on named Tschaikovsky had carried her into Romania. She had
d) rich earlier but now poor married one of the brothers, who had taken her to Berlin and left
Q679. Grandmother enjoyed a _____ life. her there, penniless and without a vocation. Unable to invoke the
a) healthy but sickly aid of her mother’s family in Germany, she had tried to drown
b) good and healthy herself. During the next few years, scores of the czar’s relatives,
c) rich but sickly ex-servants, and acquaintances interviewed her. Many of these
d) poor and healthy people said that her looks and mannerisms were evocative of the
Q680. Grandmother’s death made everyone Anastasia that they had known. Her grandmother and other
a) sad including David relatives denied that she was the real Anastasia, however.

Facebook Page- https://www.facebook.com/vishalpariharpage Youtube- https://www.youtube.com/vishalparihar


Follow
168 Vishal Sir Telegram Channel- https://t.me/englishbyvishalsirchannel Instagram- https://www.instagram.com/vishalthetrainer
By Vishal sir
Also Important For SBI CLERK, IBPS CLERK, RRB CLERK
& Other Competitive Exams

Tired of being accused of fraud, Anastasia immigrated to the Read the following passage carefully and answer the
United States in 1928 and took the name Anna Anderson. She questions given below it. Certain words are given in bold to
still wished to prove that she was Anastasia, though, and returned help you locate them while answering some of the questions.
to Germany in 1933 to bring suit against her mother’s family. In May 1966, the World Health Organization was authorized to
There she declaimed to the court, asserting that she was indeed initiate a global campaign to eradicate smallpox. The goal was to
Anastasia and deserved her inheritance. In 1957, the court eradicate the disease in one decade. Because similar projects for
decided that it could neither confirm nor deny Anastasia’s malaria and yellow fever had failed, few believed that smallpox
identity. Although it will probably never be known whether this could actually be eradicated but eleven years after the initial
woman was the Grand Duchess Anastasia, her search to establish organization of the campaign, no cases were reported in the
her identity has been the subject of numerous books, plays, and field.The strategy was not only to provide mass vaccinations, but
movies. also to isolate patients with active smallpox in order to contain
Q681. Some Russian peasants and workers ______ for social the spread of the disease and to break the chain of human
reform. transmission. Rewards for reporting smallpox assisted in
a) longed (b) cried out motivating the public to aid health workers. One by one, each
b) begged (d) hoped smallpox victim was sought out, removed from contact with
Q682. Witnesses ______ that all members of the czar’s family had others and treated. At the same time, the entire village where the
been executed. victim had lived was vaccinated. Today smallpox is no longer a
a) gave assurance threat to humanity. Routine vaccinations have been stopped
b) thought worldwide.
c) hoped
d) doubted Q686. Which of the following is the best title for the passage?
Q683. Tschaikovsky initially ______ any connection with the czar’s a) The World Health Organization
family. b) The Eradication of Smallpox
a) denied c) Smallpox Vaccinations
b) stopped d) Infectious Diseases
c) noted
d) justified Q687. What was the goal of the campaign against smallpox?
Q684. She was unable to ______ the aid of her relatives. a) To decrease the spread of smallpox worldwide
a) locate b) To eliminate smallpox worldwide in ten years
b) speak about c) To provide mass vaccinations against smallpox worldwide
c) call on d) To initiate worldwide projects for smallpox, malaria and yellow
d) identify fever at the same time
Q685. In court she _________ maintaining that she was Anastasia
and deserved her inheritance. Q688. According to the paragraph what was the strategy used to
a) finally appeared eliminate the spread of small pox?
b) spoke forcefully a) Vaccination of the entire village
c) gave testimony (d) gave evidence b) Treatment of individual victims
c) Isolation of victims and mass vaccinations
d) Extensive reporting of outbreaks

Facebook Page- https://www.facebook.com/vishalpariharpage Youtube- https://www.youtube.com/vishalparihar


Follow
169 Vishal Sir Telegram Channel- https://t.me/englishbyvishalsirchannel Instagram- https://www.instagram.com/vishalthetrainer
By Vishal sir
Also Important For SBI CLERK, IBPS CLERK, RRB CLERK
& Other Competitive Exams

676. a
677. b
678.c
679.b
680.a
681. B
682. A
683. A
684. C
685. B
686. (B)
687. (B)
688. (C)

Facebook Page- https://www.facebook.com/vishalpariharpage Youtube- https://www.youtube.com/vishalparihar


Follow
170 Vishal Sir Telegram Channel- https://t.me/englishbyvishalsirchannel Instagram- https://www.instagram.com/vishalthetrainer
By Vishal sir
Also Important For SBI CLERK, IBPS CLERK, RRB CLERK
& Other Competitive Exams

Q689. Eleven years after the initial organization of the campaign, 4.T
___________ 5.Q
a) Smallpox was no longer a threat to humanity
b) No cases were reported in the field Question: 692
c) Organization members changed the strategy Which of the following should be the THIRD sentence after
d) More number of people had been victimized from smallpox rearrangement?
Q690. It can be inferred that— 1.S
a) No new cases of smallpox have been reported this year 2.R
b) Malaria and yellow fever have been eliminated 3.Q
c) Smallpox victims no longer die when they contract the disease 4.T
d) Smallpox is not transmitted from one person to another 5.P

Directions: (Que No. 691 - 695) Question: 693


The following sentences form a paragraph. The last sentence Which of the following should be the FIFTH sentence after
of the paragraph is given. The rest of the sentences are rearrangement?
numbered as P, Q, R and S and T. These five parts are not 1.Q
given in their proper order. Read the sentences and choose 2.T
the alternative that arranges them in correct order. 3.S
P. The currency sunk to a closing low of 68.07 against the U.S. 4.R
dollar, its lowest level in 16 months. 5.P
Q. The rise in crude oil prices through this year, amidst rising
geopolitical tensions in West Asia and dwindling global supply, Question: 694
have obviously hurt the rupee and the trade balance. Which of the following should be the SECOND sentence after
R. Meanwhile, despite a depreciating currency, India’s rearrangement?
merchandise exports are stumbling instead of gaining from the 1.P
opportunity. 2.Q
S. India’s macroeconomic threats lie exposed as it grapples with 3.R
the rupee’s slide. 4.S
T. The rupee, already one of the worst performing Asian 5.T
currencies, has now weakened 6.2% in 2018.
6. April clocked a sharp decline in exports from employment- Question: 695
intensive sectors such as readymade garments and gems and Which of the following should be the FOURTH sentence after
jewellery, according to official data. rearrangement?
1.P
Question: 691 2.R
Which of the following should be the FIRST sentence after 3.S
rearrangement? 4.T
1.R 5.Q
2.S
3.P

Facebook Page- https://www.facebook.com/vishalpariharpage Youtube- https://www.youtube.com/vishalparihar


Follow
171 Vishal Sir Telegram Channel- https://t.me/englishbyvishalsirchannel Instagram- https://www.instagram.com/vishalthetrainer
By Vishal sir
Also Important For SBI CLERK, IBPS CLERK, RRB CLERK
& Other Competitive Exams

689. (A) talking about the performance of rupees and it is the third
690. (C) sentence. Now sentence Q is the next sentence and not sentence
Solution 691: 2 R because sentence Q mentioned the previous theme and starts a
Sentence 1: The first sentence of any paragraph introduces an new theme i.e. of trade. It is a transitional sentence and it will
idea or topic on which the whole paragraph talks about. It should come before R. The last sentence is given.
talk about something in general and not specific details as they The correct order of the sentence is SPTQR6.
will come later. Sentence S is a good starting sentence. Thus, option 4 is the correct answer.
Now ‘slide’ in sentence S is closely connected with ‘sunk’ in Solution 694:1
sentence P and the passage should have a smooth flow of ideas Sentence 1: The first sentence of any paragraph introduces an
hence sentence P is the second sentence. Sentence T continues idea or topic on which the whole paragraph talks about. It should
talking about the performance of rupees and it is the third talk about something in general and not specific details as they
sentence. Now sentence Q is the next sentence and not sentence will come later. Sentence S is a good starting sentence.
R because sentence Q mentioned the previous theme and starts a Now ‘slide’ in sentence S is closely connected with ‘sunk’ in
new theme i.e. of trade. It is a transitional sentence and it will sentence P and the passage should have a smooth flow of ideas
come before R. The last sentence is given. hence sentence P is the second sentence. Sentence T continues
The correct order of the sentence is SPTQR6. talking about the performance of rupees and it is the third
Thus, option 2 is correct answer. sentence. Now sentence Q is the next sentence and not sentence
Solution 692:4 R because sentence Q mentioned the previous theme and starts a
Sentence 1: The first sentence of any paragraph introduces an new theme i.e. of trade. It is a transitional sentence and it will
idea or topic on which the whole paragraph talks about. It should come before R. The last sentence is given.
talk about something in general and not specific details as they The correct order of the sentence is SPTQR6.
will come later. Sentence S is a good starting sentence. Thus, option 1 is the correct answer.
Now ‘slide’ in sentence S is closely connected with ‘sunk’ in
sentence P and the passage should have a smooth flow of ideas Solution695:5
hence sentence P is the second sentence. Sentence T continues Sentence 1: The first sentence of any paragraph introduces an
talking about the performance of rupees and it is the third idea or topic on which the whole paragraph talks about. It should
sentence. Now sentence Q is the next sentence and not sentence talk about something in general and not specific details as they
R because sentence Q mentioned the previous theme and starts a will come later. Sentence S is a good starting sentence.
new theme i.e. of trade. It is a transitional sentence and it will Now ‘slide’ in sentence S is closely connected with ‘sunk’ in
come before R. The last sentence is given. sentence P and the passage should have a smooth flow of ideas
The correct order of the sentence is SPTQR6. hence sentence P is the second sentence. Sentence T continues
Thus, option 4 is the correct answer. talking about the performance of rupees and it is the third
Solution 693:4 sentence. Now sentence Q is the next sentence and not sentence
Sentence 1: The first sentence of any paragraph introduces an R because sentence Q mentioned the previous theme and starts a
idea or topic on which the whole paragraph talks about. It should new theme i.e. of trade. It is a transitional sentence and it will
talk about something in general and not specific details as they come before R. The last sentence is given.
will come later. Sentence S is a good starting sentence. The correct order of the sentence is SPTQR6.
Now ‘slide’ in sentence S is closely connected with ‘sunk’ in Thus, option 5 is the correct answer.
sentence P and the passage should have a smooth flow of ideas
hence sentence P is the second sentence. Sentence T continues

Facebook Page- https://www.facebook.com/vishalpariharpage Youtube- https://www.youtube.com/vishalparihar


Follow
172 Vishal Sir Telegram Channel- https://t.me/englishbyvishalsirchannel Instagram- https://www.instagram.com/vishalthetrainer
By Vishal sir
Also Important For SBI CLERK, IBPS CLERK, RRB CLERK
& Other Competitive Exams

Directions: (Que No. 696- 700) 4.P


The following sentences form a paragraph. The second and 5.R
the fifth sentence of the paragraph are given. The rest of the
sentences are numbered as P, Q, R and S and T. These five Question No.698
parts are not given in their proper order. Read the sentences Which of the following should be the SIXTH sentence after
and choose the alternative that arranges them in correct rearrangement?
order. 1.T
P. The most affected will be countries closer to the region, in 2.R
particular India. 3.P
2. To be sure, the least affected will be the U.S. 4.Q
Q. For one, the International Atomic Energy Agency (IAEA) has 5.S
consistently maintained that Tehran has complied with the
strictures of the JCPOA without fail. Question No.699
R. Washington’s decision is unjustified and unreasonable for Which of the following should be the FIRST sentence after
several reasons. rearrangement?
5. Moreover, for a U.S. administration that has made it a habit of 1.Q
accusing other countries of “undermining the rules-based order”, 2.R
this action has severely undermined the rules-based global order. 3.P
S. American President Donald Trump’s decision to withdraw 4.T
from the Joint Comprehensive Plan of Action (JCPOA), 5.S
popularly called the Iran nuclear deal, is bound to have serious
implications for the international system, and for India. Question No.700
T. European Union countries will be moderately affected due to Which of the following should be the THIRD sentence after
the business ties with Iran. rearrangement?
1.P
Question No.696 2.Q
Which of the following should be the FOURTH sentence after 3.R
rearrangement? 4.S
1.T 6.T
2.P
3.S Directions: Read each sentence any to find out whether there
4.Q is any grammatical error or idiomatic error in it. The error,
5.R if any, will be in one part of the sentence. If
there is no error, the answer is No error. (Ignore errors of
Question No.697 punctuation, if any)
Which of the following should be the LAST(Seventh) sentence
after rearrangement? Q701. The judge asked the man (a)/if the bag he had lost (b)/contain five
1.Q thousand rupees. (c)/The man replied that it did. (d)/No error (e).
2.S
3.T

Facebook Page- https://www.facebook.com/vishalpariharpage Youtube- https://www.youtube.com/vishalparihar


Follow
173 Vishal Sir Telegram Channel- https://t.me/englishbyvishalsirchannel Instagram- https://www.instagram.com/vishalthetrainer
By Vishal sir
Also Important For SBI CLERK, IBPS CLERK, RRB CLERK
& Other Competitive Exams

Solution 696: 2 mandatory pair of sentences in that order and they form the 6th
Sentence 1: The first sentence of any paragraph introduces an and 7th sentences.
idea or topic on which the whole paragraph talks about. It should So, the correct order of sentences is S2TP5RQ.
talk about something in general and not specific details as they Thus, option 1 is the correct answer.
will come later. After giving a cursory read to the passage, we
know it’s about the withdrawal of American President from the Solution 698:2
Iran nuclear deal and sentence S introduces this topic. Hence Sentence 1: The first sentence of any paragraph introduces an
sentence S is our first sentence. idea or topic on which the whole paragraph talks about. It should
Sentence 2: Sentence 2 is given and it tells which one will be least talk about something in general and not specific details as they
affected. And now we have got a pair of three sentences which will come later. After giving a cursory read to the passage, we
will go together. The least affected, the moderately affected and know it’s about the withdrawal of American President from the
the most affected countries will form a series of continuous Iran nuclear deal and sentence S introduces this topic. Hence
sentences hence sentences T and P will be our 3rd and 4th sentence S is our first sentence.
sentence. Sentence 2: Sentence 2 is given and it tells which one will be least
Sentence 5 is given. Now we are left with sentence Q and R. Now affected. And now we have got a pair of three sentences which
sentence R talks about the reasons why Washington’s decision is will go together. The least affected, the moderately affected and
unjustifiable and Q lists one of those reasons so RQ is a the most affected countries will form a series of continuous
mandatory pair of sentences in that order and they form the 6th sentences hence sentences T and P will be our 3rd and 4th
and 7th sentences. sentence.
So, the correct order of sentences is S2TP5RQ. Sentence 5 is given. Now we are left with sentence Q and R. Now
Thus, option 2 is the correct answer. sentence R talks about the reasons why Washington’s decision is
unjustifiable and Q lists one of those reasons so RQ is a
Solution 697:1 mandatory pair of sentences in that order and they form the 6th
Sentence 1: The first sentence of any paragraph introduces an and 7th sentences.
idea or topic on which the whole paragraph talks about. It should So, the correct order of sentences is S2TP5RQ.
talk about something in general and not specific details as they Thus, option 2 is the correct answer.
will come later. After giving a cursory read to the passage, we
know it’s about the withdrawal of American President from the Solution 699:5
Iran nuclear deal and sentence S introduces this topic. Hence Sentence 1: The first sentence of any paragraph introduces an
sentence S is our first sentence. idea or topic on which the whole paragraph talks about. It should
Sentence 2: Sentence 2 is given and it tells which one will be least talk about something in general and not specific details as they
affected. And now we have got a pair of three sentences which will come later. After giving a cursory read to the passage, we
will go together. The least affected, the moderately affected and know it’s about the withdrawal of American President from the
the most affected countries will form a series of continuous Iran nuclear deal and sentence S introduces this topic. Hence
sentences hence sentences T and P will be our 3rd and 4th sentence S is our first sentence.
sentence. Sentence 2: Sentence 2 is given and it tells which one will be least
Sentence 5 is given. Now we are left with sentence Q and R. Now affected. And now we have got a pair of three sentences which
sentence R talks about the reasons why Washington’s decision is will go together. The least affected, the moderately affected and
unjustifiable and Q lists one of those reasons so RQ is a the most affected countries will form a series of continuous

Facebook Page- https://www.facebook.com/vishalpariharpage Youtube- https://www.youtube.com/vishalparihar


Follow
174 Vishal Sir Telegram Channel- https://t.me/englishbyvishalsirchannel Instagram- https://www.instagram.com/vishalthetrainer
By Vishal sir
Also Important For SBI CLERK, IBPS CLERK, RRB CLERK
& Other Competitive Exams

sentences hence sentences T and P will be our 3rd and the Past. Event one is written in Simple Past and the event two in
4thsentence. Past Perfect.
Sentence 5 is given. Now we are left with sentence Q and R. Now
sentence R talks about the reasons why Washington’s decision is
unjustifiable and Q lists one of those reasons so RQ is a
mandatory pair of sentences in that order and they form the 6th
and 7th sentences.
So, the correct order of sentences is S2TP5RQ.
Thus, option 5 is the correct answer.

Solution 700:5
Sentence 1: The first sentence of any paragraph introduces an
idea or topic on which the whole paragraph talks about. It should
talk about something in general and not specific details as they
will come later. After giving a cursory read to the passage, we
know it’s about the withdrawal of American President from the
Iran nuclear deal and sentence S introduces this topic. Hence
sentence S is our first sentence.
Sentence 2: Sentence 2 is given and it tells which one will be least
affected. And now we have got a pair of three sentences which
will go together. The least affected, the moderately affected and
the most affected countries will form a series of continuous
sentences hence sentences T and P will be our 3rd and 4th
sentence.
Sentence 5 is given. Now we are left with sentence Q and R. Now
sentence R talks about the reasons why Washington’s decision is
unjustifiable and Q lists one of those reasons so RQ is a
mandatory pair of sentences in that order and they form the 6th
and 7th sentences.
So, the correct order of sentences is S2TP5RQ.
Thus, option 5 is the correct answer.

Directions: Read each sentence any to find out whether there


is any grammatical error or idiomatic error in it. The error,
if any, will be in one part of the sentence. If
there is no error, the answer is No error. (Ignore errors of

Ans 701: (c) The Past Perfect refers to a time earlier than before now. It is
used to make it clear that one event happened before another in

Facebook Page- https://www.facebook.com/vishalpariharpage Youtube- https://www.youtube.com/vishalparihar


Follow
175 Vishal Sir Telegram Channel- https://t.me/englishbyvishalsirchannel Instagram- https://www.instagram.com/vishalthetrainer
By Vishal sir
Also Important For SBI CLERK, IBPS CLERK, RRB CLERK
& Other Competitive Exams

Q702. I trust you will (a)/show forbearance to me (b)/a few minutes Q 714. At my arrival(a)/ in Del hi(b)/ I went straight(c)/ to the nearest
more (c)/so that I can finish this work. (d)/ No error (e). hospital.(d)/ No error(e)

Q703. The ground outside the village, (a)/abounding with frogs and Q715. It is more better(a)/ if one of the parents(b)/ stays at home(c)/ to
snakes, (b)/the enemies of mankind, (c)/is soft and marshy. look after the children(d)/ No error(e)
(d)/No error (e).
Q716. These reports have(a)/ deterred some(b)/ women to have(c)/ the
Q704. We are all short-sighted (a)/and very often see but one side of the operation(d)/ No error(e)
matter. (b)/Our views are not extended (c)/ to all that has a
connection with it. (d)/No error (e). Q717 I answered(a)/ the questions(b)/ as best as(c)/ I could(d)/ No
error/(e)
Q 705. Just laws are no restraint with (a)/the freedom of the good, (b)/for
the good man desires nothing (c)/ which a just law interfere with. Q 718. Some of the world (a)/ largest water bodies are (b)/ drying up thus
(d)/No error (e). threatening (c)/ the livelihood of millions. (d)/ No error. (e)

Q706. Had he done (a)/his home work well (b)/he would not have Q 719. Among the many (a)/ challenges facing the country (b)/ in the
(c)/suffered this embarrassment. (d)/No error (e). next decade (c)/ is poverty and unemployment. (d)/ No error. (e)

Q707. He was angry with me (a)/because he thought my (b)/remark was Q720. According to economists (a)/ not more than five percent (b)/ of
(c)/aimed before him. education loans taken (c)/ by students are overdue. (d)/ No error.
(d)/No error (e). (e)

Q708. As you know(a)/ by my visiting card(b)/ I am now(c)/ in Q721. The two candidates share (a)/ a reputation for (b)/ competency as
Mumbai(d)/ No error(e). well as (c)/ for good communication skills. (d)/ No error. (e)

Q709. With a fresh coat(a)/ of paint(b)/ the school can(c)/ look much Q 722. His main qualification (a)/ on the job is (b)/ his extensive
nice(d)/ No error(e) experience (c)/ in foreign branches. (d)/ No error. (e)

Q710. I asked the salesman(a)/ if I could exchange(b)/ the faulty Q723. A representative of the (a)/ Reserve Bank will provide students
camera(c)/ with another one(d)/ No error(e) an (b)/ insight into the (c)/ economic future of our country. (d)/
No error. (e)
Q711. It took me(a)/ almost a hour(b)/ to fill the(c)/ application form(d)/
No error(e) Q 724. As one of the leader (a)/ insurance companies in (b)/ India they
offer (c)/ comprehensive financial services. (d)/No error. (e)
Q712. She insists(a)/ you stay(b)/ until her husband(c)/ comes home(d)/
No error(e) Q 725. There is a rumour that (a)/ this multinational company will (b)/
set up its regional headquarters (c)/ in India in short. (d)/ No error.
Q 713. I don‘t understand(a)/ how she could(b)/ treat him(c)/ so bad(d)/ (e)
No error(e)

Facebook Page- https://www.facebook.com/vishalpariharpage Youtube- https://www.youtube.com/vishalparihar


Follow
176 Vishal Sir Telegram Channel- https://t.me/englishbyvishalsirchannel Instagram- https://www.instagram.com/vishalthetrainer
By Vishal sir
Also Important For SBI CLERK, IBPS CLERK, RRB CLERK
& Other Competitive Exams

Ans 702: (b)


In place of forbearance to, we should use forbearance towards. Ans 712: (b)
forbearance (Noun): patient self-control; tolerance show you stay → you on staying, because insist is always followed by
forbearance towards somebody : exercise forbearance in dealing Prep.on.
with people.
Ans 713: (d)
Ans 703: (b) so bad should be replaced with so badly. Bad is an Adj. and its
In place of abounding with we should use abounded with. abound Adv.ial form is badly.
with/ abound in : contained, full of.
Ans 714: (a)
Ans 704: (e) No error. on/upon my arrival will replace at my arrival
Ans 715: (a)
Ans 705: (d) It is more better should be replaced with it is better. The use of
just law is in Third Person, Singular Number. So, interfere → Double Comparatives is wrong.
interferes. The sentence is in Simple Present Tense. Therefore,
which a just law interferes with is the right usage. Ans 716: (c)
The word deter is followed by Prep.-from. Therefore, women
Ans 706: (e) No error. from having is the right usage. deter from will be followed by V
+ ing form.
Ans 707: (d)
We use aim at and not aim before. Aim agrees with at (Prep.) Ans 717: (c)
as best as should be replaced with as best. as best one can (Id.) :
Ans 708: (b) not perfectly but as well as one is able to.
by → from know follows from when the source is recognised.
know from Ans 718: (a)
(Id.) : to know about something some of the world will be replaced by some of the world’s. Here
Possessive Case is used to make the sentence correct.
Ans709: (d)
look much nice → look very nice. It should be noted that very is Ans 719: (d)
used to modify Adj.s Challenges are two (poverty and unemployment) Therefore, are
and Past Participles used as Adj.s. poverty and unemployment is the right usage.

Ans 710: (d) Ans 720: (c)


another is an Indefinite Determiner which means an additional loans is Noun. An Adj. qualifies a Noun. Therefore, use
person or thing. The use of one after it is superfluous. educational (Adj.) in place of ‗education (Noun). Therefore, of
educational loans taken is the right usage.
Ans 711: (b)
almost a hour → almost an hour because h of hour is pronounced Ans 721: (d)
with Vowel sound. an (In def. Art.) is used before a Vowel sound.

Facebook Page- https://www.facebook.com/vishalpariharpage Youtube- https://www.youtube.com/vishalparihar


Follow
177 Vishal Sir Telegram Channel- https://t.me/englishbyvishalsirchannel Instagram- https://www.instagram.com/vishalthetrainer
By Vishal sir
Also Important For SBI CLERK, IBPS CLERK, RRB CLERK
& Other Competitive Exams

Prep. for has already been used before the word competency. Use
of for before good communication skills is superfluous. There
fore, “good communication skills” is the right usage.

Ans 722: (b)


The word qualification is followed by Prep.- for. Therefore,
replace on by Prep.-for.

Ans 723: (b)


When provide (Verb) means give someone something, then with
(Prep.) is used with it.

Ans 724: (a)


leading (Adj.) will replace leader (Noun).
leader (Noun) : a person who leads/is the best leading (Adj.) :
most important/most successful

Ans 725: (d)


in India in short will be replaced by in India shortly. shortly :
soon; in a short time. Look at the sentence : (i) I will be ready
shortly.

Facebook Page- https://www.facebook.com/vishalpariharpage Youtube- https://www.youtube.com/vishalparihar


Follow
178 Vishal Sir Telegram Channel- https://t.me/englishbyvishalsirchannel Instagram- https://www.instagram.com/vishalthetrainer
By Vishal sir
Also Important For SBI CLERK, IBPS CLERK, RRB CLERK
& Other Competitive Exams

Q 726. Despite taking steps to (a)/ encourage foreign investment (b)/ Q737. The survey conducted reveals (a)/ that there is a lack (b)/ of
there has been any (c)/ substantial improvement in our economy. adequate healthcare facilities (c)/ even in urban areas. (d)/ No
(d)/ No error. (e) error. (e)

Q727. We had made every effort (a)/ to ensure that a (b)/ com promise Q 738. Wheat productions in the region (a)/ will fall drastically (b)/
is reached and (c)/ that deal was signed. (d)/ No error. (e) unless we do anything (c)/ to stop climate change. (d)/ No error.
(e)
Q 728. India‘s travel and tourism (a)/ industry is poised (b)/ on growth at
seven (c)/ per cent this year alone. (d)/ No error. (e) Q739. The Indian stock market is (a)/ one of the worse (b)/ per forming
stock markets (c)/ in the recent times. (d)/ No error. (e)
Q 729. The Ministry was considered (a)/ several proposals for the (b)/
development of small and medium (c)/ enterprises during Budget Q740. With inflation at eleven per cent (a)/ companies need to come up
discussions. (d)/ No error. (e) (b)/ with innovative ways to (c)/ get customers to buy its goods
(d)/ No error. (e)
Q 720. The amount of foreign (a)/ direct investment in (b)/ the country
in 2008 is (c)/ doubled Q741. Private companies which profits (a)/ have grown due to (b)/ the
that received in 1997. (d)/No error.(e) high price of oil (c)/
should offer discounts on cooking gas. (d)/ No error. (e)
Q 731. The details of the scheme (a)/ will be made clearly (b)/ to the
public by (c)/ the Q742. Having work in (a)/ both public and private sector banks (b)/ Mr.
end of the financial year. (d)/ No error. (e) Rao is the ideal choice to (c)/ take over as chairman. (d)/ No error
(e)
Q 732. Government officials have been working (a)/ overtime to answer
queries raised (b)/ by banks on numerous of (c)/issues pertaining Q743. In order to expand, the company (a)/ plans to transform from a
to the loan waiver. (d)/ No error. (e) (b)/ family based business into (c)/ a professional managed one.
(d)/ No error (e)
Q 733. We have sought (a)/ clearance in (b)/ additional expenditure
incurred (c)/ during the project. (d)/ No error. (e) Q744. Amit started a small (a)/ construction business two years ago (b)/
when the country is (c)/ enjoying a construction boom. (d)/ No
Q 734. The Reserve Bank of India‘s decision (a)/ to waive ATM charges error (e)
(b)/ have put banks (c)/ in a difficult position. (d)/ No error. (e)
Q745. Given the current (a)/ market conditions has is (b)/ better for us
Q 735. The government is planning to (a)/ sanction grants to to invest (c)/ in the infrastcture sector. (d)/ No error (e)
pharmaceutical companies (b)/ for inventing new treatments for
(c)/ diseases about malaria and tuberculosis. (d)/ No error. (e) Q746. Many athletes have (a)/ taken on yoga (b)/ to develop their
Q736. The Finance Minister‘s view (a)/ is that the scheme will ensure concentration (c)/ and reduce stress. (d)/ No error (e)
(b)/ that millions of farm ers will be (c)/ lifted out of poverty. (d)/
No error. (e) Q747. His main aim is (a)/ to create an (b)/ interest in hockey (c)/ among
school children. (d)/ No error (e)

Facebook Page- https://www.facebook.com/vishalpariharpage Youtube- https://www.youtube.com/vishalparihar


Follow
179 Vishal Sir Telegram Channel- https://t.me/englishbyvishalsirchannel Instagram- https://www.instagram.com/vishalthetrainer
By Vishal sir
Also Important For SBI CLERK, IBPS CLERK, RRB CLERK
& Other Competitive Exams

Ans 726: (c) Ans 736: (c)


Two negatives make a positive. Therefore, there has not been any that millions of farmers will be will be replaced by that millions
/ there has been no will replace there has been any to make the of farmers are. first part of the sentence is in Future Tense.
sentence meaningful. Therefore, it will be correct to use second part in Present Tense.
Moreover, this sentence refers to a possible condition and its
Ans 727: (c) probable result. Therefore, its structure is as follows : Conditional
The Reporting Verb is in Past Tense. So, in Indirect statement, Clause ® Simple Pre sent Main Clause ® Simple Future
Past Tense should be used. Therefore, a compromise was reached Ans 737: (e) No error.
and is the right usage. Ans 738: (c)
Ans 728: (c) unless we do anything will be replaced by unless we do
an Infinitive should be used. Therefore to grow at seven will re something unless has been used in a negative sense. Therefore,
place of on growth at seven. two negatives will not be used in the sentence.
Ans 729: (a) Ans 739: (c)
It is improper to use Past Tense (PV). So, Simple Past/Present It is proper to use Superlative form of bad i.e. worst. the (Def.
Perfect will be used. Therefore, the ministry has considered is the Art.) has been used and a Superlative Degree is used with the.
right usage. Ans 740: (d)
The Subj. of the sentence is Plural that will take Plural Possessive.
Ans 730: (d) Therefore, replace get customers to buy its goods by get
doubled that received in 1997 will be replaced by ‘twice to that customers buy their goods Here to has been used wrongly.
received in 1997. Ans 741: (a)
Private companies whose profits will be the right usage.
Ans 731: (b) Ans 742: (a)
will be made clearly will be replaced by will be made clear. Look Perfect Participle (Having + Past Participle) will be used.
at the sentence : (i) I hope I made it clear to him that he was no Therefore, having worked in is the right usage.
longer welcome here. Ans 743: (d)
As an Adv. modifies an Adj. a professionally managed one is the
Ans 732: (c) right usage.
banks on numerous of → by banks on numerous. numerous : Ans 744: (c)
many. It is improper to use Prep.-of here. Look at the sentence : The sentence shows Past event. Therefore, when the country was
(i) He has been late on numerous occasions. is the right usage.
Ans 733: (b) Ans 745: (b)
for clearance of is the right usage. The sentence shows future time. Therefore, market conditions it
Ans 734: (c) will be is the right usage.
The Subj. of the sentence The Reserve Bank of India’s decision Ans 746: (b)
is in Singular Number. It will take Singular Verb-has. Therefore, taken up yoga is the right usage.
has put banks is the right usage.
Ans 735: (d) Ans 747: (e) No error.
diseases about malaria and tuberculosis will be replaced by
diseases like malaria and tuberculosis. Like (Prep.)

Facebook Page- https://www.facebook.com/vishalpariharpage Youtube- https://www.youtube.com/vishalparihar


Follow
180 Vishal Sir Telegram Channel- https://t.me/englishbyvishalsirchannel Instagram- https://www.instagram.com/vishalthetrainer
By Vishal sir
Also Important For SBI CLERK, IBPS CLERK, RRB CLERK
& Other Competitive Exams

Q748. These companies have been asked (a)/ to furnish their financial Q759. We have received many (a)/ of the letters from customers (b)/
details (b)/ and information about (c)/ its board members. (d)/ No asking us to extend (c)/ the deadline to repay their loans (d)/. No
error (e) error (e).

Q749. The scheme which will be launched (a)/during the next two years Q760. Under the terms of the new deal (a)/ the channel can broad cast
(b)/ require an additional investment (c)/ of one hundred crores. (b)/ the next cricket tournament to be (c)/ played among India and
(d)/ No error (e). Australia (d)/. No error (e).

Q750. Road developers unable (a)/ to complete their projects (b)/ on Q761. This is turned out to be (a)/ one of our most successful pro jects
time will not be (c)/ awarded new ones. (d)/ No error (e) (b)/ and we have made quite (c)/ a large profit from it (d)/. No
error (e).
Q751. We have taken on (a)/ the responsibility of (b)/ arranging the
required training (c)/ and supervise the new staff.(d)/No error (e) Q762. A non-banking financial company is a (a)/ financial institution
similarly to a bank (b)/ but it cannot issue (c)/ cheque books to
Q752. The government has signed (a)/ a memorandum of understanding customers (d)/. No error (e).
with (b)/ the company to set up (c)/ a plant in the state. (d)/ No
error (e) Q763. He has taken care to (a)/ compliance with the norms (b)/ so he
expects the proposal (c)/ to be approved without delay (d)/. No
Q753. Owing the new policy (a)/ we feel that the targets (b)/ set for this error (e).
year (c)/ may not be achieved. (d)/ No error (e)
Q764. Since I had lived there (a)/ for many years the villagers (b)/ were
Q754. Since the lack of (a)/ man power we cannot (b)/ conduct the very comfortable talked (c)/ to me about their problems (d)/No
survey (c)/ in rural areas. (d)/ No error (e) error (e).

Q 755. Palm oil is very beneficial (a)/ and is use to (b)/ make products Q765. Our equipment gets dam age (a)/ very often in summer (b)/
ranging (c)/ from soap to biodiesel. (d)/ No error (e) because there are (c)/ frequent power cuts (d)/. No error (e).

Q756. There are only (a)/ a few company (b)/ which can handle (c)/ Q766. We have been under (a)/ a lot of pressure to (b)/ open fifty new
projects of stores (c)/ by the ending of the year (d)/. No error (e).
this magnitude. (d)/ No error (e)
Q767. The company is in debt (a)/ and has been unable (b)/ to pay their
Q757. The data shows that (a)/ the unemployment rate has (b)/ raised to employees‘ salaries (c)/ for the past six months (d)/. No error (e).
6.1 per cent (c)/ the highest in five years. (d)/ No error (e)
Q768. To be a king and (a)/ wear a crown are (b)/ more glamorous to
Q758. The government has (a)/ launched many creative schemes (b)/ to (c)/ see than to bear. (d)/No error (e).
make
banking services (c)/ available to everyone (d)/. No error (e). Q769. None of the student (a)/ in the class (b)/ scored below the (c)/
given cut-off marks.
(d)/ No error (e).

Facebook Page- https://www.facebook.com/vishalpariharpage Youtube- https://www.youtube.com/vishalparihar


Follow
181 Vishal Sir Telegram Channel- https://t.me/englishbyvishalsirchannel Instagram- https://www.instagram.com/vishalthetrainer
By Vishal sir
Also Important For SBI CLERK, IBPS CLERK, RRB CLERK
& Other Competitive Exams

Ans 748: (d) Determiner-a few takes Plural Noun/ Pronoun. Therefore, a few
The Subj. of the sentence these companies is Plural. Therefore, company will be replaced by a few companies.
its board members will be re placed by their board members.
Ans 757: (c)
Ans 749: (c) raised to 6.1 per cent will be replaced by increased to 6.1 per cent.
The Subj. of the sentence is the scheme that is Singular and it will
take Singular Verb. Therefore, require an additional investment Ans 758: (e) No error.
will be replaced by requires an addition al investment. Singular
Subj. ® Ans 759: (b)
Sin gular Verb (with s) Plural Subj. ® Plural Verb (withouts) many is used before Plural Nouns. many of is used before Plural
Pronouns/Plural Noun Phrase beginning with the, these, those,
Ans 750: (a) my/their. many a is used with Singular Noun.
The road developers unable is the right usage. road developers
are specified. Therefore, the (Def. Art.) will be used before Road Ans 760: (d)
developers. When there are two things in comparison/relation, it is
appropriate to use Connective between. Therefore, played
Ans 751: (d) between India and Australia is the right usage.
Replace and supervise the new staff will be replaced by and
supervising the new staff as word arranging (Present Participle) Ans 761: (a)
has been used before Connective-and. When two This is turned out to be (Passive)
sentences/phrases are joined by and (connective), the Part of will be replaced by This turned out to be (Active).
Speech should be the same in the both the phrases/sentences.
Ans 762: (b)
Ans 752: (e) No error. financial institution similarly to a bank will be replaced by
financial institution similar to a bank. similarly is an Adv. and
Ans 753: (a) similar is an Adj.
The correct form of Prep. Is owing to that expresses reason.
Therefore, owing the new policy will be replaced by owing to the Ans 763: (b)
new Infinitive is used as to
policy. + V1 . Therefore, comply with the norms will be used in place of
compliance with the norms. Comply (Verb) : to obey a rule, an
Ans 754: (a) order, etc. compliance (Noun) : the practice of obeying
since the lack of will be replaced by due to lack of. rules/requests made by people in authority

Ans 755: (b) Ans 764: (c)


PV will be used used to will replace use to. used to + Verb is The sentence is in Past Continuous Tense. Therefore, talking will
always used in the Past Tense. use to + Verb is used when it be used in place of talked. were very comfortable talking is the
follows did/didn’t. right usage.

Ans 756: (b) Ans 765: (a)

Facebook Page- https://www.facebook.com/vishalpariharpage Youtube- https://www.youtube.com/vishalparihar


Follow
182 Vishal Sir Telegram Channel- https://t.me/englishbyvishalsirchannel Instagram- https://www.instagram.com/vishalthetrainer
By Vishal sir
Also Important For SBI CLERK, IBPS CLERK, RRB CLERK
& Other Competitive Exams

Our equipment gets damage (V1) will be replaced by our


equipment gets damaged (V3) (Passive)

Ans 766: (d)


by the ending of the year

Ans 767: (c)


Subj. company is Singular. Therefore, to pay its employees'
salaries is the right usage. Directions: Read each sentence (s) to
find out whether there is any grammatical error in it. The error, if
any, will be in one part of the sentence. If there is no error, mark
No
error. (Ignore errors of punctuation, if any.) (United Bank Of
India Clerk Ex am. 21.06.2009)

Ans 768: (b)


Here the Subj. is Singular, Therefore Singular Verb is should be
used. Therefore, wear a crown is is the right usage.

Ans 769: (a)


None of the student (Singular) will be replaced by None of the students
(Plural). A Plural Noun is used after “none of”.

Facebook Page- https://www.facebook.com/vishalpariharpage Youtube- https://www.youtube.com/vishalparihar


Follow
183 Vishal Sir Telegram Channel- https://t.me/englishbyvishalsirchannel Instagram- https://www.instagram.com/vishalthetrainer
By Vishal sir
Also Important For SBI CLERK, IBPS CLERK, RRB CLERK
& Other Competitive Exams

Q770. Ashok is among the (a)/ few people in the world (b)/ which did
not blindly follow (c)/ the path of others.(4)/ No error. (e) Q781. Although a brilliant writer, (a)/ an lying (b)/ pessimism pre vails
in (c)/ all her novels. (d)/ No error (e)
Q771. Most people like to (a)/rest after a day ‘s hard work (b)/ but he
seemed to have (c)/an inexhaustive supply of energy. (d)/No Q782. Changed social setting (a)/ demands the schools to teach (b)/
error. (e) moral and social values (c)/ among with the academic skills. (d)/
No error (e)
Q772. Ancient artefacts are (a)/ a part of global heritage (b)/ and should
not be (b)/ sold even to the highest bidder. (d)/ No error.(e) Q783. I may go to the (a)/ swimming class tomorrow (b)/ if I have
recovered (c)/ from the cold. (d)/ No error (e)
Q773. In emerging economies, (a)/ the private credit market (b)/ remains
highly segmented and thus (c)/ weaken the power of monetary Q784. The prime minister announced (a)/ that the taxes will be (b)/
policy. (d)/ No error (e) increasing from the (c)/ beginning of next year. (d)/ No error (e)

Q774. The recent election campaign (a)/has been one of (b)/ the most Q785. He is the most (a)/ intelligent and also (b)/ the very talent ed
noisiest campaigns (c)/ in the last decade. (d)/ No error (e) (c)/student of the college. (d)/ No error (e)

Q775. Wholesome strategic planning (a)/ was the focus as (b)/ the firm Q786. She immediately quit (a)/ the job in which (b)/ neither the skill
manage through a difficult period (c)/ a couple of years ago. (d)/ nor (c)/ knowledge were required. (d)/ No error (e)
No error (e)
Q787. The meteorological department (a)/ predicted that the (b)/ rains
Q776. Inspite of the best govern mental efforts, (a)/ emission of and thunderstorm may (c)/ continue throughout today. (d)/ No
greenhouse gases (b)/ and noxious chemicals (c)/ remain a cause error (e)
of worry. (d)/ No error (e)
Q788. The cruel king believed (a)/ in punishing the prisoners (b)/ until
Q777. The rate of metabolism of (a)/ a body is comparatively lowest they did not (c)/ confessed their crime. (d)/ No error (e)
when (b)/ it is at rest and is (c)/ thus optimum for examination.
(d)/ No error (e) Q789. As soon as the CEO (a)/ entered the office, the (b)/ union leaders
approached him (c)/ and report the matter to him. (d)/ No error
Q778. The opposition leader tried (a)/ to bolster his position (b)/ with (e)
the voters by pressing (c)/ corruption charges against rivals. (d)/
No error (e) Q790. Instead of being a clear winner (a)/ he was not given (b)/ the best
employee (c)/ of the year award. (d)/ No error (e)
Q779. The recently imposed dress code (a)/ in the university has enraged
(b)/ the students who will be going (c)/ on strike since tomorrow. Q791. Having been found (a)/ guilty of the theft, (b)/ Sunny was
(d)/ No error (e) sentenced (c)/ to five year‘s imprisonment. (d)/ No error (e)

Q780. Ever since he took over (a)/ as the chief minister of the state, (b)/ Q792. Instead of laying off (a)/ the workers, the company (b)/ decided
rate for unemployment (c)/ has drastically increased. (d)/ No error to ask them (c)/ to avail voluntary retirement. (d)/ No error (e)
(e)

Facebook Page- https://www.facebook.com/vishalpariharpage Youtube- https://www.youtube.com/vishalparihar


Follow
184 Vishal Sir Telegram Channel- https://t.me/englishbyvishalsirchannel Instagram- https://www.instagram.com/vishalthetrainer
By Vishal sir
Also Important For SBI CLERK, IBPS CLERK, RRB CLERK
& Other Competitive Exams

Ans 770: (c) on strike since tomorrow will be replaced by on strike from
For a person who should be used as a Relative Pronoun. tomorrow. Prep.-since is used to show a time in the past until a
Therefore, which did not blindly follow will be replaced by who later past time or until now. Look at the sentence: (i) He has been
do not blindly follow. working in a bank since leaving school. (ii) He will be joining the
bank from tomorrow.
Ans 771: (c)
but he seemed to have will be replaced by but he seems to have Ans 780: (c)
the first part of the sentence is in Present Indefinite. rate for unemployment will be replaced by rate of unemployment.

Ans 772: (e) No error. Ans 781: (b)


There is an Article related error. a lying should be used as ‗l‘
Ans 773: (d) sounds as a Consonant.
the Subj.-the private credit market is in Singular Number that will
take Singular Verb. Therefore, weakens the power of monetary Ans 782: (d)
policy is the right usage. among with the academic skills will be replaced by along with
the academic skills.
Ans 774: (c)
It is incorrect to use Double Superlatives. Therefore, the noisiest Ans 783: (c)
campaigns is the right usage. There are two possibilities in future. Therefore, the first possible
event should be expressed in Simple Present. Therefore, if I have
Ans 775: (c) recovered will be replaced by if I recover. The structure of such
The sentence shows past time. Therefore, use Simple Past here sentences is as follows : If/When + V1 ..., ... S. Fut. ... or ... Sim.
i.e. the firm managed through a difficult period is the right usage. Fut. ..., if/when + V1 ...

Ans 776: (d) Ans 784: (b)


the Verb will agree according to the Number of the word In Indirect statement, if Reporting Verb is in Past Tense then the
emission. Verb is in Reported Speech will also be in Past Tense. Therefore,
Therefore, remains a cause of worry is the right usage. emission that the taxes would be is the right usage.
of greenhouse gases and noxious chemicals is a Singular Subj.
which will take a Singular Verb (with s) Ans 785: (c)
when two/more Nouns/Adj.s refer to the same person/thing. (Arti
Ans 777: (b) cle) is used only
It will be proper to use the Positive Degree. Therefore, the rate of before the first Adj./Noun. use of the is incorrect.
metabolism of a body is comparatively low is the right usage.
Ans 786: (d)
Ans 778: (c) When we use Neither....nor, the Verb agrees with the
with the voters by pressing will be replaced by among the voters number/person of the Noun/Pronoun used after nor. Therefore,
by pressing. knowledge was required is the right usage.

Ans 779: (d) Ans 787: (c)

Facebook Page- https://www.facebook.com/vishalpariharpage Youtube- https://www.youtube.com/vishalparihar


Follow
185 Vishal Sir Telegram Channel- https://t.me/englishbyvishalsirchannel Instagram- https://www.instagram.com/vishalthetrainer
By Vishal sir
Also Important For SBI CLERK, IBPS CLERK, RRB CLERK
& Other Competitive Exams

the Reporting Verb is in Past Tense. Therefore, rains and


thunderstorm may will be replaced by rains and thunderstorm
might,

Ans 788: (c)


Until should be followed by an Affirmative sentence. Therefore,
until they confessed their crime is the right usage.

Ans 789: (d)


and report the matter to him will be replaced by and reported the
matter to him. The sentence refers to a Past event. Therefore, all
the actions will be in Past Tense (Sequence of Tenses)

Ans 790: (a)


instead of will be replaced by in spite of. instead of (Prep.) means
: in the place of some body/something inspite of (Prep.) means :
despite; even though
Look at the sentence :
(i) They went swimming in spite of all the danger signs.
(ii) Now I can walk to work instead of going by car.

Ans 791: (d)


to five year’s imprisonment will be replaced by to five-year
imprisonment‘. Remember : Numeral Adj. + hyphen + Noun
(Singular).

Ans 792: (e) No error.

Facebook Page- https://www.facebook.com/vishalpariharpage Youtube- https://www.youtube.com/vishalparihar


Follow
186 Vishal Sir Telegram Channel- https://t.me/englishbyvishalsirchannel Instagram- https://www.instagram.com/vishalthetrainer
By Vishal sir
Also Important For SBI CLERK, IBPS CLERK, RRB CLERK
& Other Competitive Exams

Q793. Some genuine issues exist (a)/ with the newly adopted (b)/ system Q801. Giving unfair _________ to one’s relatives in the matter of
and needs to (c)/ be examined seriously. (d)/ No error (e) appointments and such other benefits is __________.
(a) punishment, parochialism
Q794. Whether or not to confront (a)/ them about their role (b)/ in the (b) advantage, nepotism
matter is a decision (c)/ which is yet to take. (d)/ No error (e) (c) benefits, pragmatism
(d) preferences, chauvinism
Q795. The government is still in the (a)/ process of finalised new policy (e) leverage, communalism
(b)/ guidelines for the allocation of land (c)/ to private sector
organisations. (d)/ No error (e) Q802. The Vidhan Sabha voted to __________ the ban ___________
drinking.
(a) invoke, for
Q796. According to government estimates (a)/ at least four million
(b) revoke, of
tonnes of sugar (b)/ will have to be imported (c)/ this year because
(c) prohibit, prevented
of a poor monsoon.
(d) repeal, on
(d)/ No error (e)
(e) refrain, from

Q797. In our experience people usually (a)/ value things that they have
Q803. Our army is __________ _________ the border of the neighbour
to (b)/
ing country.
pay off more than those that (c)/ they receive free of cost. (d)/ No (a) posted, with
error (e)
(b) accumulated, for
(c) assembled, at
Q798. At present China is the (a)/ world‘s leader manufacturer (b)/ of (d) deployed, along
environment friendly products (c)/ such as electric cars and bi (e) fighting, on
cycles. (d)/ No error (e)
Q804. The teacher ___________ his students ___________ being late
Q799. Over eighty per cent from us (a)/ feel that if we had taken (b)/ to school.
some corrective measures earlier (c)/ the crisis could have been (a) shouted, at
averted. (d)/ No error (e) (b) reprimanded, for
(c) reminded, at
Q800. The manager of that city branch (a)/ cannot handle it with the help (d) narrated, of
of (b)/ only two personnel as (c)/ business has increased (e) taught, for
substantially. (d)/ No error (e)
Q805. The dimensions of the ___________ are known and the solution
Directions (801-805): Each question below has two blanks, is __________ on.
each blank indicating that something has been omitted. (a) problem, agreed (b) theory, dealt
Choose the set of words for each blank that best fits the
(c) measures, decided (d) risks, tempered
meaning of the sentence as whole.
(e) danger, looked

Facebook Page- https://www.facebook.com/vishalpariharpage Youtube- https://www.youtube.com/vishalparihar


Follow
187 Vishal Sir Telegram Channel- https://t.me/englishbyvishalsirchannel Instagram- https://www.instagram.com/vishalthetrainer
By Vishal sir
Also Important For SBI CLERK, IBPS CLERK, RRB CLERK
& Other Competitive Exams

Sol. Reprimand is a formal expression of disapproval.


Ans 793: (c) S805. Ans. (a)
Plural Subj. takes Plural Verb. (without s) Therefore, system and
need to is the right usage.

Ans 794: (d)


PV should be used. Therefore, which is yet to take will be re
placed by which is yet to be taken.

Ans 795: (b)


Gerund should be used. Therefore, process of finalised new
policy will be replaced by process of finalising new policy.

Ans 796: (a)


Definite Article-the should be used. Therefore, According to the
government estimates is the right usage.

Ans 797: (c)


pay more than those that is the right usage. pay off (Phr. V.) : to
be successful and bring good results (of a plan/an action)

Ans 798: (b)


“world’s leader manufacturer” will be replaced by world’s
leading manufacturer. The word leading is an Adj..

Ans 799: (a)


The word per cent is followed by Prep.-of. Therefore, over

Ans 800: (e) No error.

S801. Ans. (b)


Sol. Nepotism means the practice among those with power or
influence of favouring relatives or friends, especially by giving
them jobs.
S802. Ans. (d)
Sol. Repeal means revoke or annul (a law or act of parliament).
S803. Ans. (d)
Sol. Deployed means to move (troops) into position for military
action.
S804. Ans. (b)

Facebook Page- https://www.facebook.com/vishalpariharpage Youtube- https://www.youtube.com/vishalparihar


Follow
188 Vishal Sir Telegram Channel- https://t.me/englishbyvishalsirchannel Instagram- https://www.instagram.com/vishalthetrainer
By Vishal sir
Also Important For SBI CLERK, IBPS CLERK, RRB CLERK
& Other Competitive Exams

Directions (806-810): In the following passage, some of the (e) guard


words have been left out, each of which is indicated by a
number. Find the suitable word from the options given Directions (811-815): In the following passage, some of the
against each number and fill up the blanks with appropriate words have been left out, each of which is indicated by a
words to make the paragraph meaningfully complete. number. Find the suitable word from the options given
against each number and fill up the blanks with appropriate
A factor that air passengers give little thought to but which is a words to make the paragraph meaningfully complete.
serious threat to air safety is (806) maintenance. In the current
global airline boom competition is intense which compels airlines In recent years, the banking industry has been undergoing rapid
to minimise costs and maximise output. In India with a seat changes, reflecting a number of (811) developments. The most
capacity considerably (807) of the demand all airlines practise significant has been advances in communication and information
severe cost-cutting to (808). Faced with having to cut costs to the technology, which have (812) and broadened the (813) of
bone and maximise aircraft utilization, a surprising number of financial information while lowering the costs of many financial
airlines may cut (809) on aircraft maintenance, even at the risk of activities. A second key (814) for change has been the increasing
compromising safety. While commercial aircraft are designed to competition among a broad (815) of domestic and foreign
take much punishment, for example, in the event of pilots flying institutions in providing banking and related financial services.
into thunderstorms there is a limit to the punishment that even the Third, financial activity has become larger relative to overall
toughest aircraft can take when profitability takes (810) over economic activity in most economies.
safety.
811.
Q806. (a) challenging (b) subjective
(a) expedient (b) incessant (c) situated (d) underlying
(c) routine (d) laborious (e) principled
(e) poor
812.
Q807. (a) measure (b) motioned
(a) ahead (b) less (c) habituated (d) processed
(c) more (d) saturated (e) accelerated
(e) above
813.
Q808. (a) concealment (b) disagreement
(a) invest (b) survive (c) dissemination (d) sowing
(c) live (d) appraise (e) differentiation
(e) reinforce
814.
Q809. (a) force (b) impetus
(a) out (b) taxes (c) pull (d) movement
(c) across (d) corners (e) energy
(e) short
815.
Q810. (a) group (b) rank
(a) encouragement (b) influence (c) place (d) range
(c) precedence (d) cover (e) row

Facebook Page- https://www.facebook.com/vishalpariharpage Youtube- https://www.youtube.com/vishalparihar


Follow
189 Vishal Sir Telegram Channel- https://t.me/englishbyvishalsirchannel Instagram- https://www.instagram.com/vishalthetrainer
By Vishal sir
Also Important For SBI CLERK, IBPS CLERK, RRB CLERK
& Other Competitive Exams

S806.Ans.(d)
S807.Ans.(a)
S808.Ans.(e)
S809.Ans.(b)
S810.Ans.(c)
S811.Ans. (c)
S812.Ans.(e)
S813.Ans.(b)
S814.Ans.(c)
S815.Ans. (c)

Facebook Page- https://www.facebook.com/vishalpariharpage Youtube- https://www.youtube.com/vishalparihar


Follow
190 Vishal Sir Telegram Channel- https://t.me/englishbyvishalsirchannel Instagram- https://www.instagram.com/vishalthetrainer
By Vishal sir
Also Important For SBI CLERK, IBPS CLERK, RRB CLERK
& Other Competitive Exams

(a) taxi managing by aggregators within India


Directions (816-820): A part of the sentence is bold below are (b) taxis managed in aggregators of India
given alternatives to thee underlined part at (a), (b), (c) and (c) taxis managed at aggregators by India
(d) which may improve the sentence choose the correct (d) taxis managed by aggregators across India
alternative and mark your answer in the answer sheet. In case (e) No replacement required
no improvement is needed, our answer is (e).
Directions (821-825): Rearrange the following six sentences
Q816 Visiting the village is like be transported into some other (A), (B), (C), (D), (E) and (F) in the proper sequence to form
century. a meaningful paragraph and then answer the questions given
(a) alike to transport below.
(b) like being transported
(c) likely to be transporting A. Even a limb severed in an accident can now be stitched back
(d) likes transport by doctors under certain conditions using modern techniques.
(e) No correction required B. A stroke victim can be saved if he or she is rushed to a good
hospital within the golden hour.
Q817. Environmentalists has pay little heed to the 'softer' aspects of the C. But at the same time, there is a very different aspect of modern
movement, of which the need to change our culture is one of the medical practice
most important. D. Modern medicine has advanced a lot — which is indeed a boon
(a) is paying little heed to humanity.
(b) are paying little heeding E. That is seen to be manifesting itself increasingly.
(c) has paid little heeds F. One who suffers a heart attack can lead a normal life with the
(d) have paid little heed help of a bypass surgery or angioplasty
(e) No correction required
Q821. Which of the following should be the FIRST sentence after
Q818. An honest introspection should reveal the need for utilise public rearrangement?
infrastructure budgets more effectively. (a)A (b) B (c) C (d) D (e) F
(a) need for utilising Q822. Which of the following should be the SECOND sentence after
(b) needs to utilise rearrangement?
(c) need to utilise (a)A (b) B (c) C (d) D (e) F
(d) needs for utilising Q823. Which of the following should be the THIRD sentence after
(e) No correction required rearrangement?
(a)A (b) B (c) C (d) D (e) F
Q819. Despite public investments have started to gain traction, this is Q824. Which of the following should be the FOURTH sentence after
yet to reflect in the performance of investment-linked sectors. rearrangement?
(a) Although public (a)A (b) B (c) C (d) D (e) E
(b) inspite of public
(c) though public Q825. Which of the following should be the FIFTH sentence after
(d) although if rearrangement?
(e) No correction required (a)A (b) B
(c) C (d) D (e) E
Q820. The rapid growth and popularity of taxi managed by
aggregators in India is a testimony to the fact that public
transport and transit facilities remain hopelessly inadequate

Facebook Page- https://www.facebook.com/vishalpariharpage Youtube- https://www.youtube.com/vishalparihar


Follow
191 Vishal Sir Telegram Channel- https://t.me/englishbyvishalsirchannel Instagram- https://www.instagram.com/vishalthetrainer
By Vishal sir
Also Important For SBI CLERK, IBPS CLERK, RRB CLERK
& Other Competitive Exams

S816.Ans.(b)
Replace ‘like be transported’ with ‘like being transported’.
S817.Ans.(d)
Sol.
Replace ‘has pay little heed’ with ‘have paid little heed’.
S818.Ans.(c)
Sol.
Replace ‘need for utilise’ with ‘need to utilise’.
S819.Ans.(c)
Sol.
Replace ‘Despite public’ with ‘though public’.
S820.Ans.(d)
the correct sequence is DAFBCE
S821.Ans.(d)
S822.Ans.(a)
S823.Ans.(e)
S824.Ans.(b)
S825.Ans.(c)

Facebook Page- https://www.facebook.com/vishalpariharpage Youtube- https://www.youtube.com/vishalparihar


Follow
192 Vishal Sir Telegram Channel- https://t.me/englishbyvishalsirchannel Instagram- https://www.instagram.com/vishalthetrainer
By Vishal sir
Also Important For SBI CLERK, IBPS CLERK, RRB CLERK
& Other Competitive Exams

Directions (826-830): Pick out the most effective word from (a) statements, attacking
the given words to fill in the blanks to make the sentence (b) ideas, scathing
meaningfully complete. (c) pronouncements, outdated
(d) allegations, outlandish
Q826. While the technology and state of knowledge in medicine allows (e) defamations, severe
the gadgets to _____ data and give recommendation, it will be a
while before we are ready to _____ doctors for serious illnesses. Directions (831-840 ): In the following passage there are
(a) estimate, credit blanks, each of which has been numbered.
(b) evaluate, credence These numbers are printed below the passage and against each
(c) analyse, trust five words are suggested, one of which fits the blank
(d) describe, distrust appropriately. Find out the appropriate word in each case.
(e) figure out, confidence Jamshedji Tata is (831) as the path-finder of modern industrial
builders, He is known as the grand-father of the Indian industry
Q827. Just because momentum is _____ away coal in china does not for his acumen and enthusiasm. Nobody else could have (832) of
mean that the country is no longer part of the global coal _____. the new industries started by Jamshedji at the time when
industrial (833) and revolution was yet to come to India.
(a) altering, bang Jamshedji’s father Nasarvanji Tata used to trade in jute with
(b) changing, collapse China and Britain, He started (834) from India. Jamshedji started
(c) stagnating, rise a cloth mill in Nagpur more than hundred did years ago. At that
(d) shifting, boom time almost all the (835) used to come from Lancashire in
(e) staying, failure England. What Jamshedji was praise-worthy.
Jamshedji (836) very well that an industrial revolution can only
Q828. There’s plenty of _____ that transferring kids to the adult be brought in the country by setting up iron and steel
criminal justice system for trial and conviction has _____ to industry. (837) he did not live to see the industry he had in mind,
prevent repeat offences. he had done all (838) work. In fact, he laid the ground work for
it. He had (839) the entire steel city now known as Jamshedpur,
(a) data, declined (840) with streets, roads, schools, parks, play grounds, temples,
(b) information, rise mosques, churches, etc.
(c) disproof, developed
(d) indication, finished Q831. (a)
(e) evidence, failed agreed (b) empowered (c) determined (d) considered (e)
Rewarded
Q829. Healthy scepticism, a vital ___________ for any journalist, has Q832. (a)
given ______________ to credulity, or worse, naivete. thought (b) ventured (c) set (d) planned (e) absolved
(a) factor, up Q833. (a) acts (b) machinery (c) awakening (d) factories (e) imports
(b) role, them Q834. (a)
(c) attribute, way industries (b) import (c) trade (d) dispatch (e) export
(d) thing, away Q835. (a)
(e) task, chance imports (b) cloth (c) machines (d) machinery (e) goods
Q836. (a)
Q830. Kejriwal’s ________ about all kinds of people became more and advocated (b) planned (c) thought (d) knew (e) felt
more _________ and even if there was a grain of truth in any of Q837. (a)
them, there was just no evidence. although (b) Surprisingly (c) Luckily (d) Even (e) Because

Facebook Page- https://www.facebook.com/vishalpariharpage Youtube- https://www.youtube.com/vishalparihar


Follow
193 Vishal Sir Telegram Channel- https://t.me/englishbyvishalsirchannel Instagram- https://www.instagram.com/vishalthetrainer
By Vishal sir
Also Important For SBI CLERK, IBPS CLERK, RRB CLERK
& Other Competitive Exams

S826.Ans. (c)
Sol.
Analyze mean examine (something) methodically and in detail,
typically in order to explain and interpret it.

S827.Ans.(d)
Sol.
‘shifting, boom’ fits the two blanks appropriately
Boom means a loud, deep, resonant sound.

S828.Ans.(e)
Sol.
‘evidence, failed’ fits the two blanks appropriately.

S829.Ans.(c)
Sol.
Attribute means regard something as being caused by.

S830.Ans.(d)
Sol.
Outlandish means looking or sounding bizarre or unfamiliar.

S831.Ans.(d)
S832.Ans.(a)
S833.Ans.(c)
S834.Ans.(e)
S835.Ans.(b)
S836.Ans.(d)
S837.Ans.(a)

Facebook Page- https://www.facebook.com/vishalpariharpage Youtube- https://www.youtube.com/vishalparihar


Follow
194 Vishal Sir Telegram Channel- https://t.me/englishbyvishalsirchannel Instagram- https://www.instagram.com/vishalthetrainer
By Vishal sir
Also Important For SBI CLERK, IBPS CLERK, RRB CLERK
& Other Competitive Exams

Q838. (a) (a) only (i)


complete (b) trivial (c) preliminary (d) external (e) Insignificant (b) only (ii)
Q839. (a) (c) both (ii) & (iii)
planned (b)warning (c)level (d)called (e)needed (d) both (i) & (iii)
Q840. (a) (e) none of these
fully (b)complete (c)empty (d)figured (e)reacted
Q844. Inciting
Q841. Decry (i) The racist man tried to incite hatred in his children by telling
them falsehoods about minority groups.
(i) Several religious organizations have chosen to decry the (ii) The least stimulus which suffices to incite is known as the
obscene cartoon about Jesus. stimulus of threshold value.
(ii) After the immigration bill was passed, the racist groups (iii) The purpose of the terror alerts is not to incite panic but
started to decry the features of the act. rather to make people more cautious.
(iii) After conducting experiments for several years, the scientist (a) Only (i) and (iii)
was able to decry the cause of the disease. (b) Only (ii) and (iii)
(a) both (i) & (ii) (c) Only (i) and (ii)
(b) only (ii) (d) Only (ii)
(c) both (ii) & (iii) (e) All of the above
(d) both (i) & (iii)
(e) none of these Q845. Duel
(i) Frank was a generous man who was always feign to help
Q842. Illegible those in need.
(ii) I understand some of your impractical jokes, but to feign
(i) Candidates above 32 years are not illegible to apply. your own death on April Fool's Day is a little over the top.
(ii) The cashier refused to cash my check because she could not (iii) Although I was on a diet, I was feign to eat a large slice of
understand the check writer’s illegible signature. cake.
(iii) After my essay fell into the muddy puddle, it was illegible (a) Only (ii)
because of the blurred ink. (b) Only (ii) and (iii)
(c) Only (i) and (ii)
(a) only (i) (d) Only (i) and (iii)
(b) only (ii) (e) All of the above
(c) both (ii) & (iii)
(d) both (i) & (iii) Q846. Gait
(e) none of these (i) Have you ever been to the Golden Gait Bridge in San
Francisco?
Q843. Immanent (ii) The employees staged a demonstration in front of the main
(i) As a coexistence of opposites, the sacred is immanent in pure gait of the factory.
awareness, the ground of language and thought. (iii) As he strode through the hallway, everyone could tell by his
(ii) Despite the fact that Robin is an immanent expert on art, she gait that he was wealthy.
would rather let her husband choose the pieces they display in (a) Only (i)
their home. (b) Only (ii) (c) Only (iii)
(iii) They can expropriate property through taxes or the right of (d) Both (i) and (ii) (e) None of the above
immanent domain.

Facebook Page- https://www.facebook.com/vishalpariharpage Youtube- https://www.youtube.com/vishalparihar


Follow
195 Vishal Sir Telegram Channel- https://t.me/englishbyvishalsirchannel Instagram- https://www.instagram.com/vishalthetrainer
By Vishal sir
Also Important For SBI CLERK, IBPS CLERK, RRB CLERK
& Other Competitive Exams

S838.Ans.(c) S846. Ans. (c)


S839.Ans(a) Sol. Among (i), (ii) and (iii), only statement (iii) is a
S840.Ans (b) grammatically correct and contextually meaningful sentence.
In statement (i) and (ii) gait does not fits perfectly in the
S841. Ans. (a) sentence. Instead of ‘gait’, ‘gate’ makes a contextually
Sol. Among the given statements, ‘decry’ has been used meaningful and grammatically correct statement.
correctly in statements (i) & (ii) both. In the statement (iii), Gait - manner of walking; stride, step
‘decry’ can be replaced with ‘descry’, to make the statement Gate - entrance
grammatically correct and contextually meaningful. Hence, Hence, option (c) is the correct answer.
option (a) is the most suitable answer choice.
Decry - to criticize publicly, denounce
Descry - catch sight of/discern

S842. Ans. (c)


Sol. Among the given statements, ‘illegible’ has been used
correctly in statements (ii) and (iii).
Eligible - qualify, fit to be chosen
Illegible - that cannot be read
Hence, option (c) is the most suitable answer choice.

S843. Ans. (d)


Sol. Among the given statements, ‘immanent’ has been used
correctly in statements (i) and (iii). Hence, option (d) is the most
suitable answer choice.
Eminent - distinguished
Immanent - inherent: existing as inalienable part

S844. Ans. (a)

Sol. Among (i), (ii) and (iii), (i) and (iii) are grammatically
correct and contextually meaningful sentences. In statement (ii)
the use of “incite” does not makes a contextually meaningful
sentence. Here, ' incite' can be replaced with 'excite' to make a
contextually meaningful sentence.
Hence, option (a) is the correct answer.

S845. Ans. (a)


Sol. Among (i), (ii) and (iii), only statement (ii) is a
grammatically correct and contextually meaningful sentence.
Feign - pretend, give the appearance of
Fain - pleased or willing

Facebook Page- https://www.facebook.com/vishalpariharpage Youtube- https://www.youtube.com/vishalparihar


Follow
196 Vishal Sir Telegram Channel- https://t.me/englishbyvishalsirchannel Instagram- https://www.instagram.com/vishalthetrainer
By Vishal sir
Also Important For SBI CLERK, IBPS CLERK, RRB CLERK
& Other Competitive Exams

Q847. Hoard (iii) At the age of eighty, my grandmother has a zest for life that
(i) Uncle Jack loves to hoard stuff, filling his house with allows her to live like a carefree teenager.
useless objects. (a) only (i)
(ii) Digging in her garden, she uncovered a hoard of gold dating (b) only (ii)
back to the 9th century. (iii) Because many people cared about (c) both (ii) & (iii)
the historical church, a hoard horde of volunteers helped repaint (d) both (i) & (iii)
the building. (e) none of these
(a) Only (i)
(b) Only (ii) Directions (851-855): In the following questions, a word is
(c) Both (i) and (iii) given in bold followed by four sentences. Choose the
(d) Both (i) and (ii) appropriate option mentioning the sentence that uses the
(e) All (i), (ii) and (iii) highlighted word in a grammatically correct and contextually
meaningful manner.
Q848. Hue
(i) While the little boy wanted to hue trees with his big brother, Q851. Mantle
he was too small to wield an axe. (a) He had thrust his chest out, supporting his shoulders against
(ii) The pale hue of the patient’s skin was a sign of anemia. the mantle.
(iii) Nobody understands why Danielle decided to dye her hair (b) She wrapped herself in a warm, heavy mantle, prepared to
such an unusual hue. embrace the cold wind outside.
(a) Only (iii) (c) Be careful not to strike your head against the mantle.
(b) Only (ii) and (iii) (d) He set his watch to the clock on the mantle.
(c) Only (i) and (ii) (e) None of these.
(d) Only (ii)
(e) All of the above Q852. Feat
(a) Not let the grass grow under one's feat.
Q849. Ingenuous (b) An enemy who lies at thy feat begging forgiveness must not
(i) With how skilled my chess opponent was, it would take an feel thy sword
ingenuous strategy to defeat him, and a fair amount of luck (c) He stamped his feat to shake off the snow from his boots.
(ii) While I do not consider myself ingenuous, I do not mind (d) It is a rare feat when an archer hits the center target twenty
giving everyone at least one chance to do good. times in a row
(iii) Our captain’s ingenuous ingenious plan would allow us to (e) None of these.
sneak around the enemy and capture the objective without a fight.
(a) only (i) Q853. FLAIR
(b) only (ii) (a) Since Rich has a flair for storytelling, it is not surprising that
(c) both (ii) & (iii) he is a popular children’s author.
(d) both (i) & (iii) (b) She poked at the fire causing it to flair up and throw more
(e) none of these shadows across their faces.
(c) The flair from the lighter scared the small child as the flame
Q850. Zest illuminated her face.
(i) When confronted by the teacher, the student said his words (d) As the camper rubbed the two sticks together, it even shocked
were in zest and not meant to be taken seriously. him to see the flair spark
(ii) Because I have a fear of heights, I do not have a zest for flying. (e) None of these.

Facebook Page- https://www.facebook.com/vishalpariharpage Youtube- https://www.youtube.com/vishalparihar


Follow
197 Vishal Sir Telegram Channel- https://t.me/englishbyvishalsirchannel Instagram- https://www.instagram.com/vishalthetrainer
By Vishal sir
Also Important For SBI CLERK, IBPS CLERK, RRB CLERK
& Other Competitive Exams

S847. Ans. (d) Sol. ‘Feat’ is defined as something that require great courage
Sol. and skill to achieve whereas ‘Feet’ is lower extremity of the leg
Hoard - to store or hidden stock below the ankle, on which a person stands. Hence, option (d) is
Horde - in gang the correct answer.
Among (i), (ii) and (iii), (i) and (ii) are grammatically correct
and contextually meaningful sentences. In statement (iii) the use S853. Ans. (a)
of “hoard” does not makes a contextually meaningful sentence. Sol. ‘Flair’ means a special aptitude or ability for doing
Here, ‘hoard’ can be replaced with 'horde' to make a something right whereas ‘Flare’ means a sudden burst of bright
contextually meaningful sentence. flame or light. Also the use of ‘flair is grammatically incorrect
Hence, option (d) is the correct answer. instead it should be ‘flare’. Hence, option (a) is the correct
` answer.
S848. Ans. (b)
Sol. Among (i), (ii) and (iii), (ii) and (iii) statements are
grammatically correct and contextually meaningful.
Hue - colour, of different types
Hew - chop, saw down
Hence, option (b) is the correct answer.

S849. Ans. (b)


Sol. Among the given statements, ‘ingenuous’ has been used
correctly in statements (ii) only. In the statement (i) and (ii),
‘ingenuous’ can be replaced with ‘Ingenious’, to make the
statement grammatically correct and contextually meaningful.
Hence, option (b) is the most suitable answer choice.
Ingenuous - innocent, unsuspecting mostly in a childish way
Ingenious - clever, original or inventive

S850. Ans. (c)


Sol. Among the given statements, ‘zest’ has been used correctly
in statements (ii) and (iii).
Jest – Fun, quip
Zest – Keenness, avidity
Hence, option (c) is the most suitable answer choice.

S851. Ans. (b)


Sol. A ‘Mantle’ is a shawl, cloak or a covering of a specified
sort whereas ‘Mantel’ is the shelf or slab. So in option (a), (c)
and (d) the correct word should be ‘Mantel’. Hence, option (b)
is the correct answer.

S852. Ans. (d)

Facebook Page- https://www.facebook.com/vishalpariharpage Youtube- https://www.youtube.com/vishalparihar


Follow
198 Vishal Sir Telegram Channel- https://t.me/englishbyvishalsirchannel Instagram- https://www.instagram.com/vishalthetrainer
By Vishal sir
Also Important For SBI CLERK, IBPS CLERK, RRB CLERK
& Other Competitive Exams

Q854. Whet (ii) Shooting someone is sure to cause him or her a great deal of
(a) While it was still whet, I gently teased out the tangled knots corporal suffering.
in Rosie's hair. (iii) Slaves were often tied to trees and whipped as a form of
(b) We jolted along rough whet roads through an endless banana corporal punishment.
plantation. (a) only (i)
(c) The mobile phone company uses celebrity endorsements to (b) only (ii)
make consumers whet their phones (c) both (ii) & (iii)
(d) It's too whet for the children to play out today; they'll have to (d) both (i) & (iii)
amuse themselves indoors. (e) none of these
(e) None of these.
Q858. Corpse
Q855. Ley
(a) As soon as they were settled at a table, Felipa ley out her plan. (i) Lowering the corpse into the grave, the family members of the
(b) For a moment she ley waiting, and then realized he had fallen deceased wept bitterly.
asleep. (ii) If someone among the corpse of workers on the assembly line
(c) A newspaper with Allen's picture on it ley opened on the doesn’t do their job, the whole product will be worthless.
coffee table. (iii) Police found the corpse of the missing woman outside of her
(d) The clover-grass ley is then grazed for a year or two with friend’s apartment building.
sheep. (a) only (i) (b) only (ii)
(e) None of these. (c) both (ii) & (iii) (d) both (i) & (iii)
(e) none of these
Directions (856-860): A word has been given in each question
and has been used in the sentences given below. Identify the Q859. Descent
statements where the word has been used in a contextually (i) The descent into the coal mines was taken by elevator.
and grammatically correct manner. If the word has been used (ii) Marla’s dad tried to make her act descent in public, but she
incorrectly in all the statements, mark (e), “None of these”, as lacked respect or modesty.
your answer. (iii) When it came to driving down the long hill, the inexperienced
driver was not good at its descent.
Q856. Complaisant (a) Only (i) and (iii) (b) Only (ii) and (iii)
(i) I am rightfully not complaisant with low quality service. (c) Only (i) and (ii) (d) Only (ii)
(ii) Because John is too complaisant in his willingness to please (e) All of the above
others, people often take advantage of him.
(iii) If 1,000 signatures on this petition doesn't satisfy you, then Q860. Deferment
how many will make you complaisant? (i) Firefighters deserve to be granted with the utmost deferment
(a) only (i) for their loyal service.
(b) only (ii) (ii) In my first year of college I was a thousand miles from home
(c) both (ii) & (iii) with a low draft number and no student deferment.
(d) both (i) & (iii) (iii) Show some deferment to your classmates by raising your
(e) none of these hand rather than shouting out every answer.
(a) Only (ii)
Q857. Corporal (b) Only (ii) and (iii) (c) Only (i) and (ii)
(i) The sense of touch allows us to navigate safely through the (d) Only (i) and (iii) (e) All of the above
many features of the corporal world.

Facebook Page- https://www.facebook.com/vishalpariharpage Youtube- https://www.youtube.com/vishalparihar


Follow
199 Vishal Sir Telegram Channel- https://t.me/englishbyvishalsirchannel Instagram- https://www.instagram.com/vishalthetrainer
By Vishal sir
Also Important For SBI CLERK, IBPS CLERK, RRB CLERK
& Other Competitive Exams

sentence. Here, ' descent' can be replaced with ' decent' to make
S854. Ans. (c) a contextually meaningful sentence.
Sol. ‘Whet’ means to trigger or make desire and ‘Wet’ means Decent - socially acceptable, moral, hottest
covered with water. So the usage of whet in option (a) and (b) is Descent – origin, decline, downward movement
incorrect. Also ‘Whet’ in option (d) is grammatically incorrect. Hence, option (a) is the correct answer.
Hence, option (c) is the correct answer.
S860. Ans. (a)
S855. Ans. (d) Sol. Among (i), (ii) and (iii), only statement (ii) is a
Sol. ‘Ley’ means an area of open land used as a temporary grammatically correct and contextually meaningful sentence.
pasture for animals and ‘Lay’ means put down gently or Deference - respect and politeness
carefully. Hence, option (d) is the only option with correct Deferment - postponement
usage.

S856. Ans. (b)


Sol. Among the given statements, ‘complaisant’ has been used
correctly in statements (ii) only. In the statement (i) and (ii),
‘complaisant’ can be replaced with ‘complacent’, to make the
statement grammatically correct and contextually meaningful.
Hence, option (b) is the most suitable answer choice.
Complaisant – Agreeable, cooperative
Complacent – Become satisfied on own perception

S857. Ans. (c)


Sol. Among the given statements, ‘Corporal’ has been used
correctly in statements (ii) and (iii).
Corporal – Physical
Corporeal –Having a body or physical appearance
Hence, option (c) is the most suitable answer choice.

S858. Ans. (d)


Sol. Among the given statements, ‘corpse’ has been used
correctly in statements (i) and (iii). Hence, option (d) is the most
suitable answer choice.

corpse - a dead body of human being –


Corps - a unit of army for a certain cause/group of persons for a
specific activity

S859. Ans. (a)


Sol. Censure- an official reprimand
Among (i), (ii) and (iii), (i) and (iii) are grammatically correct
and contextually meaningful sentences. In statement (ii) the use
of “descent” does not makes a contextually meaningful

Facebook Page- https://www.facebook.com/vishalpariharpage Youtube- https://www.youtube.com/vishalparihar


Follow
200 Vishal Sir Telegram Channel- https://t.me/englishbyvishalsirchannel Instagram- https://www.instagram.com/vishalthetrainer
By Vishal sir
Also Important For SBI CLERK, IBPS CLERK, RRB CLERK
& Other Competitive Exams

Directions (861-870): A word has been given in each (a) only (i)
question and has been used in the sentences given below. (b) only (ii)
Identify the statements where the word has been used in a (c) both (ii) & (iii)
contextually and grammatically correct manner. If the word (d) both (i) & (iii)
has been used incorrectly in all the statements, mark (e), (e) none of these
“None of these”, as your answer.
Q864. Proscribe
Q861. Abate (i) The policies of the school district proscribe accessing social
media websites on school computers.
(i) Can you believe the community has decided to abate crime (ii) Since the obese woman agreed to begin an aerobics
by not giving the police the names of suspected gang members? program, the doctor agreed to proscribe weight loss pills.
(ii) The firemen sprayed water on the burning house to abate (iii) When my parents discover my boyfriend has an arrest
the structural damage. (iii) If you abate violence in your home, record, they are going to proscribe me from seeing him.
you should not be surprised if your children grow up to be (a) Only (i) and (iii)
abusers. (b) Only (ii) and (iii)
(a) only (i) (c) Only (i) and (ii)
(b) only (ii) (d) Only (ii)
(c) both (ii) & (iii) (e) All of the above
(d) both (i) & (iii)
(e) none of these Q865. Propriety
(i) Placing his propriety into a small suitcase, the immigrant
Q862. Affectation prepared to take the few things he had left from his old life into
the new land.
(i) Although we began dating without much affectation, with (ii) After Harold wore a clown suit to his cousin’s funeral, we
time it developed into a fully blossomed relationship. had to question his sense of propriety.
(ii) Jake’s proper manner of speaking was an affectation he put (iii) Several pieces of the woman’s jewellery were stolen in the
on when he was surrounded by the country club set. break-in, but that is the only propriety that seems to be missing.
(iii) My mother does not realize buying a pricey new dress to (a) Only (ii)
attend a party is an affectation to impress strangers. (b) Only (ii) and (iii)
(a) only (i) (c) Only (i) and (ii)
(b) only (ii) (d) Only (i) and (iii)
(c) both (ii) & (iii) (e) All of the above
(d) both (i) & (iii)
(e) none of these Q866. Quell
(i) When I visited the local park, a quell darted near the pond
Q863. Averse and pecked with its beak at the bread crumbs lying around.
(i) For those who are averse to spaghetti, you also have the (ii) “There is no comparison between the beautiful red robin,”
meal choice of baked chicken with roasted potatoes. said the ornithologist, “and the drab-looking quell.”
(ii) Averse weather conditions forced us to pull off of the road (iii) In an attempt to quell unrest among the soldiers, the
and wait until things cleared up. commander authorized an increase in leave days for all military
(iii) Although Mary knows the advantages of using a microwave personnel.
for food preparation, she is old-fashioned and averse to serving (a) Only (i) (b) Only (ii) (c) Only (iii)
meals she has not personally prepared. (d) Both (i) and (ii) (e) None of the above

Facebook Page- https://www.facebook.com/vishalpariharpage Youtube- https://www.youtube.com/vishalparihar


Follow
201 Vishal Sir Telegram Channel- https://t.me/englishbyvishalsirchannel Instagram- https://www.instagram.com/vishalthetrainer
By Vishal sir
Also Important For SBI CLERK, IBPS CLERK, RRB CLERK
& Other Competitive Exams

S861. Ans. (b) Propriety – conformity to conventionally accepted standards of


Sol. Among the given statements, ‘abate’ has been used behaviour or morals. Property – a thing or things belonging to
correctly in statements (ii) only. In the statement (i) and (ii), someone; possessions collectively.
‘abate’ can be replaced with ‘abet,’ to make the statement
grammatically correct and contextually meaningful. Hence, S866. Ans. (c)
option (b) is the most suitable answer choice. Sol. Quell - put an end to (a rebellion or other disorder),
Abate - (of something unpleasant or severe) become less intense typically by the use of force.
or widespread. Among (i), (ii) and (iii), only statement (iii) is a grammatically
Abet - encourage or assist (someone) to do something wrong, in correct and contextually meaningful sentence.
particular to commit a crime. In statement (i) and (ii) quell does not fits perfectly in the
sentence. Instead of ‘quell, ‘quail’ makes a contextually
S862. Ans. (c) meaningful and grammatically correct statement.
Sol. Among the given statements, ‘affectation’ has been used Quail - a small or medium-sized New World game bird, the
correctly in statements (ii) and (iii). male of which has distinctive facial markings.
Affectation – behaviour, speech, or writing that is pretentious Hence, option (c) is the correct answer.
and designed to impress. Affection – a gentle feeling of
fondness or liking.
Hence, option (c) is the most suitable answer choice.

S863. Ans. (d)


Sol. Among the given statements, ‘averse’ has been used
correctly in statements (i) and (iii). Hence, option (d) is the most
suitable answer choice.
Averse – having a strong dislike of or opposition to something.
Adverse – preventing success or development; harmful;
unfavourable.

S864. Ans. (a)


Sol. Censure- an official reprimand
Among (i), (ii) and (iii), (i) and (iii) are grammatically correct
and contextually meaningful sentences. In statement (ii) the use
of “proscribe” does not makes a contextually meaningful
sentence. Here, proscribe can be replaced with Prescribe to
make a contextually meaningful sentence.
Proscribe - forbid, especially by law.
Prescribe- recommend (a substance or action) as something
beneficial.
Hence, option (a) is the correct answer.

S865. Ans. (a)


Sol. Among (i), (ii) and (iii), only statement (ii) is a
grammatically correct and contextually meaningful sentence.

Facebook Page- https://www.facebook.com/vishalpariharpage Youtube- https://www.youtube.com/vishalparihar


Follow
202 Vishal Sir Telegram Channel- https://t.me/englishbyvishalsirchannel Instagram- https://www.instagram.com/vishalthetrainer
By Vishal sir
Also Important For SBI CLERK, IBPS CLERK, RRB CLERK
& Other Competitive Exams

Q867. Revel (ii) If his words are not enough, the evidence will corroborate
(i) During the wedding reception, the happy couple appeared to his claim.
revel in their new marital status. (iii) After watching the videotape, the officials were able to
(ii) In just a few days, thousands of people will go downtown to corroborate the runner’s claim of interference during the race.
revel in the city’s New Year’s Eve celebration.
(iii) Once the police officer opened the freezer in the basement, (a) only (i)
it would revel the remains of the missing girl. (b) only (ii)
(a) Only (i) (c) both (ii) & (iii)
(b) Only (ii) (d) both (i) & (iii)
(c) Both (i) and (iii) (e) none of these
(d) Both (i) and (ii)
(e) All (i), (ii) and (iii)
Directions (871-880): A word has been given in each
Q868. Sensitive question and has been used in the sentences given below.
(i) Matt’s sensitive decision to not ride in a car with his Identify the statements where the word has been used in a
intoxicated friends is the only reason he is alive today. contextually and grammatically correct manner. If the word
(ii) Although the expectant mother was only a few weeks has been used incorrectly in all the statements, mark (e),
pregnant, the highly sensitive test was able to pick up on the “None of these”, as your answer.
hormones right away.
(iii) The screen was sensitive to touch, meaning the director Q871. Wrest
only had to lightly swipe his finger over the surface to pull up (i) My wife had to wrest the candy bar from my hands in order
the film. for me to stop eating it.
(a) Only (iii) (ii) Sally did not give up her phone easily as her father had to
(b) Only (ii) and (iii) wrest it from her hands. (iii) light in my heart the evening star
(c) Only (i) and (ii) of wrest and then let the night whisper to me of love.
(d) Only (ii) (a) both (i) & (ii) (b) only (ii)
(e) All of the above (c) both (ii) & (iii)
(d) both (i) & (iii)
Q869. Soar (e) none of these
(i) After staying up so late, the night owl’s eyes were soar and
stinging from exhaustion. Q872. Wreaths
(ii) Gas prices always seem to soar in the summer when (i) When the old man had a heart attack, he fell to the floor and
families are getting ready to travel. began to wreaths around on the carpet while holding his chest.
(iii) Rubbing his soar ankle, the runner hoped that the pain (ii) The wreaths were so nearly alike that none of those who
would subside before his next meet. were with the king could point out any difference.
(a) only (i) (b) only (ii) (iii) It turned out they were preparing funeral wreaths for the
(c) both (ii) & (iii) (d) both (i) & (iii) next day.
(e) none of these (a) only (i)
(b) only (ii)
Q870. Corroborate (c) both (ii) & (iii)
(i) The software engineers will corroborate on the project to (d) both (i) & (iii)
make sure it is available to the customer on the agreed upon (e) none of these
date.

Facebook Page- https://www.facebook.com/vishalpariharpage Youtube- https://www.youtube.com/vishalparihar


Follow
203 Vishal Sir Telegram Channel- https://t.me/englishbyvishalsirchannel Instagram- https://www.instagram.com/vishalthetrainer
By Vishal sir
Also Important For SBI CLERK, IBPS CLERK, RRB CLERK
& Other Competitive Exams

S867. Ans. (d) grammatically correct and contextually meaningful. Hence,


Sol. Revel: enjoy oneself in a lively and noisy way, especially option (a) is the most suitable answer choice.
with drinking and dancing. Among (i), (ii) and (iii), (i) and (ii) Wrest - Snatch, grab
are grammatically correct and contextually meaningful Rest - Relax, pause
sentences. In statement (iii) the use of “revel” does not makes a
contextually meaningful sentence. Here, ‘revel’ can be replaced S872. Ans. (c)
with ‘reveal’ to make a contextually meaningful sentence. Sol. Among the given statements, ‘wreaths’ has been used
Reveal: make (previously unknown or secret information) correctly in statements (ii) and (iii).
known to others. Wreath – A garland, circlet.
Hence, option (d) is the correct answer. Writhe – Squirm, roll in pain.
` Hence, option (c) is the most suitable answer choice.
S868. Ans. (b)
Sol. Among (i), (ii) and (iii), (ii) and (iii) statements are
grammatically correct and contextually meaningful.
Sensitive – quick to detect or respond to slight changes, signals,
or influences.
Sensible – done or chosen in accordance with wisdom or
prudence; likely to be of benefit.
Hence, option (b) is the correct answer.

S869. Ans. (b)


Sol. Among the given statements, ‘soar’ has been used correctly
in statements (ii) only. In the statement (i) and (ii), ‘soar’ can be
replaced with ‘sore’, to make the statement grammatically
correct and contextually meaningful. Hence, option (b) is the
most suitable answer choice.
Soar – fly or rise high in the air.
Sore – (of a part of one's body) painful or aching.

S870. Ans. (c)


Sol. Among the given statements, ‘Corroborate’ has been used
correctly in statements (ii) and (iii).
Collaborate - work jointly for a specific project/activity - A
Japanese company has collaborated with an Indian firm to
develop the project for high speed trains.
Corroborate - to strengthen or support with other evidence;
make more certain.
Hence, option (c) is the most suitable answer choice.

S871. Ans. (a)


Sol. Among the given statements, ‘wrest’ has been used
correctly in statements (i) & (ii) both. In the statement (iii),
‘wrest’ can be replaced with ‘rest’, to make the statement

Facebook Page- https://www.facebook.com/vishalpariharpage Youtube- https://www.youtube.com/vishalparihar


Follow
204 Vishal Sir Telegram Channel- https://t.me/englishbyvishalsirchannel Instagram- https://www.instagram.com/vishalthetrainer
By Vishal sir
Also Important For SBI CLERK, IBPS CLERK, RRB CLERK
& Other Competitive Exams

Q873. Whet (iii) When my daughter told me she saw a unicorn in the
(i) As I walked up my grandmother’s steps, I realized the smell backyard, I had serious doubts about the verity of her statement.
of her food was enough to whet my appetite. (a) Only (i)
(ii) He does not have whet of sense while dealing, with others. (b) Only (ii)
(iii) The purpose of television commercials is to whet the (c) Only (iii)
interest of potential buyers. (a) only (i) (d) Both (i) and (ii)
(b) only (ii) (e) None of the above
(c) both (ii) & (iii)
(d) both (i) & (iii) Q877. Tampering
(e) none of these (i) Online shopping agencies send products to their customers
after sealing the parcels against any kind of tampering.
Q874. Willing (ii) All three were barred in February for tampering with urine
(i) Well, any mother-in-law who isn't willing to baby-sit her samples during out-of-competition tests.
own grandchildren isn't much of a grandmother. (iii) The hardboards are given due tampering with oil and resin
(ii) The leader lost in the election mostly tor his willing to make them more durable.
disregard for people's feelings. (iii) The open source movement (a) Only (i)
and Creative Commons licensing are examples of (b) Only (ii)
people willing to share their intellectual labor to help others. (c) Both (i) and (iii)
(a) Only (i) and (iii) (d) Both (i) and (ii)
(b) Only (ii) and (iii) (e) All (i), (ii) and (iii)
(c) Only (i) and (ii)
(d) Only (ii) Q878. Statute
(e) All of the above (i) They erected a statute to glorify the martyr.
(ii) The rapist cannot be tried for his crimes since the statute of
Q875. Virtuous limitations expired two years ago.
(i) There is a virtuous absence of moral values in modern life. (iii) Because the statute bans smoking in restaurants, we will
(ii) Natalie considered herself very virtuous because she neither have to go outside to smoke after dinner.
drank nor smoked. (iii) She had been a virtuous prisoner in the (a) Only (iii)
cabin since she had seen the snake on the porch. (b) Only (ii) and (iii)
(a) Only (ii) (c) Only (i) and (ii)
(b) Only (ii) and (iii) (d) Only (ii)
(c) Only (i) and (ii) (e) All of the above
(d) Only (i) and (iii)
(e) All of the above Q879. Stationery
(i) When the train engineer realized there was a stationery car
Q876. Verity on the tracks, he tried to stop the locomotive's engine.
(ii) A choice was easily made at the dry goods store between the
(i) The talented jewelry maker sold a verity of unique, hand- detailed embossed stationery over the plain white notecards to
crafted bracelets, rings, and necklaces and not a single design send messages to my friends and family. (iii) Because a truck
was the same. flipped over, we have been stuck in stationery traffic for over
(ii) The small bakery held a verity of cupcakes, pastries, and three hours now.
muffins in exciting flavors like cotton candy and passion fruit. (a) only (i) (b) only (ii) (c) both (ii) & (iii)
(d) both (i) & (iii) (e) none of these

Facebook Page- https://www.facebook.com/vishalpariharpage Youtube- https://www.youtube.com/vishalparihar


Follow
205 Vishal Sir Telegram Channel- https://t.me/englishbyvishalsirchannel Instagram- https://www.instagram.com/vishalthetrainer
By Vishal sir
Also Important For SBI CLERK, IBPS CLERK, RRB CLERK
& Other Competitive Exams

S873. Ans. (d) Tampering - interfering with to cause damage or unauthorized


Sol. Among the given statements, ‘whet’ has been used alteration.
correctly in statements (i) and (iii). Hence, option (d) is the most Tempering - improve hardness and elasticity, resiliency.
suitable answer choice. Hence, option (d) is the correct answer.
Whet - sharpen. `
Whit - a small amount. S878. Ans. (b)
Sol. Among (i), (ii) and (iii), (ii) and (iii) statements are
S874. Ans. (a) grammatically correct and contextually meaningful.
Sol. Among (i), (ii) and (iii), (i) and (iii) are grammatically Statue - effigy, idol
correct and contextually meaningful sentences. In statement (ii) Statute - laws, decrees
the use of “willing” does not makes a contextually meaningful Hence, option (b) is the correct answer.
sentence. Here, ' willing’ can be replaced with ‘wilful’ to make
a contextually meaningful sentence. S879. Ans. (b)
Willing - inclined, agreed Sol. Among the given statements, ‘stationery’ has been used
Wilful - intentional, deliberate/adamant correctly in statements (ii) only. In the statement (i) and (ii),
Hence, option (a) is the correct answer. ‘stationery’ can be replaced with ‘stationary,’ to make the
statement grammatically correct and contextually meaningful.
S875. Ans. (a) Hence, option (b) is the most suitable answer choice.
Sol. Among (i), (ii) and (iii), only statement (ii) is a Stationary - not moving or not changing.
grammatically correct and contextually meaningful sentence. Stationery - writing or other office materials.
Virtuous – Having good virtues of moral standards.
Virtual – Almost, nearly.

S876. Ans. (c)


Sol. Among (i), (ii) and (iii), only statement (iii) is a
grammatically correct and contextually meaningful sentence.
In statement (i) and (ii) verity does not fits perfectly in the
sentence. Instead of ‘verity,’ ‘variety’ makes a contextually
meaningful and grammatically correct statement.
Verity – Truth.
Variety – The state of being various.
Hence, option (c) is the correct answer.

S877. Ans. (d)


Sol.
Among (i), (ii) and (iii), (i) and (ii) are grammatically correct
and contextually meaningful sentences. In statement (iii) the use
of “tampering” does not makes a contextually meaningful
sentence. Here, ‘tampering’ can be replaced with ‘tempering’
to make a contextually meaningful sentence.

Facebook Page- https://www.facebook.com/vishalpariharpage Youtube- https://www.youtube.com/vishalparihar


Follow
206 Vishal Sir Telegram Channel- https://t.me/englishbyvishalsirchannel Instagram- https://www.instagram.com/vishalthetrainer
By Vishal sir
Also Important For SBI CLERK, IBPS CLERK, RRB CLERK
& Other Competitive Exams

Q880. Secrete (c) both (ii) & (iii)


(i) The secrete report incriminating the powerful leader was (d) both (i) & (iii)
leaked in local media. (ii) The pancreas is an organ that can (e) none of these
secrete insulin which is needed to regulate blood sugar.
(iii) The exocrine system uses ducts to secrete hormones to the Q883. Confidant
outside of the body. (i) Because my husband is my confidant, I feel as though I can
(a) only (i) tell him anything without the information becoming public
(b) only (ii) (ii) The salesperson had a confidant manner with plenty of
(c) both (ii) & (iii) personality.
(d) both (i) & (iii) (iii) In prison the killer told his confidant, his roommate, about
(e) none of these all the murders he had committed.
(a) only (i)
Directions (881-890): A word has been given in each (b) only (ii)
question and has been used in the sentences given below. (c) both (ii) & (iii)
Identify the statements where the word has been used in a (d) both (i) & (iii)
contextually and grammatically correct manner. If the word (e) none of these
has been used incorrectly in all the statements, mark (e),
“None of these”, as your answer. Q884. Censure
(i) The pharmacy board will censure the pharmacist for not
Q881. CANVASS properly monitoring his drug supply.
(ii) Jane has been the censure of the media since she won the
(i) Our canvass sail was so heavy, that I could barely lift a award for best supporting actress.
corner of the folded fabric. (iii) The tardy fireman is going to be given a censure by his
(ii) After a heated canvass, in which he made a series of commander.
brilliant speeches, he was beaten by a narrow margin in New (a) Only (i) and (iii)
York. (b) Only (ii) and (iii)
(iii) Because I make tents out of canvass, I need a needle strong (c) Only (i) and (ii)
enough to stitch the tough fabric. (d) Only (ii)
(a) only (i) (e) All of the above
(b) only (ii)
(c) both (ii) & (iii) Q885. Duel
(d) both (i) & (iii) (i) That marriage lacked the duel significance it should have
(e) none of these had.
(ii) The two men were ready to duel to the death and wouldn’t
Q882. Altar consider cancelling the showdown
(i) Because my current healthcare plan is not meeting my needs, (iii) The government has employed a duel strategy to achieve
I am going to altar the benefits of my policy these two objectives.
(ii) Candles and incense were lit as sacrificial gifts and placed at (a) Only (ii)
the base of the wooden altar. (b) Only (ii) and (iii)
(iii) An altar was built by those needing a platform from which (c) Only (i) and (ii)
to worship various gods and goddesses. (d) Only (i) and (iii)
(a) only (i) (e) All of the above
(b) only (ii)

Facebook Page- https://www.facebook.com/vishalpariharpage Youtube- https://www.youtube.com/vishalparihar


Follow
207 Vishal Sir Telegram Channel- https://t.me/englishbyvishalsirchannel Instagram- https://www.instagram.com/vishalthetrainer
By Vishal sir
Also Important For SBI CLERK, IBPS CLERK, RRB CLERK
& Other Competitive Exams

S880. Ans. (c) S885. Ans. (a)


Sol. Among the given statements, ‘secrete’ has been used Sol. Among (i), (ii) and (iii), only statement (ii) is a
correctly in statements (ii) and (iii). grammatically correct and contextually meaningful sentence.
Secret – Confidential, unrevealed Dual – Combination of two things or two parts
Secrete – Produce, discharge Duel – Fight or combat
Hence, option (c) is the most suitable answer choice.

S881. Ans. (b)


Sol. Among the given statements, ‘canvass’ has been used
correctly in statements (ii) only. In the statement (i) and (ii),
‘canvass’ can be replaced with ‘canvas’, to make the statement
grammatically correct and contextually meaningful. Hence,
option (b) is the most suitable answer choice.
CANVASS - propose (an idea or plan) for discussion.
CANVAS - a strong, coarse unbleached cloth made from hemp,
flax, or a similar yarn, used to make items such as sails and tents
and as a surface for oil painting.

S882. Ans. (c)


Sol. Among the given statements, ‘altar’ has been used correctly
in statements (ii) and (iii).
Altar – platform for worship or ceremonies
Alter – To change
Hence, option (c) is the most suitable answer choice.

S883. Ans. (d)


Sol. Among the given statements, ‘confidant’ has been used
correctly in statements (i) and (iii). Hence, option (d) is the most
suitable answer choice.
Confidant – Close aide
Confident – Having confidence

S884. Ans. (a)


Sol. Censure- an official reprimand
Among (i), (ii) and (iii), (i) and (iii) are grammatically correct
and contextually meaningful sentences. In statement (ii) the use
of “censure” does not makes a contextually meaningful
sentence. Here, 'censure' can be replaced with 'cynosure' to
make a contextually meaningful sentence.
Cynosure- something or someone that is the center of attention
Hence, option (a) is the correct answer.

Facebook Page- https://www.facebook.com/vishalpariharpage Youtube- https://www.youtube.com/vishalparihar


Follow
208 Vishal Sir Telegram Channel- https://t.me/englishbyvishalsirchannel Instagram- https://www.instagram.com/vishalthetrainer
By Vishal sir
Also Important For SBI CLERK, IBPS CLERK, RRB CLERK
& Other Competitive Exams

Q886. INSIDIOUS (i) Many of these holes are as round and as cleanly cut as if they
had been made with an augur.
(i) The caller claimed three insidious devices had been planted (ii) Diminishing gas prices augur a high amount of road travel
at the Vineyards. on the upcoming holiday. (iii) In a process very similar to
(ii) Sources said the explosion seemed to have been caused by extracting a cork from a wine bottle, the augur bit is twisted in
an insidious device. and extracted without rotation.
(iii) Although the medicine made Gabriel feel better at first, its (a) only (i)
effect was insidious and only lasted a short time. (b) only (ii)
(a) Only (i) (c) both (ii) & (iii)
(b) Only (ii) (d) both (i) & (iii)
(c) Only (iii) (e) none of these
(d) Both (i) and (ii)
(e) None of the above Q890. Corroborate
(i) The software engineers will corroborate on the project to
Q887. ANTICS make sure it is available to the customer on the agreed upon
(i) Because the antics of the political candidates are so date.
disturbing I will not be voting for any of them. (ii) If his words are not enough, the evidence will corroborate
(ii) My younger son never tires of watching the kitten’s antics his claim.
with the string. (iii) After watching the videotape, the officials were able to
(iii) You might have to eat your words on that purchase I made corroborate the runner’s claim of interference during the race.
of them valuable antics.
(a) Only (i) (a) only (i)
(b) Only (ii) (b) only (ii)
(c) Both (i) and (iii) (c) both (ii) & (iii)
(d) Both (i) and (ii) (d) both (i) & (iii)
(e) All (i), (ii) and (iii) (e) none of these

Q888. Appraise Directions (891-895): For each question is given a sentence or


a group of sentences with four highlighted words. Among the
(i) He has been appraised of any institution-wide issues that the four highlighted words, two words needs to be interchanged
professional body report has identified. to make the sentence grammatically and contextually correct.
(ii) The insurance company appraised the famous painting at Choose the option which mentions the correct interchange to
ten million dollars. make the stanza grammatically and contextually correct.
(iii) The prosecutor appraised the evidence and said it was not
substantial enough to go to trial. Q891. consumer (1) materials from garbage should be a high priority
(a) Only (iii) (2), considering that India is the third highest Recovering (3) of
(b) Only (ii) and (iii) materials after China and the U.S.; the Economic Survey 2019
(c) Only (i) and (ii) estimates that India’s demand for total material (4) will double
(d) Only (ii) by 2030 at current rates of growth.
(e) All of the above (a) 1-4,
(b) 2-4,
Q889. Augur (c) 1-3,
(d) 2-3, (e) None of these

Facebook Page- https://www.facebook.com/vishalpariharpage Youtube- https://www.youtube.com/vishalparihar


Follow
209 Vishal Sir Telegram Channel- https://t.me/englishbyvishalsirchannel Instagram- https://www.instagram.com/vishalthetrainer
By Vishal sir
Also Important For SBI CLERK, IBPS CLERK, RRB CLERK
& Other Competitive Exams

Corroborate - to strengthen or support with other evidence; make


S886. Ans. (c) more certain.
Sol. Insidious- something that is slowly and secretly causing Hence, option (c) is the most suitable answer choice.
harm
Among (i), (ii) and (iii), only statement (iii) is a grammatically Solution891. C
correct and contextually meaningful sentence. Phrase ‘consumer materials from garbage’ doesn’t make any
In statement (i) and (ii) insidious does not fits perfectly in the sense. But the phrase ‘recovering materials from garbage’ makes
sentence. Instead of ‘insidious’, ‘incendiary’ makes a sense. So, the interchange should be 1-3.
contextually meaningful and grammatically correct statement. Hence, the option (c) is the correct answer.
Incendiary- explosive
Hence, option (c) is the correct answer.

S887. Ans. (d)


Sol. ANTICS: foolish, outrageous, or amusing behaviour.
Among (i), (ii) and (iii), (i) and (ii) are grammatically correct
and contextually meaningful sentences. In statement (iii) the use
of “ANTICS” does not makes a contextually meaningful
sentence. Here, ‘antics’ can be replaced with 'antique' to make
a contextually meaningful sentence.
Hence, option (d) is the correct answer.
`
S888. Ans. (b)
Sol. Among (i), (ii) and (iii), (ii) and (iii) statements are
grammatically correct and contextually meaningful.
Appraise – to estimate the value of something
Apprise – to inform or tell
Hence, option (b) is the correct answer.

S889. Ans. (b)


Sol. Among the given statements, ‘augur’ has been used correctly
in statements (ii) only. In the statement (i) and (ii), ‘augur’ can be
replaced with ‘auger’, to make the statement grammatically
correct and contextually meaningful. Hence, option (b) is the
most suitable answer choice.
Augur – Become a sign for either good or bad
Auger – A tool used in carpentry

S890. Ans. (c)


Sol. Among the given statements, ‘Corroborate’ has been used
correctly in statements (ii) and (iii).
Collaborate - work jointly for a specific project/activity - A
Japanese company has collaborated with an Indian firm to
develop the project for high speed trains.

Facebook Page- https://www.facebook.com/vishalpariharpage Youtube- https://www.youtube.com/vishalparihar


Follow
210 Vishal Sir Telegram Channel- https://t.me/englishbyvishalsirchannel Instagram- https://www.instagram.com/vishalthetrainer
By Vishal sir
Also Important For SBI CLERK, IBPS CLERK, RRB CLERK
& Other Competitive Exams

Q892. Zero-budget intervention (1) is a form of natural farming which


is neither chemical-loaded nor organic with its reliance (2) on Directions: In each of the following questions a sentence is
manure. It is a form of gardening (3) as a self-sustainable given with four highlighted words. Choose the alternative,
practice with minimum external farming (4). reflecting the appropriate replacement of the words so as to
(a) 2-1, form a grammatically correct and contextually meaningful
(b) 3-4, sentence. If no replacement is required, mark (e) i.e. “no
(c) 4-1, correction required” as your answer choice.
(d) 2-4,
(e) None of these Q896. It is not difficult to (A)imagine the bewildering array of (B)
communal that the recent (C) queries riots in the nation’s capital
Q893. After all, apart from the high base effect for some companies such have (D)triggered in the minds of young people.
as Tata Consultancy Services Ltd, the industry needs to contend (a) A – D
with a global quarter (1). This double whammy (2) has already (b) B – D
hit growth (3) at Accenture Plc, which reported results for the (c) B – C
slowdown (4) ended February late last month. (d) A – C
(e) No correction required
(a) 1-4,
(b) 1-3, Q897. Despite a century of (A) removed, the quantum world still
(c) 2-4, remains (B)mysterious and far (C) research from our
(d) 3-4, (D)experiences based on everyday life.
(e) None of these (a) A – D
(b) B – D
Q894. In general, too, the budget did not slowdown (1) major problems (c) B – C
such as rationalised (2) the economy, job creation, or bringing (d) A – C
the economy out of the address (3) phase. The only welcome (e) No correction required
feature was that labour laws were being simplified and
stimulating (4). Q898. Even as China’s success in containing the epidemic is in the
(a) 1-3, (A)spotlight, its cover-up of the (B) stain until mid-January,
(b) 2-3, nearly a month after the first few (C) cases showed up, will
(c) 2-4, remain a (D) outbreak hard to erase.
(d) 4-3, (a) A – D
(e) None of these (b) B – D
(c) B – C and A – D
Q895. But how do banks figure out which product (1) to offer to which (d) A – C
customer? Your transaction patterns with a bank gives a lot of (e) No correction required
offer (2). For instance, if you are spending repeatedly (3) to buy
movie tickets using the Net banking facility, the bank is likely to Q899. The (A) tribal of Chhattisgarh laid the basis of a (B) supposed
insights (4) you a co-branded credit card that has some related shift in development, which was essentially (C) paradigm to
benefits. benefit the (D) formation population.
(a) 2-3 (a) B – D
(b) 1-3 (b) B – C
(c) 1-4 (c) A – D and B – C
(d) 2-4 (e) None of these (d) A – C (e) No correction required

Facebook Page- https://www.facebook.com/vishalpariharpage Youtube- https://www.youtube.com/vishalparihar


Follow
211 Vishal Sir Telegram Channel- https://t.me/englishbyvishalsirchannel Instagram- https://www.instagram.com/vishalthetrainer
By Vishal sir
Also Important For SBI CLERK, IBPS CLERK, RRB CLERK
& Other Competitive Exams

Solution892. c Solution898. b
Among the given options, option (c) is the correct answer. The positions of the words “stain” and “outbreak” are incorrect.
The phrase ‘sustainable practice with minimum external farming’ Hence swapping the words in the positions (B) and (D) will frame
doesn’t make sense but the phrase ‘sustainable practice with a grammatically correct and contextually meaningful sentence.
minimum external intervention’ does make sense. So, the Therefore, option (b) is the most suitable answer choice.
interchange should be between the words 4 and 1 (or 1 and 4).
Hence, the option (c) is the correct answer. Solution899. c
The positions of the words “tribal” - “formation” and “supposed”
Solution893. a – “paradigm” should be interchanged as “paradigm shift” is a
Among the given options, option (a) is the correct answer. phrase which means a fundamental change in approach or
‘with a global quarter’ should be replaced by ‘with a global underlying assumptions while to define the characteristic of
slowdown.’ population “tribal” is an appropriate word. Hence swapping the
So, the interchange should be between the words 1 and 4 (or 4 words in the positions (A) – (D) and (B) – (C) will frame a
and 1). grammatically correct and contextually meaningful sentence.
Hence, the option (a) is the correct answer. Therefore, option (b) is the most suitable answer choice.

Solution894. c
The position of the words ‘rationalised’ and ‘stimulating’ is
wrong. Interchanging the words would make the sentence
grammatically correct and contextually meaningful.
So, the option (c) is the correct answer.

Solution895. d
The presence of the word ‘offer’ at (2) and ‘insights’ at (4) is
making the sentence grammatically & contextually incorrect and
incoherent. Upon interchanging the words at (2) and (4), we get
a grammatically correct and contextually meaningful sentence.
Hence, the option (d) is the correct answer.

Solution896. c
The positions of the words “communal” and “queries” are
incorrect as the appropriate. Hence swapping the words in the
positions (B) and (C) will frame a grammatically correct and
contextually meaningful sentence. Therefore, option (c) is the
most suitable answer choice.

Solution897. d
The positions of the words “removed” and “research” are
incorrect. Hence swapping the words in the positions (A) and (C)
will frame a grammatically correct and contextually meaningful
sentence. Therefore, option (d) is the most suitable answer
choice.

Facebook Page- https://www.facebook.com/vishalpariharpage Youtube- https://www.youtube.com/vishalparihar


Follow
212 Vishal Sir Telegram Channel- https://t.me/englishbyvishalsirchannel Instagram- https://www.instagram.com/vishalthetrainer
By Vishal sir
Also Important For SBI CLERK, IBPS CLERK, RRB CLERK
& Other Competitive Exams

Q900 Understandably the (A) world pages had to be scaled (B) Sudoku (b) B – D
from three to two and the (C) down grid was moved from the (D) (c) B – C and A – D
sports page to the sports pages. (d) A – C
(a) B – D (e) No correction required
(b) B – C
(c) A – D and B – C Q905. (A) Quarantine is considered the oldest (B) onslaughts to
(d) A – C reduce the (C) rapid spread of bacterial infections and viral (D)
(e) No correction required mechanism.
(a) A – D
Q901. The state of (A) depression affects an individual’s (B) ability to (b) B – D
think rationally, feel (C) realistically, and work (D) effectively. (c) B – C and A – D
(a) B – D (d) A – C
(b) B – C (e) No correction required
(c) A – D and B – C
(d) A – C Q906. In almost all climes (a) the tortoise and the frog are among
(e) No correction required the precursors (b) and heralds of this season, and birds fly with
song and glancing (c) plumage, and plants spring and bloom,
Q902. and winds blow, to correct this slight oscillation of the poles and
In (A) traffic to the routine heavy (B) contrast on the Delhi- preserve the equilibrium (d) of nature.
Gurugram highway, (C)plying vehicles could be seen (D) fewer (a) a-d
on the road all through the day. (b) a-d and c-b
(a) a-c (c) a-a, b-c and c-d
(b) a-d and b-c (d) b-d and c-a
(c) a-b and c-d (e) No change required
(d) a-d
(e) No change required
Q907. The current COVID-19 (A) quarantine, with its closure of shops,
Q903. (A) Lessons from the successful (B) containment and (C) (B) examinations institutions and postponement of public (C)
prevention strategies employed by South Korea, Taiwan, Hong academic, has put the people in a de facto (D) crisis.
Kong, Singapore and China are (D)instructive here (a) B – D
(a) A – D (b) B – C
(b) B – D (c) A – D and B – C
(c) B – C (d) A – C
(d) A – C (e) No correction required
(e) No correction required
Q908. The president, with the (A) advice and consent of the
Q904. The case of China may be more (A) spread to India given the (B)senate, appoints judges, diplomatic agents (C) governors, of
somewhat similar capacity shortfalls, its forthwith sealing off of territories, and (D) officers of the army and navy above the rank
whole (B) transmission centres and, most importantly, the fact of colonel.
that community (C) population had already exploded before (a) A-C (b) A-D and B-C (c) A-B and C-D
authorities had a firm handle on the (D) relevant. (d) A-D
(e) No change required
(a) A – D

Facebook Page- https://www.facebook.com/vishalpariharpage Youtube- https://www.youtube.com/vishalparihar


Follow
213 Vishal Sir Telegram Channel- https://t.me/englishbyvishalsirchannel Instagram- https://www.instagram.com/vishalthetrainer
By Vishal sir
Also Important For SBI CLERK, IBPS CLERK, RRB CLERK
& Other Competitive Exams

Solution900. c Solution906. e
The positions of the words “world” - “sports” and “Sudoku” – All the highlighted words in their original positions are
“down” should be interchanged. Hence swapping the words in grammatically correct and contextually meaningful and thus
the positions (A) – (D) and (B) – (C) will frame a grammatically successfully form a coherent sentence without interchanging any
correct and contextually meaningful sentence. Therefore, option of the words from their positions. Hence, option (e) is the most
(b) is the most suitable answer choice viable answer choice.

Solution901. e Solution907. c
All the highlighted words in their original positions are The positions of the words “quarantine” - “crisis” and
grammatically correct and contextually meaningful and thus “examinations” – “academic” should be interchanged. Hence
successfully form a coherent sentence without interchanging any swapping the words in the positions (A) – (D) and (B) – (C) will
of the words from their positions. Hence, option (e) is the most frame a grammatically correct and contextually meaningful
viable answer choice. sentence. Therefore, option (c) is the most suitable answer choice.

Solution 902: (c) Solution 908: (c)


The positions of the words “traffic” - “contrast” and “plying” – The positions of the words “advice” - “senate” and “governors” –
“fewer” should be interchanged. Hence swapping the words in “officers” should be interchanged. Hence swapping the words in the
the positions (A) – (B) and (C)– (D) will frame a grammatically positions (A) – (B) and (C)– (D) will frame a grammatically correct and
correct and contextually meaningful sentence. Therefore, option contextually meaningful sentence. Therefore, option (c) is the most suitable
(c) is the most suitable answer choice. answer choice.

Solution903. c
The positions of the words “containment” and “prevention” are
incorrect as the appropriate. Hence swapping the words in the
positions (B) and (C) will frame a grammatically correct and
contextually meaningful sentence. Therefore, option (c) is the
most suitable answer choice.

Solution904. c
The positions of the words “spread” - “relevant” and
“transmission” – “population” should be interchanged. Hence
swapping the words in the positions (A) – (D) and (B) – (C) will
frame a grammatically correct and contextually meaningful
sentence. Therefore, option (c) is the most suitable answer choice.

Solution905. b
The positions of the words “onslaughts” and “mechanism” are
incorrect. Hence swapping the words in the positions (B) and (D)
will frame a grammatically correct and contextually meaningful
sentence. Therefore, option (b) is the most suitable answer
choice.
Onslaughts means a fierce or destructive attack.

Facebook Page- https://www.facebook.com/vishalpariharpage Youtube- https://www.youtube.com/vishalparihar


Follow
214 Vishal Sir Telegram Channel- https://t.me/englishbyvishalsirchannel Instagram- https://www.instagram.com/vishalthetrainer
By Vishal sir
Also Important For SBI CLERK, IBPS CLERK, RRB CLERK
& Other Competitive Exams

Q909. The National (A) clearance Authority has recommended that the (a) A – D
testing and (B) proposal of COVID-19 be included in the PM- (b) B – D
Jan Arogya Yojana (PM-JAY) but this (C) treatment is still (c) B – C
awaiting (D) Health. (d) A – C
(e) No correction required
(a) a-c
(b) a-d and b-c Q914. COVID-19 is a (A) arbitrary that came with prior (B) warning,
(c) a-b and c-d and therefore did not warrant an (C) disaster, unplanned and ill-
(d) a-d prepared (D)decision.
(e) No change required (a) A – D
(b) B – D
Q910. The rapid (A) attention of the SARS-CoV-2 virus across the (c) B – C
world has focused (B spread on the seemingly invisible (d) A – C
processes that help (C) leap originally found in wild animals (e) No correction required
make the (D) pathogens to humans.
(a) a-c Q915. The complete 21-day COVID-19 (A)shutdown of most
(b) a-d and b-c economic activity has created new (B) sectors, causing severe
(c) a-b and c-d (C)disruptive impact on both demand and supply side elements
(d) a-d across (D) roadblocks.
(e) No change required (a) A – D
(b) B – D
Q911. (A) pathogens in forests harmlessly (B) retains dangerous (c) B – C and A – D
viruses and other (C) biodiversity among a vast (D)pool of wild (d) A – C
animals, away from people. (e) No correction required
(a) a-c
(b) a-d and b-c Q916. In the case of (A) enterprises workers, many casual and (B)
(c) a-b and c-d dependent workers are directly or indirectly (C) informal on the
(d) a-d survival of small and medium (D) contract for jobs.
(e) No change required (a) B – D
(b) B – C
Q912. A yellow ball (A) ecstatic past the net, the players grunt, the grass (c) A – D and B – C
(B) twitches and under azure blue skies, the (C) applause from (d) A – C
the stands ranges from the muted to the (D) glides. (e) No correction required
(a) a-c
(b) a-d and b-c Q917. The world can ill-afford (A)delays, as the pandemic is
(c) a-b and c-d (B)predicted to stage a (C)comeback once the (D)shutdowns
(d) a-d are gradually relaxed.
(e) No change required (a) B – D
(b) B – C
Q913. (c) A – D and B – C
As (A)infections have slowly begun to rise, there is little time to (d) A – C
be lost in (B) problem both the public health (C) addressing and (e) No correction required
the lockdown’s economic (D)impact.

Facebook Page- https://www.facebook.com/vishalpariharpage Youtube- https://www.youtube.com/vishalparihar


Follow
215 Vishal Sir Telegram Channel- https://t.me/englishbyvishalsirchannel Instagram- https://www.instagram.com/vishalthetrainer
By Vishal sir
Also Important For SBI CLERK, IBPS CLERK, RRB CLERK
& Other Competitive Exams

The positions of the words “sectors” and “roadblocks” are


Solution 909: (b) incorrect. Hence swapping the words in the positions (B) and (D)
The positions of the words “clearance” - “Health” and will frame a grammatically correct and contextually meaningful
“proposal” – “treatment” should be interchanged. Hence sentence. Therefore, option (b) is the most suitable answer
swapping the words in the positions (A) – (D) and (B) – (C) will choice.
frame a grammatically correct and contextually meaningful
sentence. Therefore, option (b) is the most suitable answer Solution916. c
choice. The positions of the words “enterprises” - “contract” and
“dependent” – “informal” should be interchanged. Hence
Solution 910: (c) swapping the words in the positions (A) – (D) and (B) – (C) will
The positions of the words “attention” - “spread” and “leap” – frame a grammatically correct and contextually meaningful
“pathogens” should be interchanged. Hence swapping the words sentence. Therefore, option (b) is the most suitable answer
in the positions (A) – (B) and (C)– (D) will frame a grammatically choice.
correct and contextually meaningful sentence. Therefore, option
(c) is the most suitable answer choice. Solution917. e
All the highlighted words in their original positions are
Solution 911: (a) grammatically correct and contextually meaningful and thus
The positions of the words “pathogens” and “biodiversity” are successfully form a coherent sentence without interchanging any
incorrect. Hence swapping the words in the positions (A) and (C) of the words from their positions. Hence, option (e) is the most
will frame a grammatically correct and contextually meaningful viable answer choice.
sentence. Therefore, option (a) is the most suitable answer choice.

Solution 912: (d)


The positions of the words “ecstatic” and “glides” are incorrect.
Hence swapping the words in the positions (A) and (D) will frame
a grammatically correct and contextually meaningful sentence.
Therefore, option (d) is the most suitable answer choice.

Solution913. c
The positions of the words “problem” and “addressing” are
incorrect as the appropriate. Hence swapping the words in the
positions (B) and (C) will frame a grammatically correct and
contextually meaningful sentence. Therefore, option (c) is the
most suitable answer choice.

Solution914. d
The positions of the words “arbitrary” and “disaster” are
incorrect. Hence swapping the words in the positions (A) and (C)
will frame a grammatically correct and contextually meaningful
sentence. Therefore, option (d) is the most suitable answer
choice.

Solution915. b

Facebook Page- https://www.facebook.com/vishalpariharpage Youtube- https://www.youtube.com/vishalparihar


Follow
216 Vishal Sir Telegram Channel- https://t.me/englishbyvishalsirchannel Instagram- https://www.instagram.com/vishalthetrainer
By Vishal sir
Also Important For SBI CLERK, IBPS CLERK, RRB CLERK
& Other Competitive Exams

Q918. (c) a-b and c-d


There is a need to (A) augment easy movement of trained health (d) a-d
(B)professionals across the world to train others and (e) No change required
(C)facilitate resources wherever there are (D)shortages.
(a) a-c Q923.
(b) a-b The initial (a) equinox is taken as the vernal (b) point on
(c) a-b and c-d the equatorial (c) from which coordinates are measured in
(d) a-d the sphere (d) and ecliptic systems.
(e) No change required (a) a-c
(b) a-d and b-c
Q919. We must (A) shutdowns food shortages (B) consequent sooner (c) a-b and c-d
or later, in some part of the world, (C) occurring to the national (d) a-d
(D) anticipate. (e) No change required
(a) a-c
(b) a-d and b-c Q924. As the world (A) novel, India must plan its (B) aggressive for a
(c) a-b and c-d calibrated exit, possibly in a week, from the most (C) strategy
(d) a-d lockdown anywhere to contain the (D) watches coronavirus.
(e) No change required (a) a-c
(b) a-d and b-c
(c) a-b and c-d
Q920. (d) a-d
It is (A) isolates to sequence the virus (B) important in at least (e) No change required
three different (C)sequencing in India to ensure that (D)
institutions errors are eliminated. Q925. After three weeks or more of (A) lockdown, India, where 284
(a) a-c districts have so far been (B)affected, should institute a system
(b) a-d and b-c of testing that includes not just (C) indicative cases but
(c) a-b and c-d surveillance samples to determine the (D)extent of spread.
(d) a-d
(e) No change required (a) a-c
(b) a-d and b-c
Q921. In a (A) benefits where only one in five people are (B) eligible (c) a-b and c-d
for unemployment (C) world, lay-offs spell (D)catastrophe for (d) a-d
millions of families. (e) No change required
(a) a-c
(b) a-d and b-c Q926.
(c) a-b and c-d Urban (A) congestion for workers in the (B)absence of public
(d) a-d transport could be made possible by (C)encouraging bicycle use
(e) No change required where feasible, avoiding (D) mobility.
(a) a-c
Q922. (A)domain is often (B)mocked as an (C)inconvenience, as the (b) a-d and b-c
(D)truth of fools or saints. (c) a-b and c-d
(a) a-c (d) a-d
(b) a-d and b-c (e) No change required

Facebook Page- https://www.facebook.com/vishalpariharpage Youtube- https://www.youtube.com/vishalparihar


Follow
217 Vishal Sir Telegram Channel- https://t.me/englishbyvishalsirchannel Instagram- https://www.instagram.com/vishalthetrainer
By Vishal sir
Also Important For SBI CLERK, IBPS CLERK, RRB CLERK
& Other Competitive Exams

Solution 918: (a) correct and contextually meaningful sentence. Therefore, option
The positions of the words “augment” and “facilitate” are (b) is the most suitable answer choice.
incorrect. Hence swapping the words in the positions (A) and (C)
will frame a grammatically correct and contextually meaningful Solution 925: (b)
sentence. Therefore, option (a) is the most suitable answer choice. The positions of the words “novel” - “watches” and “aggressive”
Solution 919: (b) – “strategy” should be interchanged. Hence swapping the words
The positions of the words “shutdowns” - “anticipate” and in the positions (A) – (D) and (B)– (C) will frame a grammatically
“consequent” – “occurring” should be interchanged. Hence correct and contextually meaningful sentence. Therefore, option
swapping the words in the positions (A) – (D) and (B) – (C) will (b) is the most suitable answer choice.
frame a grammatically correct and contextually meaningful
sentence. Therefore, option (b) is the most suitable answer Solution 926: (d)
choice. The positions of the words “congestion” and “mobility” are
incorrect. Hence swapping the words in the positions (A) and (D)
Solution 920: (c) will frame a grammatically correct and contextually meaningful
The positions of the words “isolates” - “important” and sentence. Therefore, option (d) is the most suitable answer
“sequencing” – “institutions” should be interchanged. Hence choice.
swapping the words in the positions (A) – (B) and (C)– (D) will
frame a grammatically correct and contextually meaningful
sentence. Therefore, option (c) is the most suitable answer choice.

Solution921: (a)
The positions of the words “benefits” and “world” are incorrect.
Hence swapping the words in the positions (A) and (C) will frame
a grammatically correct and contextually meaningful sentence.
Therefore, option (a) is the most suitable answer choice.

Solution 922: (d)


The positions of the words “domain” and “truth” are incorrect.
Hence swapping the words in the positions (A) and (D) will frame
a grammatically correct and contextually meaningful sentence.
Therefore, option (d) is the most suitable answer choice.

Solution 923: (c)


The positions of the words “initial” - “vernal” and “equatorial”
– “sphere” should be interchanged. Hence swapping the words
in the positions (A) – (B) and (C)– (D) will frame a grammatically
correct and contextually meaningful sentence. Therefore, option
(c) is the most suitable answer choice.

Solution 924: (b)


The positions of the words “novel” - “watches” and “aggressive”
– “strategy” should be interchanged. Hence swapping the words
in the positions (A) – (D) and (B)– (C) will frame a grammatically

Facebook Page- https://www.facebook.com/vishalpariharpage Youtube- https://www.youtube.com/vishalparihar


Follow
218 Vishal Sir Telegram Channel- https://t.me/englishbyvishalsirchannel Instagram- https://www.instagram.com/vishalthetrainer
By Vishal sir
Also Important For SBI CLERK, IBPS CLERK, RRB CLERK
& Other Competitive Exams

Q927. Most global banks, fearing U.S. (A) difficult and legal Q931.
consequences, stay away from doing (B) business with Iran, About 12,000 years ago, our (A) agriculture decided to settle
which makes it (C) retaliation for the Islamic Republic to find a (B)down in various places, mostly large river deltas, and began
functional payment (D)mechanism. (C) ancestors and the (D)domestication of cattle.
(a) a-c
(b) a-d and b-c (a) A – D
(c) a-b and c-d (b) B – D
(d) a-d (c) B – C
(e) No change required (d) A – C
(e) No correction required
Q928. Except during the hot season, when the (a)extreme are off the
fields, the general (b) verdant in normal years is that of Q932. Hopefully the culture of personal (A) hygiene which includes
a (c)aspect and well-tilled but very monotonous plain, only taking showers, (B) receded hand washing, and avoiding
merging into hilly or mountainous country at the (d)crops edges (C)spitting in public places will stay even after the threat of
of the basin on the south and north. SARS-CoV-2 has (D) practicing.
(a) a-d
(b) a-d and c-b (a) A – D
(c) b-c (b) B – D
(d) b-d and c-a (c) B – C and A – D
(e) No change required (d) A – C
(e) No correction required
Q929. There is an (A)enormous potential for growth in the India-U.S.
relationship, the Trump administration has said, (B) friendship Q933. It is obvious that: our government can make quick and well-
confidence that the ongoing trade (C)negotiations could be informed decisions; our (A) solutions can put in place effective
worked through because of the (D) exuding between the two steps (B) providing the size and diversity of our country, and our
nations. scientists and doctors can lead the world in (C) despite useful and
(a) A – D timely (D) administration and information.
(b) B – D
(c) B – C and A – D (a) B – D
(d) A – C (b) B – C
(e) No correction required (c) A – D and B – C
(d) A – C
Q930. Today we are in the midst of an (A) unprecedented and (e) No correction required
concerted attack on an infection, (B) ingenuity the resources and
(C) mobilising of the entire world to (D)control (and not even Q934. All countries have tried to (A)enforce border controls to stop the
annihilate) a single virus. (B)virus, which (C) ironically also demonstrated their
(D)futility.
(a) A – D (a) B – D
(b) B – D (b) B – C
(c) B – C (c) A – D and B – C
(d) A – C (d) A – C
(e) No correction required (e) No correction required

Facebook Page- https://www.facebook.com/vishalpariharpage Youtube- https://www.youtube.com/vishalparihar


Follow
219 Vishal Sir Telegram Channel- https://t.me/englishbyvishalsirchannel Instagram- https://www.instagram.com/vishalthetrainer
By Vishal sir
Also Important For SBI CLERK, IBPS CLERK, RRB CLERK
& Other Competitive Exams

Solution 927: (a) The positions of the words “solutions” - “administration” and
The positions of the words “difficult” and “retaliation” are “providing” – “despite” should be interchanged. Hence
incorrect. Hence swapping the words in the positions (A) and (C) swapping the words in the positions (A) – (D) and (B) – (C) will
will frame a grammatically correct and contextually meaningful frame a grammatically correct and contextually meaningful
sentence. Therefore, option (a) is the most suitable answer choice. sentence. Therefore, option (b) is the most suitable answer
choice.
Solution 928: (b)
The positions of the words “extreme” - “crops” and “verdant” Solution934. E
– “aspect” should be interchanged. Hence swapping the words in All the highlighted words in their original positions are
the positions (A) – (D) and (C)– (B) will frame a grammatically grammatically correct and contextually meaningful and thus
correct and contextually meaningful sentence. Therefore, option successfully form a coherent sentence without interchanging any
(b) is the most suitable answer choice. of the words from their positions. Hence, option (e) is the most
viable answer choice.
Solution929. 2
The positions of the words “exuding” and “friendship” are
incorrect. Hence swapping the words in the positions (B) and (D)
will frame a grammatically correct and contextually meaningful
sentence. Therefore, option (b) is the most suitable answer
choice.

Solution930. 3
The positions of the words “ingenuity” and “mobilising” are
incorrect as the appropriate. Hence swapping the words in the
positions (B) and (C) will frame a grammatically correct and
contextually meaningful sentence. Therefore, option (c) is the
most suitable answer choice.

Solution931. D
The positions of the words “agriculture” and “ancestors” are
incorrect. Hence swapping the words in the positions (A) and (C)
will frame a grammatically correct and contextually meaningful
sentence. Therefore, option (d) is the most suitable answer
choice.

Solution932. B
The positions of the words “receded” and “practicing” are
incorrect. Hence swapping the words in the positions (B) and (D)
will frame a grammatically correct and contextually meaningful
sentence. Therefore, option (b) is the most suitable answer
choice.

Solution933. C

Facebook Page- https://www.facebook.com/vishalpariharpage Youtube- https://www.youtube.com/vishalparihar


Follow
220 Vishal Sir Telegram Channel- https://t.me/englishbyvishalsirchannel Instagram- https://www.instagram.com/vishalthetrainer
By Vishal sir
Also Important For SBI CLERK, IBPS CLERK, RRB CLERK
& Other Competitive Exams

Q935. As it fights COVID-19 with its (A) halt healthcare resources, (c) a-b and c-d
India has chosen to bring the (B)economy to a near (C) meagre (d) a-d
with no clear idea of how many (D)lives can be saved in this (e) No change required
manner.
(a) a-c Q940.
(b) a-b Dean considered his poking (a) options as he used half a cake
(c) a-b and c-d (b) of soap to scrub away (c) the stink of the mine (d).
(d) a-d
(e) No change required (a) a-c
(b) a-d and b-c
Q936. The economic cost of (A) large COVID-19 can be reduced by (c) a-b and c-d
combining (B) proposed testing and isolation, a strategy (C) (d) a-d
aggressive by economist Paul Romer for the U.S. For it to work, (e) No change required
people must be tested in (D) combating numbers.
(a) a-c Directions (941-945): In each of the following questions a
(b) a-d and b-c sentence is given with four highlighted words. Choose the
(c) a-b and c-d alternative, reflecting the appropriate replacement of the
(d) a-d words so as to form a grammatically correct and contextually
(e) No change required meaningful sentence. If no replacement is required, mark (e)
i.e. “no correction required” as your answer choice.
Q937. Loan (A) bankruptcy and cash transfers can fend off (B)
moratoriums and defaults for a few months and buy (C) banks Q941. Despite (A)criticism from Democrats that his comments about
on non-performing assets in (D) time. the four minority Congresswomen are racist, Mr. Trump went on
(a) a-c a 20-minute (B) policies about them, saying they were welcome
(b) a-d and b-c to leave the country if they do not like his (C) diatribe on issues
(c) a-b and c-d such as immigration and (D)defending Israel.
(d) a-d
(e) No change required (a) A – D
(b) B – D
Q938. If the public health (A) versus can be the economy’s main (c) B – C
(B)engine for six months, the public health (C) sector economic (d) A – C
health trade-off can be (D)resolved. (e) No correction required
(a) a-c
(b) a-d and b-c Q942. While the risk of (A) announcement spread remains high the risk
(c) a-b and c-d outside the region (B)remains low, WHO chief said after the (C)
(d) a-d regional in Geneva. “The (international emergency) should not
(e) No change required be used to (D)stigmatise or penalise the very people who are
most in need of our help,” he said.
Q939. Having a colleague (A) community very early in the pandemic (a) A – D
will (B) hamper the (C)morale of not only the health workforce (b) B – D
but also the (D) succumb. (c) B – C
(a) a-c (d) A – C
(b) a-d and b-c (e) No correction required

Facebook Page- https://www.facebook.com/vishalpariharpage Youtube- https://www.youtube.com/vishalparihar


Follow
221 Vishal Sir Telegram Channel- https://t.me/englishbyvishalsirchannel Instagram- https://www.instagram.com/vishalthetrainer
By Vishal sir
Also Important For SBI CLERK, IBPS CLERK, RRB CLERK
& Other Competitive Exams

The positions of the words “diatribe” and “policies” are incorrect


Solution 935: (a) as the appropriate. Hence swapping the words in the positions (B)
The positions of the words “halt” and “meagre” are incorrect. and (C) will frame a grammatically correct and contextually
Hence swapping the words in the positions (A) and (C) will frame meaningful sentence. Therefore, option (c) is the most suitable
a grammatically correct and contextually meaningful sentence. answer choice.
Therefore, option (a) is the most suitable answer choice.
Solution942. D
Solution 936: (b) The positions of the words “regional” and “announcement” are
The positions of the words “large” - “combating” and incorrect. Hence swapping the words in the positions (A) and (C)
“proposed” – “aggressive” should be interchanged. Hence will frame a grammatically correct and contextually meaningful
swapping the words in the positions (A) – (D) and (B) – (C) will sentence. Therefore, option (d) is the most suitable answer
frame a grammatically correct and contextually meaningful choice.
sentence. Therefore, option (b) is the most suitable answer
choice.

Solution 937: (c)


The positions of the words “bankruptcy” - “moratoriums” and
“banks” – “time” should be interchanged. Hence swapping the
words in the positions (A) – (B) and (C)– (D) will frame a
grammatically correct and contextually meaningful sentence.
Therefore, option (c) is the most suitable answer choice.

Solution 938: (a)


The positions of the words “versus” and “sector” are incorrect.
Hence swapping the words in the positions (A) and (C) will frame
a grammatically correct and contextually meaningful sentence.
Therefore, option (a) is the most suitable answer choice.

Solution 939: (d)


The positions of the words “community” and “succumb” are
incorrect. Hence swapping the words in the positions (A) and (D)
will frame a grammatically correct and contextually meaningful
sentence. Therefore, option (d) is the most suitable answer
choice.

Solution940. E
All the highlighted words in their original positions are
grammatically correct and contextually meaningful and thus
successfully form a coherent sentence without interchanging any
of the words from their positions. Hence, option (e) is the most
viable answer choice.

Solution941. C

Facebook Page- https://www.facebook.com/vishalpariharpage Youtube- https://www.youtube.com/vishalparihar


Follow
222 Vishal Sir Telegram Channel- https://t.me/englishbyvishalsirchannel Instagram- https://www.instagram.com/vishalthetrainer
By Vishal sir
Also Important For SBI CLERK, IBPS CLERK, RRB CLERK
& Other Competitive Exams

Q943. There is an (A)enormous potential for growth in the India-U.S.


relationship, the Trump administration has said, (B) friendship Q947. To (A) bear the case for various (B)precautionary measures, we
confidence that the ongoing trade (C)negotiations could be must (C) assess in mind the dual (D)motive for taking
worked through because of the (D) exuding between the two precautions.
nations. (a) A – D
(a) A – D (b) B – D
(b) B – D (c) B – C
(c) B – C and A – D (d) A – C
(d) A – C (e) No correction required
(e) No correction required
Q948. (A) services that help poor people in their hour of (B) function
Q944. The (A) tribal of Chhattisgarh laid the basis of a (B) supposed without creating a major health (C) hazard should continue to
shift in development, which was essentially (C) paradigm to (D) need as far as possible.
benefit the (D) formation population. (a) A – D
(a) B – D (b) B – D
(b) B – C (c) B – C and A – D
(c) A – D and B – C (d) A – C
(d) A – C (e) No correction required
(e) No correction required
Q949. An (A) workplace list of essential (B) guidelines (already
Q945. The state of (A) depression affects an individual’s (B) ability to available in some States) and official (C) services on coronavirus
think rationally, feel (C) realistically, and work (D) effectively. readiness at the (D) explicit would be a good start.
(a) B – D (a) B – D
(b) B – C (b) B – C
(c) A – D and B – C (c) A – D and B – C
(d) A – C (d) A – C
(e) No correction required (e) No correction required

Directions (946-950): In each of the following questions a Q950. The (A) urgent need for effective (B)social security measures
sentence is given with four highlighted words. Choose the (C) makes it all the more (D) important to avoid a loss of nerve.
alternative, reflecting the appropriate replacement of the (a) B – D
words so as to form a grammatically correct and contextually (b) B – C
meaningful sentence. If no replacement is required, mark (e) (c) A – D and B – C
i.e. “no correction required” as your answer choice. (d) A – C
(e) No correction required
Q946. In terms of (A) casualties, the health crisis is still very (B) people
(seven deaths in a country where eight million (C) confined die Q951.
every year), but the numbers are (D)growing fast. As the COVID-19 threat (A) regularly, doctors, healthcare
(a) A – D (B)professionals and state institutions have been and (C) looms
(b) B – D issuing guidelines on the (D)precautions to be taken.
(c) B – C (a) a-c
(d) A – C (b) a-b (c) a-b and c-d
(e) No correction required (d) a-d (e) No change required

Facebook Page- https://www.facebook.com/vishalpariharpage Youtube- https://www.youtube.com/vishalparihar


Follow
223 Vishal Sir Telegram Channel- https://t.me/englishbyvishalsirchannel Instagram- https://www.instagram.com/vishalthetrainer
By Vishal sir
Also Important For SBI CLERK, IBPS CLERK, RRB CLERK
& Other Competitive Exams

Solution943. B Solution949. C
The positions of the words “exuding” and “friendship” are The positions of the words “workplace” - “explicit” and
incorrect. Hence swapping the words in the positions (B) and (D) “guidelines” – “services” should be interchanged. Hence
will frame a grammatically correct and contextually meaningful swapping the words in the positions (A) – (D) and (B) – (C) will
sentence. Therefore, option (b) is the most suitable answer frame a grammatically correct and contextually meaningful
choice. sentence. Therefore, option (b) is the most suitable answer
choice.
Solution944. C
The positions of the words “tribal” - “formation” and “supposed” Solution950. E
– “paradigm” should be interchanged as “paradigm shift” is a All the highlighted words in their original positions are
phrase which means a fundamental change in approach or grammatically correct and contextually meaningful and thus
underlying assumptions while to define the characteristic of successfully form a coherent sentence without interchanging any
population “tribal” is an appropriate word. Hence swapping the of the words from their positions. Hence, option (e) is the most
words in the positions (A) – (D) and (B) – (C) will frame a viable answer choice.
grammatically correct and contextually meaningful sentence.
Therefore, option (b) is the most suitable answer choice. Solution 951: (a)
The positions of the words “regularly” and “looms” are incorrect.
Solution945. E Hence swapping the words in the positions (A) and (C) will frame
All the highlighted words in their original positions are a grammatically correct and contextually meaningful sentence.
grammatically correct and contextually meaningful and thus Therefore, option (a) is the most suitable answer choice.
successfully form a coherent sentence without interchanging any
of the words from their positions. Hence, option (e) is the most
viable answer choice.

Solution946. C
The positions of the words “confined” and “people” are incorrect
as the appropriate. Hence swapping the words in the positions (B)
and (C) will frame a grammatically correct and contextually
meaningful sentence. Therefore, option (c) is the most suitable
answer choice.

Solution947. D
The positions of the words “assess” and “bear” are incorrect.
Hence swapping the words in the positions (A) and (C) will frame
a grammatically correct and contextually meaningful sentence.
Therefore, option (d) is the most suitable answer choice.

Solution948. B
The positions of the words “need” and “function” are incorrect.
Hence swapping the words in the positions (B) and (D) will frame
a grammatically correct and contextually meaningful sentence.
Therefore, option (b) is the most suitable answer choice.

Facebook Page- https://www.facebook.com/vishalpariharpage Youtube- https://www.youtube.com/vishalparihar


Follow
224 Vishal Sir Telegram Channel- https://t.me/englishbyvishalsirchannel Instagram- https://www.instagram.com/vishalthetrainer
By Vishal sir
Also Important For SBI CLERK, IBPS CLERK, RRB CLERK
& Other Competitive Exams

Q952. Directions (956-960): In each of the following questions a


The (A) quarantined of COVID-19 has led to a peculiar (B) sentence is given with four highlighted words. Choose the
afford wherein those who can otherwise (C) scenario private alternative, reflecting the appropriate replacement of the
healthcare are now relying on government facilities to be (D) words so as to form a grammatically correct and contextually
advent and tested. meaningful sentence. If no replacement is required, mark (e)
i.e. “no correction required” as your answer choice.
(a) a-c
(b) a-d and b-c Q956. The G20, with (A) present of other affected (B) serve, itself
(c) a-b and c-d might (C) countries the purpose for the (D) co-option.
(d) a-d (a) B – D
(e) No change required (b) B – C
(c) A – D and B – C
Q953. (d) A – C
The (A) flew became grave after a family that (B) situation down (e) No correction required
from Italy (C) virus positive for the (D) tested.
(a) a-c Q957. The world can ill-afford (A)delays, as the pandemic is
(b) a-d and b-c (B)predicted to stage a (C)comeback once the (D)shutdowns
(c) a-b and c-d are gradually relaxed.
(d) a-d (a) B – D
(e) No change required (b) B – C
(c) A – D and B – C
Q954. (d) A – C
Each life is (A) number, and any drop in our (B) guard can (e) No correction required
quickly raise the (C) precious of the infected, thereby
(D)undermining the gains achieved. Q958.
There is a need to (A) augment easy movement of trained health
(a) a-c (B)professionals across the world to train others and
(b) a-d and b-c (C)facilitate resources wherever there are (D)shortages.
(c) a-b and c-d (a) a-c
(d) a-d (b) a-b
(e) No change required (c) a-b and c-d
(d) a-d
Q955. (e) No change required
(A) humanity the world must (B)stand together in the (C) battle
for the safety and wellbeing of (D)today. Q959.
We must (A) shutdowns food shortages (B) consequent sooner
(a) a-c or later, in some part of the world, (C) occurring to the national
(b) a-d and b-c (D) anticipate.
(c) a-b and c-d (a) a-c
(d) a-d (b) a-d and b-c
(e) No change required (c) a-b and c-d
(d) a-d
(e) No change required

Facebook Page- https://www.facebook.com/vishalpariharpage Youtube- https://www.youtube.com/vishalparihar


Follow
225 Vishal Sir Telegram Channel- https://t.me/englishbyvishalsirchannel Instagram- https://www.instagram.com/vishalthetrainer
By Vishal sir
Also Important For SBI CLERK, IBPS CLERK, RRB CLERK
& Other Competitive Exams

Solution 952: (b) will frame a grammatically correct and contextually meaningful
The positions of the words “quarantined” - “advent” and “afford” sentence. Therefore, option (a) is the most suitable answer choice.
– “scenario” should be interchanged. Hence swapping the words
in the positions (A) – (D) and (B) – (C) will frame a Solution 959: (b)
grammatically correct and contextually meaningful sentence. The positions of the words “shutdowns” - “anticipate” and
Therefore, option (b) is the most suitable answer choice. “consequent” – “occurring” should be interchanged. Hence
swapping the words in the positions (A) – (D) and (B) – (C) will
Solution 953: (c) frame a grammatically correct and contextually meaningful
The positions of the words “flew” - “situation” and “virus” – sentence. Therefore, option (b) is the most suitable answer
“tested” should be interchanged. Hence swapping the words in choice.
the positions (A) – (B) and (C)– (D) will frame a grammatically
correct and contextually meaningful sentence. Therefore, option
(c) is the most suitable answer choice.

Solution 954: (a)


The positions of the words “number” and “precious” are
incorrect. Hence swapping the words in the positions (A) and (C)
will frame a grammatically correct and contextually meaningful
sentence. Therefore, option (a) is the most suitable answer choice.

Solution 955: (d)


The positions of the words “humanity” and “today” are incorrect.
Hence swapping the words in the positions (A) and (D) will frame
a grammatically correct and contextually meaningful sentence.
Therefore, option (d) is the most suitable answer choice.

Solution956. C
The positions of the words “present” - “co-option” and “serve” –
“countries” should be interchanged. Hence swapping the words
in the positions (A) – (D) and (B) – (C) will frame a
grammatically correct and contextually meaningful sentence.
Therefore, option (b) is the most suitable answer choice.

Solution957. E
All the highlighted words in their original positions are
grammatically correct and contextually meaningful and thus
successfully form a coherent sentence without interchanging any
of the words from their positions. Hence, option (e) is the most
viable answer choice.

Solution 958: (a)


The positions of the words “augment” and “facilitate” are
incorrect. Hence swapping the words in the positions (A) and (C)

Facebook Page- https://www.facebook.com/vishalpariharpage Youtube- https://www.youtube.com/vishalparihar


Follow
226 Vishal Sir Telegram Channel- https://t.me/englishbyvishalsirchannel Instagram- https://www.instagram.com/vishalthetrainer
By Vishal sir
Also Important For SBI CLERK, IBPS CLERK, RRB CLERK
& Other Competitive Exams

Q960. It is (A) isolates to sequence the virus (B) important in at least


three different (C)sequencing in India to ensure that (D) Q965. He __________ the identities of new visitors and stopped any
institutions errors are eliminated. hawker who would _____ into the building during the quiet
(a) a-c afternoons.
(b) a-d and b-c (a) verified, wander (b) checked, roaming
(c) a-b and c-d (c) scrutiny, running (d) investigated, wanderer
(d) a-d (e) None of these
(e) No change required
Directions : In each of the questions below, there is a word
Directions (961-965): In following questions, a sentence is given in bold which is followed by five options. In each of the
given with two blanks. From the given alternatives, choose the options, a pair of words is given which is either the pair of
most appropriate set of words that fills the blank forming a synonyms or antonyms or synonym & antonym of the word
grammatically correct and contextually meaningful sentence. given in bold. Choose that pair as your answer.
Q961. Some thinkers hold that mathematics is a kind of language--a
systematic _________ of signs, the criteria for the authority of Q966. AFFINITY
which are internal ________, elegance, and depth. (a) rapport : abolish
(a) impetus, alliance (b) original : sympathy
(b) seditious, imprecision (c) partiality : absurd
(c) rigorous, nuance (d) affection : sympathy
(d) contrivance, coherence (e) hatred : absolve
(e) radical, congruence
Q967. SCRUTINIZE
Q962. Gmail is very effective at filtering spam emails but _________ (a) analyze : oversight
seem to have figured out a new way to _________ the spam (b) inspect : adapt
filters and send emails that land right in the user’s inbox. (c) omission : amend
(a) estrangement, perturbate (b) conformity, disdain (d) neglect : antidote
(c) discord, affinity (d) spammers, bypass (e) examine : apposite
(e) perseverance, fervor
Q968. CONGRUOUS
Q963. Schools need boost air ventilation to create a safe and (a) agreement : accordance
_________ environment for students and increase the amount of (b) application : harmony
(c) appreciation : unison
_________ air that circulates through their HVAC systems.
(d) apprehension : compatibility
A.healthy, outdoor B.sound, ground (e) arrogance : discordance
C.health, surface D.perfect, creative
E.secure, circulation Q969. IMPEDIMENT
(a) subvert : convert
Q964. The policy ________ child labor exists but lack of enforcement (b) depose : deflate
of labor restrictions _________ child labor. (c) assistance : facilitation
(d) attribute : diverse
(a) curbing, perpetuates (b) emancipating, untamed
(e) compress : distort
(c) generating, tortures (d) reclaiming, assort
(e) ambulating, indignant

Facebook Page- https://www.facebook.com/vishalpariharpage Youtube- https://www.youtube.com/vishalparihar


Follow
227 Vishal Sir Telegram Channel- https://t.me/englishbyvishalsirchannel Instagram- https://www.instagram.com/vishalthetrainer
By Vishal sir
Also Important For SBI CLERK, IBPS CLERK, RRB CLERK
& Other Competitive Exams

Solution 960: (c) sense. In the 2nd blank, only ‘prevent, gives the exact meaning,
The positions of the words “isolates” - “important” and as before this blank ‘aggressive’ has been mentioned. grown,
“sequencing” – “institutions” should be interchanged. Hence convert; cultivated, reach; poked, make ----- these pairs have no
swapping the words in the positions (A) – (B) and (C)– (D) will connection with the sentence grammatically and contextually.
frame a grammatically correct and contextually meaningful Hence, these do not fit in the sentence
sentence. Therefore, option (c) is the most suitable answer choice. 964Ans. (a)
961Ans. (d) ‘curbing, perpetuates’ best suits the purpose.
‘contrivance, coherence’ are best fit for the blank. Curbing means to limit or control something, especially
Seditious means inciting or causing people to rebel against the something bad
authority of a state or monarch. Assort means place in a group; classify.
Rigorous means extremely thorough and careful. Ambulating means walk; move about.
Nuance means a subtle difference in or shade of meaning, Indignant means feeling or showing anger or annoyance at what
expression, or sound. is perceived as unfair treatment.
Contrivance means the use of skill to create or bring about 965Ans. (a)
something, especially with a consequent effect of artificiality. ‘verified, wander’, fits in the sentence correctly. Other options
Congruence means agreement or harmony; compatibility. do not fit in the sentence.
962Ans. (d) S966. Ans. (d)
‘spammers, bypass’ is the correct set of words making the Sol. “Affinity” means a natural liking for and understanding of
sentence correct and meaningful. someone or something. Therefore, “affection : sympathy” is the
Estrangement means the fact of no longer being on friendly set of words that expresses the meaning of the given highlighted
terms or part of a social group. word. Hence, option (d) is the most suitable answer choice.
Perturbate means anxiety; mental uneasiness. Rapport means a close and harmonious relationship in which the
Discord means disagreement between people. people or groups concerned understand each other's feelings or
Perseverance means persistence in doing something despite ideas and communicate well.
difficulty or delay in achieving success. Abolish means formally put an end to (a system, practice, or
Fervor means intense and passionate feeling. institution).
963Ans. (c) Absurd means wildly unreasonable, illogical, or inappropriate.
‘reared, prevent’, fits in the sentence correctly. After S967. Ans. (a)
‘commercially’ and before ‘chicken’, ‘reared’ gives the exact

Facebook Page- https://www.facebook.com/vishalpariharpage Youtube- https://www.youtube.com/vishalparihar


Follow
228 Vishal Sir Telegram Channel- https://t.me/englishbyvishalsirchannel Instagram- https://www.instagram.com/vishalthetrainer
By Vishal sir
Also Important For SBI CLERK, IBPS CLERK, RRB CLERK
& Other Competitive Exams

Sol. “Scrutinize” means examine or inspect closely and


thoroughly. Therefore, the set of words that consists of its S969. Ans. (c)
synonym and antonym is “analyze : oversight”. Hence, option (a) Sol. “Impediment” means a hindrance or obstruction in doing
is the most suitable answer choice. something. Therefore, the set of words that reflect the antonyms
Analyze means examine (something) methodically and in detail, of the given highlighted word is “assistance : facilitation”. Hence,
typically in order to explain and interpret it. option (c) is the most suitable answer choice.
Oversight means an unintentional failure to notice or do Subvert means undermine the power and authority of (an
something. established system or institution).
Adapt means make (something) suitable for a new use or purpose; Depose means remove from office suddenly and forcefully.
modify. Facilitation means the action of facilitating something.
Amend means make minor changes to (a text, piece of legislation, Distort means pull or twist out of shape.
etc.) in order to make it fairer or more accurate, or to reflect
changing circumstances.
Antidote means a medicine taken or given to counteract a
particular poison.
Apposite means apt in the circumstances or in relation to
something.
S968. Ans. (a)
Sol. “Congruous” means in agreement or harmony. Therefore, the
set of words that reflect the synonyms of the given highlighted
word is “agreement : accordance”. Hence, option (a) is the most
suitable answer choice.
Accordance means in a manner conforming with.
Harmony means the state of being in agreement or concord.
Unison means simultaneous performance or utterance of action
or speech.
Apprehension means understanding; grasp.
Arrogance means the quality of being arrogant.
Discordance means unconformity.

Facebook Page- https://www.facebook.com/vishalpariharpage Youtube- https://www.youtube.com/vishalparihar


Follow
229 Vishal Sir Telegram Channel- https://t.me/englishbyvishalsirchannel Instagram- https://www.instagram.com/vishalthetrainer
By Vishal sir
Also Important For SBI CLERK, IBPS CLERK, RRB CLERK
& Other Competitive Exams

Q970. Which of the following options best explain the meaning of (b) appease
‘missives’? (c) mollify
(a)Address (d) exacerbate
(b)Invitations (e) slump
(c)Letters
(d)Notebooks Directions: Read each sentence any to find out whether there
(e)None of the above is any grammatical error or idiomatic error in it. The error,
if any, will be in one part of the sentence. If
Q971. Which of the following words has a meaning CLOSER to the there is no error, the answer is No error. (Ignore errors of
opposite to the word ‘INTRUIGING’? punctuation, if any)
(a)Interesting
(b) Fascinating Q976. The bus was (a) / hired by (b) / the ladies (c) / for its picnic. (d)
(c) Attracting / No Error (e)
(d) Boring Q977. Within an hour’s (a) / journey we (b) / will approach near (c) /
(e) Tempting Patna. (d) / No Error. (e)
Q972. Which of the following words has a meaning CLOSER to the Q978. We had swam (a) / across the river (b) / before (c) / the sun set.
word ‘CELESTIAL’? (d) / No Error. (e)
(a) Propitiate
(b) Astronomical Q979. Neither Rakesh(a) / nor (b) / I are leaving (c) / for Hyderabad.
(c) Pacify (d) / No Error (e)
(d) Constitute
(e) Mollify Q980. Death (a) / is (b) / preferable (c) / than life. (d) / No Error (e)
Q973. Which of the following options best explains the meaning of Q981. If (a) / it snowed tomorrow (b) / we’ll go (c) / skating. (d) / No
‘surge’? Error (e)
(a)Escalate
(b)Ecstasy Q982. You may rest assured (a) / that Madhav knows (b) / more than I
(c)Eclectic (c) / in this matter. (d) / No Error. (e)
(d)Emulation
(e) None of the above Q983. Every one of us (a) / knows that given (b) / an option he prefers
(c) / cold drink than coffee. (d) / No Error. (e)
Q974. Which of the following options best explains the meaning of
‘reverberating’? Q984. He has no affection (a) / with anybody (b) / including the
(a)Retrospect members (c) / of his own family. (d) / No Error. (e)
(b)Resentment
(c)Resonate Q985. I am glad (a) / to hear that (b) / you narrowly escaped (c) / being
(d)Revive run over yesterday. (d) / No Error. (e)
(e) None of the above
Q986. Both the girls (a) / helped one another (b) / when both of them
Q975. Which of the following words has a meaning which is SIMILAR (c) / were in difficulty. (d) / No Error. (e)
to the meaning of the word, ‘aggravated’?
(a) placate

Facebook Page- https://www.facebook.com/vishalpariharpage Youtube- https://www.youtube.com/vishalparihar


Follow
230 Vishal Sir Telegram Channel- https://t.me/englishbyvishalsirchannel Instagram- https://www.instagram.com/vishalthetrainer
By Vishal sir
Also Important For SBI CLERK, IBPS CLERK, RRB CLERK
& Other Competitive Exams

S970. Ans. (c) Hence option (c) is the correct answer choice for the given
Sol. Missives- a letter, especially a long or official one. question.
Hence option (c) is the correct answer.
S975. Ans. (d)
Sol. aggravated [verb] means ‘make (a problem, injury, or
S971. Ans. (d) offense) worse or more serious’;
Sol. Among the given options, the option (d) has a meaning which Placate [verb] means ‘make (someone) less angry or hostile’;
is OPPOSITE to the word ‘intriguing’ and rest are the options Appease [verb] means ‘pacify or placate (someone) by acceding
which have the same meaning as the given word, and Hence, to their demands; assuage or satisfy (a demand or a feeling)’;
option (d) is the correct answer choice. Mollify [verb] means ‘appease the anger or anxiety of
(someone)’;
S972. Ans. (b) Exacerbate [verb] means ‘make (a problem, bad situation, or
Sol. Celestial- positioned in or relating to the sky, or outer space negative feeling) worse’;
as observed in astronomy. Slump [verb] means ‘fail or decline substantially; undergo a
Propitiate- win or regain the favour of (a god, spirit, or person) sudden severe or prolonged fall in price, value, or amount; sit,
by doing something that pleases them. lean, or fall heavily and limply’;
Among the given options, the option (b) has a meaning which is From above, it can be deduced that the word exacerbate, has a
closer to the word ‘Celestial’ and hence, is the correct answer. meaning which is SIMILAR to the word ‘aggravate’. Hence, the
option (d) is the correct answer.
S973. Ans. (a) Q976.Ans. (c)
Sol. Surge- a sudden powerful forward or upward movement, The Use of ‘near’ is superfluous as ‘approach’ itself means ‘to
especially by a crowd or by a natural force such as the tide. come close’.
Ecstasy- an overwhelming feeling of great happiness or joyful Q977.Ans. (a)
excitement ‘swum’ will be used in place of ‘swam’ as the past form of
Eclectic deriving ideas, style, or taste from a broad and diverse ‘swim’ is ‘swam’ and its past participle form is ‘swum’ and
range of sources moreover after have/ has/ had/ having, past participle form of
Hence option (a) is the correct answer choice for the given verb means V3 is used.
question. Q978.Ans. (c)
Use ‘am’ in place of ‘are’ as with ‘neither… nor’, ‘either… or’,
S974. Ans. (c) ‘not only… but also’, the verb is used according to the subject
Sol. Reverberating- (of a loud noise) be repeated several times as close to it.
an echo Ex. Either you or he has committed a blunder.
Retrospect a survey or review of a past course of events or period Q979.Ans. (d)
of time. ‘to’ will be used in place of ‘than’ as after ‘preferable, senior,
Resentment bitter indignation at having been treated unfairly junior, superior, inferior, prior, anterior, posterior’, ‘to’ is used.
Resonate- produce or be filled with a deep, full, reverberating Ex. She is junior to you.
sound. This is preferable

Facebook Page- https://www.facebook.com/vishalpariharpage Youtube- https://www.youtube.com/vishalparihar


Follow
231 Vishal Sir Telegram Channel- https://t.me/englishbyvishalsirchannel Instagram- https://www.instagram.com/vishalthetrainer
By Vishal sir
Also Important For SBI CLERK, IBPS CLERK, RRB CLERK
& Other Competitive Exams

Q980.Ans. (b)
‘snows’ is the correct use instead of ‘snowed’. There are two
incidents of future (tomorrow, we’ll go) hence simple present
tense is used in subordinate clause (if, till, until, before, after).
Ex. If it rains, we shall not go out.
Q981.Ans. (a)
‘You may be rest assured’ will be used in place of ‘you may rest
assured’ as ‘may+ V1’ is used in active voice whereas ‘may +be
+ V3’ is used in passive voice.
Ex. I may assure you that you will succeed. (Active)
You may be assured that you will succeed. (Passive)
Q982.Ans. (d)
Use ‘to’ in place of ‘than’ as after ‘prefer’, preposition ‘to’ is
used not ‘than’.
Q983.Ans. (b)
Use ‘for’ in place of ‘with’.
Q984.Ans. (e)
The sentence is grammatically correct.
Q985.Ans. (b)
‘Each other’ will be used in place ‘one another’ as ‘one another’
is used for two or more than two whereas ‘each other’ is used
for only two.
Ex. The three sisters love one another.
The two sisters love each other.
Q986.Ans. (a)
‘will talk’ will be used in place of ‘talked’ as the sentence
contains two actions of future.
Ex. He will go to his sister and then he will go to his friend.

Facebook Page- https://www.facebook.com/vishalpariharpage Youtube- https://www.youtube.com/vishalparihar


Follow
232 Vishal Sir Telegram Channel- https://t.me/englishbyvishalsirchannel Instagram- https://www.instagram.com/vishalthetrainer
By Vishal sir
Also Important For SBI CLERK, IBPS CLERK, RRB CLERK
& Other Competitive Exams

Q987. They talked for a while (a)/ and then they will (b)/ continue to Rome as a gift for her. One of the most ___(1000)___ women of
play the game (c)/ till tomorrow morning. (d)/ No error. (e) her time, Ingrid Bergman was never anything ___(1001)___ her
supremely simple self. At the peak of her ___(1002)___, Ingrid
Q988. Some skills are (a)/ more difficult than others (b)/ so it takes a
insisted on taking screen tests and refused leads in favour of lesser
longer time (c)/ to master them. (d)/ No error. (e)
but more ___(1003)___ roles. When the director told his
Q989. The guide told us (a)/ that where the island was (b)/ and went on prospective star that they would have to change her name, cap her
(c)/ narrating its history. (d)/ No error. (e) ___(1004)___ And pluck her eyebrows, Ingrid threatened to
return to Sweden. And so the famous ___(1005)___ look was
Q990. Even after hearing him (a)/ for a fairly long time (b)/ we failed born.
to understand (c)/ that he was harping on. (d)/ No error. (e)

Q991. Prabhakar arrives in the office (a)/ as fresh as he starts (b)/ from Q 996. (a) shoes (b) hat (c) jacket
home even though (c)/ he commutes a long distance. (d)/ No (d) gloves (e) make-up
error. (e)
Q 997. (a) increased (b) decreased (c) lessened
Q992. Jagdish met me yesterday (a)/ but he did not tell me (b)/ that he (d) remained steady (e) indifferent
has (c)/ resigned from the factory. (d)/ No error. (e)
Q 998. (a) brought (b) bought (c) sold
Q993. It is unfortunate (a)/ but still a reality (b)/ that poverty and
(d) boarded (e) borrowed
development (c)/ goes hand in hand (d)/ No error. (e)

Q994. I know who (a)/ this job should be (b)/ entrusted to (c)/ for Q 999. (a) accountant (b) administrator (c) accomplice
smooth handing. (d)/ No error. (e) (d) acrobat (e) admirer

Q995. The Supervisor noticed that (a)/ one of the employees were (b)/ Q 1000.(a) non-descript (b) average (c) self-effacing
involved in (c)/ unfair activities outside the office. (d)/ No error.
(d) glamorous (e) unknown
(e)
Q 1001.(a) rather (b) because (c) less than
Directions (996-1005): In the following passage, some of the (d) rather than (e) but
words have been left out, each of which is indicated by a
number. Find the suitable word from the options given Q 1002.(a) freedom (b) kingdom (c) famous
against each number and fill up the blanks with appropriate (d) stardom (e) dukedom
words to make the paragraph meaningfully complete.
Q 1003.(a) monotonous (b) humdrum (c) boring
When she appeared on the screen without ___(996)___cosmetic
(d) challenging (e) similar
sales declined. When she played a nun, convent enrolments
Q 1004.(a) nose (b) cheeks (c) ears
___(997)___. Letters were delivered to her addressed simply
(d) teeth (e) elbows
“Ingrid Bergman”. Industrialist Howard Hughes once
___(998)___ every available air ticket from New York to Los
Q 1005.(a) unnatural (b) artificial (c) natural
Angeles to be sure she would accept a seat in his private plane.
(d) make-up (e) dramatic
An ___(999)___ walked a sheep all the way from Sweden to

Facebook Page- https://www.facebook.com/vishalpariharpage Youtube- https://www.youtube.com/vishalparihar


Follow
233 Vishal Sir Telegram Channel- https://t.me/englishbyvishalsirchannel Instagram- https://www.instagram.com/vishalthetrainer
By Vishal sir
Also Important For SBI CLERK, IBPS CLERK, RRB CLERK
& Other Competitive Exams

Q987.Ans. (e)
The sentence is grammatically correct.
Q988.Ans. (b)
The use of ‘that’ before ‘where’ is superfluous.
Q989.Ans. (d)
Use ‘what’ in place of ‘that’. ‘Harping on’ means talk or write .
persistently and tediously on.
Q990.Ans. (e)
The sentence is grammatically correct.
Q991.Ans. (c)
‘had’ will be used in place of ‘has’ as the sentence is in past
tense.
Q992.Ans. d)
‘go’ will be used in place of ‘goes’ as the subject of ‘that- clause’
is plural, therefore plural verb is used.
Ex. Oil and water do not mix
Q993.Ans. (a)
‘who’ should be replaced by to ‘whom’
Q994.Ans. (a)
‘was’ will be used in place of ‘were’ as the subject of ‘that-
clause’ is ‘one’ Ex. One of these girls is very beautiful.
Q995.Ans. (c)
‘depends’ will be used in place of ‘depend’ as the subject of the
sentence is singular.
Q996.Ans.(e)
Q997.Ans.(a)
Q998.Ans.(b)
Q999.Ans.(e)
Q1000.Ans.(d)
Q1001.Ans.(e)
Q1002.Ans.(d)
Q1003.Ans.(d)
Q1004.Ans.(d)
Q1005.Ans.(e)

Facebook Page- https://www.facebook.com/vishalpariharpage Youtube- https://www.youtube.com/vishalparihar


Follow
234 Vishal Sir Telegram Channel- https://t.me/englishbyvishalsirchannel Instagram- https://www.instagram.com/vishalthetrainer
By Vishal sir
Also Important For SBI CLERK, IBPS CLERK, RRB CLERK
& Other Competitive Exams

Facebook Page- https://www.facebook.com/vishalpariharpage Youtube- https://www.youtube.com/vishalparihar


Follow
235 Vishal Sir Telegram Channel- https://t.me/englishbyvishalsirchannel Instagram- https://www.instagram.com/vishalthetrainer

You might also like